Deja Review USMLE Step 1, Second Edition

217

Transcript of Deja Review USMLE Step 1, Second Edition

DEJA REVIEW™USMLE Step 1

NOTICE

Medicine i s an ever-changing science. As new research and cl inica l experience broaden our knowledge, changes in treatment and drugtherapy are required. The authors and the publ i sher of this work have checked with sources bel ieved to be rel iable in thei r efforts to provideinforma on that i s complete and genera l ly in accord with the s tandards accepted at the me of publ ica on. However, in view of theposs ibi l i ty of human error or changes in medica l sciences , nei ther the authors nor the publ i sher nor any other party who has been involved inthe prepara on or publ ica on of this work warrants that the informa on conta ined herein i s in every respect accurate or complete, and theydiscla im a l l respons ibi l i ty for any errors or omiss ions or for the resul ts obta ined from use of the informa on conta ined in this work. Readersare encouraged to confirm the informa on conta ined herein with other sources . For example and in par cular, readers are advised to checkthe product informa on sheet included in the package of each drug they plan to adminis ter to be certa in that the informa on conta ined inthis work i s accurate and that changes have not been made in the recommended dose or in the contra indica ons for adminis tra on. Thisrecommendation i s of particular importance in connection with new or infrequently used drugs .

DEJA REVIEW™

USMLE Step 1

Second Edition

John H. Naheedy, MDFel low, Pediatric Radiology

Department of RadiologyChi ldren’s Hospi ta l Boston

Harvard Medica l SchoolBoston, Massachusetts

Daniel A. Orringer, MDChief Res ident

Department of NeurosurgeryUnivers i ty of Michigan Medica l School

Ann Arbor, Michigan

Copyright © 2010, 2006 by John H. Naheedy and Daniel A. Orringer. Al l rights reserved. Except as permi ed under the United States Copyright Actof 1976, no part of this publ ica on may be reproduced or dis tributed in any form or by any means , or s tored in a database or retrieva l system,without the prior wri tten permiss ion of the publ i sher.

ISBN: 978-0-07-163786-2

MHID: 0-07-163786-9

The materia l in this eBook a lso appears in the print vers ion of this ti tle: ISBN: 978-0-07-162718-4, MHID: 0-07-162718-9.

Al l trademarks are trademarks of thei r respec ve owners . Rather than put a trademark symbol a er every occurrence of a trademarked name,we use names in an edi toria l fashion only, and to the benefit of the trademark owner, with no inten on of infringement of the trademark.Where such des ignations appear in this book, they have been printed with ini tia l caps .

McGraw-Hi l l eBooks are ava i lable at specia l quan ty discounts to use as premiums and sa les promo ons , or for use in corporate tra iningprograms. To contact a representative please e-mai l us at bulksa les@mcgraw-hi l l .com.

TERMS OF USE

This i s a copyrighted work and The McGraw-Hi l l Companies , Inc. (“McGraw-Hi l l ”) and i ts l i censors reserve a l l rights in and to the work. Use ofthis work i s subject to these terms. Except as permi ed under the Copyright Act of 1976 and the right to s tore and retrieve one copy of the work,you may not decompi le, disassemble, reverse engineer, reproduce, modi fy, create deriva ve works based upon, transmit, dis tribute,disseminate, sel l , publ i sh or subl icense the work or any part of i t wi thout McGraw-Hi l l ’s prior consent. You may use the work for your ownnoncommercia l and personal use; any other use of the work i s s trictly prohibi ted. Your right to use the work may be terminated i f you fa i l tocomply with these terms.

THE WORK IS PROVIDED “AS IS.” McGRAW-HILL AND ITS LICENSORS MAKE NO GUARANTEES OR WARRANTIES AS TO THE ACCURACY, ADEQUACY ORCOMPLETENESS OF OR RESULTS TO BE OBTAINED FROM USING THE WORK, INCLUDING ANY INFORMATION THAT CAN BE ACCESSED THROUGH THEWORK VIA HYPERLINK OR OTHERWISE, AND EXPRESSLY DISCLAIM ANY WARRANTY, EXPRESS OR IMPLIED, INCLUDING BUT NOT LIMITED TO IMPLIEDWARRANTIES OF MERCHANTABILITY OR FITNESS FOR A PARTICULAR PURPOSE. McGraw-Hi l l and i ts l i censors do not warrant or guarantee that thefunc ons conta ined in the work wi l l meet your requirements or that i ts opera on wi l l be uninterrupted or error free. Nei ther McGraw-Hi l l nori ts l i censors sha l l be l iable to you or anyone else for any inaccuracy, error or omiss ion, regardless of cause, in the work or for any damagesresul ng there from. McGraw-Hi l l has no respons ibi l i ty for the content of any informa on accessed through the work. Under no ci rcumstancesshal l McGraw-Hi l l and/or i ts l i censors be l iable for any indirect, incidenta l , specia l , puni ve, consequen al or s imi lar damages that resul tfrom the use of or inabi l i ty to use the work, even i f any of them has been advised of the poss ibi l i ty of such damages . This l imita on ofl iabi l i ty sha l l apply to any cla im or cause whatsoever whether such cla im or cause arises in contract, tort or otherwise.

To my family and friends, for their love and encouragement; and to my parents, for being an example of everything I want to be.—John

To my mother and my family.—Dan

Contents

ContributorsStudent ReviewersPrefaceAcknowledgments

Chapter 1 BASIC PRINCIPLESMolecular Biology, Biochemistry, and GeneticsNutri tionGenera l EmbryologyBas ic PathologyBas ic Pharmacology

Chapter 2 IMMUNOLOGYImmunology Bas icsPathology Immune System

Chapter 3 MICROBIOLOGY AND INFECTIOUS DISEASESBacteriologyAntibioticsVirologyAntivi ra l AgentsMycologyAnti fungalsParas i tology

Chapter 4 NEUROSCIENCEEmbryologyNeuroanatomy and Neurophys iologyPathology of the Nervous SystemPharmacology of the Nervous System

Chapter 5 CARDIOVASCULAREmbryologyAnatomyPhys iologyCardiovascular Pathology and Pathophys iologyPharmacology

Chapter 6 PULMONARYEmbryologyAnatomyPhys iologyPathologyOther PathologyPharmacology

Chapter 7 GASTROENTEROLOGYEmbryologyAnatomyPhys iologyPathologyPharmacology

Chapter 8 REPRODUCTION AND ENDOCRINOLOGYEmbryologyAnatomyPhys iologyPathologyPharmacology

Chapter 9 RENAL AND GENITOURINARYEmbryologyAnatomyPhys iologyFunctional Regions of the NephronEndocrine Functions of the KidneyPathologyPharmacology—Renal

Chapter 10 HEMATOLOGY AND ONCOLOGYEmbryologyPathologyPharmacology

Chapter 11 SKIN AND CONNECTIVE TISSUESEmbryology/Anatomy/His tologyPathology

Chapter 12 MUSCULOSKELETALAnatomyPhys iologyMusculoskeleta l and Connective Tissue Pathology

Chapter 13 BEHAVIORAL SCIENCESLi fe CyclePhys iologySubstance AbusePsychologyPsychiatryPharmacology—PsychopharmacologyMedica l EthicsBiostatis ticsPubl ic Heal th/Epidemiology

Chapter 14 MAKE THE DIAGNOSISNutri tionMicrobiology and Infectious DiseasesNeuroscienceCardiovascularPulmonaryGastroenterologyReproductive/EndocrineRenal and GenitourinaryHematology/OncologySkin and Connective TissuesMusculoskeleta lBehaviora l Science

Appendix ABBREVIATIONS

Index

Contributors

Alexander ChooMedica l StudentClass of 2010Univers i ty of Ca l i fornia , San DiegoLa Jol la , Ca l i forniaChapters: Pulmonary, Skin and Connective Tissues, Musculoskeletal

Charlotte GoreMedica l StudentClass of 2010Univers i ty of Ca l i fornia , San DiegoLa Jol la , Ca l i forniaChapters: Basic Principles, Neuroscience, Gastroenterology, Hematology and Oncology, Behavioral Science, Make the Diagnosis

Kristen A. KippsMedica l StudentClass of 2010Univers i ty of Ca l i fornia , San DiegoLa Jol la , Ca l i forniaChapters: Immunology, Microbiology and Infectious Diseases

Brian S. PugmireMedica l StudentClass of 2010Univers i ty of Ca l i fornia , San DiegoLa Jol la , Ca l i forniaChapter: Renal and Genitourinary

Krishna C. RaviMedica l StudentClass of 2010Univers i ty of Ca l i fornia , San DiegoLa Jol la , Ca l i forniaChapter: Cardiovascular

Omeed SaghafiMedica l StudentClass of 2010Univers i ty of Ca l i fornia , San DiegoLa Jol la , Ca l i forniaChapter: Reproduction and Endocrinology

Student Reviewers

Betty ChungMD/PhD CandidateUnivers i ty of Medicine and Dentis try of New JerseySchool of Osteopathic MedicineClass of 2012

Sarah FabianoSUNY Upstate Medica l Univers i tyClass of 2010

Ali Rizvi, MPHLake Erie Col lege of Osteopathic MedicineClass of 2010

Preface

Déjà Review™ USMLE Step 1 has been scru nized and edi ted to produce a second edi on that i s even higher yield and eas ier to use than thefirst. Outstanding medica l s tudents who have recently taken Step 1 have revised the origina l text to ensure the materia l covered herein i scomplete and current. The authors , now with a combined 30 years of experience in the medica l field, have a lso edi ted the manuscript toemphas ize the relevant core concepts covered in the USMLE Step 1 examina on. We are confident that our efforts have produced one of themost useful guides for Step 1 review ava i lable today.The main objec ve of a medica l s tudent preparing for Step 1 of the United States Medica l Licens ing Examina on (USMLE) i s to commit a vastbody of knowledge to memory. Having recently prepared for Step 1, we rea l i ze how daun ng this task can be. We feel there are two mainprinciples that wi l l a l low you to be success ful in your prepara ons for Step 1: (1) repe on of key facts and (2) us ing review ques ons togauge your comprehens ion and memory. The Déjà Review™ series i s a unique resource that has been des igned to a l low you to review theessen al facts and determine your level of knowledge on the subjects tested on Step 1. We a lso know, from experience, that bui lding a sol idfoundation in the bas ic sciences wi l l a l low you to make a smooth trans i tion into the cl inica l years of medica l school .

ORGANIZATION

Al l concepts are presented in a ques on and answer format that covers the key facts on hundreds of commonly tested USMLE Step 1 topics .The materia l i s divided into chapters organized by body systems. Specia l emphas is has been placed on the molecular and gene c bas is ofpathology, as this area has become increas ingly emphas ized in recent examinations .This question and answer format has severa l important advantages :

It provides a rapid, s tra ightforward way for you to assess your s trengths and weaknesses .It a l lows you to efficiently review and commit to memory a large body of information.It offers a break from tedious , convoluted multiple-choice questions .The “Make the Diagnos is” section exposes you to the prototypic presentation of diseases class ica l ly tested on the USMLE Step 1.It serves as a quick, las t-minute review of high-yield facts .

The compact, condensed des ign of the book i s conducive to s tudying on the go, especia l ly during any downtime throughout your day.

HOW TO USE THIS BOOK

This text has been sampled by a number of medica l s tudents who found i t to be an essen al part of thei r prepara on for Step 1, in addi onto their course examina ons . Remember, this text i s not intended to replace comprehens ive textbooks , course packs , or lectures . It i s s implyintended to serve as a supplement to your s tudies during your firs t 2 years of medica l school and throughout your prepara on for Step 1. Weencourage you to begin us ing this book early in your firs t year to reinforce topics covered in your course examina ons . We a lso recommendhaving the book spi ra l bound to make i t more portable and eas ier to use outs ide of the l ibrary or classroom. You may cover up the answerswith the included bookmark and quiz yoursel f or even your classmates . For a greater cha l lenge, try covering up the questions!However you choose to s tudy, we hope you find this resource helpful throughout your precl inica l years and during your prepara on for theUSMLE Step 1. Best of luck!

John H. Naheedy, MDDaniel A. Orringer, MD

Acknowledgments

The authors would l ike to thank the fol lowing individuals for thei r inva luable contribu ons to this text and their efforts in making this auseful , accurate resource for s tudents :Stephen M. Baird, MDProfessor Emeri tus of PathologySchool of MedicineUnivers i ty of Ca l i fornia , San DiegoVeterans Affa i rs San Diego Heal thcare SystemLa Jol la , Ca l i forniaVi Q. Bowman, MDFel low, Department of Internal MedicineDivis ion of Infectious DiseasesUnivers i ty of Ca l i fornia San Diego, Medica l CenterSan Diego, Ca l i forniaDavid L. Clark, PhDAssociate ProfessorDivis ion of AnatomyCol lege of MedicineOhio State Univers i tyColumbus , OhioJohn J. Curry III, PhDDirector, Problem-Based Learning PathwayAssociate Professor, Departments of Phys iology & Obstetrics and GynecologyCol lege of MedicineOhio State Univers i tyColumbus , OhioRobert M. DePhilip, PhDAssociate ProfessorDivis ion of AnatomyCol lege of MedicineOhio State Univers i tyColumbus , OhioRichard Fertel, PhDAssociate ProfessorDepartment of PharmacologyCol lege of MedicineOhio State Univers i tyColumbus , OhioHeather L. Hofflich, DODeparment of Internal MedicineDivis ion of Endocrinology, Diabetes and Metabol i smUnivers i ty of Ca l i fornia , San DiegoLa Jol la , Ca l i forniaJohn H. Hughes, PhDAssociate ProfessorDepartment of Molecular Vi rology, Immunology, and Medica l GeneticsCol lege of MedicineOhio State Univers i tyColumbus , OhioCarolyn J. Kelly, MDProfessorDept. of Internal MedicineAssociate Dean for Admiss ions and Student Affa i rsUnivers i ty of Ca l i fornia , San DiegoLa Jol la , Ca l i forniaDavid E. Krummen, MDAss is tant Professor of MedicineDivis ion of Electrophys iologyUnivers i ty of Ca l i fornia San DiegoVeterans Affa i rs San Diego Heal thcare SystemLa Jol la , Ca l i forniaNora D. Laiken, PhDAss is tant Dean of Educational Support ServicesSchool of MedicineUnivers i ty of Ca l i fornia , San DiegoLa Jol la , Ca l i forniaJess Mandel, MDProfessorDepartment of Internal MedcineDivis ion of Pulmonary & Cri tica l CareAssociate Dean for Undergraduate Medica l EducationUnivers i ty of Ca l i fornia , San DiegoLa Jol la , Ca l i forniaKhashayar Mohebali, MDChief Res ident, Cl inica l InstructorDivis ion of Plastic and Reconstructive Surgery

Department of SurgeryDepartment of SurgeryUnivers i ty of Ca l i fornia , San FranciscoSan Francisco, Ca l i forniaAlexandra Schwartz, MDAssociate Cl inica l ProfessorDepartment of Orthopedic SurgeryUnivers i ty of Ca l i fornia , San Diego Medica l CenterSan Diego, Ca l i forniaPal L. Vaghy, MDAssociate ProfessorDepartment of Molecular and Cel lular BiochemistryCol lege of MedicineOhio State Univers i tyColumbus , OhioThe authors would l ike to recognize the facul ty and s taff at the Ohio State Univers i ty Col lege of Medicine and dedicate this edi on to thememory of the late Dr. John M. Stang, for his constant encouragement and his endless commitment to educa on. We would a lso l ike to thankthe s tudents who used this text in prepara on for thei r boards and provided cri ca l feedback. Fina l ly, a specia l thanks to Li l ly Ghahremani ofFul l Ci rcle Li terary for her guidance, and to our managing edi tor Ki rs ten Funk for her patience and guidance throughout this project.

CHAPTER 1Basic Principles

MOLECULAR BIOLOGY, BIOCHEMISTRY, AND GENETICS

DNA, Genes, and Chromosomes

Which nucleotide bases are purines and which are pyrimidines?

“CUT the PY”: Cytos ine, Uraci l , and Thymine = PYrimidines ; “PURe As Gold”: PURines = adenine and guanineWhich proteins make up the core of a nucleosome?

Histones : H2A, H2B, H3, and H4Which proteins are associated with DNA between nucleosomes?

Histone H1Name the type of mutations described below:

Type of mutation that does not result in a change in amino acid sequenceSi lentType of mutation that results in a change in amino acid sequenceMissenseType of mutation that results in a stop codonNonsense “NO sense”Type of mutation that changes the reading frameFrameshi ftType of mutation in which a portion of DNA is lostDeletionType of mutation in which a single base is exchangedPoint

Name the type of cytogenetic disorders described below:Failure of chromosomes to disjoin properly during cell divisionNondis junctionLoss of a portion of a chromosomeDeletionTwo internal chromosomal breaks with inverted reincorporation of a portion of the chromosomeInvers ionSingle breaks in two chromosomes resulting in the exchange of segments between chromosomes without loss of genetic materialBalanced reciproca l trans locationSingle breaks in two acrocentric chromosomes resul ng in one large chromosome and one small chromosome accompanied by the loss of some gene cinformation, hereditary form of Down syndromeRobertsonian trans locationMitotic error in early development leading to the development of two karyotypically distinct populations of cells in an organismMosais icmWhat term is used to describe the AT-rich sequences in the genome where DNA replication begins?Origin of repl ication

DNA Replication, Transcription, and Translation

Name the protein(s) involved in replication or DNA repair with the functions listed below:

Stabilize single-stranded DNASingle-s tranded DNA-binding proteinsRecognition of AT-rich sequences at the origin of replication and separation of DNA strands in bacteria (prokaryotes)DnaA proteinUnwinding DNA double helixDNA hel icasesPrevention of supercoiling during replicationDNA topoisomerasesPlacement of RNA primer at site where replication is initiatedPrimase, and an RNA polymeraseRemoval of RNA primers from DNA synthesized discontinuouslyDNA polymerase I (speci fica l ly, the 5’-3’ exonuclease activi ty)DNA chain elongation in prokaryotesDNA polymerase II IProofreading of newly synthesized DNA strandDNA polymerase II I (speci fica l ly, the 3’-5’ exonuclease activi ty)Repair UV damage to DNAUV-speci fic endonuclease, exonuclease, and DNA l igaseRemoval of damaged bases from DNAApurinic or apyrimidinic endonuclease, exonuclease, and DNA l igase

What term is used to describe the DNA strand synthesized continuously toward the replication fork?Leading s trand

What term is used to describe the DNA strand synthesized discontinuously away from the replication fork?Lagging s trand

What are the three stop codons?UGA, UAA, UAG (U Go Away, U Are Away, U Are Gone)

In which direction are DNA and RNA synthesized?5’ → 3’

What is the start codon?AUG

Name the type of RNA responsible for each of the following functions:Largest RNA moleculemRNAMost abundant type of RNArRNASmallest RNA moleculetRNAPortion of RNA transcript encoding information for protein synthesisExons (“exons are expressed”)Portion of RNA transcript that is found between sequences of RNA encoding information for protein synthesisIntronsType of RNA covalently bound to a single amino acidtRNAName the term used to describe the region of genomic DNA where RNA polymerase and transcription factors bind to regulate transcription:PromoterName the term used to describe the region of genomic DNA where transcription factors activators bind to enhance transcription:EnhancerName the term to describe the region of genomic DNA where repressors bind.Si lencer

Name the enzyme responsible for each of the following functions:Synthesis of rRNARNA polymerase ISynthesis of mRNARNA polymerase IISynthesis of tRNARNA polymerase II I

Name three major regulatory mechanisms of transcription in eukaryotes:1. Regulation by transcription factors at the level of the promoter2. Regulation by his tones binding to speci fic genomic regions3. Regulation of DNA structure (including methylation, gene rearrangement, and ampl i fi cation)

What genetic structure regulates transcription in prokaryotes?An operon

Name the elements of an operon responsible for each of the following functions:Region where proteins bind to enable transcriptionPromoter regionMolecule that binds at the promoterActivator or repressorSequence in DNA where regulatory protein bindOperatorMolecule that binds the operator to regulate transcriptionRepressor

What are three modifications made to an RNA transcript before it leaves the nucleus?1. 5’ Capping with 7-methylguanos ine2. 3’ Polyadenylation3. Spl icing of introns

Which small molecule provides the energy for charging a tRNA with its amino acid?Adenos ine triphosphate (ATP)

Which small molecule provides the energy for binding tRNA to the ribosome and for translocation?Guanos ine triphosphate (GTP)

Which molecules, central to the discipline of molecular biology, recognize and cleave specific sequences of a DNA molecule?Restriction enzymes

Molecular Biology Techniques

Name the molecular biology techniques described below:

Method of separating molecules based on movement through a gel placed in an electric fieldGel electrophores isTechnique for detecting specific DNA sequences using restriction enzymes and a radiolabeled DNA probeSouthern blotTechnique for detecting specific RNA sequences using restriction enzymes and a radiolabeled DNA probeNorthern blotTechnique for detecting specific protein sequences using radiolabeled antibodiesWestern blotA rapid technique for amplifying a specific DNA sequence in vitroPolymerase chain reactionTechnique for detecting different alleles at a gene of interest using restriction enzymesRestriction fragment length polymorphism analys isTechnique for detecting the presence of antigen or antibody using radiolabeled antibodiesRadioimmunoassay (RIA)Technique for detecting the presence of antigen or antibody using antibodies linked to enzymes with detectable activityEnzyme-l inked immunosorbent assay (ELISA)

Inherited Diseases

Name inheritance patterns described below:

Twenty-five percent of offspring from two carrier parents affectedAutosomal recess ive (AR)Commonly cause defects in structural genesAutosomal dominant (AD)Commonly cause defects in enzymesARDefect seen in multiple generations in both sexesADDefects not typically seen in consecutive generationsARDisease is not observed in females.X-l inked (XL) recess iveDisease is transmitted by mother.Mitochondria l inheri tanceHalf of male offspring from affected mother will manifest disease.XL recess iveDisease manifestations commonly present after puberty.AD

What are the conditions for a population to be in Hardy-Weinberg equilibrium?1. No mutation at locus of interest2. No selection for a l lele at locus of interest3. Random mating4. Closed population (no migration)

What are the two Hardy-Weinberg equations?1. p2 + 2pq + q2 = 12. p + q = 1 (p and q are separate a l leles and pq i s the heterozygote frequency)

Name the disease or condition associated with each of the following statements:Lack of UV-specific endonuclease causing dry skin and malignant melanomaXeroderma pigmentosaLack of aldolase B causing hypoglycemia, jaundice, and cirrhosisFructose intoleranceLack of fructokinase causing fructosemia and fructosuriaEssentia l fructosuriaLack of galactose-1-phosphate uridyltransferase causing cataracts, hepatosplenomegaly (HSM), and mental retardationGalactosemiaDeficiency of lactase causing bloating, flatulence, and diarrhea on consumption of dairy productsLactose intoleranceLactic acidosis and neurologic deficits in an alcoholicPyruvate dehydrogenase deficiencyHemolytic anemia in patients of Mediterranean descent after eating fava beans or taking antimalarial medicationGlucose-6-phosphate dehydrogenase deficiencyHemolytic anemia due to deficiency in glycolysisHexokinase, glucose-phosphate i somerase, a ldolase, triose-phosphate i somerase, phosphate glycerate kinase, enolase, or pyruvatekinase deficiencyInappropriate hepatocellular accumula on of glycogen caused by a deficiency of glucose-6-phosphatase, associated with severe fas ng, hypoglycemia,lactic acidosis, hyperlipidemia, and impaired fructose metabolismvon Gierke disease/type I glycogen s torage diseaseInappropriate accumulation of glycogen in the liver, heart, and muscle caused by a deficiency of lysosomal α-1,4-glucosidase, resulting in cardiomegalyPompe disease/type II glycogen s torage diseaseInappropriate accumula on of glycogen in liver and heart due to deficiency of α-1,6-glucosidase, a debranching enzyme o en leading to muscularhypotoniaCori disease/type II I glycogen s torage diseaseInappropriate accumula on of glycogen in skeletal muscle fibers due to deficiency of glycogen phosphorylase, leading to myalgia and myoglobinuriawith exerciseMcArdle disease/type V glycogen s torage diseaseDefect in cystathione synthase leading to the presence of homocysteine in the urineHomocystinuriaDefect of renal tubular amino acid transporter for cysteine, ornithine, lysine, and arginineCystinuriaInadequate catabolism of branched-chain amino acids (Ile, Val, and Leu) due to lack of α-ketoacid dehydrogenase leading to mental retardationMaple syrup urine diseaseLack of phenylalanine hydroxylase (PAH) leading to a buildup of phenylalanine resul ng in mental retarda on, hypopigmenta on, eczema, and a mousyodorPhenylketonuria (can a lso get Phe bui ldup due to deficiency of tetrahydrobiopterin, BH4, cofactor of PAH)Lack of homogentisic acid oxidase leading to a buildup of homogentisate, causing darkening of the urine and connective tissuesAlkaptonuriaLack of tyrosinase leading to a lack of melaninAlbinismLack of adenosine deaminase inhibits DNA synthesis by causing the accumula on of metabolites in the purine salvage pathway; one of the causes ofsevere combined immunodeficiency syndromeAdenos ine deaminase deficiencyLack of hypoxanthine-guanine phosphoryltransferase (HGPRTase) causing an overproduc on of uric acid leading to neurologic deficits, hyperuricemia,and behavioral abnormalities, including self-mutilationLesch-N yhan S yndrome (L acks N ucleotide S a lvage)Trisomy 21 → mental retarda on, slanted palpebral fissures, hypertelorism, macroglossia, atrial septal defect (ASD), duodenal atresia, early-onsetAlzheimer disease, and multiple visceral anomaliesDown syndrome

Expansion of unstable region of X chromosome (abnormal FMR1 gene with CGG expansion) leading to mental retarda on, enlarged testes, andcraniofacial anomaliesFragi le X syndromeXL recessive deficiency of α -galactosidase A → buildup of ceramide trihexoside which causes pain in the extremi es, ocular abnormali es,angiokeratomas, cardiovascular disease, and renal failureFabry diseaseAR deficiency of galactosylceramide β -galactosidase leading to cerebral accumula on of galactocerebroside, which causes progressive neurologicdegenerationKrabbe disease (aka globoid cel l leukodystrophy)AR deficiency of β-glucocerebrosidase leading to the accumula on of glucocerebroside in the brain, bone marrow, liver, spleen →HSM, asep c necrosisof femur, and neurologic dysfunctionGaucher diseaseAR deficiency of sphingomyelinase on Chromosome 11 leading to buildup of sphinogmyelin and cholesterol in his ocytes of the liver, spleen, andlymphatic system, resulting in cortical atrophy, cherry red spot on macula, HSMNiemann-Pick diseaseAR deficiency of hexosaminidase A on Chromosome 18, leading to the accumula on of GM2 ganglioside within lysosomes resul ng in neurologicdegeneration and developmental delay, and cherry red spot on maculaTay-Sachs diseaseAR deficiency of arylsulfatase A, leading to an accumula on of cerebroside sulfate and dysfunc on and demyelina on of the central and peripheralnervous systems resulting in ataxia and dementiaMetachromatic leukodystrophyXL recessive deficiency of iduronate sulfatase, leading to an accumula on of heparan and dermatan sulfate resul ng in mental retarda on, coarse facialfeatures, and short statureHunter syndromeAR deficiency of α -l-iduronidase, leading to the accumula on of par ally degraded glycosaminoglycans within lysosomes resul ng in dysmorphic,gargoyle-like facies, corneal clouding, HSM, and skeletal abnormalitiesHurler syndromeAR deficiency of UDP-N-acetylglucosamine: N-acetylglucosaminyl-L-phosphotransferase, loss of protein tagging with mannose-6-phosphate, leading todefective trafficking of enzymes into lysosomes → developmental delay and coarse facial featuresI-cel l disease (mucol ipidos is type II)

Name the genetic disease associated with each of the following clinical or pathologic findings:Cherry red spot of the maculaTay-Sachs disease and Niemman-Pick diseaseCells containing “crinkled paper” cytoplasm and glycolipid-laden macrophagesGaucher diseaseCorneal cloudingHurler syndrome

Carbohydrate, Protein, and Lipid Metabolism

Name the major metabolic pathway regulated by the following enzymes:

Isocitrate dehydrogenaseCitric acid cyclePhosphofructokinaseGlycolys isFructose-1,6-bisphosphataseGluconeogenes isGlycogen synthaseGlyconeogenes isGlycogen phosphorylaseGlycogenolys isGlucose-6-phosphate dehydrogenasePentose phosphate pathwayAcetyl-coenzyme A (CoA) carboxylaseLipogenes isCarnitine acyltransferaseLipolys is (β-oxidation)HMG-CoA reductaseCholesterol synthes is

Name the major activators and/or inhibitors for each of the following enzymes:Citrate synthaseActivator: no major activatorInhibi tors : ATP, NADH, succinyl -CoA, and acyl -CoA derivatives of fatty acidsPhosphofructokinase-1Activator: AMP, fructose-2,6-bisphosphate (l iver)Inhibi tor: ci trate, ATP, and cAMPPyruvate dehydrogenaseActivator: CoA, nicotinamide adenine (NAD), adenos ine diphosphate (ADP), and pyruvateInhibi tor: acetyl -CoA, NADH, and ATPPyruvate carboxylaseActivator: acetyl -CoAInhibi tor: ADPFructose-1,6-bisphosphataseActivator: cAMPInhibi tor: AMP and fructose-2,6-bisphosphateGlycogen synthaseActivator: glucose-6-phosphateInhibi tor: no major inhibi tor

Glycogen phosphorylaseActivators : cAMP and Ca 2+ (muscle)Inhibi tors : glucose, glucose-6-phosphate, and ATPGlucose-6-phosphate dehydrogenaseActivator: NADP+

Inhibi tor: NADPHAcetyl-CoA carboxylaseActivator: ci trateInhibi tor: malonyl -CoA, pa lmitoyl -CoA, and cAMPCarnitine acyltransferaseActivator: no major activatorInhibi tor: malonyl -CoAHMG-CoA reductaseActivator: no major activatorInhibi tor: cholesterol and cAMP

Describe the effect of insulin on the following metabolic processes:Glycogen synthesis in muscle and liverIncreaseGluconeogenesis in the liverDecreaseGlycogenolysis in the liverDecreaseGlucose uptake in muscle and adipose tissueIncreaseTriacylglycerol degradationDecreaseTriacylglycerol synthesisIncreaseProtein synthesisIncrease

Describe the effect of glucagon on the following metabolic processes:Glycogenolysis in the liverIncreaseGluconeogenesis in the liverIncreaseβ-Oxidation of fatty acids in the liverIncreaseAmino acid uptake by liverIncrease

Which small molecule accepts reducing equivalents and is typically involved in catabolic processes?NAD+

Which small molecule donates reducing equivalents and is typically involved in anabolic processes?Nicotinamide adenine dinucleotide phosphate (NADPH)

Name the cellular compartment (cytosol, mitochondria, or both) where each of the following processes occur:Citric acid cycleMitochondriaFatty acid oxidationMitochondriaFatty acid synthesisCytosolGluconeogenesisBothGlycolysisCytosolHeme synthesisBothHexose monophosphate (HMP) shuntCytosolProtein synthesisCytosol (rough ER)Steroid synthesisCytosol (smooth ER)Urea cycleBoth

Hexokinase or glucokinase?Found in most tissuesHexokinaseFound exclusively in the liver and β-islet cellsGlucokinaseHigh affinity for glucoseHexokinaseFaster reaction velocityGlucokinaseInhibited by glucose-6-phosphateHexokinase

Name the molecule(s) that serve as the primary source of energy for the following organs:LiverAmino acids , l ipids , glucose, fructose, and lactate

Central nervous system (CNS)GlucoseHeartLipids , ketone bodies , lactate, and glucoseAdipose tissueGlucose and l ipidsType I skeletal muscle fibers (slow twitch, “red”)Lipids and ketone bodiesType II skeletal muscle fibers (fast twitch, “white”)Glucose

What are the four major enzymes that function in gluconeogenesis?1. P yruvate carboxylase2. P EP carboxykinase3. F ructose-1,6-bisphosphatase4. G lucose-6-phosphatase (Pathway Produces Fresh Glucose)

Name four important cellular processes that require reducing equivalents supplied primarily by the pentose phosphate pathway:1. Fatty acid and s teroid biosynthes is2. Glutathione-mediated reduction of H2O2

3. Cytochrome P-450 system4. Respiratory burst in phagocytes

Which molecule serves as donor of methyl groups in many metabolic processes?S-adenosylmethionine

Name four biosynthetic pathways that require methyl group donation by S-adenosylmethionine:Synthes is of1. Chol ine2. Creatine3. Epinephrine4. 7-Methylguanos ine

Which metabolic process is responsible for transferring reducing equivalents from RBCs and muscle tissue to the liver?Cori cycle

Which two amino acids play key roles in the transport of nitrogen from the periphery to the liver?1. Alanine2. Glutamine

What are the cofactors for the pyruvate dehydrogenase complex?B1, B2, B3, B5, and l ipoic acid

Name the eight intermediates of the citric acid cycle:1. C i trate (Cindy)2. I socitrate (Is)3. α-Ketoglutarate (Kinky)5. Succinyl -CoA (So)4. S uccinate (She)6. F umarate (Fornicates)7. M a late (More)8. O xa loacetate (Often)

Name a toxin that directly blocks the flow of electrons through the electron transport chain:Cyanide

Cyanide is a by-product of the metabolism of what antihypertensive?Nitropruss ide

Name a toxin that directly inhibits mitochondrial ATPase:Ol igomycin

Name a toxin that uncouples oxidative phosphorylation by increasing the permeability of the inner mitochondrial membrane:2,4-Dini trophenol (DNP)

What are the primary metabolic substrates released by the liver in the fed state and the fasting state?Very low-dens i ty l ipoprotein (VLDL) (fed); glucose and ketone bodies (fasting)

Name the apolipoprotein or lipoprotein responsible for each of the following functions:Activates lecithin-cholesterol acyltransferaseApol ipoprotein A-IBinds to low-density lipoprotein (LDL) receptor, mediates VLDL secretionApol ipoprotein B-100Delivery of cholesterol from the tissues to the liverHigh-dens i ty l ipoprotein (HDL)Delivery of cholesterol produced by the liver to the tissuesLDLDelivery of triglycerol absorbed in the small intestine to tissuesChylomicronDelivery of triglycerol produced by the liver to tissuesVLDLMediates chylomicron remnant uptakeApol ipoprotein EServes as a cofactor for lipoprotein lipaseApol ipoprotein C-II

Which enzyme, found in the liver and bone marrow, catalyzes the rate-limiting step of heme synthesis?Aminolevul inate synthase

What are the major intermediates in the degradation of heme?Heme → bi l iverdin → bi l i rubin → bi l i rubindiglucuronide (conjugated bi l i rubin)

Conjugated bilirubin is excreted in which bodily fluid?Bi le

Which compound produced by intestinal bacterial degradation of conjugated bilirubin gives urine its yellow color?Urobi l inogen

Which common physical finding in severe liver disease results from the accumulation of bilirubin (hyperbilirubinemia)?Jaundice

Name the essential amino acids:“PriVaTe TIM HALL”: Phe, Va l , Trp, Thr, I le, Met, His , Arg, Leu, Lys

Which amino acids (AAs) are exclusively ketogenic?Leu and Lys

Which AAs can be ketogenic or glucogenic?Tyr, I le, Phe, and Trp

Which AAs are exclusively glucogenic?Al l the amino acids not l i s ted in the above two i tems

Which AAs are negatively charged at physiologic pH (7.4)?Asp, Glu (the two acidic AAs)

Which AAs are positively charged at physiologic pH (7.4)?Arg, Lys (the two bas ic AAs), His a lso bas ic but weakly charged at pH 7.4

Which AA is used to carry ammonium from the muscles to the liver?Alanine

Which small molecule, essential to the excretion of ammonium, is derived from the removal of ammonium from glutamine?α-Ketoglutarate

What are the intermediates of the urea cycle?1. Orni thine (Ordinarily)2. Carbamoyl phosphate (Careless)3. Ci trul l ine (Crappers)4. Aspartate (Are)5. Arginosuccinate (Also)6. Fumarate (Frivolous)7. Arginine (About)8. U rea (Urination)

What is the limiting reagent for hepatic ethanol (EtOH) catabolism?NAD+

Which metabolic abnormality in chronic alcoholics results from depletion of NAD+ in the liver?Hypoglycemia (due to inhibi tion of gluconeogenes is )

Depletion of NAD inhibits which two major metabolic processes?1. The convers ion of pyruvate to lactate2. Oxaloacetate to malate

In what metabolic state does the liver commonly produce ketone bodies?During s tarvation (or diabetic ketoacidos is )

G Proteins

What class of G protein is associated with the following cell surface receptors:

α1

Qα2

Iβ1

Sβ2

SM1

QM2

IM3

QM4

SD1

SD2

IH1

QH2

SV1

QV2

SWhich two second messengers are increased by the activation of a Gq protein?

1. IP32. Diacylglycerol (DAG)

Which enzyme is induced by the activation of a Gq protein?Protein kinase C

Which enzyme is induced by the activation of a Gs protein?Protein kinase A

Which second messenger is increased by the activation of a Gs protein?cAMP

Which enzyme is inhibited by the activation of a Gi protein?Protein kinase A

Which second messenger is decreased by the activation of a Gi protein?cAMP

Enzymes

What is the shape of the plot of reaction velocity against substrate concentration for enzymes following Michaelis-Menten kinetics?

Hyperbol icFor an enzyme following Michaelis-Menten kinetics, how does halving enzyme concentration affect Vmax??

Vmax wi l l be ha lved. (Vmax i s di rectly proportional to enzyme concentration for a l l substrate concentrations .)What is the shape of a plot of reaction velocity against substrate concentration for enzyme with a single allosteric regulator?

Sigmoid or “S” shapedName two important parameters of the cellular environment that directly affect enzyme kinetics:

1. pH2. Temperature

The inverse of Vmax ((the maximum reaction velocity) for a given enzyme is represented by what point on a Lineweaver-Burk plot?The y intercept

The inverse of Km (the Michaelis-Menten constant) for a given enzyme is represented by what point on a Lineweaver-Burk plot?The x intercept

An enzyme with a small Km will have a high or low affinity for its substrate?High affini ty (Km = 1/2Vmax)

How does a competitive inhibitor affect Km?Increases

How does a noncompetitive inhibitor affect Km?No effect

How does a competitive inhibitor affect Vm?No effect

How does a noncompetitive inhibitor affect Km?Decreases

Cell Cycle

Name the phase of the cell cycle associated with each of the following cellular events:

QuiescenceG0

Centrosome duplicationSRNA, protein, organelle synthesisG0

DNA, RNA, histone synthesisSRelease of E2F from RbG1/S trans i tionCyclin-dependent kinase (Cdk)-cyclin A and Cdk-cyclin B are active.G2/M trans i tionCdk-cyclin D and Cdk-cyclin E are active.G1/S trans i tionChromatin condensation, mitotic spindle formationM: prophaseKinetochore assemblyM: prometaphaseCentrosomes move to opposite poles.M: prophaseNuclear envelope and nucleolar disappearanceM: prometaphaseChromosome alignment at metaphase plateM: metaphaseKinetochore separationM: anaphaseNuclear envelope and nucleolar formationM: telophaseCytoplasmic division (cytokinesis)M: cytokines isMost variable phase of cell cycleG1

Which two important molecules are involved in the G1 to S checkpoint?Rb, p53

What is the significance of p53?Al lows cel l to detect DNA defects and repair them before proceeding with repl ication

Cell Membranes

What are the major categories of molecules that make up the cell membrane?

Cholesterol , phosphol ipids , sphingol ipids , glycol ipids , and proteinsWhere in the cell membrane are glycoproteins found?

Exclus ively in the noncytoplasmic s ideWhat is the effect of increasing the cholesterol content of a cell membrane?

Membrane fluidi ty i s decreased.

NUTRITION

Name the fat-soluble vitamins:

Vitamins A, D, E, and KWhere are these vitamins absorbed?

I leumWhat conditions cause fat-soluble vitamin deficiencies?

Malabsorption syndromes (eg, cystic fibros is , cel iac disease, i lea l disease, i lea l resection)Why do toxicities occur more commonly with fat-soluble vitamins?

Fat-soluble vi tamins can accumulate in fa y ssues ; varying fat content with age leads to different thresholds of toxici ty for chi ldrenversus elderly.

Which prolonged dietary deficiency of protein and calories is characterized by retarded growth and cachexia in children?Marasmus

Which disease, characterized by protein deficiency with adequate caloric intake, results in retarded growth, anemia, and severe edema?Kwashiorkor

Name the vitamin(s) associated with each of the following statements:Composes NAD+ and NADP+

Vitamin B3 (niacin)Remains in the body with stores lasting up to 3 to 5 yearsVitamin B12 (cobalamin)Important in purine/pyrimidine synthesisFolateImportant part of visual pigments and epithelial cell differentiationVitamin A (retinol )Antioxidant that delays cataracts and atherosclerosisVitamin E (α-tocopherol )Component of CoA and fatty acid synthaseVitamin B5 (pantothenate)Cofactors for pyruvate-dehydrogenase complexVitamins B1 (thiamine), B2 (riboflavin), B3 (niacin), and B5 (pantothenic acid)Found only in animal products; Schilling test used to detect deficiencyVitamin B12 (cobalamin)Cofactor for norepinephrine (NE) synthesis and collagen cross-linkage; ↑ Fe absorptionVitamin C (ascorbic acid)Cofactor in transamination reactionsVitamin B6 (pyridoxine)Toxicity causes nausea, stupor, and hypercalcemiaVitamin DImportant in methionine synthesis and isomerization of methylmalonyl-CoAVitamin B12 (cobalamin)Toxicity causes skin changes, arthralgias, and premature epiphyseal closure, the first sign of toxicity usually CNS changes from edema.Vitamin A (retinol )Catalyzes γ-carboxylation of coagulation factors, synthesized by GI floraVitamin KThe most toxic vitamin in overdoseVitamin DThe most common vitamin deficiency in the United StatesFolate

Name the vitamin deficiency associated with each of the following findings:Wernicke-Korsakoff syndrome, beriberiVitamin B1 (thiamine); B1 = Ber1Ber1Rickets, osteomalacia, and hypocalcemic tetanyVitamin DNeonatal hemorrhage and ↑ prothrombin time (PT)Vitamin KMegaloblastic anemia with neurologic dysfunctionVitamin B12 (cobalamin)Dermatitis, diarrhea, and dementia (pellagra)Vitamin B3 (niacin)Deficiency caused by long-term raw egg ingestionBiotin (avidin in egg whites binds biotin)Megaloblastic anemia without neurologic dysfunctionFolateDry skin, dry eyes, and night blindnessVitamin A (retinol )Hemolysis (from RBC fragility) and ataxiaVitamin E (α-tocopherol )EEG abnormalities and peripheral neuropathy; caused by isoniazid (INH) and oral contraceptives

Vitamin B6 (pyridoxine)Cheilosis, corneal vascularization, and angular stomatitisVitamin B2 (riboflavin)Neural tube defects during pregnancyFolateScurvy, hemorrhages, and impaired wound healingVitamin C (ascorbic acid)Deficiency caused by Diphyllobothrium latum infection, sprue, pernicious anemia, and Crohn’s diseaseVitamin B12 (cobalamin)

Name the trace element associated with each of the following statements:Important in protein synthesis; deficiency causes acrodermatitis and ↑ sense of taste/smellZincInvolved in hemoglobin synthesis; excess caused by ceruloplasmin deficiencyCopperCofactor for glutathione peroxidase; deficiency causes cardiomyopathySeleniumInvolved in collagen cross-linkage; excess causes pulmonary fibrosisSi l i conInvolved in methionine metabolism; deficiency mimics vitamin B12 deficiencyCobalt (consti tuent of cobalamin)Reduces insulin resistance, glucose tolerance factorChromium

GENERAL EMBRYOLOGY

Name the embryonic structure described below:

Consists of the inner cell mass which ultimately gives rise to the fetusEmbryoblastConsists of the outer cell mass which ultimately gives rise to the placentaTrophoblastDerived from embryoblast; clefts form the amniotic cavityEpiblastDerived from embryoblast; ultimately forms the yolk sacHypoblastBorder between future mouth and pharynx; formed by both hypoblast and epiblast cellsBuccopharyngeal membraneProduces β-human chorionic gonadotropin (hCG)SyncytiotrophoblastForms the lining of the cytotrophoblastExtraembryonic somatic mesodermConsists of the syncytiotrophoblast, cytotrophoblast, and extraembryonic somatic mesodermChorionForms the covering of the yolk sacExtraembryonic viscera l mesoderm

The following developmental milestones occur how long after contraception?ImplantationWithin 1 weekBilaminar discWithin 2 weeksGastrulationWithin 3 weeksFormation of the primitive streak and neural plateWithin 3 weeksOrganogenesis, peak of susceptibility to teratogensWeeks 3 to 8Limb formationWeek 4Cardiac contractions beginWeek 4Male and female genitals can be distinguished.Week 10

Name the abnormality/abnormalities caused by the following teratogens:Angiotensin-converting enzyme (ACE) inhibitorsRenal dysgenes is → ol igohydramnios , pulmonary hypoplas ia , and l imb contracturesTetracyclineYel low teeth and enamel hypoplas iaAminogylcosidesEighth crania l nerve damage → deafnessOral hypoglycemicsNeonata l hypoglycemiaWarfarinCraniofacia l (nasa l hypoplas ia) and CNS defects , s ti l lbi rthDilantinFeta l hydantoin syndrome: craniofacia l and l imb defects , menta l deficienciesValproic acidSpina bi fidaLithium

Cardiac (Ebstein) anomalyIsotretinoinCraniofacia l (smal l ears ), CNS, cardiac, and thymus defectsIndomethacinConstriction of ductus arteriosusDiethylstilbesterol (DES)Clear cel l vagina l cancer and cervica l/uterine mal formations in female offspringThalidomideLimb reduction defectsAlcoholFeta l a lcohol syndrome: craniofacia l defects (absent phi l trum, fla ened nasa l bridge, and microphthalmia), growth restric on, and bra in,cardiac, and spina l defectsTobaccoGrowth restriction, prematuri ty, low bi rth weightRadiationGrowth restriction, CNS defects , and leukemia

Name the embryonic layer that gives rise to each of the following tissues:Adrenal cortexMesodermAnterior pituitaryEctoderm (ora l ectoderm/Rathke pouch)Aorticopulmonary septumEctoderm (neura l crest)Autonomic nervesEctoderm (neura l crest)Long bones and vertebraeMesodermFacial bonesEctoderm (neura l crest)CNS neurons and astrocytesEctoderm (neura l tube)Connective tissueMesodermEpidermisEctoderm (surface ectoderm)Epithelial lining of the GI tractEndodermMyocardiumMesodermKidneysMesodermLens of eyeEctoderm (surface ectoderm)Liver parenchymaEndodermMammary glandsEctoderm (surface ectoderm)MelanocytesEctoderm (neura l crest)Striated muscleMesodermNucleus pulposusMesoderm (notochord)PancreasEndodermParafollicular cells of the thyroidEctoderm (neura l crest)RetinaEctoderm (neura l tube)Schwann cellsEctoderm (neura l crest)SpleenMesodermParathyroidEndodermPosterior pituitaryEctoderm (neura l tube)ThymusEndodermThyroidEndodermKidneys, ureters, and gonadsMesoderm

BASIC PATHOLOGY

Cellular Adaptations

Give the appropriate term for each of the following definitions:

Complete failure of cell productionAplas iaRelative decrease in cell productionHypoplas iaIncrease in cell sizeHypertrophyReplacement of one adult (differentiated) cell (epithelial or mesenchymal) type with another adult cell typeMetaplas iaDecrease in cell substance results in a decrease in cell size; may result in decreased tissue/organ size.AtrophyIncrease of organ/tissue size due to an increase in the number of cellsHyperplas ia

Cell Injury

Name the mechanism of cell injury characterized by each of the following statements:

Mitochondrial dysfunction →↓ cellular ATP → failure of Na+/K+ ATPase, failure of protein synthesisIschemic/hypoxic cel l injuryDissociation of ribosomes and polysomesATP depletionCauses lipid peroxidation of membranesReactive oxygen species (O2-free radica ls )Associated with ionizing radiation, UV light, and reperffusion after ischemic injuryReactive oxygen species (O2-free radica ls )Prevented by enzymes such as glutathione peroxidase, catalase, and superoxide dismutaseGeneration of reactive oxygen species (O2-free radica ls )

Which molecules are released in response to mitochondrial damage?Cytochrome c and H+

Cytoskeletal abnormalities, ATP depletion, and cell swelling are associated with what key event in cell injury?Defective membrane permeabi l i ty

What is the effect of increased cytoplasmic calcium ions in a cell undergoing apoptosis or necrosis?Activation of ATPase, endonuclease, phosphol ipase, and proteases

Necrosis/Apoptosis

Classify the following as features of apoptosis or necrosis:

Cellular blebbing and cell shrinkageApoptos isInvolves many contiguous cellsNecros isPhysiologic, programmed cell removalApoptos isActive form of cell death (requires energy consumption)Apoptos isGross, irreversible cellular injuryNecros isInvolves single cells or groups of cellsApoptos isInvolution and shrinkage of affected cells and fragmentsApoptos isMarked inflammatory reactionNecros isInhibited by bcl-2; facilitated by bax, p53Apoptos is

State the function of each of the following molecules during apoptosis:Caspases (cysteine protease)Protein cleavageEndonucleasesDNA cleavagePhosphatidylserine and thrombospondinCel l surface molecules recognized by phagocytesTumor necrosis factor (TNF)-α receptor and FAS (CD95)Death receptorsBcl-2 and Bcl-xMajor antiapoptotic proteinsBad and BaxMajor proapoptotic proteinsCytochrome cActivation of procaspase 9Apoptosis-activating factor-1 (Apaf-1)Cytoplasmic receptor for cytochrome cTNF-αBindings of this l igand to i ts receptor induce association with a death domain.Granzyme B

Activation of the caspase cascade (released by cytotoxic T cel l s )Name the type of necrosis characterized by each of the following features:

Enzymatic degradation of tissue seen in abscessesLiquefactive necros isCommonly seen in tuberculous granulomasCaseous necros isFibrin-like, proteinaceous deposition in arterial wallsFibrinoid necros isLipase-induced autodigestion of adipose tissue → saponificationFat necros isInterruption of blood supply to organs supplied by end arteries; architecture well preservedCoagulative necros isResults from vascular occlusion; most commonly affects lower extremities or bowelGangrenous necros is

Cell Changes/Accumulations

What type of cellular change is characterized by excess accumulation of intracellular triglycerides?

Fatty change (s teatos is )What type of calcification is caused by hypercalcemia?

Metastatic ca lci fi cationWhat type of calcification occurs in previously damaged tissues?

Dystrophic ca lci fi cationName four endogenous pigments that accumulate in cells:

1. Melanin2. Bi l i rubin3. Hemos iderin4. Lipofuscin

Name four diseases associated with protein misfolding:1. Alzheimer disease2. α-1-Anti tryps in deficiency3. Cystic fibros is4. Amyloidos is

Name the cellular pigment described below:Accumulates in jaundiceBi l i rubinIdentified with Prussian blue dye; can result in organ damage or simple depositionHemos iderinYellowish, fat-soluble “wear-and-tear” pigmentLipofuscinFormed in the epidermis from tyrosineMelanin

Inflammation

Which three classes of adhesion molecules are involved in inflammation?

1. Selectins (E, P, and L)2. Immunoglobul in (Ig) fami ly (intercel lular adhes ion molecule [ICAM], platelet cel l adhes ion molecule [PCAM])3. Integrins

What are the five steps in the extravasation of inflammatory leukocytes?1. Margination2. Pavementing3. Tumbl ing (rol l ing)4. Adhes ion5. Transmigration

Which two groups of cell adhesion molecules pair mediate tumbling?1. Selectins on endothel ia l cel l s2. Sia lylated glycoproteins (eg, s ia lyl -Lewis -X) on leukocytes

Which two groups of cell adhesion molecules pair mediate leukocyte adhesion to endothelial cells?1. ICAM and vascular cel l adhes ion molecule (VCAM) (Ig superfami ly) on endothel ia l cel l s2. Integrins on leukocytes

Which factors induce endothelial expression of P-selectin?Platelet activation factor (PAF), his tamine, and thrombin

Which factors induce endothelial expression of ICAM and VCAM?Interleukin (IL)-1 and TNF

Name five chemotactic factors for neutrophils:1. Bacteria l products2. C5a3. LTB4

4. Chemokines (IL-8)5. Fibrin spl i t (degradation) products

Name five functional responses of leukocytes following their activation:1. Eicosanoid production2. Cytokine secretion3. Generation of reactive oxygen species4. Degranulation5. Al tered cel l adhes ion molecule express ion

6. Upregulation of receptors (tol l -l ike, G protein-coupled, opsonin, cytokines)Name two receptors that function in binding and phagocytosis of bacteria:

1. Mannose2. Scavenger receptors

What term is used to describe the process of coating substances (with Ig or C3b) to facilitate phagocytosis?Opsonization

Which neutrophil intracellular microbicidial mechanism uses the HMP shunt to generate an oxidative burst?H2O2-myeloperoxidase (MPO)-ha l ide system of bacteria l ki l l ing

Name two processes associated with impaired leukocyte adhesion:1. Recurrent bacteria l infections2. Al tered wound heal ing

Decide whether each of the following substances causes vasoconstriction or vasodilation:BradykininVasodi lationThromboxane (TXA)VasoconstrictionProstacyclin (PGI2)Vasodi lationLeukotrienes (LTC4, LTD4, and LTE4)VasoconstrictionProstaglandins (PGD2, PGE2, and PGF2)Vasodi lationRemember: most substances a lso cause analogous effects on bronchia l tone.

Name seven substances that increase vascular permeability:1. H i s tamine2. Serotonin3. C3a and C5a4. Leukotrienes (LTC4, LTD4, and LTE4)5. Bradykinin6. Ni tric oxide (NO)7. PAF (low concentration)

Which two enzymes stimulate the release of arachidonic acid from membrane phospholipids?1. Phosphol ipase A2

2. Phosphol ipase CWhat are the two major pathways in arachidonic acid metabolism?

1. Cyclooxygenase (COX)2. Lipoxygenase

What signaling molecules are produced by the COX pathway?TXA2 (in platelets ), PGI2 (in endothel ia l cel l s ), and other prostaglandins (in other ti s sues)

What are the two major enzymes involved in prostaglandin production?1. COX-12. COX-2

Which cycloxygenase enzyme serves in homeostatic functions?COX-1

What products does the lipooxygenase pathway produce?HPETEs and leukotrienes

Which arachidonic acid metabolite is thought to sensitize nerve endings to pain mediators?PGE2 (↓ PGE2 → analges ic effects )

Name four acute-phase responses of inflammation.1. Systemic effects (fever and leukocytos is )2. Hepatic synthes is of acute-phase reactants (C-reactive protein [CRP] ferri tin, complement, and prothrombin)3. Synthes is of adhes ion molecules4. Neutrophi l degranulation

Which substance links the kinin, coagulation, plasminogen, and complement systems?Factor XIIa (Hageman factor)

Which group of plasma proteins participates in immune-mediated lysis of cells?Complement system

Which substance produced by endothelial cells relaxes smooth muscle and inhibits platelet aggregation?Nitric oxide (NO)

Which factor is a pyrogen and causes fever?IL-1 → PGE2

Name four possible outcomes of acute inflammation:1. Complete resolution2. Abscess/ulcer/fi s tula formation3. Heal ing by fibros is and scarring4. Progress ion to chronic inflammation

Which pattern of chronic inflammation is characterized by nodular collections of epithelioid histocytes and multinucleated giant cells?Granulomatous inflammation

Name three etiologies of granulomatous inflammation:1. Infectious (Mycobacterium tuberculosis, Histoplasma, catscratch disease, leprosy, syphi l i s )2. Foreign bodies3. Idiopathic (eg, sarcoidos is , Crohn’s disease)

Tissue Repair

What are the four factors determining the size of a cell population?

1. Prol i feration

2. Cel l death3. Cel l di fferentiation4. Replacement by s tem cel l s

What are the three categories of cells based on their inherent proliferative activity?1. Permanent (cardiac myocytes , neurons)2. Quiescent (hepatocytes , endothel ia l cel l s , lymphocytes )3. Labi le (epidermis , GI, and respiratory tract epi thel ia l cel l s ; bone marrow; ha i r fol l i cles )

Name five factors that mediate cellular proliferation during the process of tissue repair:1. Platelet-derived growth factor (PDGF)2. Epidermal growth factor (EGF) and transforming growth factor-a lpha (TGF-α)3. Fibroblast growth factors (FGFs)4. Hepatyocyte growth factor5. Vascular endothel ia l growth factor (VEGF)

What highly vascular, newly formed connective tissue fills defects left by removal of cellular debris?Granulation ti ssue

What are the four key components to the orderly formation of a scar?1. Angiogenes is and granulation ti ssue formation2. Fibroblast emigration and prol i feration3. Depos i tion of extracel lular matrix4. Maturation and remodel ing → scar

Name five factors that delay or impede tissue repair:1. Impaired ci rculation2. Pers is tent infection3. Retention of debris or foreign body4. Nutri tional deficiency (eg, proteinand vi tamin C)5. Metabol ic disorders (eg, diabetesmel l i tus )

Hemodynamic Dysfunction

Name five causes of edema:

1. ↑ Hydrostatic pressure2. ↑ Capi l lary permeabi l i ty3. ↓ Oncotic pressure4. ↑ Na+ retention (renal disorders or congestive heart fa i lure [CHF])5. Lymphatic obstruction

Describe the contents of a transudate.Low-protein content (SG 1.012), few cel l s , and l i ttle protein

What type of fluid accumulation forms as a result of increased vascular permeability due to endothelial cell destruction?Exudate

What is the composition of exudate?High protein (SG >1.020) ↑ inflammatory leukocytes , and ↑ protein

Name two organs commonly affected by chronic passive congestion and their related pathologic findings:1. Lungs: hemos iderin-laden macrophages or “heart-fa i lure cel l s” (from left heart fa i lure)2. Liver: nutmeg l iver (from right heart fa i lure)

Name the two types of infarcts and several examples of each:1. Anemic (white) infarcts : heart, spleen, and kidneys2. Hemorrhagic (red) infarcts : lungs , testes , and GI tract (occurs in areas with col latera l ci rculation)

Thrombosis

What is Virchow triad?

The three primary influences on thrombus formation:1. Endothel ia l injury2. Stas is3. Hypercoagulabi l i ty

Name the three components necessary for hemostasis:1. Vascular endothel ium2. Platelets3. Coagulation cascade

Name five factors that promote platelet aggregation:1. ADP2. Thrombin3. TXA2

4. Col lagen5. Platelet-activating factor

Which product of the COX pathway limits further platelet aggregation?PGI2

Where are proteins C and S made?Endothel ia l cel l s

Classify the following as features of the extrinsic or the intrinsic pathway:Involves activation of factor VIIExtrins ic pathwayClinically monitored by the PTExtrins ic pathwayInitiated by activation of factor XIIIntrins ic pathwayMonitored by partial thromboplastin time (PTT)

Intrins ic pathwayName the component(s) of the coagulation cascade associated with each of the following features:

Vitamin K-dependent coagulation factorsFactors I I , VII , IX, X, proteins C and SIon necessary for proper function of the coagulation cascadeCalciumFactors inhibited by antithrombin III (AT III)Thrombin, factors IX, X, XI, and XIIComplex that activates proteins C and SThrombomodul in-thrombin complexThe most important fibrinolytic proteasePlasminConverts plasminogen to plasminTissue plasminogen activator (t-PA), a l so urokinase plasminogen activatorCleaved by activated protein C (APC) → inhibition of coagulationFactors Va and VIIIa

Which molecule dramatically enhances the activity of AT III?Heparin

What is the most frequent cause of hereditary thrombophilia?Factor V (Leiden) mutation (2%-15% of white population)

Hereditary thrombophilia can also be caused by deficiency of what major antithrombotic proteins?AT II I , protein C, and protein S

How does the Leiden mutation confer hypercoagulability?Renders mutant factor V res is tant to cleavage by APC

Which prothrombotic disorder is characterized by autoantibodies that induce platelet activation?Antiphosphol ipid antibody syndrome

Name the type of thrombus associated with each of the following:Reddish-blue cast; usually in lower extremitiesVenous thrombos isLines of ZahnArteria l thrombusSterile vegetations of heart valves in patients with hypercoagulable statesNonbacteria l thrombotic endocardi ti sNoninfective heart valve vegetations from circulating immune complexesVerrucous (Libman-Sacks ) endocardi ti s

Embolism

Name the type of embolism described by each of the following statements:

Venous thrombus that gains access to arterial circulation through a right-to-left shuntParadoxica l embolusAssociated with decompression sicknessAir embolusOccurs at parturition; can lead to disseminated intravascular coagulation (DIC) and deathAmniotic fluid embolusEmbolus obstructing the bifurcation of the pulmonary arterySaddle embolusOften arises from one wall of a heart chamber, especially in the context of atrial fibrillationMural thrombus → arteria l embol iImportant cause of death in immobilized, post-op patientsPulmonary embolusOccur after severe, multiple long bone fracturesFat embol i

Shock

Name the type of shock described by each of the following statements:

Associated with gram-negative endotoxemiaSeptic shockCirculatory collapse from pump failure of left ventricle (LV)Cardiogenic shockIgE-mediated systemic vasodilation with ↑ vascular permeabilityAnaphylactic shockCaused by severe hemorrhage or fluid lossHypovolemic shockAssociated with severe trauma causing reactive peripheral vasodilationNeurogenic shockWhich two mediators are associated with systemic vasodilation in septic shock?1. NO2. PAF

Name the characteristic manifestations of shock on each of the following organs:LungsPulmonary edemaLiverSteatos is and centri lobular necros isColonPatchy mucosa l hemorrhages

AdrenalsLipid depletion of cortexKidneysAcute tubular necros is

BASIC PHARMACOLOGY

Absorption/Distribution

In which part of the GI tract are most oral drugs absorbed?

DuodenumName three factors that influence absorption of drugs:

1. Chemica l properties (active transport vs pass ive di ffus ion)2. pH (percent of drug in the uncharged s tate determines rate of absorption)3. Phys ica l factors (blood flow, surface area, and contact time with absorptive surfaces)

What is first-pass metabolism?Hepatic degradation/a l teration of an ora l drug after absorption, before i t enters the genera l ci rculation

Complete the following formulas:(Rate of drug elimination)/(plasma drug concentration) =Clearance (CL)(Amount of drug in body)/(plasma drug concentration) =Volume of dis tribution (Vd)(Cp X Vd)/F =Loading dose (a lso defined as the amount of drug necessary to rapidly ra ise a des i red plasma concentration of the drug)(Cp X CL)/F =Maintenance dose (a lso defined as the amount of drug necessary to mainta in a des i red plasma concentration of the drug)(0.693 X Vd)/CL =Cp= target plasma concentration, F = bioavailabilityHal f-l i fe (t1/2)

If a pa ent were known to be a rapid metabolizer, would the loading dose or maintenance dose have to be adjusted to maintain a desired plasmaconcentration of the drug?

The loading dose would be unchanged; the maintenance dose would need to be increased.How do dosage calculations change for patients with impaired renal/hepatic function?

Loading dose s tays the same, maintenance dose decreases .List four conditions that alter drug distribution:

1. Edematous s tate (eg, CHF and nephrotic syndrome)2. Pregnancy (↑ intravascular volume)3. Obes i ty (accumulation of l ipophi l i c agents in fat cel l s )4. Hypoalbuminemia (no a lbumin to bind drugs →↑ avai labi l i ty)

Name the key mechanism of drug-drug interactions:Drug displacement from a lbumin (combined adminis tration of classes I and II drugs)

Metabolism/Elimination

Name four clinical situations that would result in increased drug half-life.

1. Prerenal s tate (↓ renal plasma flow)2. Renal disease → decreased extraction ratio3. Adding a second drug that displaces the fi rs t from a lbumin, thus ↑ Vd

4. ↓ Metabol i sm (hepatic insufficiency or drug interaction)Classify each of the following statements as characteristic of phase I or phase II metabolism:

Produces slightly polar, water-soluble metabolitesPhase IInvolves mixed-function oxidase (P-450)Phase IInvolves conjugation reactions (acetylation, glucuronidation, and sulfation)Phase IIProduces very polar, inactive metabolites that are excreted by the kidneysPhase IIReduction, oxidation, and hydrolysisPhase IPhase that may be compromised first in geriatric patientsPhase IPhase that may be compromised in neonatesPhase I

Classify each of the following statements as characteristic of first-or zero-order drug elimination:Constant fraction of drug eliminated per unit of timeFi rs t-order el iminationConstant amount of drug eliminated per unit of timeZero-order el iminationPlasma concentration decreases linearly with timeZero-order el iminationPlasma concentration decreases exponentially with timeFi rs t-order el iminationElimination rate is independent of concentration

Zero-order el imination

Pharmacodynamics

What term describes the maximum effect a drug can produce?

EfficacyWhat term describes the measure of the amount of drug needed to produce a given result?

PotencyWhat term describes the dose of a drug that produces the desired effect?

Effective dose (ED)What term describes the dose of a drug that produces death?

Lethal dose (LD)What term describes the measure of the safety of a drug?

Therapeutic indexHow is therapeutic index calculated?

LD50/ED50 (LD50 = dose letha l in 50% of the population, ED50 = dose effective in 50% of the population)How does a competitive antagonist affect the dose-response curve?

Shi fts i t to the right (↑ ED50)How does a noncompetitive antagonist affect the dose-response curve?

Shi fts i t downward (↓ maximal response)How does a partial agonist differ from a full agonist?

Acts l ike an agonis t when an agonis t i s not present; acts l ike an antagonis t i f an agonis t i s present

Drug Development

Name the phase of drug development described by each of the following statements:

Measures effect of the drug in patients with a diseasePhase II cl inica l testingMeasures whether the drug is safe in healthy volunteersPhase I cl inica l testingPostmarketing surveillancePhase IVLarge, multicenter clinical trials to prove efficacy and safetyPhase II I cl inica l tria l s

Tox icology

Name the antidote for each of the following toxins:

IronDeferoxamineCopper/gold/arsenicPenici l lamineLeadCaEDTA, succimer, dimercaprol (BAL in oi l ), and ora l penici l lamineArsenic/mercuryDimercaprolCarbon monoxide100% O2 and hyperbaric O2

CyanideNitri te, vi tamin B12, and thiosul fateMethemoglobinMethylene blueMethanol/ethylene glycolETOH, fomepizole, and dia lys isAcetaminophen (Tylenol)N-acetylcysteineAspirin (salicylates)Alka l inze urine (promotes excretion) and dia lys isOpioidsNaloxone (IV, IM, and SQ) or na l trexone (PO)BenzodiazepinesFlumazeni lOrganophosphates, anticholinesterasesAtropine and pra l idoxime (PAM)HeparinProtamine sul fateWarfarinVitamin K, fresh frozen plasma (FFP) for acute reversa ltPAAminocaproic acidDigitalisAntidig Fab fragments (fi rs t s top drug and s tabi l i ze K+, Mg2+, and l idoca ine)CyclophosphamideMesna

P-450

Classify each of the following drugs as inhibitors or inducers of P-450:

Inducers : “Queen Barb takes Phen-phen and Refuses Greasy Carbs”Inhibi tors : “Inhibi tors Stop Cyber-K ids from Eating Grapefrui ts”BarbituratesInducerINHInhibi torSpironolactoneInhibi torRifampinInducerCimetidineInhibi torKetoconazoleInhibi torPhenytoin and carbamazepineInducerQuinidineInducerDisulfiramInhibi torSulfonamidesInhibi torSteroidsInhibi torMacrolides (erythromycin)Inhibi torChloramphenicolInhibi torGriseofulvinInducerGrapefruitInhibi torVerapamilInhibi torChronic EtOH useInducerAcute EtOH useInhibi tor

CHAPTER 2Immunology

IMMUNOLOGY BASICS

Cells of the Immune System

Name the type of immune cell that fits each description given below:

Major cells involved in innate immunityMonocytes , macrophages , neutrophi l s , natura l ki l ler (NK) cel l sCell-mediated immune responseT lymphocyteHumoral immunityB lymphocytePrimary phagocytic cell in acute inflammationNeutrophi lType of cell necessary for transplant rejectionT lymphocyteContains myeloperoxidase and lysozymeNeutrophi lMajor mediator of a type 1 hypersensitivity reactionMast cel lMajor mediator of the antiparasitic responseEos inophi lGranules contain histamine and heparinBasophi l and mast cel l sMajor antigen-presenting cells in tissuesMacrophages , dendri tic cel l s , and B cel l sDemonstrates a multilobed (“hypersegmented”) nucleus in vitamin B12 or folate deficiencyNeutrophi lSecretes interleukin (IL)-1 to promote T cell activityMacrophageExpresses IgE receptors on its cell membrane to mediate the allergic responseBasophi lExpresses high levels of major histocompatibility complex (MHC) class II on its cell membraneMacrophageMacrophage precursorMonocyteCell type increased in atopic asthmaEos inophi lRecognizes antigen presented in the context of MHC class II moleculesT-helper cel lAntibody-producing cell with abundant rough endoplasmic reticulumPlasma cel lMajor source of IL-2 productionT-helper cel l (speci fica l ly TH1 cel l s )Major cell of the humoral immune responseB lymphocyte

Immunoglobulins

Name the type of immunoglobulin (Ig) associated with the following features:

Most abundant type of IgIgGFirst class of Ig produced in an immune response upon exposure to antigenIgMAble to fix complementIgG and IgMFound on the lining of mucous membranes and in secretions, including breast milk and salivaIgAAble to cross placentaIgGType of Ig commonly occurring as a dimerIgAType of Ig commonly occurring as a pentamerI§MIg elevated in patients with asthma and allergiesIgEResponsible for long-term immunityIgGCauses mast cells and basophils to release histamine when triggered by antigen

IgEType of Ig found embedded in the cell membrane of developing B cellsIgDTotal levels and concentration of this antibody can be estimated using radioimmunosorbent test (RIST) and radioallergosorbent test (RAST).IgE

What term is used to describe the portion of a molecule that serves as an antigenic determinant?Epitope

What term is used to describe a small molecule that can serve as an antigenic determinant only if it is attached to a larger carrier molecule?Hapten

What type of chemical bonds are critical in linking the heavy and light chains of Igs?Disul fide bonds

Which products result when papain digests an Ig?Two Fab fragments (each capable of binding antigen) and one Fc fragment

Which products are produced following pepsin digestion of an Ig?One F(ab’)2 fragment and one Fcfragment

What term is used to describe the region in an antibody that determines antigen specificity?The hypervariable region or complementari ty determining region (CDR)

Name five mechanisms by which antibody diversity is created:1. Mutations in the genes encoding the CDR region2. Random VJ recombination in l ight cha ins3. Random VDJ recombination in heavy cha ins4. Random assembly of l ight and heavy cha ins5. Imperfect recombination of VDJ genes

T Cells

Name the T-lymphocyte cell surface protein associated with the f ollowing features:

Antigen-specific receptor on 95% of T cellsαβ T-cel l receptor (αβ TCR)Antigen-specific receptor on 5% (or less) of T cellsγδ T-cel l receptor (γδ TCR)Signal transduction protein always associated with TCRCDST-cell marker expressed in immature T cellsCD2Responds to MHC class II molecule expressed by antigen-presenting cellsCD4Responds to MHC class I molecule expressed on all cellsCD8Found specifically on T-helper cellsCD4Found specifically on cytotoxic T cellsCD8Cells with this surface marker ↓ in HIV/AIDSCD4Type of T cell that destroys virally infected cellsCD8

HLA Subtypes

Name the HLA haplotype(s) associated with the following diseases:

Ankylosing spondylitisHLA-B27Type 1 diabetes mellitusHLA-DR3/DR4Multiple sclerosisHLA-DR2Rheumatoid arthritisHLA-DR4Screening for abacavir hypersensitivityHLA-B*5701

Cytokines

Name the cytokine described below:

Endogenous pyrogenIL-1 (produced by macrophages)Promotes IgA synthesisIL-5Induces IL-2 production by T cellsIL-1High concentrations induce cell death in some tumors and cause cachexia.Tumor necros is factor (TNF)-αInduces T- and B-cell activity during the initial stages of an immune responseIL-1Induces production of IgE and IgG

IL-4 (produced by T-helper cel l s )Induces differentiation of eosinophils and promotes growth in B cellsIL-5 (produced by T-helper cel l s )Chemotactic factor for neutrophilsIL-8 (produced by monocytes and endothel ia l cel l s )Secreted by activated T cells and induces maturation of bone marrow stem cellsIL-3Inhibits production of interferon-gamma (IFN-γ) by T-helper cellsIL-10 (produced by TH2 cel l s )Inhibits production of IFN-γIL-4Inhibits differentiation of TH1 cellsIL-10Stimulates TH1 differentiationIL-12 (produced by macrophages and B cel l s )Activates T-helper, T-cytotoxic, natural killer, and B cellsIL-2 (produced by T-helper cel l s )Promotes production of IFN-γ T-helper cellsIL-12Low concentrations promote neutrophil activity and IL-2 receptor expression.TNF-αInhibits growth and function of T and B cells and promotes collagen secretion during tissue repairTransforming growth factor-beta (TGF-β)

Gel and Coombs Hypersensitivity Reactions

Name the Gel and Coombs hypersensitivity reaction described below:

Mediated by IgE bound to mast cells and basophilsType 1 (immediate or anaphylactic hypersens i tivi ty)Antibody-mediated cytotoxic reactionType 2 (cytotoxic reaction)Occurs in response to environmental allergiesType 1Antigen-sensitized T cells release cytokines, which induce an inflammatory response up to 48 hours after initial contact with antigen.Type 4 (delayed-type hypersens i tivi ty)Associated with the release of histamine, platelet-activating factor, leukotrienes, prostaglandins, and thromboxanesType 1Binding of cytotoxic T cells or complement to Fc portion of antibody causes target cell lysisType 2Immune complex deposition in tissues results in an inflammatory responseType 3 (immune complex reaction)

Name the type of hypersensitivity reaction responsible for the following diseases or conditions:Arthus reactionType 3AsthmaType 1Chronic transplant rejectionType 4Contact dermatitis, reaction to poison ivyType 4Drug allergiesType 1Environmental allergiesType 1Erythroblastosis fetalisType 2Goodpasture syndromeType 2Graves diseaseType 2Hemolytic anemiaType 2Immune complex-mediated glomerulonephritisType 3Lambert-EatonType 2Myasthenia gravisType 2Multiple sclerosisType 4Pernicious anemiaType 2Purified protein derivative (PPD)/tuberculin skin testType 4Rheumatic feverType 2Rheumatoid arthritis

Type 3Systemic anaphylaxisType 1Serum sicknessType 3Systemic lupus erythematosus (SLE)Type 3Transfusion reaction due to ABO incompatibilityType 2

Complement

Which class of bacteria is particularly susceptible to complement-mediated lysis?

Gram-negative organismsWhich antibody isotypes activate the classic pathway of the complement cascade?

IgG and IgMWhich molecules activate the alternative pathway of the complement cascade?

Aggregated IgA, endotoxin, and other components of the bacteria l cel l wal lName the component(s) of the complement cascade responsible for the following functions:

Neutralization of virusesCl to C4OpsonizationC3bNeutrophil and macrophage chemotaxisC5aSynthesis of membrane attack complex (MAC)C5b to C9Formation of C3 convertaseC3b, Bb (a l ternative pathway) or C4b, C2b (class ic pathway)

Name the disease or condition caused by a deficiency of the following complement components:Cl esterase inhibitorHereditary angioedemaC3Sinus and upper respiratory tract infectionsC5b to C9Recurrent Neisseria infectionsDecay accelerating factorParoxysmal nocturnal hemoglobinuria

Transplant Rejection and MHC Molecules

For each of the following descriptions, name the type of transplant rejection:

Preformed antibodies in host react against graft antigensHyperacute rejection (minutes to hours )Activation of previously sensitized T cellsAccelerated rejection (hours to days )Involves T-cell activation, differentiation, and antibody productionAcute rejection (days to weeks)Immune complex deposition combined with subacute cell cytotoxicityChronic rejection (months to years )

What are the four different classes of grafts?1. Autograft (from sel f)2. Syngeneic (from identica l twin or clone)3. Al lograft (from same species )4. Xenograft (from di fferent species )

The activity of cytotoxic T cells against tumor or virally infected cells requires which cell surface receptor for antigen presentation?MHC class I

The activity of cytotoxic T cells against pathogens phagocytosed by macrophages requires which cell surface receptor for antigen presentation?MHC class I I

PATHOLOGY IMMUNE SYSTEM

Autoantibodies

Describe the disease associated with the following autoantibodies:

Antinuclear antibodies (ANA)SLE (sens i tive but not speci fic for SLE)Antiacetylcholine esterase (ACh)Myasthenia gravisAntibasement membraneGoodpasture diseaseAnticentromereCREST syndrome (Calcinos is , Raynaud, Esophageal dysmoti l i ty, Sclerodactyly, Telangiectas ias )Anti-dsDNA

SLE (highly speci fic for SLE)Antiepithelial cellPemphigus vulgarisAntigliadinCel iac sprueAntihistoneDrug-induced lupusAnti-IgG Fc (rheumatoid factor)Rheumatoid arthri ti sAnti-JolMyos i ti sAntimicrosomalHashimoto thyroidi ti sAntimitochondrialPrimary bi l iary ci rrhos isAntinuclear ribonucleoprotein (nRNP)Mixed connective ti ssue diseaseAntiplateletIdiopathic thrombocytopenic purpuraAnti-Scl-70 (DNA topoisomerase 1)Systemic scleros isAnti-SmithSLE (highly speci fic for SLE)Anti-SS-A (Ro) and anti-SS-B (La)Sjögren syndromeAntithyroglobulinHashimoto thyroidi ti sAnti-thyroid-stimulating hormone receptor (TSHr)Graves diseaseAnti-voltage-gated calcium channelLambert-Eaton syndromeCytoplasmic pattern of antineutrophil cytoplasmic antibodies (c-ANCA)Wegener granulomatos isPerinuclear pattern of antineutrophil cytoplasmic antibodies (p-ANCA)Microscopic PolyANgi i ti s , Polyarteri ti s Nodosa (PAN), and Churg-Strauss syndromeList the four types of nuclear antigens against which ANAs are directed:1. DNA2. His tones3. Nonhis tone proteins4. Nucleolar antigens

Systemic Lupus Erythematosus

What pathologic finding is common to all tissues affected by SLE?

Acute necrotizing vascul i ti s of smal l arteries and arterioles caused by immune complex depos i tionDescribe the effect of SLE on each of the following organs:

SkinMalar rash, discoid rash, photosens i tivi tyJointsArthri ti s and arthra lgiasBrainNeuropsychiatric changes or seizures (2° to cerebra l vascul i ti s ), cogni tive dys functionEyesCotton-wool spots , retina l hemorrhagesHeartPericardi ti s , Libman-Sacks Endocardi ti s (SLE → LSE)LungsPleuri ti s , pulmonary fibros isGastrointestinal (GI)Oral and nasopharyngeal ulcersSpleenSplenomegaly and onion skinning of splenic vesselsKidneysWire-loop glomerular les ions and mesangia l immune complex depos i ts → glomerulonephri ti sHematologyHemolytic anemia, leukopenia , thrombocytopenia , antiphosphol ipid antibody syndromeBlood vesselsRaynaud phenomenon

Libman-Sacks endocarditis causes sterile vegetations to form on both sides of which cardiac valve?Mitra l va lve

Name five medications capable of inducing a lupus-like syndrome:1. Hydra lazine2. Isonicotinic acid (INH)3. Phenytoin4. Proca inamide5. Penici l lamine

Name the disease related to SLE that is characterized by immune complex deposition at the dermal-epidermal junction:Discoid lupus erythematosus

Immunodeficiencies

Name the immunodeficiency associated with the following clinical and pathologic features:

B-cell deficiency causing recurrent respiratory tract bacterial infections in boys >6 months of ageX-l inked (XL) agammaglobul inemiaT-cell deficiency due to failure of development of the third and fourth pharyngeal pouchesThymic aplas ia (DiGeorge syndrome)Defec ve B and T cells, most cases caused by XL recessive muta on in γ-chain of cytokine receptors or autosomal recessive (AR) muta on in adenosinedeaminaseSevere combined immunodeficiencyAR disease characterized by IgA deficiency, cerebellar dysfunction, and conjunctival telangiectasiasAtaxia-telangiectas iaXL deficiency of nicotinamide adenine dinucleotide phosphate (NADPH) oxidase activity, resulting in an impaired neutrophil respiratory burst and leadingto increased bacterial and fungal infectionsChronic granulomatous diseaseRecurrent bacterial infections early in life due to a defect in CD40 ligand that prevents B-cell class switchingHyper-IgM syndromeAR microtubule defect resulting in decreased phagocytosis, partial albinism, and neuropathyChediak-Higashi diseaseAR syndrome characterized by failure of neutrophil chemotaxis-associated eczema, elevated IgE, noninf lamed staphylococcal abscessesJob (hyper-IgE) syndromeXL recessive disease characterized by recurrent infections, thrombocytopenia, eczemaWiskott-Aldrich syndromeMutation of the Btk tyrosine kinase gene resulting in the underproduction of all classes of antibodiesXL agammaglobul inemiaDefect in the receptor for IL-7Severe combined immunodeficiencyAssociated with Staphylococcus aureus, Streptococcus pneumoniae, and Haemophilus influenzae respiratory infec ons and persistent GiardialambliainfectionsXL agammaglobul inemiaAssociated with recurrent GI and pulmonary infectionsIsolated IgA deficiencyFrequent viral and fungal infections in a patient with hypocalcemia due to low PTH levelsThymic aplas ia (DiGeorge syndrome)Associated with tetany and congenital defects of the heart and aortaThymic aplas ia (DiGeorge syndrome)Small thymus devoid of lymphocytes, hypoplastic lymph nodes, and splenic white pulpSevere combined immunodeficiency↑ IgA, normal IgE, and ↓ IgM levelsWiskott-Aldrich syndromeHypogammaglobulinemia commonly due to failure of T-cell-mediated B-cell maturationCommon variable immunodeficiency

Autoimmune Connective Tissue Disorders

Name the autoimmune disease of connective tissue associated with the f ollowing clinical and pathologic findings:

Keratoconjunctivitis sicca or xerophthalmia, xerostomia, and evidence of other connective tissue diseaseSjögren syndromeAutoimmune inflammatory disorder associated with malignancy; frequently caused muscle weaknessPolymyos i ti sHeliotrope rashDermatomyos i ti sRapidly progressive diffuse fibrosis of skin and involved organs including the heart, GI tract, kidney, lung, muscle, and skinDiffuse sclerodermaCREST syndrome of calcinosis, Raynaud phenomenon, esophageal dysfunction, sclerodactyly, telangiectasiasLoca l i zed scleroderma↑ Serum creatine kinase (CK) levelsPolymyos i ti s

What is the most common cause of death due to scleroderma?Renal cri s i s (accounts for 50% of deaths related to scleroderma)

Which autoimmune disease of connective tissue lacks renal involvement?Mixed connective ti ssue disease

Amyloidoses

Name the group of disorders characterized by extracellular deposition of protein in a β-pleated sheet conformation.

AmyloidosesWhich stain is used to identify the presence of amyloid in tissue that exhibits apple green bif ringence under polarized light?

Congo redWhich molecular configuration is common to all forms of amyloid?

Cross -β-pleated sheetName the effect of amyloidosis on each of the following organs:

KidneysGlomerular, peri tubular, and vascular hya l inizationSpleenSago spleen (tapioca-l ike amyloid depos i ts in fol l i cles ) or lardaceous spleen (amyloid depos i tion in splenic pulp)

LiverHepatomegaly with amyloid depos i tion in the space of DisseHeartRestrictive cardiomyopathyTongueHypertrophy due to amyloid depos i tion

CHAPTER 3Microbiology and Infectious Diseases

BACTERIOLOGY

Taxonomy

Name two genera of gram-positive cocci:

1. Streptococcus2. Staphylococcus

Name four genera of gram-positive bacilli:1. Clostridium2. Listeria3. Bacillus4. Corynebacterium

Which of these gram-positive bacilli are spore forming?Clostridium and Bacillus

Which gram-positive cocci are catalase positive (+)?Staphylococcus

Which of these is also coagulase (+)?Staphylococcus aureus

Name the streptococci that typically show the following pattern of hemolysis:α (green/partial hemolysis) “αlmost hemolytic”Streptococcus pneumoniae and vi ridans group (eg, Streptococcus mutans)β (clear hemolysis) “βetter hemolysis”Group A (Streptococcus pyogenes) and group B (Streptococcus agalactiae)γ (no hemolysis)Group D (Enterococcus and Peptostreptococcus)

Laboratory Evaluation of Gram-Positive Bacteria

How are S. pneumoniae and viridans streptococci differentiated in the laboratory?

Streptococcus pneumoniae i s bi le soluble and optochin sens i tive.How can capsulated S. pneumoniae bugs be detected in the laboratory?

Quel lung pos i tive (capsule swel l s when antisera i s added)What determines the Lancef ield grouping of streptococci?

C-carbohydrate in the bacteria l cel l wal lHow are groups A and B differentiated in the laboratory?

Group A i s baci tracin sens i tive.How are spores from gram-positive rods killed?

Autoclave (spores are res is tant to heat and most chemica ls )Name the gram-negative organisms associated with each of the following statements:

Three pathogenic gram-negative cocci1. Neisseria meningitides2. Neisseria gonorrhea3. Moraxella catarrhalisSix gram-negative coccobacilli1. Haemophilus influenzae2. Pasteurella3. Brucella4. Bordetella pertussis5. Francisella6. LegionellaThree clinically important gram-negative rods that are typically lactose fermenting1. Enterobacter2. Escherichia coli3. Klebsiella (a l l impl icated in urinary tract infections [UTIs ])Two obligate intracellular organisms1. Chlamydia (s tea ls adenos ine triphosphate [ATP] from host)2. Rickettsia (lacks coenzyme A [CoA] and nicotinamide adenine [NAD] → cannot produce own ATP)Four obligate aerobes1. Nocardia2. Pseudomonas3. Mycobacterium tuberculosis4. BacillusThree obligate anaerobes1. Clostridium2. Bacteroides3. Actinomyces (no cata lase and/or superoxide dismutase → susceptible to oxidative damage)

How are the pathogenic Neisseria species differentiated in the laboratory?Neisseria meningitidis ferments maltose.

How can Pseudomonas be rapidly differentiated from many lactose nonfermenters in the laboratory?

Pseudomonas i s oxidase pos i tive.Provide culture requirements or conditions for each of the following bacteria:

Corynebacterium diphtheriaeTel luri te agar or Loeffler mediaBordetella pertussisBordet-Gengou potato blood agarNeisseria gonorrhoeaThayer-Martin (VCN—vancomycin, col i s tin, and nystatin) and a selective mediumLegionella pneumophilaCharcoal yeast agar with i ron and L-cysteineMycoplasma pneumoniaeEaton agarMycobacterium tuberculosisLowenstein-Jensen agarVibrio speciesThiosul fate-Ci trate-Bi le Sa l ts -Sucrose (TCBS) agarEnterococcus40% bi le and 6.5% NaClHaemophilus influenzaeChocolate agar (conta ins factor V [NAD] and X [hematin])Lactose-fermenters (Klebsiella, Escherichia, etc)MacConkey agar → pink coloniesFungiSabouraud agar

Bacteriology Basics

Which cell membrane structure is unique to gram-positive organisms?

Teichoic acidWhich molecule, unique to the bacterial cell wall, provides rigid support and resistance against osmotic pressure?

PeptidoglycanWhich heat-stable lipopolysaccharide (LPS) is found in the cell wall of gram-negative bacteria?

EndotoxinWhich is the only gram-positive organism with LPS-lipid A?

Listeria monocytogenesName five important systemic effects of endotoxin (particularly, lipid A):

1. ↑Interleukin (IL)-1 → fever2. ↑ Tissue necros is factor (TNF) → hemorrhagic ti s sue death3. ↑ Nitric oxide → hypotens ion and shock4. Activation of a l ternate complement pathway →↑ C3a (edema) and C5a (polymorphonuclear [PMN] chemotaxis )5. Activation of factor XII → coagulation cascade → DIC

Which has a higher toxicity, endotoxins or exotoxins?Exotoxins : fata l dose on the order of 1 μg (vs hundreds of micrograms for endotoxins )

Name the mechanism of DNA transfer characterized by the following statements:DNA is taken up directly from the environment by competent cellsTransformation (can occur in eukaryotic cel l s , too)Medica l ly important natura l transformers : HHSNG: “Here, Have Some New Genes”: H. pylori, H. influenzae, S. pneumoniae, N. gonorrhoeaPlasmid or chromosomal DNA transferred from one bacterium to another via cell-to-cell contactConjugationDNA transferred by a virus from one cell to another; can be generalized or specializedTransductionDNA segments able to excise and reincorporate into different locationsTransposons

Name the bacterium whose exotoxin has the following effects:Superantigen that induces IL-1 and IL-2 synthesis in toxic shock syndrome; also leads to food poisoningStaphylococcus aureusα-Toxin is a lecithinase → gas gangreneClostridium perfringensPrevents the release of the neurotransmitter (NT) glycine from Renshaw cells in spinal cord → paralysisClostridium tetani↑ Adenylate cyclase by adenosine diphosphate (ADP) ribosylation → whooping coughBordetella pertussisExotoxin encoded by β-prophage; α subunit → inac vates elonga on factor 2 (EF-2) hal ng protein synthesis; β subunit → permits entry into cardiacand neural tissueCorynebacterium diphtheriasErythrogenic superantigen → rash in scarlet feverStreptococcus pyogenesPrevents release of acetylcholine (ACh) → central nervous system (CNS) paralysis; spores in canned food and honey, construction sitesClostridium botulinumHeat-stable toxin ↑ guanylate cyclase; heat-labile toxin ↑ adenylate cyclase by ADP ribosylation of G protein → watery diarrheaEscherichia coliInactivates the 60S ribosome → kills intestinal cellsShigella dysenteriaePermanent ADP ribosylation of G protein →↑ adenylate cyclase →↑ Cl- and H2OO in gut → voluminous stoolsVibrio choleraeExotoxin A inhibits protein synthesis by blocking EF-2Pseudomonas

Which virulence factor allows organisms to colonize mucosal surfaces?

IgA protease (eg, S. pneumoniae and H. influenzae)Name the bacterial structure associated with each antigenic classification given below:

K-antigenCapsule (related to vi rulence of the bacteria)K = KapsuleO-antigenOuter portion of the polysaccharide of endotoxinO = OuterH-antigenFlagel la (seen in moti le species )

Name the key virulence f actor(s) associated with each of the following organisms:Group A streptococcusM-protein, s treptokinase, and hya luronidaseStaphylococcus aureusProtein A (prevents complement fixation and phagocytos is ), penici l l inase, and hya luronidaseStreptococcus viridansExtracel lular dextran → helps bind to heart va lvesYersinia pestisF1 capsular antigen (antiphagocytic) and protease (degrades clots )Haemophilus influenzaeCapsule: s ix types (a-f) and IgA proteaseBorreliaAntigenic variationMycobacterium tuberculosisMycos ides (cord factor, wax D, and sul fatides )

Infectious Diseases

Name the organism(s) associated with each of the following characteristics:

Gram-positive rods with metachromatic granulesCorynebacterium diphtheriaeThree urease (+)Helicobacter pylori, Proteus, and Ureaplasma urealyticumAerosol transmission from environmental water source (eg, air conditioner)Legionella pneumophilaContain mycolic acid in membranesMycobacterium and NocardiaPeptidoglycan wall lacks muramic acidChlamydiaeProduces pyocyanin (blue-green) pigmentPseudomonas aeruginosaProduces yellow-gold pigmentStaphylococcus aureusProduces reddish pigmentSerratia marcescensOnly bacterial membrane containing cholesterolMycoplasma pneumoniaeFilamentous, branching rods in a cervicofacial infectionActinomyces ismeliiTwo forms: elementary and reticulate bodiesChlamydiaePleomorphic gram-negative rods in “school of fish” patternHaemophilus ducreyiClue cells on wet mountGardnerella vaginalisHigh liter of cold agglutinins (IgM)Mycoplasma pneumoniaeTwo fungi-like bacteriaActinomyces israelii and Nocardia asteroides

Name the organism(s) associated with the following pathology:Fitz-Hugh and Curtis syndromeChlamydia trachomatis or N. gonorrheaInvades gastrointestinal (GI) mucosa → diarrhea; motile; can disseminate hematogenouslySalmonellaInfected dog or cat bites (or scratches)Pasteurella multocidaGhon complexMycobacterium tuberculosis (1° tuberculos is [TB]). Note: hi lar nodes plus Ghon focus usual ly in lower lobeMeningitis and pneumonia in neonatesHaemophilus influenzaeAtypical pneumonia with avian reservoirChlamydia psittaciGas gangrene in traumatic open woundsClostridium perfringensInfects skin and superficial nervesMycobacterium lepraeFibrocaseous cavitary lung lesionMycobacterium tuberculosis (2° TB). Note: usual ly at apex because ↑ affini ty for ↑ O2 envi ronments

Mycobacterium causing disseminated disease in acquired immunodeficiency syndrome (AIDS) patientsMycobacterium avium-intracellulareMycobacterium causing cervical lymphadenitis in kidsMycobacterium scrofulaceum

For each of the following clinical findings, name the organism responsible and the drug(s) of choice:Oral/facial abscesses with sulfur granules in sinus tractsActinomyces israelii——penici l l in G (IV)Currant jelly sputumKlebsiella—firs t- or second-generation cephalosporinsWoolsorter’s diseaseBacillus anthracis—penici l l in G or ciprofloxacinScarlet fever, impetigo, and pharyngitisStreptococcus pyogenes—penici l l inPontiac feverLegionella pneumophila—macrol ide (erythromycin and azi thromycin)Gram-positive coccus causing sepsis/meningitis in a newbornStreptococcus agalactiae—ampici l l in (Note: group B, think Babies )Acute epiglottitis, meningitis, otitis, and pneumoniaHaemophilus influenzae—second-generation cephalosporins (treat meningi ti s with ceftriaxone, plus ri fampin for contacts )Gastritis and ~90% of duodenal ulcersHelicobacter pylori—triple therapyWaterhouse-Friderichsen syndromeNeisseria meningitidis—ceftriaxonePneumonia in cystic fibrosis and burn patientsPseudomonas cepacia—bactrim or ciprofloxacinBacterial vaginosis with discharge and fishy odorGardnerella vaginalis—metronidazoleBurn and wound infections with fruity odorPseudomonas aeruginosa—aminoglycos ide plus antipseudomonal (eg, piperaci l l in and tazobactam)Acute postinf ectious glomerulonephritisStreptococcus pyogenes—penici l l in GPseudomembranous enterocolitisClostridium difficile—metronidazole or ora l vancomycinAtypical “walking” pneumonia in young adultMycoplasma pneumoniae—erythromycin or doxycycl ineUrethritis/pelvic inflammatory disease (PID), neonatal conjunctivitis, and pneumoniaChlamydia tmchomatis types D to K—erythromycin eye drops in neonates , azi thromycin for urethri ti s , pneumoniaLyme diseaseBorrelia burgdorferi—doxycycl ineMalignant, vesicular papules covered with black eschar → bacteremia and even deathBacillus anthracis—penici l l in G or ciprofloxacinPneumonia, sepsis, otitis externa, UTIs, hot-tub f olliculitis, osteomyelitisPseudomonas aeruginosa—aminoglycos ide plus antipseudomonal piperaci l l in and tazobactamUndulant fever, Bang diseaseBrucella sp.—doxycycl ine plus gentamicin or ri fampin (pasteurize mi lk to prevent)Bubonic plagueYersinia pestis—gentamicinRocky Mountain spotted feverRickettsia rickettsii—tetracycl ine /doxycycl ineTrench fever (lasts 5 days; recurs in 5-day cycles)Bartonella quintana—gentamicin/doxycycl ineTabes dorsalis, aortitis, and gummasTreponema pallidum (3° syphi l i s )—penici l l in GQ fever (acute)Coxiella burnetii—doxycycl ineWeil diseaseLeptospira interrogans—penici l l in GYawsTreponema pertenue—penici l l in GPott diseaseMycobacterium tuberculosis (disseminated)—four-drug anti -tuberculous therapy, including ri fampin plus i soniazid (INH)Dental cariesStreptococcus mutans—amoxici l l in or amoxici l l in/clavulonic acid (prevention with topica l fluoride/chlorhexidine)Rheumatic feverStreptococcus pyogenes—penici l l in GScalded skin syndrome and toxic shock syndromeStaphylococcus aureus—penici l l in agent (vancomycin i f methici l l in-res is tant S. aureus[MRSA])Hansen diseaseMycobacterium leprae—dapsone plus clofazimine or ri fampin

What is the differential for a rash affecting the palms and soles?Rocky Mounta in spotted fever, 2° syphi l i s , hand-foot-and-mouth disease (coxsackie A), and Kawasaki syndrome

Name the mode of transmission and reservoir(s) for each of the following bacteria:Brucella sp.Contact with animals or da i ry products ; cowsFrancisella tularensisTick or deerfly bi te; rabbits and deerPasteurella multocidaAnimal bi te/scratch; cats and dogsBorrelia burgdorferi

Ixodes tick bi te; l i ves on deer and miceYersinia pestisFlea bi te; rodents (eg, pra i rie dogs)Rickettsia rickettsiiTick bi te; dogs , rabbits , and rodents (endemic to eastern United States )Rickettsia prowazekiiHuman body louse; humans and flying squirrels

Name the laboratory test described below:Detects antirickettsial antibodiesWei l -Fel ix reaction (cross -reacts with proteus)Sensitive for treponemesFluorescent treponemal an body-absorp on test (FTA-ABS) (+) (earl ies t and longest, used as confirmatory test for syphi l i s i f RPR i sreactive)Useful in screening for TBPuri fied protein derivative (PPD) testName the screening test for syphilis and four biological false positives:VDRL tes t1. Vi ruses (mononucleos is and hepati ti s )2. Drugs (narcotics )3. Rheumatoid arthri ti s /fever4. Leprosy and lupus

Name the normal, dominant flora for each of the f ollowing locations:NoseStaphylococcus aureusOropharynxGroup D s treptococci (vi ridans)Dental plaquesStreptococcus mutansColonBacteroides fmgilis > E. coliVaginaLactobacillus; colonized by E. coli and group B s treptococcusSkinStaphylococcus epidermidis

Name the nosocomial pathogen(s) associated with each of the following:Urinary catheterEscherichia coli and Proteus mirabilisRespiratory therapy equipment, ventilatorsPseudomonas aeruginosaWound infectionsStaphylococcus aureusWater aerosolsLegionella sp.

Tuberculosis

Decide whether each of the f ollowing statements is more closely associated with 1° or 2° TB:

Radiographic finding = Ghon complex; classically affects lower lobes1°TBMiliary TB2°TBFibrocaseous cavitary lung lesion; classically affects apical lungs (↑ affinity for ↑ O2 environment)2°TBSymptoms of cough/hemoptysis, fever, night sweats, and weight loss2°TB

What type of hypersensitivity reaction is seen after infection with M. tuberculosis?Type IV or delayed-type hypersens i tivi ty (bas is for PPD test)What unique type of cell is seen in association with caseating granulomas in TB?Langerhans giant cel l

What is the mode of transmission of M. tuberculosis!Respiratory droplets

What term describes the lymphatic and hematogenous spread of TB, causing numerous small foci of infection in extrapulmonary sites?Mil iary TB

Name five common sites of extrapulmonary TB:1. CNS (tuberculous meningi ti s )2. Vertebra l bodies (Pott disease)3. Psoas major muscle → abscess4. GI tract (l iver and cecum)5. Cervica l lymph nodes →scrofuloderma

What is an effective screening tool for latent TB?PPD test

How is active TB infection diagnosed?Cl inica l and radiologic s igns of 2° TB and acid-fast baci l l i in sputum

What is the management of PPD + latent TB?Treatment with INH + pyridoxine (vi tamin B6) for 9 months

What is the treatment for active TB?Respiratory i solation and ini tia l four-drug therapy (RIPE: Ri fampin, INH, Pyrizinamide, Ethambutol )

What is the major toxicity of most TB drugs?

Hepatotoxici ty; INH → vi tamin B 6deficiency; ethambutol → optic neuri ti s

ANTIBIOTICS

Name the drug(s) whose mechanism of action is described below:

Binds penicillin-binding proteins →inhibits transpeptidase → blocks cell wall synthesis; also releases autolytic enzymes (bactericidal)β-Lactam antibiotics (penici l l in, cephalosporins , cephalomycins , carbapenems, and monobactams)Forms reactive cytotoxic metabolites inside cellMetronidazoleBinds and inactivate β-lactamase → protects antibioticβ-Lactamase inhibi torsInhibits 50S peptidyl transferaseChloramphenicolBlocks entry of aa-tRNA to 305 ribosomal complexTetracycl inesBlocks transpeptidation of D-ala Inhibits dihydrof olate reductaseVancomycinInhibits dihydrofolate reductaseTrimethoprim (TMP)Para-aminobenzoic acid (PABA) antimetabolites →↓ dihydropteroate synthaseSul fonamidesBinds to 30S subunit → block formation of 70S initiation complex → misreading of mRNAAminoglycos idesBinds to 50S subunit → inhibit translocaseMacrol ides (erythromycin and azi thromycin)Blocks DNA topoisomerase (gyrase)Quinolones (ciprofloxacin and levofloxacin)Blocks 50S peptide bond formationCl indamycinInhibits DNA-dependent RNA polymeraseRi fampinInterferes with mycolic acid synthesisINHBind to bacterial/fungal cell membranes → disrupt osmotic propertiesPolymyxinsPABA antagonist → blocks purine synthesisSul fones (dapsone and sul foxone)

Name the antibacterial drug(s) associated with each of the following unique toxicities:Kernicterus in infantsSul fonamides and ceftriaxoneInterstitial nephritisPenici l l insDisulfiram-like reactionsMetronidazole, second-generation cephalosporinsPhotosensitivity rashDoxycycl ineGray baby syndromeChloramphenicolMegaloblastic anemiaIMPHemolytic anemia in G6PD-def icient patientSul fonamides , chloramphenicol , ni trofurantoin, and INHHepatotoxicity, vitamin B6deficiency, lupuslike syndromeINH (Note: ↑ t1/2 in s low acetylators )Pseudomembranous colitisCl indamycin (most common)Fanconi syndromeTetracycl ine (ingestion of expired drug)Ototoxicity and nephrotoxicityAminoglycos idesRed, pruritic rash on torso with rapid IV infusion (red man syndrome)VancomycinReversible cholestatic hepatitis; ↑ GI motilityErythromycinAchilles tendonitis; cartilage damage in laboratory animalsFluoroquinolonesRed-orange discoloration of bodily secretionsRi fampinDiscolors teeth; suppresses bone growth in kidsTetracycl ineAplastic anemia (dose independent)ChloramphenicolNeurotoxicity and nephrotoxicityPolymyxins

Name six uses for metronidazole:1. Giardia2. Entamoeba

3. Trichomonas4. Gardnerella vaginalis5. Anaerobes (C. difficile, bacteroides)6. Helicobacter pylori (part of triple therapy)

Which drug is used as solo prophylaxis for TB?INH

How do organisms develop resistance against vancomycin?D-lac (or D-ser) replaces terminal D-a la in cel l wal l → ↓ affini ty of vancomycin for cel l wal l

VIROLOGY

Taxonomy/Basics

Name six medically important DNA viral families:

HHAPPPy:1. Hepadnaviridae2. Herpesvi ridae3. Adenoviridae4. Poxvi ridae5. Parvovi ridae6. Papovaviridae

Name two families of circular DNA viruses:1. Papovaviridae2. Hepadnaviridae

Name the only ssDNA viral genome:Parvovirus

Name the only DNA virus that replicates in the cytoplasm:Poxviridae (carries i ts own DNA-dependent RNA polymerase)

Name three naked DNA viruses:PAP:1. Parvo2. Adeno3. Papov

Name the only dsRNA viral genome:Reoviruses (eg, rotavi rus )

Name the family of the smallest RNA viruses:Picornavi ruses

Name four families of naked RNA viruses:CRAP:1. Ca l icivi rus2. Reovirus3. Astrovi rus4. Picornavi rus

Name two families of RNA viruses that do not replicate solely in the cytoplasm:1. Influenza vi ruses2. Retrovi ruses

Where do most enveloped viruses acquire their membranes?From plasma membrane (except herpesvi ruses—nuclear membrane)

Name four families of segmented viruses:BOAR (a l l RNA vi ruses )1. Bunyaviridae2. Orthomyxoviridae (influenza vi ruses )3. Arenaviridae4. Reoviridae

Name the only diploid viruses:Retrovi ruses

What types of nucleic acids do not require special enzymes to be infectious?Those with same s tructure as host nucleic acids (eg, pos i tive-s tranded ssRNA and most naked dsDNA)

Name the type of viral genetic strategy described below:The virus contains its own genetic material but is coated with surface proteins from another virus, which determine its inf ectivityPhenotypic mixingOccurs when viruses exchange segments of their genomesReassortmentOccurs when a nonmutated virus assists a mutated one by making a functional gene product that serves both itself and the mutated virusComplementationExchanging oligonucleotides by crossing-over within base sequencesRecombination

What type of antigenic change in the influenza virus causes epidemics?Antigenic dri ft (minor changes from random mutation)

What type of antigenic change in the influenza virus causes pandemics?Antigenic shi ft (reassortment of genome, including animal acquis i tion)(Dri fting i s a Slow Process , Shi fting i s a Rapid Process )

For each of the following vaccines, state whether it is live attenuated or killed:Sabin polioLive attenuatedSalk polio

Ki l led (Sa lK = Ki l led)May revert to virulenceLive attenuated (very rare)

For whom is it dangerous to receive live vaccines?Immunocompromised hosts (ie, transplant recipients , AIDS patients , and pregnant women)

Infectious Diseases

Name the virus(es) associated with each of the following statements:

Tzanck prep shows multinucleated giant cellsHerpes s implex vi rus (HSV)-l , HSV-2, and varicel la -zoster vi rus (VZV)Viral culture with buffy coatCytomegalovi rus (CMV)Transmitted by bat, raccoon, and skunk bitesRabies vi rusTransmitted by arthropodsArbovirusesCowdry type A inclusion bodiesHerpesvi ruses (HSV-1, CMV, and VZV)Number 1 cause of diarrhea in kids 3 years old (y/o)Rotavi rusNumber 1 cause of viral pneumonia in infants 6 monthsRespiratory syncytia l vi rus (RSV)Severe (but rare) seqealae include giant cell pneumonia and subacute sclerosing panencephalitis (SSPE)Meas les vi rus (rubeola)Councilman bodies in liverYel low fever vi rusReactivation of virus in brain of AIDS patient → demyelination, deathJC vi rus → progress ive multi foca l leukoencephalopathy (PML)Bullet-shaped, helical nucleocapsid; travels up nerve axons to CNS in retrograde fashionRabies vi rusKoplik spotsMeas les vi rus (rubeola)Dane particleHepati ti s B vi rusAtypical lymphocytesEpstein-Barr vi rus (EBV) and CMVNosocomial infection associated with the newborn nurseryCMV and RSVNegri bodies in neuronsRabies vi rusIncomplete RNA virus → requires envelopeHepati ti s D vi rusPositive heterophile antibody testEBVNecrosis of large motor neurons in anterior horn spinal cord → flaccid paralysisPol iovi rus

For each of the following clinical findings, name the associated virus:ShinglesVZVSuboccipital lymphadenopathyRubel la vi rusHerpangina, hand-foot-and-mouth diseaseCoxsackie A vi rusGingivostomatitis, keratitis, and temporal lobe encephalitisHSV-1Genital and neonatal infectionsHSV-2Fever, hepatosplenomegaly pharyngitis, and posterior auricular lymphadenopathy; “kissing disease”EBV (mononucleos is )Common coldRhinoviruses (>100 serotypes ; associated with 85% of cases )PMLJC polyomavirusChickenpoxVaricel laSmall, pearly, umbilicated papular epidermal growths near genitalsMol luscum contagiosum vi rusGerman measles and congenital infectionsRubel la vi rusExplosive gastroenteritis; recent epidemics on cruiseships and schoolsNorwalk vi rusExanthem subitum (roseola)Human herpesvi rus (HHV)-6Contains hemagglutinin and neuraminidase virulence factor; undergoes antigenic shift and responsible for pandemicsInfluenza AFever, black vomitus, and jaundice; transmitted by Aedes mosquitoYel low fever vi rus

Pericarditis, myocarditis, and pleurodyniaCoxsackie B vi rusAcute viral hepatitisHepati ti s A and BMononucleosis, congenital infection, and pneumoniaCMVSubacute sclerosing panencephalitis (SSPE)Meas les (rubeola) vi rusIntussusception from hyperplasia of Peyer patchesAdenovirusesKaposi sarcomaHHV-8Aseptic meningitis, orchitis, and parotitisMumps vi rusBarking cough and laryngeal swellingPara influenza vi ruses (croup)Hepatitis from IV drug abuse or blood transfusionHepati ti s C vi rusHydrophobia, seizures, and fatal encephalitisRabies vi rusCause of common and genital wartsHuman papi l lomavirus (HPV)Tropical spastic paresisHuman T-cel l leukemia/lymphoma vi rus (HTLV)Hepatitis with high mortality rate in pregnant womenHepati ti s E vi rusReye syndromeInfluenza vi ruses (occurs with aspi rin [ASA] ingestion)Epidemic keratoconjunctivitis; childhood URIsAdenovirusesCough, coryza, and conjunctivitisMeas les (rubeola) vi rusErythema infectiosum (fifth disease); transient aplastic anemic crisisParvovirus B19

Name the oncogenic virus associated with the following cancers:Burkitt lymphomaEBVHepatocellular carcinomaHepati ti s B and C vi rusesHairy cell leukemiaHTLV-2Adult T-cell lymphomaHTLV-1Nasopharyngeal carcinomaEBVKaposi sarcomaHHV-8Cervical, penile, and anal carcinomaHPV types : 16,18, 31, 33, and 45

Name the family of viruses characterized by the following:Smallpox, molluscum contagiosum, and vacciniaPoxviridaeHantavirus and California encephalitisBunyaviridaeMarburg and Ebola hemorrhagic feverFi lovi ridaeLassa fever and lymphocytic choriomeningitisArenaviridae

Human Immunodeficiency Virus

Which continent has the highest incidence (and prevalence) of AIDS?

AfricaWhat are the three major routes of HIV transmission?

1. Sexual contact2. Vertica l (mother to newborn) transmiss ion3. Parentera l

What is the most common mode of HIV transmission on a global basis?Heterosexual contact

What type of virus is HIV?Human retrovi rus of the lentivi rus fami ly

What test is used to screen for HIV infection?Enzyme-l inked immunosorbent assay (ELISA) looks for AB to vi ra l proteins ; ↑ sens i tivi ty.

What test is used to confirm HIV(+) screening results?Western blot assay (high fa lse negative within 2 months of infection); ↑ speci fici ty

Which enzyme creates dsDNA from RNA for integration into host genome?Reverse transcriptase (RT)

What test is used to monitor the effects of antiretroviral therapy?

HIV RT-polymerase chain reaction (RT-PCR) (measures vi ra l load)What is the strongest measure of disease progression in an HIV(+) patient?

CD4+ T-cel l countName two key glycoproteins on the surface of the HIV viral envelope:

1. gp41 (fus ion)2. gp120 (attachment) proteins ; together = gp160

Name two key HIV viral core proteins:1. p24 (nucleocaps id)2. p17 (matrix protein)

Name three key HIV retroviral enzymes contained in the core:1. RT2. Integrase3. Protease (a l l encoded by pol gene)

Which viral antigen peaks within 2 months of infection, then rises again years later?p24

What are the two cell surface molecules to which gpl20 must bind?1. CD42. A chemokine receptor (CCR5 or CXCR4)

HIV infects which three cell types?1. CD4+T cel l s2. Monocytes/macrophages3. Dendri tic cel l s

The induction of what cellular transcription factor during an immune response leads to activation of transcription of HIV proviral DNA?Nuclear factor-kappa B (NF-KB)

Protease inhibitors of HIV prevent cleavage of the protein product of what viral genes?Gag and pol genes

What are the three mechanisms by which HIV-infected CD4+ T cells are lost?1. HIV cytopathic effect2. Apoptos is3. HlV-speci fic cytotoxic T-cel l ki l l ing

What are the three stages of HIV infection?1. Acute retrovi ra l infection2. Chronic phase3. AIDS

What is the surrogate measure of viral load in an HIV(+) patient?HIV-1 RNA

What is the strongest measure of disease progression in AIDS?CD4+ T-cel l count

Which tissues are the major reservoirs of HIV-infected T cells and macrophages in patients?1. Lymph nodes2. Spleen3. Tons i l s

Which cell type in the brain is infected by HIV?Microgl ia l cel l s

List the major immune abnormalities in AIDS:1. Decreased number of CD4+ T cel l s2. Decreased T-cel l function3. Polyclonal activation of B cel l s4. Al tered macrophage function

What is the clinical picture of direct viral disease from HIV?Cons tu onal symptoms (weight loss , fever, fa gue, and night sweats ) and/or neurologic symptoms (encephalopathy with demen a andaseptic meningi ti s )

How is AIDS defined?CD4+ 200 or AIDS-defining i l lness , regardless of CD4+ count

Name the common AIDS opportunistic organisms or infections/diseases associated with the following:Four fungal infections1. Candidias is (GI tract)2. Cryptococcos is (meningi ti s )3. His toplasmos is (disseminated)4. Coccidioidomycos is (disseminated)Five bacterial infections1. Mycobacterium tuberculosis (lung/disseminated)2. Mycobacterium avium-intracellulare (lung)3. Nocardia (lung/CNS/disseminated)4. Salmonella (disseminated)5. Encapsulated organismsFour viral infections1. HSV2. VZV (shingles )3. CMV (retini ti s or col i ti s )4. JC vi rus (PML)Three protozoal infections1. Pneumocystis (lung or disseminated)2. Toxoplasm (lung/CNS)3. Cryptosporidium (GI)

State the typical CD4+ count associated with each of the following HIV complications:Opportunistic infections are typically seen, especially Pneumocystis jiroveci pneumonia

200 cel l s /mLMycobacterium avium complex (MAC), CMV, and cryptosporidiosis50 cel l s /mLToxoplasmosis100 cel l s /mLTB becomes more common400 cel l s /mL

List four common neoplasms in patients with AIDS:1. Kapos i sarcoma2. Non-Hodgkin B-cel l l ymphoma3. CNS lymphoma4. Squamous cel l carcinoma of the cervix or anus

What has been shown to minimize the risk of perinatal HIV transmission?Zidovudine (AZT) given to pregnant women, cesarean del ivery, and avoiding breast feeding

Prions

Which infectious agents lack both DNA and RNA?

Prions (made of proteins only)What are symptoms of prion diseases?

Rapidly progress ing dementia , psychiatric dis turbances , and cerebel lar symptoms (ataxia , myoclonis ). Al l prion diseases are fata l .Name four prion diseases:

1. Creutzfeldt-Jacob disease (rapidly progress ive dementia)2. Mad cow disease3. Kuru4. Fata l fami l ia l insomnia

What type of histopathologic change is seen in these diseases?Spongi form encephalopathy

ANTIVIRAL AGENTS

For each of the following drugs, provide:1. The mechanism of action (MOA)2. Indication(s) (IND)3. Significant side effects and uniquetoxicity (TOX) (if any):

Acyclovir and valacyclovirMOA: guanos ine analog; activated by herpes thymidine kinase → inhibi ts vi ra l DNA polymeraseIND: HSV (treatment and prophylaxis for ora l , geni ta l , and ocular herpes), VZV (chickenpox and shingles )TOX: neurotoxic (del i rium and tremors) and nephrotoxicGanciclovirMOA: guanos ine analog; activated by human thymidine kinase → inhibi ts CMV DNA polymeraseIND: CMV (retini ti s , pneumonia , col i ti s ), especia l ly in immunocompromised peopleTOX: bone marrow suppress ion, nephrotoxic, and ↓ spermatogenes is (toxici ty → acyclovi r because activated by human enzyme)FoscarnetMOA: pyrophosphate analog; inhibi ts vi ra l DNA polymerase (no activation required)IND: CMV, HSV (refractory infections)TOX: revers ible nephrotoxici ty and anemiaNucleoside RT inhibitors (zidovudine—azidothymidine [AZT], didanosine—ddl, zalcitabine—ddC, lamivudine-3TC, and stavudine—d4T)MOA: nucleos ide analogs ; activated by phosphorylation → inhibi ts RT → prevents incorporation of vi ra l genome into host DNAIND: part of combination therapy for HIVTOX: bone marrow suppress ion, periphera l neuropathy, pancreati ti s (especia l ly ddl ), lactic acidos is , and macrocytic anemia (AZT)Nonnucleoside RT inhibitors (nevirapine, delavirdine, and efavirenz)MOA: binds di rectly to and inhibi ts HIV RT → prevents incorporation of vi ra l genome into host DNAIND: part of combination therapy for HIVTOX: rash (including Steven-Johnson), ↑ l iver enzymes , inhibi ts P-450, vivid dreams/CNS changes (with ef avi renz)Protease inhibitors (saquinavir, ritonavir, indinavir, nelf inavir, and amprenavir)MOA: blocks protease enzyme → inhibi ts assembly of vi ra l core and new vi rusesIND: part of combination therapy for HIVTOX: GI upset, insul in res is tance, ↑ l ipids , fat redis tribution syndromes, inters ti tia l nephri ti s , and thrombocytopenia (indinavi r)Amantadine and rimantadineMOA: inhibi ts vi ra l penetration and uncoating; releases dopamine (DA) from intact nerve terminalsIND: influenza A treatment/prophylaxis , Parkinson diseaseTOX: CNS effects : confus ion, ataxia , and s lurred speech (less with rimantadine); teratogenes isZanamivir and oseltamivirMOA: neuraminidase inhibi tor → a l ters vi rion aggregation and releaseIND: influenza A and B treatment and prophylaxis (osel tamivi r)TOX: bronchospasm in patients with asthma /COPD) (zanamivi r)RibavirinMOA: guanos ine analog; activated by phosphorylation → inhibi ts inos ine-5’-monophosphate (IMP) dehydrogenaseIND: RSV, hantavi rus , and chronic hepati ti s CTOX: hemolys is (when given IV)Interferon-αMOA: human glycoproteins that interfere with abi l i ty of vi ruses to repl icate (block protein synthes is and degrade mRNA)IND: chronic hepati ti s B and C, geni ta l warts , Kapos i sarcoma, and ha i ry cel l leukemiaTOX: bone marrow suppress ion

What constitutes highly active antiretroviral therapy (HAART)?Two nucleos ide RT inhibi tors and a protease inhibi tor or nonnucleos ide RT inhibi tor. Note: no pa ent should ever be on monotherapy as

res is tance i s invariable.When is HAART typically initiated?

CD4+ 500 cel l s /mL or very high vi ra l loadWhich drug is used to prevent vertical transmission during pregnancy?

Zidovudine (AZT)

MYCOLOGY

Which four endemic mycoses can mimic TB?

1. His toplasmos is2. Coccidioidomycos is3. Paracocidioidomycos is4. Blastomycos is

What is a dimorphic fungus?Lives in two forms: cold = mold (37°C), heat = yeast

Name the fungus associated with each of the following statements:“Spaghetti and meatball” appearance on KOH prepMalassezia furfurContains cancer-causing aflatoxinsAspergillus flavusDimorphic fungus living on rose thorns and splintersSporothrix schenckiiUrease (+), stains with India ink, and latex agglutination (+)Cryptococcus neoformansOrganism found inside macrophages; spread in pigeon and bat droppingsHistoplasma capsulatumBudding yeast, pseudohyphae; germ tubes at 37°CCandida albicansWide angle (90°) branching of irregular nonseptated hyphaeMucor and Rhizopus sp.Big, broad-based budding dimorphic fungusBlastomycosisB-B-B-B-B (Big, Broad-Based Budding Blasto)45° angle branching and septated hyphae; fruiting bodiesAspergillus sp.Narrow-based unequal budding yeasts with capsular haloCryptococcus neoformansEndemic to Ohio and Mississippi river valleysHistoplasma capsulatum”Flying-saucer” appearance of silver stainPneumocystis carinii (a l so P. jiroveci)”Captain’s wheel” appearance; endemic to rural Latin AmericaParacoccidioidomycosis

For each of the f ollowing diseases, name the fungus/yeast responsible and the drug of choice:Rose gardener disease with ascending lymphangitisSporothrix schenckii—potass ium iodideTinea nigraPhaeoannellomyces werneckii—topica l sa l icyl i c acidThrush in an immunocompromised patientCandida albicans—nystatin (swish and swal low) and amphotericin B i f sys temic infectionSan Joaquin (desert valley) fever; endemic to southwest United States, CaliforniaCoccidioides immitis—fluconazole and amphotericin BInterstitial pneumonia of the immunocompromisedPneumocystis carinii (or P. jiroveci)—TMP-sul famethoxazole (SMX) or pentamidine (prophylaxis when CD4+200 cel l s /mL)Tinea (pityriasis) versicolorMalassezia furfur—topica l miconazole or selenium sul fide

Meningitis from pigeon droppingsCryptococcus neoformans—amphotericin + flucytos ine for 2 weeks fol lowed by fluconazoleTinea cruris/capitis/corporis/unguium/pedisTrichophyton, Epidermophyton, or Microsporum sp.—topica l miconazole, ora l gri seofulvin for capi ti s and unguiumFungus ball in lungs or invasive diseaseAspergillus fumigatus—voriconazole

ANTIFUNGALS

Name the drug whose mechanism of action is described below:

Blocks ergosterol (unique to fungi) synthesis by inhibiting P-450Azole fami ly (ketoconazole, fluconazole, miconazole, voriconazole, and posaconazole)Binds ergosterol → produces membrane poresAmphotericin B and nystatinBlocks ergosterol synthesis by blocking squalene epoxidaseTerbinafineInterferes with microtubule formation → inhibits mitosisGriseofulvinConverted to 5-fluorouracil (5-FU) → blocks formation of purinesFlucytos ine

Name the antifungal drug(s) associated with each of the following unique toxicities:Rigors, acute febrile reaction, nephrotoxicity, and arrythmiasAmphotericin B—fol low BUN/creatinine da i ly. Note: newer l ipid formulations (ie, AmBisome less nephrotoxic)Photosensitivity, mental confusion, bone marrow suppression, and induces P-450GriseofulvinAntiandrogenic effects, adrenal suppression, and liver dysfunctionAzole fami ly (especia l ly ketoconazole)Bone marrow suppression and alopeciaFlucytos ine

PARASITOLOGY

Protozoa

Name the protozoan associated with each of the following statements:

Transmitted by Tsetse fly; shows antigenic variationTrypanosoma brucei (gambiense and rhodesiense)Transmitted by Anopheles mosquitoPlasmodiumTransmitted by Reduviid bugTrypanosoma cruziTransmitted by cysts in meat or cat fecesToxoplasmaTransmitted by sandflyLeishmaniaObligate intracellular parasite; cysts on acid-fast stainCryptosporidiumMaltese “X” cross shapeBabesiaPear-shaped, binucleate, flagellated trophozoiteGiardia lambliaBlood smear shows trophozoites and schizontsPlasmodiumMacrophages containing amastigotesLeishmania donovani

For each of the following clinical findings, name the associated protozoan and the treatment:Megacolon, megaesophagus, and cardiomegaly (with apical atrophy)Trypanosoma cruzii (Chagas disease)—nifurtimoxCyclic fever, headache, anemia, and splenomegalyPlasmodium—chloroquine for erythrocyte forms and primaquine for latent forms (Plasmodium vivax and Plasmodium ovale)Chloroquine-resistant malariaPlasmodium falciparum—mefloquine or quinine and pyramethamine/sul fadoxineBloody diarrhea with “flask-shaped” ulcers, liver abscesses, and trophozoites in stoolEntamoeba histolytica—metronidazole and iodoquinolBlack fever or “kala-azar”Leishmania donovani—sodium stibogluconate (pentava lent antimony)Severe, watery diarrhea in AIDS patientCryptosporidium—supportive (hydration, improve immune s tatus )Flatulence, bloating, and foul-smelling diarrheaGiardia lamblia—metronidazoleAfrican sleeping sicknessTrypanosoma brucei (gambiense and rhodesiense)—suramin (acutely) or melarsoprol (for CNS symptoms)Encephalitis with brain abscesses in immunocompromised host; congenital defectsToxoplasma gondii—pyrimethamine and sul fadoxine or cl indamycinVaginitis with foul-smelling, frothy dischargeTrichomonas vaginalis—metronidazoleFever and hemolytic anemia after Ixodes tick biteBabesia sp.—quinine and cl indamycin

Why are many Africans resistant to P. vivax infection?They carry s ickle cel l tra i t and/or they lack antigens Duffy a and b on RBCs .

What causes the cyclic symptoms in malaria?Immune response to burst RBCs that release merozoi tes

Helminths

For each of the following clinical findings, name the associated helminth and the treatment:

Larvae penetrate skin of feet; GI infection → anemiaAncylostoma duodenale or Necator amencanus (hookworms)—mebendazole or pyrantel pamoateWorms visibly crawling in conjunctiva; spread by deerflyLoa loa—diethylcarbamazineFever, periorbital edema, and myositis after ingesting raw porkTrichinella spiralis—thiabendazoleRiver blindness; spread by female blackfliesOnchocerca volvulus—ivermectin (rIVERmectin for RIVER bl indness )Elephantiasis from lymphatic blockage

Wuchereria bancrofti—diethylcarbamazineLarvae penetrate skin → autoinfection; GI infectionStrongyloides stercoralis—(threadworm)-thiabendazole or ivermectinIntestinal infection and anal pruritis; ↑ incidence in children; positive “tape test”Enterobius vermicularis (pinworms)—mebendazole or pyrantel pamoateFluke associated with squamous cell carcinoma of the urinary tractSchistosoma haematobium—praziquantelGranulomatous hepatitis and chorioretinitisToxocara canis—diethylcarbamazineGI infection; competes for food → malnutrition in children; eggs visible in fecesAscaris lumbricoides—mebendazole or pyrantel pamoateInflammation and 2° bacterial infection of lungs from undercooked crab meatParagonimus westermani—praziquantelBiliary tract inflammation from undercooked fishOpisthorchis (Clonorchis) sinensis—praziquantelUndercooked pork larval worm → mass lesions in brain; cysticercosisTaenia solium—praziquantel or niclosamide, a lbendazole for cysticercos isHydatid liver cysts from eggs in dog feces → anaphylaxis if antigens released from cystsEchinococcus granulosus—albendazole, careful surgica l removal of cysts (preinjected with ETOH)Fish tapeworm causing vitamin B12deficiencyDiphyllobothrium latum—praziquantelCercariae penetrate skin → granulomas and inflammation of liver and spleen; snails are hostsSchistosoma sp.—praziquantel

Which cell count is elevated during many helminth infections and can be detected by routine CBC?Eosinophils

Which is the most common helminth infection in the United States?Enterobius vermicularis

Name the antiparasitic drug(s) associated with each of the following unique toxicities:CinchonismQuinineHemolytic anemia in G6PD-deficient personChloroquine, primaquine, quinine, and TMP-SMXMazzotti reaction (pyrexia, hypotension, and respiratory distress caused by death of parasites)Diethylcarbamazine (with Onchocerca)

CHAPTER 4Neuroscience

EMBRYOLOGY

Name the structure(s) in the adult nervous system that arise from the following embryonic components:

Alar plateSensory neuronsBasal plateMotor neuronsTelencephalonCerebra l hemispheres and latera l ventriclesDiencephalonThalamus, optic nerves , and thi rd ventricleMesencephalonMidbra in and aqueductMetencephalonPons , cerebel lum, and superior fourth ventricleMyelencephalonMedul la and inferior fourth ventricleNeural crest cellsPeriphera l sensory and autonomic nerves and sensory gangl ia

What is the level of the conus medullaris in a newborn and in an adult?L2 or L3 (newborn), L1 (adult)

NEUROANATOMY AND NEUROPHYSIOLOGY

Organization of the Nervous System

What are the divisions of the autonomic nervous system?

Sympathetic, parasympatheticWhere are the preganglionic cell bodies of the sympathetic nervous system?

Intermediolatera l horn of the spina l cord from T1 to L3Where are the preganglionic cell bodies of the parasympathetic nervous system located?

Bra instem (crania l nerve nuclei ) and spina l cord from S2 to S4Which is the primary neurotransmitter (NT) of both sympathetic and parasympathetic ganglia?

Acetylchol ine (ACh)Which is the primary type of cholinergic receptor of both sympathetic and parasympathetic ganglia?

NicotinicWhich NT mediates the transmission of impulses from sympathetic neurons to effector organs?

Norepinephrine (NE)Which NT mediates the transmission of impulses from parasympathetic neurons to effector organs?

AChWhich NT mediates the transmission of impulses from somatic neurons to skeletal muscle?

AChWhat types of receptors are present on the effector organs innervated by the sympathetic nervous system?

α1, α2, β1, and β2

What type of receptor is present on the effector organs innervated by the parasympathetic nervous system?Muscarinic

What type of receptor is present on muscle innervated by the somatic nervous system?Nicotinic

Sympathetic Nervous System

Name the effect of the sympathetic nervous system on the f ol lowing organ systems and the type of receptor which mediates each effect:

EyesPupi l lary di lation (α1)Salivary glandsIncreased thick, vi scous secretionsBronchiolesBronchodi lation (β2), ↑ secretionsHeartTachycardia (β1), ↑ contracti l i ty (β1), ↑ AV nodal conduction (β1)Vascular smooth muscleVasoconstriction of cutaneous mucous membrane and splanchnic vessels (α1); vasodi lation in skeleta l muscle (β2)Gastrointestinal (GI) tract↓ Muscle moti l i ty and tone (β2), contraction of sphincters (α1)Male sex organsEjaculation (α2)Uterus

Relaxation (β2), contraction (α1)Bladder and uretersRelaxation of detrusor (β2) contraction of trigone and sphincter (α1)Sweat glands↑ Secretions (muscarinic)Kidneys↑ Renin secretionAdipocytes↑ Lipolys is (β1)Pancreas↓ Insul in secretion (α2), ↑ insul in secretion (β2)

Parasympathetic Nervous System

What is the effect of the parasympathetic nervous system on the f ollowing organ systems:

EyesPupi l lary constrictionBronchiolesBronchoconstrictionHeartBradycardia , ↓ contracti l i ty, ↓ AV nodal conductionGI tract↑ Moti l i ty, relaxation of sphinctersMale sex organsErectionBladder and uretersContraction of detrusor, relaxation of sphincters and trigone

What type of cholinergic receptor mediates all of the effects on the organs above?Muscarinic

Motor and Sensory Fibers?

What type of motor fiber innervates extrafusal muscle fibers?

A-a lpha (A-α)What type of motor fiber innervates intrafusal muscle fibers?

A-gamma (A-γ)Name the function of each of the following types of sensory fibers:

Ia (A-α)Proprioception, muscle spindlesIbProprioception, Golgi tendon organsII (A-β)Touch, pressure, and vibration; secondary afferents of muscle spindlesIII (A-δ)Touch, pressure, fas t pa in, and temperatureIV (c)Slow pain and temperature (unmyel inated)

What types of sensory fibers have the largest diameter and consequently the fastest conduction velocity?Ia and Ib

What type of motor fibers have the largest diameter and consequently the fastest conduction velocity?A-α

What type of sensory fibers have the smallest diameter and consequently the slowest conduction velocity?C

What is the electrochemical effect of an inward Na+ current on a sensory fiber?Depolarization

Name the function of each of the following components of a sensory pathway:Sensory receptorTrans lates envi ronmenta l s timulus into an electrica l impulseFirst-order neuronCarries impulse from sensory receptor into centra l nervous system (CNS)Second-order neuronCarries impulse from primary neuron to the tha lamusThird-order neuronCarries impulse from second-order neuron to the cerebra l cortexFourth-order neuronCarries impulses from thi rd-order neurons to appropriate somatosensory area of cerebra l cortex

Name the type of mechanoreceptor described below:Onion-like subcutaneous receptors that respond to vibration and tappingPacinian corpusclePrimary receptors of the dermal papillae that mediate two-point tactile discriminationMeissner corpuscleEncapsulated receptor that responds to pressureRuffini corpuscleDisc-shaped touch receptor of the deep dermisMerkel tacti le disc

Rods or cones?

Sensitive to low-intensity lightRodsSensitive to high-intensity lightConesReceptor used primarily for night visionRodsReceptor used primarily for day visionConesPresent in foveaConesHigh visual acuityConesReceptor which adjusts to low light conditions most rapidlyConesReceptor capable of color visionCones

Name the type of muscle sensor for each of the following functions:Detection of static and dynamic changes in muscle lengthMuscle spindlesDetection of muscle tensionGolgi tendon organsDetection of vibrationPacinian corpusclesDetection of painFree nerve endings

What type of motoneuron is responsible for ensuring that a muscle will respond appropriately throughout contraction, despite changes in tension?γ-Motoneurons

What type of muscle reflex, mediated by type Ia afferent fibers, causes muscle contraction in response to muscle stretch?Stretch or myotatic reflex

What type of muscle reflex, mediated by type Ib afferent fibers, causes muscle relaxation in response to muscle contraction?Golgi tendon reflex

What type of muscle reflex, mediated by types II, III, and IV afferent fibers, causes ipsilateral flexion and contralateral extension?Flexor withdrawal reflex

What are the components of the afferent limb of a myotatic reflex arc?Muscle spindle receptor → Ia fiber → dorsa l root gangl ion

What comprises the efferent limb of a myotatic reflex arc?Ventra l motor neuron

For each of the following muscle stretch reflexes, name the muscle group and spinal level tested:Ankle jerkGastrocnemius (S1)Knee jerkQuadriceps (L2-L4)Biceps jerkBiceps (C5-C6)Forearm jerkBrachioradia l i s (C5-C6)Triceps jerkTriceps (C7-C8)

What type of posturing is caused by a transecting lesion above the level of the medulla but below the midbrain?Decerebrate rigidi ty

What type of posturing is caused by a transecting lesion above the level of the red nucleus (midbrain)?Decorticate rigidi ty

What are the three layers of the cerebellar cortex?1. Granular layer (innermost)2. Purkinje layer (middle)3. Molecular layer (outermost)

Which is the major NT of cerebellar Purkinje cells?γ-Aminobutyric acid (GABA).Note: The output of Purkinje cel l s i s a lways inhibi tory.

Meninges

What are the three layers of the meninges?

“The meninges PAD the CNS”1. P ia2. A rachnoid3. D ura

What meningeal space, which lies between the pia and arachnoid, contains the cerebrospinal fluid (CSF)?Subarachnoid space

What structure produces CSF?The choroid plexus of the latera l , thi rd, and fourth ventricles

What structures reabsorb CSF into venous circulation?The arachnoid granulations

Trace the flow of CSF from the choroid plexus into venous circulation.Choroid plexus → latera l ventricles → interventricular foramina (of Monro) → thi rd ventricle → cerebra l aqueduct → fourth ventricle →latera l foramina (of Luschka) or median foramen (of Magendie) → subarachnoid space → arachnoid granula ons → superior sagi a ls inus

What are the three major functions of CSF?

1. To provide support and protection to the CNS2. To remove metabol ic waste products3. To transport hormones and cytokines throughout the CSF and to the systemic ci rculation

Vasculature of the Central Nervous System

Name the blood vessel that supplies each of the following structures:

Anterior two-thirds of the spinal cord, the medullary pyramids, medial lemniscus, and root fibers of cranial nerve (CN) XIIAnterior spina l arteryRetinaCentra l artery of the retina (a branch of the ophthalmic artery)Lateral geniculate body, globus pallidus, posterior limb of internal capsuleAnterior choroida l artery (an important branch of internal carotid artery)Hypothalamus and ventral thalamusPosterior communicating arteryLeg-foot area of motor and sensory corticesAnterior cerebra l artery (ACA)Anterior putamen, caudate nucleus, and anteroinferior internal capsuleMedia l s triate arteries (branches of the ACA)Broca (expressive) and Wernicke (receptive) speech areas, face and arm areas of motor cortices, frontal eye fieldMiddle cerebra l artery (MCA)Internal capsule, caudate nucleus, putamen, globus pallidusLatera l s triate arteries (branches of the MCA)Nucleus ambiguus and the inferior surface of the cerebellumPosterior inferior cerebel lar arteryCaudal lateral pontine tegmentum (including portions of the nuclei of CN V, VII) and the inferior cerebellar surfaceAnterior inferior cerebel lar arterySuperior surface of cerebellum, cerebellar nuclei, and the cochlear nucleiSuperior cerebel lar arteryMajority of midbrain, por ons of the thalamus, lateral and medial geniculate bodies, occipital lobe, inferior aspect of the temporal lobes, and thehippocampusPosterior cerebra l artery (PCA)Majority of the duraMiddle meningeal artery

Name the cerebral veins that drain directly into the superior sagittal sinus:Bridging veins

Name the cerebral vein that drains deep cerebral veins into the straight sinus:Vein of Galen

CN III, CN V1, CN V2, CN VI, postganglionic sympathetic fibers, and both internal carotid arteries all pass through which structure?Cavernous s inus

Axonal Transport

Name the cytoplasmic structure in the nerve cell body and dendrites that are involved in protein synthesis:

Niss l substanceName the type of axonal transport described below:

Transport responsible for delivery of synthesized NTs away from the cell bodyFast anterograde axonal transportTransport responsible for delivery of cytoskeletal and cytoplasmic components away from the cell bodySlow anterograde transportTransport responsible for returning material to the cell body for degradationFast retrograde transportKinesin-dependent transportFast anterograde axonal transportDynein-dependent transportFast retrograde transportTransport responsible for carrying nerve growth factor, viruses, and toxins to cell bodiesFast retrograde transport

Name the process of anterograde axonal and myelin degeneration accompanied by Schwann cell proliferation:Wal lerian degeneration

Supporting Cells of the Nervous System

Name the type of cell described below:

Primary supportive cell type of the CNSAstrocyteMyelin-producing cell type of the CNSOl igodendrocyteCNS scavenger cell typeMicrogl iaCSF-producing cell typeEpendymal cel lMyelin-producing cell type of the peripheral nervous system (PNS)Schwann cel l s

What type of intercellular connections are responsible for maintaining the integrity of the blood-brain barrier?Tight junctions

What proteins are commonly used to identify astrocytes?Gl ia l fibri l lary acidic protein (GFAP) and glutamine synthetase

Pigments and Inclusions

Name the process or disease associated with the following neuronal histopathologic findings:

Lipofuscin granulesAgingDepletion of neuromelanin in substantia nigra and Lewy bodiesParkinson diseaseNegri bodiesRabiesAmyloid plaques and neurofibrillary tanglesAlzheimer diseaseCowdry type A inclusion bodiesHerpes s implex encephal i ti s

Spinal Tracts

Name the spinal tract responsible for each of the following functions:

Voluntary control of skeletal muscleLatera l corticospina l/pyramidal tractSensation of pain and temperatureLatera l spinothalamic tractTwo-point discrimination and vibratory sensationDorsa l column-media l lemniscus tractControl of facial musclesCorticobulbar tractCoordination of muscle tone, posture, balance, and motor activityDentothalamic tract

Describe the major difference between the innervation of the lower and upper facial muscles.Corticobulbar fibers innervate the lower facia l muscles uni latera l ly, whi le upper facia l muscles are innervated bi latera l ly

Name the structure in the spinal cord composed of ascending fibers of the dorsal column-medial lemniscus pathway originating in the upper extremities:Cuneate fasciculus

Name the structure in the spinal cord composed of ascending fibers of the dorsal column-medial lemniscus pathway originating in the lower extremities:Graci le fasciculus (media l to cuneate fasciculus )

At what level of the brainstem do fibers of the dorsal column-medial lemniscus pathway cross?Caudal medul la

What type of receptors provide input to the lateral spinothalamic tract?Free nerve endings

At what level do fibers of the spinothalamic tract cross?At the same level or 1 to 2 levels above/below where they enter the spina l cord

Name the structure where fibers of the lateral spinothalamic tract cross the midline:Ventra l white commissure/anterior commisure

Where do fibers of the dorsal column-medial lemniscus pathway, the trigeminothalamic, and lateral spinothalamic tract all terminate?The sensory cortex (Brodmann areas 3, 1, and 2)

What part of the cortex gives rise to the fibers of the lateral corticospinal and corticobulbar tracts?The motor, premotor, and sensory areas of the cortex (Brodmann areas 6, 4, and 3, 1, 2)

Fibers of the lateral corticospinal tract pass through which limb of the internal capsule?Posterior l imb

Name the structure where fibers of the lateral corticospinal tract cross the midline:Medul lary pyramids

Classify each of the following clinical findings as upper motor neuron (UMN) or lower motor neuron (LMN) signs:Spastic paresisUMNFlaccid paralysisLMNBabinski sign (upgoing toes)UMNFasciculations and fibrillationsLMNAreflexiaLMNAtrophyLMNHyperreflexiaUMN

Cranial Nerves

Name the cranial foramen that each of the cranial nerves below pass through:

ICribi form plateIIOptic canalIII, IV, V1, VI

Superior orbi ta l fi s sure. Note: a l l of these nerves pass through the cavernous s inus as wel l .V2

Foramen RotundumV3

Foramen Ova le (for divis ions of the trigeminal nerve think “S tanding R oom O nly”)VII, VIIIInternal acoustic meatusIX, X, XIJugular foramenXIIHypoglossa l canal

Name the function(s) for each cranial nerve listed below:I: olfactorySmel lII: opticVis ionIII: oculomotor1. Eye movement2. “Parasympathetic” ci l iary and pupi l lary sphincter mmNote: mm is used as the abbreviation for “muscles”.IV: trochlearContraction of superior obl ique muscleV1: trigeminal—ophthalmic branchSensation from nose to foreheadV2: trigeminal—maxillary branchSensation from latera l nose, upper l ip, superior bucca l areaV3: trigeminal—mandibular branch1. Sensation from areas of the lower face not covered by V1 and V2

2 . Movement of the Muscles of Mas ca on ( Masseter, te Mpora l i s , Media l , and latera l pterygoids ), tensor vel i pa la ni , and tensortympaniVI: abducensContraction of latera l rectus muscleVII: facial1. Parasympathetic—lacrimal , submandibular, and subl ingual glands2. Mm of facia l express ion, s tapedius , s tylohyoid, and the posterior bel ly of the digastric muscle3. Taste—anterior two-thi rds of tongue4. Sensation—skin of external earVIII: vestibulocochlearHearing and sense of ba lanceIX: glossopharyngeal1. Parasympathetic—parotid gland2. Motor—stylopharyngeus mm3. Taste—posterior one-thi rd tongue4. Sensation—parotid gland, carotid body and s inus , pharynx, and middle ear5. Cutaneous sensation—external ear canalX: vagus1. Parasympathetic—trachea, bronchi , heart, GI tract2. Contraction of laryngeal , pharyngeal , and esophageal s triated mm3. Taste—epiglotti s and pa late4. Sensation—trachea, GI tract5. Cutaneous sensation—external earXI: accessoryMovement of s ternocleidomastoid and trapezius musclesXII: hypoglossalContraction of muscles of tongue

Which three cranial nerves are purely sensory nerves?1. CN I2. CN II3. CN VIII

Which five cranial nerves are purely motor nerves?1. CNIII2. CNIV3. CNVI4. CNXI5. CNXII

Which four cranial nerves have both motor and sensory components?1. CNV2. CNVII3. CNIX4. CNX

Which two cranial nerves are rostral to the midbrain?1. CNI2. CNII

Which two cranial nerve nuclei are located in the midbrain?1. CNIII2. CNIV

Which four cranial nerves have at least a portion of their nuclei in the pons?1. CNV

2. CNVI3. CNVII4. CNVIII

Which seven cranial nerves have at least a portion of their nuclei in the medulla?1. CN V2. CN VI3. CN VII4. CNVIII5. CN IX6. CNX7. CNXII

Name the only cranial nerve that crosses the midline and exits the brainstem posterior to the ventricular system:CN IV—exits the bra instem posteriorly and crosses the midl ine after exi ting the caudal midbra in

What nucleus serves as the origin of preganglionic parasympathetic fibers projecting to the ciliary ganglion?Edinger-Westphal nucleus of CN II I

What visceral sensory nucleus, located in the medulla, is a relay center for taste, sensory input from the carotid sinus, carotid body, and the vagus nerve?Nucleus Sol i tarius

What visceral motor nucleus, located in the medulla, is involved in coordinating swallowing and speech?Nucleus aMbiguus

Which are the afferent and efferent limbs of the corneal reflex?CN V1 and CN VII

Which are the afferent and efferent limbs of the pupillary light reflex?CN II and CN II I

Which are the afferent and efferent limbs of the gag reflex?CN IX and CN X

Name the site of a lesion, within the visual tract, capable of causing each of the following deficits:Ipsilateral blindnessTransection of the optic nerveBinasal hemianopiaBi latera l latera l compress ion of optic chiasmBitemporal hemianopiaMidsagi tta l transection or midl ine pressure on the optic chiasm (often caused by a pi tui tary tumor)Right hemianopia without macular sparingTransection of the left optic radiationRight upper quadrantanopiaTransection of the lower divis ion of the left optic radiationRight lower quadrantanopiaTransection of the upper divis ion of the left optic radiationRight hemianopia with macular sparingDestruction of the left vi sua l cortex

What are five key structures of the pupillary light reflex pathway?1. Gangl ion cel l s of the retina2. Pretecta l nucleus of the midbra in3. Edinger-Westphal nucleus4. Ci l iary gangl ion5. Postgangl ionic parasympathetic fibers of CN II I

What are four key structures of the pupillary dilation pathway?1. Paraventricular nucleus of the hypothalamus2. Ci l iospina l center of Budge at the level of T1 to T23. Superior cervica l gangl ion4. Postgangl ionic sympathetic fibers travel ing a long the internal carotid artery and i ts branches to the eye

What part of the cortex is responsible for voluntary eye movements?Fronta l eye field (Brodmann area 8)

What side will a patient’s eyes deviate toward if there is a lesion of the right frontal eye field?Right s ide (“Look toward the les ion of fronta l eye fields”)

What structure connects the nucleus of CN VI and the nucleus of CN III?Media l longi tudina l fasciculus (MLF)

What type of lesion will result in medial rectus palsy (inability to adduct the eye) on attempted lateral gaze but normal adduction on accommodation?Intranuclear ophthalmoplegia (a les ion of the MLF)

What classic idiopathic lesion is characterized by ptosis, miosis, and anhydrosis?Horner syndrome

Name the condition characterized by a pupil that will accommodate but cannot react to light:Argyl l -Robertson pupi l (associated with tertiary syphi l i s , lupus , and diabetes mel l i tus )

Name the condition caused by a lesion in the afferent fibers of the light reflex pathway:Marcus Gunn pupi l

What are the primary sensory receptors of the auditory pathway?Inner ha i r cel l s of the organ of Corti

Where does the auditory pathway terminate?Bi latera l input from both auditory tracts terminates in primary auditory cortex in superior tempora l gyrus (Brodmann areas 41 and 42)

What type of cells are responsible for relaying auditory stimuli from the organ of Corti to the cochlear nuclei?Bipolar cel l s of the spi ra l or cochleargangl ion

What thalamic nucleus plays a key role in relay of impulses from the cochlear nuclei to higher cortical centers?Media l geniculate body of the tha lamus

What pontine nucleus plays a key role in sound localization?Superior ol ivary nucleus

What are key structures of the hearing pathway?Cochlear → cochlear nucleus → decussating fibers in Trapezoid body → superior ol ivary nucleus → latera l lemniscus → inferior col l i cul i →media l geniculate nucleus → primary auditory cortex

Conduction deafness is caused by a lesion of which components of the auditory system?

External audi tory canal , tympanic membrane, or the middle earSensorineural deafness is caused by a lesion of which components of the auditory system?

Cochlea, cochlear nerve, or the cochlear nucleiPatients with presbycusis have trouble hearing what types of sounds?

High-frequency soundsWhich cells of the vestibular system respond to angular acceleration and deceleration?

The ha i r cel l s of the three semici rcular canalsWhat structures of the vestibular system respond to linear acceleration and deceleration?

The ha i r cel l s of the utricleWhat type of cells are responsible for relaying vestibular stimuli from the hair cells to the vestibular nuclei?

Bipolar cel l s of the vestibular gangl ionWhat structures provide input to the vestibular nuclei?

Hair cel l s of the semici rcular canal , ha i r cel l s of the utricle, and the flocculonodular lobe of the cerebel lumWhat structures receive signals from the vestibular nuclei?

The tha lamus, spina l cord, cerebel lum, and CNs II I , IV, and VI

Cerebellum, Thalamus, Hypothalamus

What are the three primary functions of the cerebellum?

1. Maintenance of posture and equi l ibrium2. Control of muscle tone3. Coordination of voluntary muscle activi ty

What type of tremor may result from a cerebellar lesion?Intention tremor

A positive Romberg sign (loss of balance when the eyes are closed) suggests a lesion to which tract of the CNS?Dentothalamic tract (the main cerebel lar pathway) or dorsa l column (tabes dorsa l i s in neurosyphi l i s )

Name the thalamic nucleus/nuclei responsible for the relay of impulses for each modality listed below:VisionLatera l geniculate nucleus (“Latera l to Look”)HearingMedia l geniculate nucleus (“Media l for Mus ic”)Proprioception, pain, pressure, touch, vibrationLatera l portion of ventra l posteriornucleus (VPL, “Posterior for Proprioception, Pain”)Facial sensationMedia l portion of ventra l posterior nucleus (VML)MotorVentra l anterior/latera l nucleiLimbic functionDorsomedia l , anterior nuclei

Name the largest thalamic nucleus:Pulvinar

What is the function of the pulvinar?Integration of vi sua l , audi tory, and somesthetic input

Which portion of the internal capsule contains fibers of the corticobulbar tract?The genu

Which portion of the internal capsule contains fibers of the corticospinal, spinothalamic, visual, and auditory tracts?The posterior l imb

Which arteries supply the posterior limb of the internal capsule?Perforating branches of the anteriorchoroida l artery and lenticulostriate arteries

Name the major hypothalamic nucleus (or nuclei) responsible for each function listed below:Regulation of the release of gonadotropic hormonesMedia l preoptic nucleus (which conta ins the sexual ly dimorphic nucleus)Regulation of circadian rhythmsSuprachiasmatic nucleusRegulation of body temperatureAnterior nucleus (les ion resul ts in hyperthermia) and posterior nucleus (les ion resul ts in poiki lothermia)Regulation of water balance, synthesis of antidiuretic hormone, oxytocin, and corticotropin-releasing factorParaventricular and supraoptic nucleiRegulation of appetiteVentroMedia l nucleus (les ion resul ng from ea ng Ve ry Much [hyperphagia , obes i ty]) and latera l hypothalamic nucleus (les ions causeanorexia and s tarvation)Regulation of hypothalamusArcuate or infundibular nucleusRegulation of emotional expressionMammil lary nucleus (a component of the l imbic system)

What are the major structures of the Papez circuit?Septa l area, mammil lary body, anteriornucleus of tha lamus, cingulate gyrus , entorhina l cortex, and hippocampal formation

What is the most epileptogenic part of the cerebrum?The hippocampus

What system within the CNS plays a central role in the initiation and coordination of somatic motor activity?The s triata l or extrapyramidal motor system

What are the major components of the striatal motor system?Neocortex, basa l gangl ia (s triatum [caudate + putamen], globus pa l l idus , subthalamic nucleus , substantia nigra), and tha lamus

Neurotransmitters

Name the NT described below:

Major NT of the PNSAChNT which is increased in the CNS of patients with schizophreniaDopamineMajor NT of the parasympathetic nervous systemAChNT believed to cause panic attacks when released suddenly by the locus coeruleusNEMajor NT of the preganglionic sympathetic nervous systemAChNT highly concentrated in the substantia nigra that plays a key role in pain transmissionSubstance PMajor NT of the postganglionic sympathetic neurons supplying sweat glands and certain blood vesselsAChNT which is depleted from the basal nucleus of Meynert in Alzheimer diseaseAChNT which is depleted from the substantia nigra in patients with Parkinson diseaseDopamineNT that causes renal vasodilationDopamineTwo NTs believed to be depleted in depression1. NE2. SerotoninPowerful analgesic NT found exclusively in the hypothalamusβ-EndorphinOpiate peptides which play a role in pain suppressionEnkephal insNT that regulates release of GH and TSH; markedly ↓ Alzheimer diseaseSomatostatinMajor inhibitory NT of the cortexGABAMajor inhibitory NT of the spinal cordGlycineMajor excitatory NT of the brainGlutamateGaseous, vasoactive NT involved in memoryNitrous oxideNT important in the initiation of sleepMelatoninNT which inhibits the reticular activating center, thereby increasing total sleep timeACh

Which two amino acids can serve as a precursor for the synthesis of catecholamines?1. Phenyla lanine2. Tyros ine

Cerebral Cortex

What are the s ix layers of neocortex?

1. Layer I: molecular layer2. Layer I I : external granular layer3. Layer I I I : external pyramidal layer4. Layer IV: internal granular layer5. Layer V: internal pyramidal layer6. Layer VI: multi form layer

Name the site of a lesion, within the cortex, capable of causing each of the following deficits:Right-sided flaccid hemiparalysisLeft primary motor area (Brodmann area 4)Left-sided pronator driftRight primary motor area (Brodmann area 4)Loss of abstract thought and self-restraintBi latera l loss of fronta l lobes anterior to the fronta l eye fieldsSlowed speech without any impairment of language comprehensionBroca speech area (Brodmann areas 44, 45; a lways in the dominant hemisphere, usual ly left)Loss of right-sided tactile sensation and proprioceptionLeft somesthetic area (Brodmann areas 3, 1, 2)Cortical deafnessBi latera l destruc on of the auditory areas (Brodmann areas 41, 42); uni latera l destruc on of the auditory area causes a s l ight ↓ inhearing.Inability to understand spoken language and verbalize coherent thoughtsWernicke speech area (Brodmann area 22) in the dominant hemisphere, usual ly leftIpsilateral anosmia (inability to smell)Primary ol factory area (Brodmann area 34)Alexia and agraphia (inability to read and write)Angular gyrus (Brodmann area 39)Loss of ability to transfer information from short-term to long-term memoryBi latera l destruction of the hippocampal cortexPsychic blindness, hyperphagia, docility, and hypersexuality (Klüver-Bucy syndrome)Bi latera l destruction of the anterior tempora l lobes (amygdala)

Loss of ability to recognize facesInferomedia l right occipi totempora l areaLoss of vision in the right visual field with macular sparingDestruction of the left primary visua l area (Brodmann area 17)

Name the term used to describe a deficit in the ability to draw a geometric figure:Construction apraxia

Name the term used to describe a “magnetic gait,” commonly seen in normal-pressure hydrocephalus:Gait apraxia

What part of the nervous system is involved in maintaining wakefulness?Reticular activating system and bi latera l cortex

PATHOLOGY OF THE NERVOUS SYSTEM

Congenital Disorders

Name the type of neural tube defect with the following features:

Failure of posterior vertebral arch closure (not evident on clinical examination)Spina bi fida occul taFailure of posterior vertebral arch closure accompanied by herniation of the meningesSpina bi fida cysticaHerniation of the meninges outside of the spinal canalMeningoceleHerniation of nervous tissue and meninges outside of the spinal canalMyelomeningoceleComplete cerebral agenesis due to lack of closure of the anterior neuroporeAnencephalyDiverticulum of malformed CNS tissueEncephalocele

What factor is used to screen pregnant mothers for neural tube defects?α-Fetoprotein

What is the most common cause of mental retardation?Feta l a lcohol syndrome, often associated with cardiac and facia l anomal ies

What are two common chromosomal genetic causes of mental retardation?1. Tri somy 212. Fragi le X

Name the condition characterized by an excess of CSF in the crania l cavi ty:Hydrocephalus

What type of hydrocephalus is characterized by obstruction in the flow of CSF through the ventricular system and subarachnoid space?Noncommunicating hydrocephalus

What type of hydrocephalus i s characterized by free flow of CSF but abnormal CSF absorption?Communicating hydrocephalus

What congenita l mal forma on of the CNS i s characterized by hernia on of the cerebel lar tons i l s and medul la through the foramen magnum(which may resul t in obstruction of CSF ci rculation)?

Arnold-Chiari mal formation (type 1)What congenital malformation of the CNS is associated with syringomyelia (central cavitation of the spinal cord)?

Arnold-Chiari mal formation (type 2)What congenital malformation of the CNS is characterized by cystic dilation of fourth ventricle, agenesis of vermis, and associated with hydrocephalus?

Dandy-Walker mal formationWhat complication of premature babies usually results in hypoxic/ischemic injuries of brain?

Subependymal germinal matrix bleed

Stroke

Describe the artery that has been occluded in each of the following stroke syndromes:

Paresis and sensory loss of contralateral lower extremityACAHemiparesis, contralateral hemisensory loss, homonymous hemianopsia, aphasiaMCA supplying the dominant hemisphere, usual ly left hemisphereLoss of consciousness, hemisensory loss, homonymous hemianopsia with macular sparingPCAAmaurosis fugaxOphthalmic arteryVertigo, cranial nerve palsies, impaired level of consciousness, dysarthriaVertebrobas i lar arterySensory neglect and apraxiaMCA supplying the nondominant hemisphere, usual ly right hemisphereUrinary incontinence, suck and grasp reflexesMiddle or ACA supplying the fronta l lobeIpsilateral loss of pain and temperature for face, contralateral pain and temperature for bodyPICA—Wal lenberg syndrome (latera l medul la )

What are the most frequent sites of thrombotic occlusion in the cerebral vasculature?Carotid bi furcation, MCA, and bas i lar artery

What is the most frequent site of embolic occlusion in the cerebral vasculature?MCA

Which cardiac arrhythmia is associated with embolic stroke?

Atria l fibri l lationWhat type of stroke, associated with HTN, causes the formation of small, cystic, moon-shaped pits?

Lacunar infarctionsWhere do lacunar strokes usually occur?

Basa l gangl ia , tha lamus, internal capsule, white matter, pons , cerebel lumName the term used to describe small aneurysms of the cerebral vasculature, caused by long-standing HTN, that may result in intracerebral hemorrhage:

Charcot-Bouchard aneurysmsWhat are the most common locations for Charcot-Bouchard aneurysms?

Thalamus and basa l gangl iaWithin the cerebral vasculature, what are the most common sites of berry aneurysm formation?

At the bi furcations of the ci rcle of Wi l l i sWhat is the most common complication of berry aneurysms?

Rupture caus ing subarachnoid hemorrhageWhat are three disorders that predispose to the formation of berry aneurysms?

1. Polycystic kidney disease2. Ehlers -Danlos syndrome3. Marfan syndrome

Which cranial nerve palsy is associated with internal carotid or posterior communicating artery aneurysms?CN III pa lsy caus ing papi l lary di lation

Name the term used to describe paroxysmal, self-limiting episodes of neurologic deficit, commonly including transient aphasia.Trans ient i schemic attack

Which syndrome is characterized by loss of all motor function except that of CN III and IV?Locked-in syndrome (usual ly a resul t of infarction or tumor at the base of the pons)

Seizures

Name the type of seizure associated with the following clinical findings:

Loss of consciousness followed by loss of postural control, a tonic phase of muscle contraction and clonic limb jerkingTonic-clonic seizureA child who appears to be daydreaming in class and is found to have a 3-second spike-and-wave pattern on EEGAbsence seizureSudden, brief muscle contractionsMyoclonic epi lepsyMotor, sensory, visual, psychic, or autonomic phenomena with preserved level of consciousnessSimple partia l seizureSeizure begins with behavioral arrest, which is followed by auditory or visual hallucination, automatisms, and finally by postictal confusionComplex partia l seizureWhat disorder is characterized by paroxysmal episodes of sharp, shooting facial pain in the distribution of one or more branches of CN V?Trigeminal neura lgia

What is the drug of choice for trigeminal neuralgia?Carbamazepine

What is the triad of cerebellar dysfunction?1. Loss of ba lance (disequi l ibrium)2. Hypotonia3. Loss of coordinated muscle activi ty (dyssynergia)

Name the terms used to describe traumatic injury to the cortex at the site of impact and opposite the side of impact:Coup injury (at the s i te of impact); contrecoup injury (oppos i te the s i te of impact)

Intracranial Hemorrhage

Name the type of intracranial hemorrhage associated with the following features:

Bloody or xanthrochromic CSF on lumbar punctureSubarachnoid hemorrhageHematoma following the contour of a cerebral hemisphere on computed tomography (CT) scanSubdura l hematomaLaceration of bridging cerebral veinsSubdura l hematomaLaceration of middle meningeal artery due to fracture of the temporal boneEpidura l hematomaLucid interval followed by rapid decline in mental statusEpidura l hematomaMost common type of intracranial hemorrhage resulting from traumaSubdura l hematomaRuptured berry aneurysm or arteriovenous malformationSubarachnoid hemorrhageSeen in patients with long-standing, poorly controlled HTNIntraparenchymal hemorrhageLens-shaped hematoma on CT scanEpidura l hematomaCrescentic hematoma on CT scanSubdura l hematomaSeen more commonly in alcoholics and the elderlySubdura l hematoma

What is the most common cause of subarachnoid hemorrhage?Trauma

Meningitis/Encephalitis

Name the type of meningitis associated with the following CSF findings:

Greater than 1000 polymorphonuclear mononuclear leukocytes, ↓ glucose, increased proteinBacteria l meningi ti sIncreased lymphocytes, minor elevation in protein, normal CSF pressureVira l meningi ti sIncreased lymphocytes, minor elevation in protein, elevated CSF pressureFungal meningi ti s

Perivascular cuffing, inclusion bodies, and glial nodules may be seen in what cerebral infection?Vira l meningoencephal i ti s

Name the three most common bacteria1. causing neonatal meningitis:1. Group B Streptococcus2. Escherichia coli3. Li s teria

Name the parasite spread from cats to humans that causes periventricular calcifications and congenital disorders in offspring of infected mothers:Toxoplasma gondii

Name the fungal meningitis most commonly associated with diabetics in DKA:Mucormycos is

Bullet-shaped intracytoplasmic inclusions, Negri bodies are characteristic of which CNS viral infection?Rabies

Hemorrhagic necrosis of temporal lobes is most commonly associated with which viral encephalitis?HSV-1

Which infec ous disease is characterized by neuronal vacuoliza on leading to small cysts in the gray ma er of the brain without an associated inflammatoryresponse?

Spongi form encephalopathyWhich disease is characterized by progressive ataxia, dementia, and spongiform gray matter changes?

Creutzfeldt-Jacob disease

Demyelinating Disorders

Name the demyelinating disorder associated with the following clinical and pathologic features:

Most common demyelinating disorderMultiple scleros isAssociated with JC virus infection in AIDS patientsProgress ive multi foca l leukoencephalopathyPeriventricular calcification; spinal Multiple sclerosis lesions typically in the white matter of the cervical cordMultiple scleros isPostviral autoimmune syndrome causing demyelination of peripheral nerves, especially motor fibersGui l la in-Barré syndromeTriad of intention tremor, scanning speech, and nystagmusMultiple scleros isMay present with intranuclear ophthalmoplegia (MLF syndrome) or sudden visual loss due to optic neuritisMultiple scleros isAscending paralysis, facial diplegia, and autonomic dysfunctionGui l la in-Barré syndromeOligoclonal bands in the CSFMultiple scleros isAlbuminocytologic dissociation (↑ CSF protein with normal cell count)Gui l la in-Barré syndrome↑ CSF protein, normal glucose, ↑ lymphocytesMultiple scleros is

Leukodystrophies and Neurocutaneous Syndromes

Name the type of leukodystrophy associated with the following features:

Globoid bodies in white matterKrabbe diseaseDeficiency of β-galactocerebrosidaseKrabbe diseaseDeficiency of arylsulfatase AMetachromatic leukodystrophyNervous tissue demonstrates loss of myelin and appears yellowish brown, build up of cerebroside in myelin sheathMetachromatic leukodystrophyPeroxisomal deficiency, demyelination starts in occipital lobe and moves anteriorlyAdrenoleukodystrophy (X-l inked)

Name the type of neurocutaneous syndromes associated with the following features:Cutaneous and plexiform neurofibromas, Lisch nodules on iris, cafe au lait spots, axillary frecklingNeurofibromatos is (NF1)Bilateral schwanomas, meningioma, ependyomaNF2Capillary hemagiomas, hemagioblastomas in cerebellum and retina, increased incidence of renal cell carcinomavon Hippel -LindauNasolabial subcutaneous angiofibroma, epilepsy, subependymal nodules, ungual fibroma, shagreen patch, ash leaf spotsTuberous scleros is

What is the inheritance pattern of neurocutaneous syndromes?Autosomal dominant (AD)

What is the inheritance pattern of leukodystrophies?

Autosomal recess ive (AR)

Neurodegenerative Disorders

What are the two most common causes of dementia in the elderly?

Alzheimer dementia and multi -infarct dementiaName the neurodegenerative disorder associated with the following clinical and pathologic features:

Hirano bodies, neurofibrillary tangles, senile plaques (accumulations of β-amyloid protein)Alzheimer dementiaStepwise dementia in a patient with focal neurologic deficitsMulti -infarct dementiaThiamine deficiency from alcohol abuse causing ophthalmoplegia, ataxia, nystagmusWernicke encephalopathyLong-term alcohol abuse causing retrograde and anterograde amnesia, confabulation, and shrunken, petechial hemorrhage in mammillary bodiesKorsakoff psychos isProgressive dementia with predominantly frontal and temporal gliosis and neuronal lossPick diseaseDegeneration of the caudate nucleusHuntington diseaseLewy bodies and depigmentation of the substantia nigraParkinson diseaseParkinsonian symptoms with autonomic dysfunction, including orthostatic hypotensionShy-Drager syndromeResting tremor, cogwheel rigidity, akinesia, postural instabilityParkinson diseaseCan be caused by MPTP useParkinson diseaseSlowly progressive ataxia, dysarthria associated with kyphoscoliosis, diabetes, arrythmias, and myocarditis due to triplet repeat GAA on chromosome 9Friedreich ataxiaAutosomal dominant (AD) inheritance and an cipa on (worsening of disease in future genera ons) due to increasing number of CAG repeats onchromosome 4Huntington diseaseUMN and LMN signs due to loss of myelinated fibers of the corticospinal tractAmyotrophic latera l scleros is (ALS)Viral infection → inflammatory response in anterior horn of the spinal cord → LMN lossPol iomyel i ti sChildhood ataxia associated with telangiectasias of the skin and conjunctiva associated with ATM gene mutationAtaxia-telangiectas iaFloppy baby (hypotonia) due to LMN degeneration, tongue fasciculationWerdnig-Hoffman disease

Which protein gives rise to the amyloid fibrils of Alzheimer disease?A-β

Which is the conformation of A-β protein in neuritic plaques?β-Pleated sheet

The A-β protein is derived from processing of which larger molecule?Amyloid precursor protein (APP)

Brain Tumors

Where are the majority of adults versus children CNS tumors found?

Supratentoria l for adul ts , infratentoria l for chi ldrenName the brain tumor associated with each of the following clinical or pathologic findings:

Most common pediatric intracranial tumorJuveni le pi locytic as trocytomaMost common pituitary tumorPitui tary adenomaMost common pituitary adenomaProlactinomaMost common pediatric supratentorial tumorCraniopharyngiomaMost common primary brain tumorGl ioblastoma multi formeMost common intracranial tumorMetastasesMalignant pediatric tumor which metastases through CSF pathwaysMedul loblastomaMalignant pediatric tumor found exclusively in the posterior fossaMedul loblastomaVascular tumor of cerebellum and retina in patients with von Hippel-Lindau syndromeHemangioblastomaAbundant capillaries and vacuolated foam cellsHemangioblastomaType of tumor that is found bilaterally in patients with neurofibromatosis IIVestibular schwannomaTumor of the dorsal root that may grow in a dumbbell configuration through a vertebral foramenSchwannomaTumor which originates from the vestibular division of CN VIII

SchwannomaTumor which grows in a mixture of Antoni A or Antoni B patternsSchwannomaSmall round blue cell tumorMedul loblastomaBipolar cells, Rosenthal fibers, and microcystsJuveni le pi locytic as trocytomaVerocay bodiesSchwannomaTumor derived from Rathke pouchCraniopharyngiomaTwo tumors often presenting with bitemporal hemianopiaPitui tary adenoma and craniopharyngiomaTumor characterized by concentric whorls and calcified psammoma bodiesMeningiomaTumor arising from ependymal lining of ventricular systemEpendymomaTumor commonly arising in the pineal region causing obstructive hydrocephalus by compromising the aqueduct of SylviusGerminomaEBV positive B-cell tumor of the CNS in AIDS patientsCNS lymphomaTumor of the foramen of Monro causing obstructive hydrocephalusCol loid cyst of the thi rd ventricleBenign tumor characterized by calcifications and cells with fried-egg appearance or perinuclear halosOl igodendromaTumor characterized by highly malignant cells bordering necrotic areasGl ioblastoma multi formeBenign tumor derived from arachnoid cap cells with well-delineated marginsMeningioma

Disorders of the Spinal Cord

Name a disease of the spinal cord associated with each of the following neurologic findings:

Loss of all spinal modalities except tactile discrimination, vibratory sensation, and proprioceptionVentra l spina l artery occlus ionImpaired tactile discrimination, vibratory sensation, and proprioceptionTabes dorsa l i sLoss of pain and temperature sensation in a cape-like distribution and flaccid paralysis of the intrinsic muscles of the handSyringomyel iaImpaired tactile discrimination, vibratory sensation and proprioception, UMN signs, and ataxiaVitamin B12 deficiency

Miscellaneous

What complication affecting the brainstem can be caused by rapid correction of hyponatremia?

Centra l pontine myel inolys isDescribe how transtentorial herniation causes contralateral hemiparesis.

Compresses the right crus cerebri → corticospina l and corticobulbar fibers are compromised (Kernohan notch)Describe how transtentorial herniation causes pupillary dilation.

Tens ion on CN II I causes pupi l lary di lation.Name a life-threatening complication of transforaminal or tonsillar herniation:

Compress ion of the medul lary respiratory center → respiratory insufficiencyWhat is the term used to describe brainstem hemorrhages caused by transtentorial and transforaminal herniation?

Duret hemorrhagesWhat is the systemic response to increased intracranial pressure (Cushing triad)?

1. HTN2. Bradycardia3. Irregular respi rations

What is the most common reversible cause of dementia in the elderly?Normal pressure hydrocephalus

Normal pressure hydrocephalus is a common complication of what type of intracranial pathology?Subarachnoid hemorrhage

What is the triad of normal-pressure hydrocephalus?1. Ataxic, magnetic ga i t (Wobbly)2. Dementia and/or short-term memory loss (Weird)3. Urinary incontinence (Wet)

What diuretic is commonly used to manage increased intracranial pressure?Mannitol (provides osmotic diures is )

What is the protein change associated with prion diseases?Conformational change in PrPc (α-hel ix i soform) to PrPsc (β-pleated sheet i soform)

PHARMACOLOGY OF THE NERVOUS SYSTEM

Antiepileptics

For each of the following drugs, name:1. The mechanism of action (MOA)2. Indication(s) (IND)3. Significant side effects or important toxicity (TOX) (if any)

CarbamazepineMOA: Na + channel blockerIND: Tonic-clonic, partia l , and Jacksonian seizuresTOX: ↑ LFT, agranulocytos is , a plastic anemiaEthosuximideMOA: May block T-type Ca 2+ channels in tha lamusIND: Absence seizuresTOX: GI upset, Stevens-Johnson syndromeDiazepamMOA: Faci l i tates GABA action by ↑ frequency of Cl channel openingIND: Status epi lepticusTOX: SedationLamotrigineMOA: Blocks Na + channelsIND: Adjuvant antiepi leptic agentTOX: Li fe-threatening rash and Stevens-Johnson syndromePhenytoinMOA: Na + channel blockerIND: Tonic-clonic, partia l , and s tatusTOX: Nystagmus, ataxia , gingiva lhyperplas ia , hi rsutism, megalobasticanemia, teratogenicPhenobarbitalMOA: Faci l i tates GABA action by ↑ duration of Cl channel openingIND: Tonic-clonic seizuresTOX: Induces P-450, drows inessValproic acidMOA: Unknown—may faci l i tate GABA actionIND: Myoclonic seizuresTOX: Hepatotoxici ty, GI toxici ty, inhibi ts P-450, thrombocytopenia

Name the drug(s) of choice for each of the following types of epilepsy:Simple and complex partialPhenytoin, carbamazepineAbsenceEthosuximideFebrilePhenobarbi ta lMyoclonicValproic acid, clonazepamStatus epilepticusPhenytoin, diazepamTonic-clonicPhenytoin, carbamazepine

Antiparkinsonian Agents

For each of the following drugs, name:1. The mechanism of action (MOA)2. Indication(s) (IND)3. Significant side effects or important toxicity (TOX) (if any)

AmantidineMOA: May enhance dopamine releaseIND: Helpful for rigidi ty and bradykines iaTOX: Acute psychos is (rare)BenztropineMOA: AntimuscarinicIND: Adjuvant therapyTOX: Simi lar to atropineBromocriptineMOA: Dopamine receptor agonis tIND: Used with levodopaTOX: Hypotens ion, confus ion, ha l lucinations , nauseaLevodopaMOA: Dopamine precursor converted to dopamine in CNSIND: Combined with carbidopa, levodopa i s the most efficacious regimen for Parkinson diseaseTOX: Nausea, tachycardia , hypotens ion, ha l lucinations , dyskines iasCarbidopaMOA: Inhibi tion of dopamine decarboxylase →↑ levodopa ava i labi l i ty in CNSIND: Used with levodopaTOX: spasms of the eyel id, i rregular heartbeat, confus ion, agi tation, ha l lucinationsSelegilineMOA: Inhibi tion of MAOb →↑ dopamine levels in CNSIND: Used as adjuvant to levodopaTOX: HTN

Anesthetics

For each of the following drugs, provide:1. The mechanism of action (MOA)2. Indication(s) (IND)3. Significant side effects or important toxicity (TOX) (if any)

HalothaneMOA: CNS depress ionIND: Prototype genera l anesthetic; potent anesthetic but weak analges icTOX: Arrhythmias , ↓ cardiac output, hypotens ion, hepatotoxici tyNitrous oxideMOA: CNS depress ionIND: Weak genera l anesthetic, s trong analges icTOX: Anoxia , vi tamin B12 deficiency (with chronic use)ThiopentalMOA: Prolongs inhibi tory postsynaptic potentia ls by ↑ GABA levels (s imi lar to phenobarbi ta l )IND: Surgica l anesthes iaTOX: LaryngospasmBenzodiazepines (diazepam, midazolam)MOA: Faci l i tates GABA action by ↑ frequency of Cl - channel openingIND: Sedative, hypnotic, anxiolyticTOX: SedationKetamineMOA: PCP analogIND: Genera l anestheticTOX: Postoperative ha l lucinations , amnes ia , respi ratory depress ionPropofolMOA: CNS depress ionIND: Genera l anesthetic—rapid onset and clearanceTOX: Cannot be given to patients with egg or soybean a l lergiesLocal anesthetics (procaine, cocaine, tetracaine, lidocaine, bupivacaine)MOA: Block Na + channelsIND: Anesthetic for minor procedures , spina l blocksTOX: Arrhythmias , HTN; cardiotoxici ty (bupivaca ine), seizuresSuccinylcholineMOA: Depolarizing neuromuscular blockerIND: Rapid sequence inductionTOX: Mal ignant hyperthermia when given with ha logenated inhaled anesthe c; contra indicated in pa ents with glaucoma because of ↑intraocular pressure (IOP)TubocurarineMOA: Nondepolarizing neuromuscular blockerIND: Adjuvant to genera l anesthes iaTOX: Hypotens ion

Why is epinephrine commonly combined with local anesthetics?To prolong the duration of the anesthetic effect by caus ing loca l vasoconstriction

What types of fibers are affected most by local anesthetic agents?Pain > temperature > touch > pressure; smal l unmyel inated fibers most affected; large, myel inated fibers least a ffected

Which drug is used to reverse the effects of the nondepolarizing muscle blockers?Neostigmine

Which drug is used to treat malignant hyperthermia?Dantrolene

Analgesics

For each of the following drugs, provide:1. The mechanism of action (MOA)2. Indication(s) (IND)3. Significant side effects and unique toxicity (TOX) (if any)

AcetaminophenMOA: COX inhibi torIND: Pa in, fever (but not used as anti -inflammatory)TOX: Overdose causes hepatic necros isAspirinMOA: Irrevers ible inhibi tion of COX-1 and COX-2IND: Analges ic, antipyretic, anti -inflammatory, antiplatelet drugTOX: GI ulcers , platelet dys function, hypersens i tivi ty reactions , bronchoconstriction, tinni tus , Reye syndrome in chi ldrenCelecoxibMOA: COX-2 inhibi torIND: Osteoarthri ti s , rheumatoid arthri ti sTOX: Simi lar to aspi rin but less GI toxici tyGabapentinMOA: Structura l ana log of GABAIND: Neuropathic pa inTOX: Sedation, movement disordersIndomethacinMOA: Revers ible inhibi tion of COX-1 and COX-2

IND: Acute gout, neonata l patent ductus arteriosusTOX: GI upset, headacheMeperidineMOA: μ opioid receptor agonis tIND: Analges icTOX: Seizures ; s ide effects s imi lar tomorphineMorphineMOA: μ opioid receptor agonis t; opioid receptor binding activates G proteins and adenylyl cyclase.IND: Analges ic, cough suppressant.TOX: Constipation, emes is , sedation, respi ratory depress ion, mios is , urinary retention.Note: these symptoms are typica l for heroin overdose.NalbuphineMOA: Opioid mixed agonis t-antagonis t analges ic that activates K and weakly blocks μ receptorsIND: Analges ic with less abuse potentia lNaloxoneMOA: μ opioid receptor antagonis tIND: Used to reverse the effects of opioid agonis tsTOX: CNS depress ion

What two types of opioid receptors mediate analgesia, respiratory depression, and physical dependence?1. μ2. δ

What type of opioid receptors mediate spinal analgesia and the sedative effects of opioids?K

CHAPTER 5Cardiovascular

EMBRYOLOGY

What are the five dilatations of the primitive heart tube?

1. Truncus arteriosus2. Bulbus cordis3. Primitive ventricle4. Primitive atrium5. Sinus venosus

Name the structures in the mature heart that are derived from the following embryonic structures:Truncus arteriosusAscending aorta and pulmonary trunkBulbus cordisSmooth parts of left (aortic vestibule) and right (conus arteriosus) ventriclePrimitive ventricleTrabeculated parts of left and right ventriclePrimitive atriaTrabeculated parts of left and right atriaLeft horn of sinus venosusCoronary s inusRight horn of sinus venosusSmooth part of right atrium (s inus venarum)Transient common pulmonary veinSmooth part of left atriumRight common cardinal vein and right anterior cardinal veinSuperior vena cava

Which embryonic layer gives rise to most of the cardiovascular system?Mesoderm

What structure divides the truncus arteriosus and bulbus cordis?Aorticopulmonary septum

Name the structure between the atria that develops from the walls of the septum primum and septum secundum:Foramen ova le

Name the three physiologic shunts in the fetal circulation and the structures they shunt between:1. Foramen ova le (right to left atrium)2. Ductus arteriosus (pulmonary artery to aortic arch)3. Ductus venosus (umbi l i ca l vein to IVC)

ANATOMY

What are the two anatomic divisions of the pericardium?1. Serous pericardium (made of vi scera l epicardia l layer and parieta l layer)2. Fibrous pericardium

Which nerve lies between the fibrous paricardium and mediastinal pleura?Phrenic nerve (runs with pericardiophrenic vessels )

Name the chamber associated with each heart surface:Sternocostal surfaceRight ventriclePosterior surface (base)Left atriumDiaphragmatic surfaceRight and left ventriclesPulmonary surface and apexLeft ventricle

Name the structures that compose each heart border:Right borderRight atriumLeft borderLeft ventricle, left auricleInferior borderRight ventricleSuperior borderRight and left auricles , great vessels

Name the major artery that commonly supplies each of the following structures:Right atrium and right ventricleRight coronary artery (RCA)Sinoatrial (SA) and atrioventricular (AV) nodesRCALeft atrium and left ventricleLeft coronary artery (LCA)Interventricular septum1. Anterior 2/3

2. Posterior 1/31. Left anterior descending (LAD)2. Posterior interventricular artery

Which artery determines dominance of cardiac blood supply?Posterior interventricular arteryWhat are the most common sites for coronary occlusion?Left anterior descending (LAD) > RCA > ci rcumflexTrace the general pathway of venous drainage from myocardium.Great, middle, and smal l cardiac veins → coronary s inus → right atriumTrace the conduction pathway of a cardiac impulse.SA node → AV node → bundle of His → right and left bundle branches → Purkinje fibersWhich nerve supplies parasympathetic input to heart?Vagus nerveWhich syndrome is characterized by arm claudication, syncope, vertigo, nausea, and a supraclavicular bruit?Subclavian s tea l syndrome (occlus ion in subclavian artery proximal to take vertebra l artery take-off → “stea l ing” of blood from vertebra lartery to dis ta l subclavian artery)

Describe the best location for auscultation of the following cardiac valves:Tricuspid valveLeft s ternal border, fi fth intercosta l spacePulmonary valveLeft s ternal border, second intercosta l spaceMitral valveApex of heart, fi fth intercosta l spaceAortic valveRight s ternal border, second intercosta l space

PHYSIOLOGY

Cardiac Electrophysiology

Name the electrical event in the heart associated with each feature of a normal electrocardiogram:

P waveAtria l depolarizationPR intervalAtria l depolarization and conduction delay through AV nodeQRS complexDepolarization of the ventriclesT waveVentricular repolarization

Which ion primarily dictates the resting membrane potential of a myocyte?Potass ium. Membrane has high K+ permeabi l i ty through K+ channels .

Which membrane protein maintains the ion gradient?Sodium-potass ium ATPase

What is the effect of potassium efflux from a myocardial cell?Hyperpolarization

What is the effect of potassium influx into a myocardial cell?Depolarization

Describe the electrochemical events that cause the following phases of the cardiac myocyte action potential:Phase 0 (the upstroke)Influx of Na + into cel lPhase 1 (initial repolarization)Efflux of K+ out of cel l and ↓ Na + influxPhase 2 (the plateau)Influx of Ca 2+ into cel l , efflux of K+ out of cel lPhase 3 (repolarization)Efflux of K+ out of cel lPhase 4 (resting membrane potential)Equi l ibrium potentia l , ba lance between K+ leak current and Na +/K+ ATPase

What unique electrochemical feature of SA node allows it to act as a pacemaker for the heart?Phase 4 depolarization caus ing automatici ty

What ion and channel are responsible for automaticity?Na + conductance via I f channel .

Describe the electrochemical events that cause the following phases of the SA nodal action potential:Phase 0 (the upstroke)Influx of Ca 2+ into cel lPhase 3 (repolarization)Efflux of K+ out of cellPhase 4 (slow depolarization)Increas ing Na + influx into cel l

What phase 4 characteristic determines heart rate?Slope which represents rate of depolarization

Name the component of the cardiac conduction system where phase 4 depolarization is fastest:SA node

Name the component of the cardiac conduction system where phase 4 depolarization is slowest:

Bundle of His and Purkinje fibersDefine conduction velocity.

The rate at which an impulse spreads throughout cardiac ti s sueWhat determines conduction velocity?

Rate of depolarization (phase 0 upstroke)Where is conduction velocity fastest?

The Purkinje systemWhere is conduction velocity slowest?

The AV nodeWhat is the significance of the conduction delay at the AV node?

The delay in conduction a l lows for ventricular fi l l ing

Cardiac Contractility and Output

Describe the function of each myocardial cellular component:

SarcomereContracti le uni tIntercalated disksCel l adhes ionGap junctionElectrochemica l communication between myocardia l fibersT tubulesCarry action potentia ls into the cel l interiorSarcoplasmic reticulumStorage and release of ca lcium

Which ion determines the magnitude of tension in a contracting myocardial cell?Amount of intracel lular ca lcium

Describe the effect of each of the following on contractility:Increased heart rateIncreased contracti l i tyCatecholaminesIncreased contracti l i tyDigoxinIncreased contracti l i tyAcetylcholine (ACh)Decreased contracti l i ty

What is the Frank-Starling relationship?The greater the end-diastol ic volume (preload), the greater i s the s troke volume.

How does contractility affect cardiac output?Cardiac output increases as contracti l i ty increases .

Name the four factors that determine myocardial oxygen consumption:1. Afterload2. Size of heart (wal l tens ion)3. Contracti l i ty4. Heart rate

Provide formulas for each of the following:Stroke volumeEnd-diastol ic volume − end-systol ic volume =Cardiac outputStroke volume × heart rate =Ejection fractionStroke volume/end-diastol ic volume =Stroke workAortic pressure × s troke volumeCardiac output based on Fick principleO2 consumption/(arteria l O2 - venous O2)

Heart Sounds

Name the event associated with eachheart sound:

S1

Closure of the AV (tricuspid, mitra l )va lvesS2

Closure of semi lunar (aortic, pulmonary) va lvesS3

Flow of blood from atria into ventriclesduring diastole (o en seen with largeventricular volumes , ie, CHF; o enbenign in youth andtra ined athletes )S4

Flow of blood from atria into ventriclesduring atria l sys tole (often present inpatient with s ti ffened ventricle)

Maintenance of Blood Pressure

What is the site of highest resistance in the cardiovascular system?

ArteriolesWhich vascular bed has the largest cross-sectional and surface areas?

Capi l lariesWhich vascular bed contains the largest volume of blood at any given time?

VeinsDefine the following terms:

Systolic blood pressureHighest arteria l blood pressure during acardiac cycleDiastolic blood pressureLowest arteria l blood pressure during acardiac cyclePulse pressureDifference between systol ic and diastol icblood pressure

What is the most important determinant of pulse pressure?Stroke volume

What is the effect of aging on pulse pressure?Aging widens pulse pressure due to ↓ capaci tance of blood vessels

How is left atrial pressure estimated clinically?Pulmonary capi l lary wedge pressure

Where are the carotid baroreceptors located?Bi furcation of common carotid arteries

What is the function of the carotid baroreceptors?Minute to minute regulation of blood pressure

Which nerve carries information from baroreceptors to the vasomotor center in the brainstem?Crania l nerve IX

Describe the phases of the Valsalva maneuver and changes occurring with BP and heart rate.Phase 1: forceful expiration aga inst closed glotti s (↑ BP with ↑ intrathoracic pressure, ↓ HR with baroreflex)Phase 2: accommodation (↓ BP with reduced venous return, ↑ HR with baroreflex)Phase 3: breathe in (↓ BP with ↓ intrathoracic pressure, ↑ HR with baroreflex)Phase 4: recovery (↑ BP with normal venous return, marked ↓ HR with baroreflex overshoot)

Name two ways that the vasomotor center responds to decreased mean arterial pressure:1. ↓ Parasympathetic output2. ↑ Sympathetic output

Name the hormone(s) responsible for each of the following functions:Long-term regulation of blood pressureRenin-angiotens in-a ldosterone systemStimulation of aldosterone secretion and arterial vasoconstrictionAngiotens in I IWater retention and direct arteriolar vasoconstriction causing an increase in blood pressureVasopress inInhibition of renin release, stimulation of salt and water excretion, and vascular smooth muscle relaxationAtria l natriuretic peptide

Name three stimuli for renin secretion:1. ↓ Renal blood pressure2. ↓ Na + del ivery to macula densa of JGA3. ↑ Sympathetic tone

What is the effect of cerebral ischemia on blood pressure and heart rate?Cushing reflex: blood pressure ↑ and heart rate ↓

What is the mechanism of increased blood pressure in cerebral ischemia?Chemoreceptors in the vasomotor center s timulate increased sympathetic outflow

What is the mechanism of decreased heart rate in cerebral ischemia?Baroreceptor reflex to increase in BP leads to increased parasympathetic outflow to heart

Where are the carotid chemoreceptors located?Bi furcation of common carotid arteries and the aortic arch at carotid body

What do chemoreceptors sense?Oxygen, CO2, pH, and temperature

What is the Starling equation?Jv = Kf [(Pc - Pi) - (πc − πi)]

Describe the effect of each of the following on capillary filtration:Increased capillary hydrostatic pressureIncreased fluid fi l trationIncreased interstitial hydrostatic pressureDecreased fluid fi l tration

Increased capillary oncotic pressureDecreased fluid fi l tration

Increased interstitial oncotic pressureIncreased fluid fi l tration

Define autoregulation.The capaci ty of an organ to mainta in constant blood flow despi te changes in mean arteria l pressure

Define active hyperemia.The capaci ty of an organ to increase blood flow in response to metabol ic demands

Name the primary mechanism of blood flow regulation in the following tissues:Coronary arteriesLoca l metabol ic controlCerebral vasculatureLoca l metabol ic controlMuscleLoca l metabol ic controlSkinSympathetic controlPulmonary

Loca l metabol ic controlName five key metabolites which cause vasodilation:

1. Lactate2. K+

3. Adenos ine4. CO2

5. H+Name the primary vasoactive metabolite in the following tissues:

Coronary arteriesO2, adenos ineCerebral vasculatureCO2 (most important), H+

MuscleLactate, K+, adenos inePulmonaryO2

CARDIOVASCULAR PATHOLOGY AND PATHOPHYSIOLOGY

Murmurs

Name the valvular defect causing each murmur described below:

Harsh midsystolic murmur in the left second intercostal space at the left sternal borderPulmonic s tenos isHarsh midsystolic murmur in the right second intercostal space at the right sternal border, radiating to the neck (carotid arteries) and apexAortic s tenos isHarsh midsystolic murmur at the le third and fourth interspaces radia ng down the le sternal border; murmur louder with decreased preload (ie, onValsalva); S4 and biphasic apical impulse often presentHypertrophic cardiomyopathyBlowing holosystolic murmur at apex radiating to the left axilla with increased apical impulseMitra l regurgi tationBlowing holosystolic murmur at the lower left sternal border radiating to the right of the sternum; may ↑ with inspirationTricuspid regurgi tationSoft, late systolic murmur at the left sternal border or apex, accompanied by midsystolic clickMitra l va lve prolapseHarsh holosystolic murmur at the lower left sternal border, accompanied by a thrillVentricular septa l defect (VSD)Blowing, high-pitched diastolic murmur at the left second to fourth interspaces radiating to the apexAortic regurgi tationLow-pitched diastolic murmur at the apex that gets louder prior to S1; an opening snap is often present just after S2

Mitra l s tenos isSystolic flow murmur at left upper sternal border; fixed splitting of S2

Atria l septa l defect (ASD)What congenital valvular defect is associated with aortic stenosis?

Bicuspid aortic va lveWhat are two common manifestations of aortic stenosis?

Angina and syncopeWhich disorder results from myxomatous degeneration of the mitral valve?

Mitra l va lve prolapsePatients with mitral valve prolapse are at increased risk of which infection?

Infective endocardi ti s

Heart Failure

What are the most common causes of left-sided heart failure?

Ischemic heart disease, HTN, mitra l and aortic va lvular disease, myocardia l disease (cardiomyopathy, myocardi ti s )Name four key mechanisms of compensation in congestive heart failure (CHF):

1. Hypertrophy2. Ventricular compensation3. Blood volume expans ion4. Tachycardia

Name two major clinical signs/symptoms of left-sided heart failure:1. Pulmonary congestion2. Pulmonary edema caus ing dyspnea and orthopnea

Name three consequences of suboptimal renal perfusion:1. RAA axis activation leading to sa l t and water retention2. Ischemic acute tubular necros is (ATN)3. Prerenal azotemia

What is the consequence of impaired cerebral perfusion in CHF?Hypoxic encephalopathy

What is the most common cause of right-sided heart failure?Left-s ided heart fa i lure

Name two major pulmonary causes of right-sided heart failure:

1. Inters ti tia l fibros is2. Pulmonary HTN

Name four key clinical signs of right-sided heart failure:1. Porta l , sys temic, periphera l congestion, and edema2. Hepatomegaly3. Congestive splenomegaly4. Renal congestion

Describe the phases of pathologic change in the liver that result from chronic right-sided heart failure.Nutmeg appearance → centri lobular necros is → centra l hemorrhagic necros is → cardiac scleros is (ci rrhos is )

What is the most common cause of acute right-sided heart failure in a patient with a deep venous thrombosis?Mass ive pulmonary embolus

What EKG findings may be seen with acute right-sided heart stress?S1Q3T3: S wave in lead 1, Q wave in lead 3, T-wave invers ion in lead 3

Ischemic Heart Disease

Name four important manifestations of ischemic heart disease:

1. Angina2. Myocardia l infarction (MI)3. Letha l arrhythmia caus ing sudden cardiac death4. Chronic CHF

What are the two major etiologies of myocardial ischemia?1. ↓ Coronary perfus ion2. ↑ Myocardia l O2 demand

What conditions compound the consequences of impaired myocardial perfusion?Anemia, advanced lung disease, cigarette smoking, congenita l heart disease

Name the type of angina:Pain precipitated by exertion, relieved by rest/vasodilatorsStable anginaParoxysmal chest pain at rest in patient with or without coronary risk factorsPrinzmeta l anginaSevere substernal pain/pressure at restUnstable angina

Describe the underlying pathology for each type of angina:Stable anginaGreater than 75% occlus ion of coronary arteryPrinzmetal anginaCoronary vasospasmUnstable anginaPlaque dis ruption with resul ting occlus ive thrombos is within a coronary artery

Describe the associations between myocardial ischemia, injury, and infarction.1. Ischemia—insufficient blood supply to the myocardium, occurs fi rs t2. Injury—resul ts when the i schemic process i s prolonged3. Infarction—describes necros is or death of myocardia l cel l s

Name the type of myocardial damage described below:Full-thickness infarction caused by complete occlusion of a coronary arteryTransmural infarctionInfarction of the inner half (or less) of the ventricular wall supplied by a partially occluded coronary arterySubendocardia l infarctionT-wave inversion on ECGTransmural i schemiaST depression on ECGSubendocardia l injuryST elevation on ECGTransmural injuryQ waves present on ECGTransmural infarction

What is the initial event in the development of a transmural infarction?Acute plaque dis ruption

What are the most common symptoms of a myocardial infarct?Crushing retrosternal chest pa in or pressure, dyspnea, pa in radiating into left arm or neck, diaphores is , nausea

What type of necrosis is seen in myocardium within 24 hours of infarction?Coagulative necros is

What type of inflammatory cells are seen in the myocardium within 24 hours of infarction?Neutrophi l s

What is the most common type of inflammatory cell seen in myocardium from the 2nd to 10th day after an infarction?Macrophages

When is the risk of myocardial rupture greatest and why?At 4 to 7 days . Tissue i s weakest fol lowing phagocytos is of debris by macrophages and prior to growth of granulation ti ssue.

How many days does it take to form granulation tissue in a region of infarcted myocardium?7 to 10 days

How many weeks does it take to form contracted scar tissue in a region of infarcted myocardium?7 weeks

What is the diagnostic test of choice in a patient with suspected MI?ECG

What two classic ECG changes are seen during an MI?1. Q waves2. ST elevation

When does CK-MB begin to rise, peak, and return to normal?Rise: 3 to 8 hoursPeak: 10 to 24 hoursReturn to normal : 2 to 3 days

When does troponin I begin to rise, peak, and return to normal?Rise: 3 to 8 hoursPeak: 24 to 48 hoursReturn to normal : 5 to 10 days

What are the advantages and disadvantages of the CK-MB serum cardiac marker?Al lows diagnos is of re-infarction as levels quickly return to normal ; may be fa lsely elevated with skeleta l muscle injury

What are the advantages of the Troponin test?Very speci fic to cardiac injury; a l lows diagnos is of late presenting MI

What are the two most common complications of MI?1. Cardiac arrhythmia2. CHF

List five less common but severe complications of MI:1. Cardiogenic shock2. Ventricular aneurysm or rupture3. Papi l lary muscle rupture4. Mura l thrombos is with resul ting periphera l embol ism5. Dress ler syndrome

Cardiomyopathy

How does the left ventricle respond to long-standing HTN?

Concentric hypertrophyWhat are the three types of cardiomyopathy?

1. Di lated or congestive2. Hypertrophic3. Restrictive

What are the most common nongenetic etiologies of dilated cardiomyopathy?“ABCDE”Al cohol abuseBeriberi (thiamine deficiency)Coxsackie Bmyocardi ti s , Cocaine, Chagas diseaseDoxorubicin toxici typrEgnancy

Name the type of cardiomyopathy associated with the following clinical and pathologic features:30% to 40% of cases are geneticDi lated100% of cases are geneticHypertrophicAssociated with alcoholism and thiamine deficiencyDi latedAssociated with coxsackie virus B and with Trypanosoma cruziDi latedAssociated with doxorubicinDi latedAssociated with eosinophiliaRestrictive (Loeffler endocardi ti s )Associated with pregnancyDi latedAsymmetric septal hypertrophy, banana-shaped left ventricle without dilatationHypertrophicCan be caused by sarcoidosis, amyloidosis, scleroderma, hereditary hemochromatosis, endocardial fibroelastosis, radiation-induced fibrosisRestrictiveCardiomyopathy most commonly caused by endomyocardial fibrosisRestrictiveCauses sudden death in young, otherwise healthy athletesHypertrophicCommonly inherited in an autosomal-dominant (AD) fashionHypertrophicFour-chamber hypertrophy and dilationDi latedLeft ventricular outflow obstructionHypertrophicMyocyte tangles, disorientationHypertrophicSymptoms relieved by squattingHypertrophic

Pericarditis and Cardiac Tumors

Name the type of pericarditis based on the following exudates descriptions:

Clear, straw colored, minimal inflammation, decreased fibrinSerous pericardi ti sFibrin rich

Fibrinous pericardi ti sBloodyHemorrhagic pericardi ti s

What are the most common etiologies of serous pericarditis?Uremia, systemic lupus erythematosus (SLE), rheumatic fever

What are the most common etiologies of fibrinous pericarditis?Uremia, SLE, rheumatic fever, coxsackie vi ra l infection, MI, trauma

What are the most common etiologies of hemorrhagic pericarditis?Trauma, mal ignancy, tuberculos is

What ECG and BP findings are seen in pericarditis?ECG: di ffuse ST elevations in a l l leadsBP: pulsus paradoxus

What is the most common cardiac tumor of adults?Metastases (eg, melanoma)

What is the most common primary cardiac tumor of adults?Left-s ided atria l myxoma

What is the most common primary cardiac tumor in children?Rhabdomyoma, commonly associated with tuberous scleros is

Congenital Heart Disease

Name the congenital heart defect associated with each of the following statements:

Three most common causes of R->L shunting1. Atria l septa l defect (ASD)2. VSD3. Patent ductus arteriosus (PDA)Two most common defects1. VSD2. PDAFive defects causing cyanosis at birth“5 T’s”:1. Tetra logy of Fa l lot2. Transpos i tion of the great vessels3. Truncus arteriosus4. Tota l anomalous pulmonary venous return5. Tricuspid atres iaContinuous machinery-like murmurPDAPulmonic stenosis, right ventricular hypertrophy, overriding aorta, VSD (“PROVe”)Tetra logy of Fa l lotBoot-shaped cardiac silhouetteTetra logy of Fa l lotDefect causing lower body cyanosisPreducta l coarctationDefect causing upper extremity HTN and diminished lower extremity pulsesPostducta l coarctation (s tenos is dis ta l to the ductus arteriosus)Associated with Turner syndromeCoarctation of the aortaAssociated with Down syndromeASDs, VSDs , and AV va lve abnormal i ties (due to endocardia l cushion abnormal i ties )Associated with RubellaPatent ductus arteriosusAssociated with 22q11 syndromeTruncus arteriosus , Tetra logy of Fa l lotCommon cyanotic congenital heart defect in children born to diabetic mothersTranspos i tion of the great vesselsAorta arises from right ventricle and pulmonary trunk arises from left ventricleTranspos i tion of the great vesselsValvular defect associated with increased risk of infective endocarditis and calcificationBicuspid aortic va lve

What is the consequence of leaving a left-to-right shunting untreated?Right → left shunting

What type of infection are patients with VSD at an increased risk for?Infective endocardi ti s

What drug is used to induce closure of a PDA?Indomethacin

What drug is used to prevent closure of a PDA?Prostaglandins

What is the most common genetic cause of congenital heart disease?Trisomy 21

Cardiac Infectious Disorders

What type of infection is responsible for causing rheumatic fever?

Group A s treptococca l pharyngi ti sHow does streptococcal pharyngitis cause rheumatic heart disease?

Antis treptococca l antibodies cross -react with a cardiac antigen

What serologic test is elevated in rheumatic heart disease:Antis treptolys in antibodies (ASO)

Name the five major Jones criteria for rheumatic heart disease:1. Joints : migratory polyarthri ti s2. ♥ (o) = pancardi ti s3. Nodules : subcutaneous nodules4. Erythema marginatum5. Syndeham chorea

Name five minor Jones criteria for rheumatic heart disease:1. Fever2. Arthra lgia3. Elevated ESR/CRP4. Leukocytos is5. Heart block on ECG

What term is used to describe the foci of pink collagen surrounded by lymphocytes and Anitschkow cells (macrophages) that are pathognomonic forrheumatic heart disease?

Aschoff bodiesWhich valve is most commonly affected in rheumatic heart disease?

Mitra l va lveWhat is the most commonly observed valvular deformity in rheumatic heart disease?

Fishmouth or buttonhole s tenos is of the mitra l va lveWhat are the three major categories of endocarditis?

1. Infective2. Nonbacteria l thrombotic or marantic3. Libman-Sacks

What is the most common valve affected by bacterial endocarditis?Mitra l va lve

What is the most common valve affected by bacterial endocarditis in IV drug users?Tricuspid va lve

Name the type of endocarditis described in each of the following vignettes:25-year-old (y/o) IV drug user with rapid onset of high fever, rigors, malaise, and tricuspid regurgitationAcute infective endocardi ti s60-y/o woman with mitral valve prolapse, who has recently undergone dental extraction, presents with low-grade fever and flu-like symptomsSubacute infective endocardi ti s65-y/o man with metastatic colon cancer and new murmur consistent with mitral regurgitationNonbacteria l thrombotic endocardi ti s30-y/o woman with SLELibman-Sacks endocardi ti s

Which organism most often causes acute infective endocarditis?Staphylococcus aureus

Which organism most often causes subacute infective endocarditis?Streptococcus viridians

What are the clinical signs of bacterial endocarditis?FeverRoth spotsOs ler nodesMurmurJaneway les ionsAnemiaNai l bed hemorrhagesEmbol i

Define the following eponyms used to describe signs of bacterial endocarditis:Osler nodesTender, ra ised les ions on finger and toe padsJaneway lesionsSmal l , erythematous les ions on pa lms and solesRoth spotsErythematous spots with white centers on retina

What are some sequelae to bacterial endocarditis?Valvular injury, renal injury (glomerulonephri ti s ), septic embol i to bra in, kidneys caus ing infarction or abscess

Atherosclerosis

What are the major risk factors for coronoary heart disease?

Age (men > 45, women > 55 or with premature menopause)Fami ly his tory of premature heart attacks (MI or sudden cardiac death in men 55, women 65)Cigarette smokingHypertens ion (HTN)HDL 40, HDL > 60 negates one ri sk factor

What are the two main histopathologic components of an atheroma?Superficia l fibrous cap overlying a necrotic core

What are the components of the fibrous cap?Smooth muscle cel l s , macrophages , foam cel l s , lymphocytes , col lagen, and elastin.

What is the pathologic precursor to atheroma?Fatty s treak

What is the term for ulcerated, calcified, hemorrhagic atheromas that predispose to thrombosis?Compl icated plaques

What is the underlying pathologic basis for the initiation of atherosclerosis?

Endothel ia l injuryWhat may produce this endothelial injury?

Hypercholesterolemia, mechanica l injury, HTN, immune mechanisms, toxins , etc.How are foam cells created?

Macrophages that ingest oxidized LDL that has accumulated in the vessel wal l become foam cel l s .Which lipids are most commonly found in an atheromatous plaque?

Cholesterol and cholesterol es ters (LDL)What is the function of smooth muscle cells in an atheroma?

Smooth muscle cel l s when ac vated by growth factors prol i ferate and migrate into in ma. They then secrete extracel lular matrixglycoproteins .

What are the major cell types associated with the formation of an atheroma?Macrophages (foam cel l s ), T lymphocytes (attracted to zone of injury), smooth muscle cel l s , and endothel ia l cel l s

What are the most common locations for atherosclerotic disease?Abdominal aorta , coronary arteries , popl i tea l arteries , and carotid arteries

What are the four major clinical manifestations of atherosclerosis?1. Arteria l insufficiency (ie, s troke, MI, periphera l vascular disease, i schemic bowel disease)2. Thrombus formation due to plaque rupture3. Atheroembol ism4. Aneurysm, dissection, or rupture of a major vessel

Arteriolosclerosis

Name the type of arteriolosclerosis associated with the following clinical and pathologic features:

Hyaline deposits causing thickening of arteriolar wallsHyal ine arterioloscleros isOnion skin arteriolar thickeningHyperplastic arterioloscleros isDeposition of calcium in the medial coat of arteries of the lower extremitiesMonckeberg arterioscleros isPipestem arteries with ringlike calcificationsMonckeberg arterioscleros isTwo types of arteriolosclerosis that occur in patients with long-term HTN1. Hyal ine arterioloscleros is2. Hyperplastic arterioloscleros is

Hypertension

HTN is the most important risk factor in which two vascular diseases?

1. Coronary artery disease2. Cerebrovascular disease

What percentage of hypertensive patients have essential HTN?90% to 95%

What are two common renal causes of secondary HTN?1. Renal artery s tenos is2. Renal parenchymal disorders

What are some common endocrine causes of secondary HTN?1. Pheochromocytoma2. Conn syndrome (primary a ldosteronism)3. Hyperthyroidism4. Ora l contraceptive use5. Cushing syndrome6. Acromegaly

Name two common cardiac complications of long-standing HTN:1. Left ventricular hypertrophy2. Left-s ided heart fa i lure (a resul t of secondary to left ventricular hypertrophy)

Define hypertensive urgency and emergency.Urgency: blood pressure > 200/120 without symptomsEmergency: blood pressure > 200/120 with symptoms or evidence of end organ damage

List the possible effects of malignant HTN on each organ system below:HeartAcute LV fa i lure, MIAortaDissectionLungsPulmonary edemaKidneysAcute renal fa i lureEyesPapi l ledema, fundal hemorrhages , blurred vis ionBrainHeadache, encephalopathy, seizure, hemorrhagic cerebrovascular accident (CVA)

What is the most common cause of death in patients with untreated HTNCoronary artery disease

Aneurysms and Dissection

What is the most common site for atherosclerotic aneurysm?Abdominal aorta , below renal arteries and above i l iac bi furcation

What is the most common site for syphilitic aneurysm?Thoracic aorta (often ascending)

What is the mechanism of syphilitic aneurysms?Obl i teration of the arteries supplying the aorta (endarteri ti s obl i terans), leading to necros is of the media

Name the type of aneurysm referred to in the following clinical vignettes:55-y/o man, who is a smoker with HTN, diabetes, and coronary artery disease, is found to have a pulsatile midline abdominal mass.Atherosclerotic, abdominal aortic aneurysm35-y/o with a family history of polycystic kidney disease presents with the worst headache of his life.Berry Aneursym rupture leading to subarachnoid hemorrhage40-y/o prostitute with angina and shortness of breath (SOB) is found to have wide pulse pressure and a high-pitched blowing diastolic murmur.Syphi l i ti c/luetic, di lation of aortic root leads to aortic regurgi tation

What are the most common etiologies for the development of aortic dissection?HTN and connective ti ssue disorders (such as Marfan syndrome)

Name two characteristic histopathologic findings in the aorta in a patient with Marfan syndrome:1. Elastic ti s sue fragmentation2. Cystic media l degeneration

What is the genetic defect in Marfan disease?Defect in fibri l l in-1 which forms a scaffold for elastic fibers .

What is the classic clinical presentation of aortic dissection?Sudden onset of severe tearing pa in radiating to the back and descending as the dissection progresses

What is the underlying mechanism in the development of varicose veins?Venous di lation or deformation leading to venous va lvular incompetence

State two complications of varicose veins:1. Varicose ulceration2. Stas is dermati ti s

Vasculitides

Name the type of vasculitis associated with the following clinical and pathologic features:

Presence of C-ANCAWegener granulomatos isPresence of P-ANCAMicroscopic PolyANgi i ti s , PolyArteri ti s Nodosa (PAN), and Churg-Strauss syndromeMost common vasculitis in the United StatesTempora l or giant cel l arteri ti sFibrous thickening of origins of the great vessels leading to absent pulsesTakayasu arteri ti sWell-demarcated, segmental fibrinoid necrosis of the arterial wall and diffuse neutrophilic infiltrates in medium-sized arteriesPANCommonly affecting the coronary arteries and may result in coronary aneurysmKawasaki diseaseSegmental inflamma on/thrombosis of arteries and adjacent veins, nerves, and connec ve ssue; results in painful ischemic disease and associatedwith smokingBuerger disease (Thromboangi i ti s obl i terans)Vascular and mesangial deposition of IgA complexes with abdominal pain, palpable purpura especially on buttocks, glomerulonephritis, and arthritisHenoch-Schonlein purpura

Which vessels are most commonly involved in temporal arteritis?External carotid artery and i ts branches

What is the triad of Wegener granulomatosis?1. Foca l necrotizing vascul i ti s of upper a i rways and lungs2. Necrotizing granulomas of upper and lower respiratory tract3. Necrotizing glomerul i ti s

What are the six clinical signs of acute arterial occlusion?“Six P’s”:1. Pain2. Pal lor3. Poiki lothermia4. Paresthes ia5. Para lys is6. Pulselessness

Name the type of vasculitis associated with each of the following clinical vignettes:70-y/o white man with constitutional symptoms, headache, jaw claudication, acute onset of blindness; W/U: ↑ ESRTempora l (giant cel l ) arteri ti s30-y/o Korean woman with constitutional symptoms, arthritis; physical examination (PE): loss of carotid and radial pulsesTakayasu arteri ti s35-y/o hepatitis B positive man with fever, HTN; W/U: neutrophilia, P-ANCA ⊕PAN12-y/o girl recovering from a viral infec on presents with exer onal chest pain; PE: febrile, conjunc val injec on, cervical lymphadenopathy, strawberrytongue, diffuse erythematous rash, edema of hands and feetKawasaki disease40-y/o man with asthma; PE: palpable purpura, diffuse wheezes; CBC: eosinophilia; transbronchial biopsy: upper airway granulomas and elevated P-ANCA and ESRChurg-Strauss syndrome8-y/o boy presents with abdominal pain, hematuria, and recent history of URI; PE: palpable purpura, heme ⊕ stools; W/U: vascular biopsy showsperivascular granulocytes, IgA depositionHenoch-Schonlein purpura

55-y/o man presents with chronic cough, rhinorrhea, ulcerations of nasal septum; W/U: RBC casts in urine and C-ANCA ⊕Wegener granulomatos is21-y/o woman smoker with pallor and cyanosis of fingertips after exposure to coldRaynaud disease

Vascular Tumors

Name the vascular disorder associated with the following clinical or pathologic features:

Most common tumor of infancyPort-wine s ta in bi rthmarkCutaneous vascular tumor commonly seen in patients with end-stage liver disease (hyperestrinism)Spider telangiectas iaAD condition is characterized by a localized dilation of venules and capillaries in skin and mucous membrane leading to epistaxis and GI bleedingHereditary hemorrhagic telangiectas ia (Rendu-Os ler-Weber) syndromeAD condi on is characterized by the presence of mul ple cavernous hemangiomas of skin, liver, pancreas, and spleen and hemangioblastomas ofcerebellum with ↑ risk of renal cell carcinomavon Hippel -Lindau diseaseVascular tumor associated with thorium contrast material and polyvinyl chlorideHemangiosarcomaVascular tumor commonly seen in men of eastern European descent and in those infected with HIV consis ng of red or purple cutaneous plaques on thelower extremitiesKapos i sarcomaVascular tumor caused by human herpesvirus (HHV) 8 or HIVKapos i sarcomaSyndrome is characterized by cyanosis and edema of upper extremities, head, and neck in a patient with bronchogenic carcinomaSuperior vena cava syndrome

PHARMACOLOGY

Antianginal Agents

What are the three classes of antianginal drugs?

1. Organic ni trates2. β-Blockers3. Ca lcium channel blockers

How do nitrates cause vasodilation?Nitrates are metabol i zed to ni tric oxide (NO) → NO causes ↑ in cycl ic guanos ine monophosphate (cGMP) in endothel ia l cel l s →vasodi lation

Describe how nitrates reduce angina.1. Vasodi lation causes venous pool ing, which reduces preload and consequently myocardia l oxygen consumption.2. Coronary vasodi lation improves oxygen del ivery to the myocardium.

What is the most common side effect of nitrates?Headache

Describe how each of the following drugs reduces angina:β-BlockersBy ↓ contracti l i ty and heart rateNifedipineCoronary arteriaolar vasodi lationVerapamilDecreased heart rate and contracti l i ty, s lowed conduction (especia l ly through AV node)

What is the antianginal drug of choice for Prinzmetal angina?Di l tiazem

What antianginal drug is contraindicated in patients with asthma and COPD?β-Blockers

Inotropic Support

Name the only oral inotropic agent:

DigoxinDescribe the three-step mechanism by which digitalis potentiates myocardial contractility.

1. Inhibi tion of Na +/K+ ATPase.2. Bui ldup of intracel lular Na +.3. High intracel lular Na + impairs Na +-Ca 2+ antiport, caus ing increased intracel lular Ca 2+.

What is the unique side effect of digoxin on the heart?Dysrhythmia

Which agent can be used to counteract the cardiac toxicity of digoxin? By what mechanism?K+ supplementation. Digoxin binds near K+ s i te on Na +/K+ ATPase. When K+ i s low, digoxin has better access and vice versa .

What are the unique neurotoxicities of digoxin?Headache, nausea, a l tered color perception, blurred vis ion, tinni tus

For which arrhythmia is digoxin commonly used?Atria l fibri l lation

What is the β-agonist of choice for inotropic support in heart failure?Dobutamine

Which antiarrhythmic is useful for abolishing torsades de pointes and digoxin toxicity?Mg2+

By which mechanism do β-agonists and phosphodiesterase inhibitors potentiate myocardial contractility?These agents increase cAMP which promotes increased intracel lular Ca 2+.

Antiarrhythmics

For each class of antiarrhythmic agent, state the mechanism of action. (Note: many of these agents have multiple mechanisms of action.):

IANa + channel blocker, some K+ blockadeIBNa + channel blockerICNa + channel blockerIIβ+ blockadeIIIK+ channel blockerIVCa 2+ channel blocker

For each class of antiarrhythmic agent, describe how the myocyte action potential is affected:IASlows phase 0 depolarization, s lows phase 3 repolarizationIBShortens phase 3 repolarizationICSlows phase 0 depolarizationIISuppresses phase 4 depolarizationIIIProlongs phase 3 depolarizationIVShortens duration of action potentia l

For each class of antiarrhythmic agent, name several commonly used agents:IAQuinidine, amiodarone, proca inamide, disopyramideIBMeLT: Mexi l i tine, Lidoca ine, TocainamideICFleca inide, enca inide, propafenoneIIPropranolol , esmolol , metoprolol , atenololIIISota lol , ibuti l ide, bretyl ium, amiodaroneIVVerapami l , di l tiazem

For each subtype of class I antiarrhythmics, describe binding affinity for Na channels and the effect on APDIAIntermediate binding affini ty to Na + channels , a l so blocks K+ channels ; prolongs AP durationIBRapid rate of binding and release; shortens AP durationICTight binding and s low release; minimal change in AP duration

For each antiarrhythmic agent, name the unique toxicity/toxicities:QuinidineCinchonism—headache, tinni tus , thrombocytopenia , torsades de pointesProcainamideRevers ible drug-induced lupusLidocaineCNS depress ion, cardiac depress ion, loca l anestheticFlecainideProarrhythmicPropranololSedation and fatigue, erecti le dys function, exacerbation of as thma and COPD, masks effects of hypoglycemiaBretyliumSevere postura l hypotens ionAmiodaroneInters a l pulmonary fibros is , thyroid dys func on, hepatotoxici ty, tremor, ataxia , neuropathy, bluish discolora on of skin, cornealdepos i ts

What is the drug of choice for abolishing acute supraventricular tachycardia (SVT)?Adenos ine

Antihypertensives

For each of the following drugs, provide:

1. The mechanism of action (MOA)2. Indication(s) (IND)3. Significant side effects and unique toxicity (TOX) (if any)

β-BlockersMOA: P-blockade ↓ cAMPIND: intravenous agent for the short-term management of HTNTOX: impotence, as thma, mask s igns of hypoglycemia, CV effects (bradycardia , CHF, AV block), sedationPrazosin, terazosin, doxazosinMOA: α-blockerIND: essentia l HTN, benign prostatic hypertrophyTOX: fi rs t-dose hypotens ionPhentolamineMOA: revers ible a-blockerIND: diagnos is of pheochromocytomaPhenoxybenzamineMOA: i rrevers ible a-blockerIND: diagnos is of pheochromocytomaNifedipineMOA: dihydropyridine Ca 2+ blockers , selective for vascular smooth muscleIND: essentia l HTNTOX: bradycardia , hypotens ion, metabol ic acidos is

Diltiazem, verapamilMOA: nondihydropyridine Ca 2+ blockers , verapimi l i s cardioselec ve, di l azem has intermediate affinity for heart and vascular smoothmuscleIND: essentia l HTNTOX: mi ld LFT abnormal i ty, sexual dys function

ClonidineMOA: centra l ly acting α2-agonis t that decreases sympathetic outflowIND: mi ld-to-moderate HTNTOX: dry mouth, rebound hypotens ion with sudden withdrawal

α-MethyldopaMOA: converted to α-methyl norepinephrine (NE) to act centra l ly as an a-agonis t and decrease sympathetic outflowIND: pregnant patients with HTNTOX: pos i tive Coombs test, drows iness

HydralazineMOA: causes arteriolar vasodi lation by increas ing cGMP in vascular smooth muscleIND: essentia l HTNTOX: drug-induced lupus , angina

MinoxidilMOA: causes arteriolar vasodi la on by caus ing K+ channels to open on vascular smooth muscle hyperpolarizing the membrane anddecreas ing vol tage-gated Ca 2+ currentIND: HTN, a lopeciaTOX: hypertrichos is , pericardia l effus ion

NitroprussideMOA: metabol i sm of ni tropruss ide releases NO which causes vasodi lation via cGMP.IND: mal ignant HTN.TOX: cyanide toxici ty (avoided i f drug i s mixed just prior to adminis tration).

For each condition listed below, select the best antihypertensive agent(s):Angina pectorisβ-Blockers , Ca 2+ channel blockersDiabetesAngiotens in-converting enzyme inhibi tors (ACEi ), Ca 2+ blockers , β-blockersHyperlipidemiaACEi , Ca 2+ blockerSVTβ-BlockersCHFDiuretics , ACEi , β-blockersHistory of MIβ-blockers , ACEiChronic renal failureDiuretics , Ca 2+ blockersAnxietyβ-BlockersBenign prostatic hyperplasiaα1-Selective antagonis tPheochromocytomaPhenoxybenzamine, phentolamineHypertrophic obstructive cardiomyopathyPhenoxybenzamine, phentolamineAsthma, COPDDiuretics , Ca 2+ blockersHyperthyroidismβ-BlockersModerate bradycardiaβ-Blockers with intrins ic sympathomimetic activi ty: pindolol and acebutolPregnancy

α-MethyldopaMigraine headachesβ-Blockers

For each condition listed below, list the antihypertensive agent that should be avoided:CHFVerapami lAsthma, COPDβ-Blockers

What class of diuretics is especially useful in black and elderly patients?Thiazide diuretics

What class of diuretics is useful in patients who have not responded to thiazides?Loop diuretics

What class of antihypertensives is useful in patients who cannot tolerate the side effects of ACEi?Angiotens in receptor blockers (ARBs)

What classes of antihypertensive agents are absolutely contraindicated in pregnant patients?ACEi and ARBs

List three agents useful in the management of malignant HTN:1. Sodium ni tropruss ide2. Hydra lazine3. Labeta lol

Lipid-Lowering Agents

Cholestyramine

MOA: bi le acid-binding res in, ↓ bi le acid s tores , ↑ catabol i sm of plasma LDLIND: adjuvant therapy for patients with fami l ia l hypercholesterolemia, ↓ LDLTOX: constipation, GI discomfort, may interfere with intestina l absorption of other drugs , LFT changes , myalgiasStatinsMOA: hydroxymethylglutaryl -CoA (HMG-CoA) reductase inhibi tors , rate-l imiting s tep of XOL synthes isIND: hypercholesterolemia: to ↓ LDLTOX: hepatotoxici ty, rhabdomyolys isNiacinMOA: reduces release of fatty acids from adipose; ↓ LDL synthes is in l iverIND: hypercholesterolemia: to ↑ HDL and ↓ LDLTOX: flushing, pruri tus (both revers ible with aspi rin), hepatotoxici ty, GI upset, paresthes iasGemfibrozil, clofibrateMOA: s timulate l ipoprotein l ipase to increase catabol i sm of VLDL and triglycerides (↓ VLDL, ↓ TGs)IND: hypercholesterolemia; dramatica l ly ↓ TGs , ↑ HDLTOX: myos i ti s , hepatoxici ty

Autonomics

Direct cholinergic agonists

AChMOA: phys iologic chol inergic agonis tIND: no cl inica l use; ACh decreases heart rate, cardiac output, and blood pressureTOX (for a l l di rect chol inergic agonis ts ):1. Diarrhea2. Diaphores is3. Mios is4. Nausea5. Urinary urgency

BethanecholMOA: binds primari ly at muscarinic receptors ; s timulates bowel and bladder smooth muscle contractionIND: postoperative i leus and urinary retentionPilocarpineMOA: di rect chol inergic agonis t that activates ci l iary muscle of the eye and pupi l lary sphincterIND: glaucoma

Indirect cholinergic agonists (anticholinesterases)Neostigmine, physostigmineMOA: inhibi tor of acetylchol inesterase → accumulation of ACh at synapsesIND: postoperative i leus , urinary retention, myasthenia gravis , reversa l of neuromuscular blockade; atropine overdose (physostigmine)TOX: genera l i zed convuls ions (rare)Edrophonium, pyridostigmineMOA: inhibi tor of acetylchol inesteraseIND: diagnos is (edrophonium) and treatment (pyridostigmine) of myasthenia gravisEchothiophateMOA: i rrevers ible inhibi tor of acetylchol inesteraseIND: treatment of organophosphate overdose, ga lucoma

List the toxicities of acetylcholinesterase poisoning (caused by parathion or organophosphates):“DUMBELS”DiarrheaUrinationMios isBronchoconstrictionExci tation of skeleta l musclesLacrimation

Sweating and Sal ivationAtropineMOA: nonselective muscarinic antagonis tIND: pupi l lary di la on, reduc on of gastric acid secre on, reduc on of GI mo l i ty, reduc on of a i rway and sa l ivatory secre ons ,organophosphate/chol inergic poisoningTOX:”Dry as a bone, Red as a beet, Blind as a bat, Mad as a hatter, Hot as a hare.”Dry as a bone: ↓ perspi ration, lacrimation, sa l ivationRed as a beet: dry, red skinBlind as a bat: blurry vis ionMad as a hatter: ha l lucinations , del i riumHot as a hare: hyperthermia

Describe how atropine acts on the following organs or systems:EyesPupi l lary di lation (bel la donna a lka loid), mydrias is , cycloplegiaAirwaysBlocks bronchia l secretion, bronchodi lationSalivary glandsBlocks sa l ivary secretionHeartBradycardia ini tia l ly; tachycardia ul timatelyScopolamineMOA: nonselective muscarinic antagonis tIND: motion s icknessIpratropiumMOA: nonselective muscarinic antagonis tIND: bronchodi lator used for asthma or COPDBenztropineMOA: centra l ly acting antimuscarinic agentIND: adjuvant therapy for Parkinson diseaseHexamethoniumMOA: competi tive nicotinic gangl ion blockerIND: used for HTN in the pastSuccinylcholineMOA: depolarizing neuromuscular blocker (nicotinic antagonis t)IND: neuromuscular blockade during genera l anesthes iaTOX: mal ignant hyperthermia when combined with ha logenated anesthetic agents in susceptible patients (treat with dantrolene)Tubocurarine, atracurium, vecuroniumMOA: nondepolarizing neuromuscular blockerIND: neuromuscular blockade during genera l anesthes ia ; atracurium and vecuronium are degraded in the plasma and therefore preferredin patients with renal fa i lureTOX: may cause his tamine release resul ting in bronchospasm, skin wheals , and hypotens ionPralidoximeMOA: chol inesterase regeneratorIND: used for organophosphate poisoning

State the MOA for each of the following agents:HemicholiniumInhibi ts reuptake of chol ine in chol inergic neurons (rate-l imiting s tep in ACh synthes is )BotulinumInhibi ts release of ACh in chol inergic neuronsReserpineInhibi ts VMAT transporter of dopamine into s torage ves icles in adrenergic neuronsGuanethidineInhibi ts release of NE from adrenergic neuronsAmphetamineStimulates release of NE from adrenergic neurons (indi rect-acting amines)Cocaine, tricyclic antidepressantsInhibi t reuptake of NE from synapse

Direct adrenergic agonistsEpinephrineMOA: agonis t at αx, α2, β1, β2 receptorsIND: acute, refractory asthma (bronchodi lator), open angle glaucoma, anaphylac c shock, used with loca l anesthe c to ↑ dura on ofaction (loca l vasoconstriction)TOX: CNS dis turbance, HTN, arrhythmia, pulmonary edema

Describe the effect of epinephrine on each of the following organs or tissues:MyocardiumPos i tive inotropic and chronotropic effects (β1)Blood vesselsOvera l l ↑ blood pressure (vasoconstric on of cutaneous mucous membrane and viscera l vasculature; vasodi la on of l iver and skeleta lmuscle vasculature) (α1 > β2)LungsBronchodi lation (β2)LiverIncreased glycogenolys is , increased insul in release (β2)Adipose tissueIncreased l ipolys is (α2, β1, β2, β3)PupilsMydrias is (α1 radia l di lator muscle contraction); accommodation for far vi s ion (β2 ci l iary muscle relaxation)Skin

Pi loerection(α1)NorepinephrineMOA: Selec ve adrenergic agonis t. Strong α -agonis t that a lso s mulates β 1 receptors . NE produces only minimal s mula on of β 2

receptors .IND: Useful in mainta ining blood pressure in shock.Compare the effects of NE and epinephrine (EPI) at the different adrenergic receptors.α1 EPI ≥ NEα2 EPI ≥ NEβ1 EPI = NEβ2 EPI >> NE *creates di fferencePhenylephrineMOA: di rect adrenergic agonis t; s timulates α1 > α2

IND: nasa l decongestantTOX: causes HTNClonidineMOA: di rect adrenergic agonis t; s timulates α2 > α1

IND: HTN (especia l ly in pregnant patients ), nicotine, heroin, and coca ine withdrawalTOX: rebound HTN, dry mouth, drows inessIsoproterenolMOA: di rect adrenergic agonis t; s timulates β1 and β2 receptors equal lyIND: Bronchodi lator used in asthmaDobutamineMOA: di rect adrenergic agonis t; s timulates β1 > β2

IND: pos i tive inotropic agent used to improve cardiac output in heart fa i lureAlbuterol, terbutalineMOA: di rect adrenergic agonis t; s timulates β2 > β1

IND: bronchodi lator used in asthma; s lows preterm labor by inhibi ting uterine contraction (terbuta l ine)TOX: tremor, tachycardiaDopamineMOA: di rect adrenergic agonis t; s timulates D1 and D2 receptors equal lyIND: maintenance of blood pressure in shock

What is the toxicity of most adrenergic agonists?1. CNS dis turbance—fear, anxiety, tens ion, headache2. HTN3. Arrhythmias4. Pulmonary edema

Indirect adrenergic agonistsAmphetamineMOA: s timulates release of NE from adrenergic neuronsIND: a ttention defici t hyperactivi ty disorder, narcolepsy, appeti te controlTOX: psychos is , anxietyTyramineMOA: s timulates release of NE from adrenergic neuronsIND: no cl inica l indications ; found in fermented foodsTOX: severe vasopressor effects in combination with MAOIsEphedrineMOA: s timulates release of NE from synaptic ves icles in adrenergic neuronsIND: nasa l decongestant, enhances athletic performanceTOX: causes HTNCocaineMOA: inhibi tion of catecholamine reuptakeIND: loca l anestheticTOX: arrhythmias , MI, seizures

Adrenergic antagonistsPhenoxybenzamineMOA: i rrevers ible adrenergic antagonis t; blocks α1 > α2

IND: HTN 2° to pheochromocytomaTOX: nasa l congestion, postura l hypotens ion, compensatory tachycardiaPhentolamineMOA: revers ible adrenergic antagonis t; blocks α1 and α2 receptors equal lyIND: diagnos is of pheochromocytoma; management of intraoperative HTNPrazosin, terazosinMOA: adrenergic antagonis t; selectively blocks α1 receptorsIND: treatment of HTN and benign prostatic hypertrophyTOX: orthostatic hypotens ionPropranololMOA: adrenergic antagonis t, blocks both P1 and P2 receptorsIND: HTN, angina prophylaxis , antiarrhythmic, anxiety, thyroid s torm, migra ine prophylaxisTOX: bronchoconstriction, arrhythmias , sexual dys function, CNS sedation, and fatigue

Describe the effect of propranolol on each of the following tissues:HeartDecreased cardiac outputBlood vesselsReflex periphera l vasoconstrictionKidneysIncreased Na + retention

LungsBronchoconstrictionMetoprolol, atenolol, esmololMOA: adrenergic antagonis t; selectively blocks P1 receptorsIND: HTN, angina prophylaxis , MI, superventricular tachycardia (esmolol )TOX: Hypotens ion, bradycardia , hyperglycemia, ventricular dysrhythmias

List two β-blockers with intrinsic sympathomimetic activity:1. Acebutolol2. Pindolol

Which β-blocker also blocks α1receptors?Labeta lol

What are the six common side effects of β-blockers?1. Bronchoconstriction/asthma attack2. Arrhythmia3. Sexual dys function4. Fasting hypoglycemia, masking of hypoglycemic s igns5. CNS sedation, fatigue6. Hypotens ion

Describe the effect of dopamine on each of the following organs or tissues:MyocardiumPos i tive inotropic and chronotropic effectsBlood vesselsVasoconstriction →↑ BPKidneysIncreases renal blood flow at low and moderate doses

CHAPTER 6Pulmonary

EMBRYOLOGY

From what structure does the lung bud arise?

ForegutFrom which embryonic layer is the lining of the respiratory tract derived?

EndodermFrom which embryonic layer is the muscle and connective tissue of the respiratory tract derived?

MesodermWhat is the consequence of incomplete separation of the lung bud from the esophagus?

Tracheoesophageal fi s tula (TEF)What is the most typical type of TEF?

Esophageal atres ia (ends in bl ind pouch)What other constellation of developmental abnormalities should you look for with TEF?

VATER syndrome: V ertebra l , A na l , T racheal , E sophageal , R adia l/R ena lAt what gestational age is a human capable of respiration?

25 weeksIn what period do the majority of alveoli develop?

PostpartumWhat histology is typical of cells lining the conducting airway?

Pseudocolumnar ci l iated cel l sWhich cells produce surfactant?

Type II pneumocytesList the structures the diaphragm is derived from:

“Severa l Parts Bui ld Diaphragm”— Septum transversum, Pleuroperi toneal folds , Body wal l , Dorsa l mesentery of esophagusWhich disorder results from abnormal development of the diaphragm?

Diaphragmatic or hiata l herniaWhere can pain from the diaphragm refer?

To the shoulder

ANATOMY

Which nerve provides motor innervation to the diaphragm?

Phrenic nerve (“C3, C4, C5 keep the diaphragm a l ive”)Which muscles are involved in respiration?

Diaphragm (most important), external intercosta ls , s ternocleidomastoids , anterior and media l sca leneWhat area of the left lung is most similar to the right middle lobe?

Lingula (technica l ly part of the left upper lobe)Which lobe does foreign body aspiration most commonly affect when supine?

Superior segment of right lower lobe (because right main s tem bronchus i s wider, more ver ca l , and the superior segment os um isposteriorly located)

The bifurcation of the trachea occurs at the level of which vertebral body?Intervertebra l disk T4 to T5

What four structures make up a bronchopulmonary segment?1. Segmenta l bronchus2. Branch of the pulmonary artery3. Branch of the bronchia l artery4. Tributaries of the pulmonary vein

Which syndrome can result from a malignant tumor in the region of the superior pulmonary sulcus?Pancoast syndrome (lower trunk brachia l plexopathy and Horner syndrome)

PHYSIOLOGY

Lung Volumes and Capacities

Name the lung volume defined below:

Expired volume with each normal breathTida l volume (TV)Volume that can be inspired beyond the TVInspi ratory reserve volume (IRV)Volume that can be expired beyond the TVExpiratory reserve volume (ERV)Volume that remains in the lungs after maximal expirationRes idual volume (RV)

Name the lung capacity defined below:TV + IRVInspi ratory capaci ty (IC)ERV + RV

Functional res idual capaci ty (FRC)TV + IRV + ERVVita l capaci ty (VC)Volume which can be forcibly expired in 1 second after maximal inspirationForced expiratory volume (FEV1)Total volume which can be forcibly expiredForced VC (FVC)TV + IRV + ERV + RVTota l lung capaci ty (TLC)

What is the formula for minute ventilation?TV × respiratory rate (breaths per minute)

Lung Compliance and Resistance

What is the relationship between compliance and elasticity?

Compl iance and elastici ty are inversely related.Name four common causes of decreased lung compliance:

1. Pulmonary fibros is2. Atelectas is3. Acute respiratory dis tress syndrome4. Neonata l respi ratory dis tress syndrome

What is a common smoking-related cause of increased lung compliance?Emphysema, normal aging lungs

What is the effect of surfactant on alveolar surface tension and lung compliance?Surfactant decreases surface tens ion to a l low smal l a lveol i to s tay open and increases compl iance.

What is the law of Laplace?Pressure = 2 × surface tens ion/radius . The larger the radius of the vessel/a i rway, the more s table and less l ikely i t i s to col lapse.

What is the relationship between airflow and resistance?Airflow is inversely proportional to a i rway res is tance.

What is the relationship between airway radius and resistance?Res is tance i s inversely proportional to the fourth power of the radius (Poiseui l le law).

For each factor below, state whether airway resistance will be increased or decreased:BronchoconstrictionIncreasedParasympathetic stimulationIncreasedSympathetic stimulationDecreasedHigh lung volumesDecreasedLow lung volumesIncreased

Hemoglobin

For each of the factors below, determine whether the hemoglobin dissociation curve will be shifted to the left or to the right?

Increased PCO2

RightIncreased pHLeftDecreased temperatureLeftIncreased 2,3-bisphosphoglycerate (2,3-BPG) concentrationRightFetal hemoglobinLeftcarbon monoxide poisoningLeftNote: ↑ in any factor (except pH) resul ts in right shi ft.

What is the mnemonic for right shift?CADET face RIGHT (C O2, Acid/Al ti tude, D PG (2,3-D PG), E xercise, T emperature

Which quaternary conformation of Hb has the highest affinity for oxygen?The R (relaxed form)

How does hemoglobin’s affinity for O2compare to that of cO?The affini ty of CO is 200 times as great as that of O2.

What are the signs and symptoms of carbon monoxide toxicity?Headache, nausea/vomiting, loss of consciousness , confus ion

What are the three ways in which cO2 is transported from the tissues to the lungs?

1. As HCO3- in erythrocytes

2. Bound to hemoglobin (carbaminohemoglobin)3. Dissolved in blood

Which enzyme catalyzes the conversion of CO2 into HCO3-?

Carbonic anhydraseWhich protein is the main buffer within erythrocytes?

Deoxyhemoglobin

Acclimatization to High Altitude

For each of the values below, determine how they are affected by high altitude acutely:

VentilationIncreasedPAO2, PaO2

Decreased (due to decreased partia l pressure of atmospheric O2)PACO2, PacO2

Decreased (due to hyperventi lation)Systemic arterial pHIncreasedHemoglobin saturationDecreasedHemoglobin concentrationNo change

For each of the values below, determine how they are affected by high altitude chronically:VentilationIncreasedPAO2, PaO2

Decreased (due to decreased partia l pressure of atmospheric O2)PACO2, PaCO2

Decreased (due to hyperventi lation)Systemic arterial pHIncreased (but closer to normal due to renal excretion of bicarbonate)Hemoglobin saturationDecreasedHemoglobin concentrationIncreased (polycythemia due to increased erythropoietin production)

Lung Zones

Choose the zone or region of the lungs that fits each description below:

Alveolar pressure > arterial pressure > venous pressureZone 1Arterial pressure > alveolar pressure > venous pressureZone 2Arterial pressure > venous pressure > alveolar pressureZone 3V/Q ≈ 3 → wasted ventilationApexV/Q ≈ 0.6 → wasted perfusionBase

What is the effect of hypoxia on pulmonary blood vessels?Vasoconstriction. Note: hypoxia causes vasodi lation in a l l other vascular beds .

What is the quotient of V/Q in a lung where the pulmonary artery is completely occluded (shunt-unventilated blood)?0

What is the quotient of V/Q in complete airway obstruction (physiologic dead space)?∞

Neurologic Regulation of Respiration

Describe the function of each of the following neural structures in the control of respiration:

Medullary respiratory centerDetermines the rhythm of breathing by control l ing inspi ration and expirationPontine apneustic centerStimulates inspi rationCerebral cortexVoluntary control of respi rationLung stretch receptors in bronchial smooth muscleReflexive s lowing of respi ratory frequency (Hering-Breuer reflex)Irritant receptors in airway epithelial cellsIni tiate coughingJuxtacapillary (J) receptors in alveolar wallsCause rapid sha l low breathing in congestive heart fa i lureJoint and muscle receptorsStimulate respiration at the ini tiation of exerciseMedullary chemoreceptorsIncrease breathing rate in response to low pHPeripheral chemoreceptors in the carotid and aortic bodiesIncrease breathing rate in response to low arteria l PO2, low pH, and high PCO2

PATHOLOGY

Pulmonary Edema, ARDS, and Neonatal RDS

Which syndrome is characterized by diffuse alveolar damage, pulmonary edema, and respiratory failure?

Adult respi ratory dis tress syndrome (ARDS)What type of material is seen in the alveoli of a patient in the acute phase of ARDS?

Hyal ine membranes formed by a fibrinous exudate and necrotic cel lular debrisWhat are some of the causes of ARDS?

Shock of any e ology, fat embol ism, gram-nega ve seps is , severe bacteria/vi ra l infec ons , near-drowning, aspi ra on of GI contents ,acute pancreati ti s , heroin overdose, oxygen toxici ty, cytotoxic drugs

What is the most common cause of respiratory failure in the newborn and also of death in premature infants 28 weeks gestational age?Neonata l respi ratory dis tress syndrome (hya l ine membrane disease)

What is the typical clinical presentation of neonatal respiratory distress syndrome?Preterm infants with ini tia l ly normal respi rations fol lowed by cyanos is , tachypnea, and s igns of respi ratory dis tress

What is the pathogenesis of neonatal respiratory distress syndrome?Deficiency of pulmonary surfactant leading to increased surface tens ion

What type of cells produce pulmonary surfactant?Type II pneumocytes

What is the predominant chemical in pulmonary surfactant?Dipalmitoyl phosphatidylchol ine (leci thin)

What is the diagnostic test of choice to judge fetal lung maturity?Amniotic fluid leci thin: sphingomyel in ratio (a ratio of 2:1 impl ies adequate surfactant i s present)

What are the three strongest risk factors for neonatal respiratory distress syndrome?1. Prematuri ty2. Maternal diabetes3. Del ivery by caesarean section

What is the classic pathologic finding in the alveoli of an infant with neonatal respiratory distress syndrome?Intra-a lveolar hya l ine membranes

Infants surviving an initial bout of neonatal respiratory distress syndrome are at risk for which five complications?1. Bronchopulmonary dysplas ia (resul ting in part from oxygen therapy)2. Retinopathy of prematuri ty (resul ting from oxygen therapy)3. Patent ductus arteriosus4. Intraventricular cerebra l hemorrhage5. Necrotizing enterocol i ti s

What medications can you give to prevent neonatal respiratory distress syndrome?Glucocorticoids given to the mother can accelerate feta l lung maturation and reduce the ri sk of neonata l respi ratory dis tress syndrome.

Pulmonary Embolism

What are the most common clinical presentations of PE?

Tachycardia , tachypnea, dyspnea, pleuri tic chest pa in, or hemoptys isWhat is the etiology of 95% of pulmonary emboli?

Dis lodged deep venous thromboses (DVT) from the deep veins of the thigh or pelvisWhat factors favor the development of a DVT?

Virchow triad: (1) s tas is , (2) hypercoagulabi l i ty, (3) endothel ia l damageHow do you diagnose a PE radiographically?

V/Q scan or CT angiogram (1st l ine), pulmonary angiography (gold s tandard)What type of tumors commonly cause a DVT by inducing a hypercoagulable state?

AdenocarcinomasWhat is the most common genetic disease that predisposes to the development of DVT?

Factor V LeidenWhat type of embolism can develop uniquely in a peripartum woman?

Amniotic fluid embol ismFor what type of embolism is a patient with a long bone fracture at risk?

Fat embol i from bone marrow, pulmonary thromboembi lsm due to trauma and s tas isWhat type of infarction results from a PE?

Hemorrhagic infarction (appears as wedge-shaped opaci ty on chest x-ray [CXR])What therapy is indicated for high-risk patients during the workup of PE and for patients diagnosed with PE?

Ful l treatment doses of heparin (unfractionated or low molecular weight)What is used for long-term prophylaxis for patients at risk of developing DVT?

WarfarinWhat is an alternative to warfarin for outpatient DVT prophylaxis?

Subcutaneous heparin (low molecular weight or unfractionated)

Obstructive Lung Disease

What are the most common disease categories causing pulmonary hypertension?

1. Chronic obstructive pulmonary disease (COPD)2. Left-s ided heart disease3. Recurrent PE4. Autoimmune disorders

How is the heart affected by long-standing pulmonary hypertension?Pulmonary hypertens ion causes right ventricular hypertrophy and di latation.

What is the reversal of a left-to-right shunt to a right-to-left shunt due to long-standing pulmonary hypertension?Eisenmenger syndrome

What is the most common cause of COPD?Cigarette smoking

What is the effect of COPD on hematocrit (Hct)?Chronic hypoxia leads →↑ Hct

What is the effect of COPD on FEV1/FVC?↓ FEV1/FVC 0.8

Name the obstructive pulmonary disorder described by the following pathologic features:Smooth muscle and goblet cell hyperplasia; mucus-plugged airways containing Curschmann spirals, eosinophils, and Charcot-Leyden crystalsBronchia l as thmaHyperplasia of bronchial submucosal glands, hypersecretion of mucus, squamous metaplasia or dysplasia of bronchial epitheliumChronic bronchi ti sPermanent airway dilation and scarring; inflammation and necrosis of bronchial walls and alveolar fibrosisBronchiectas isAlveolar enlargement due to alveolar wall destruction; ↓ lung elasticityPulmonary emphysemaDiagnosed clinically by productive cough for > 3 consecutive months for 2+ yearsChronic bronchi ti s

Name the chronic disorder characterized by increased airway reactivity, resulting in paroxysmal bronchial contraction:Bronchia l as thma

Name the two types of bronchial asthma:1. Atopic or immune asthma2. Nonatopic or nonimmune asthma

What type of hypersensitivity response is seen in atopic asthma?Type I hypersens i tivi ty response mediated by IgE and mast cel l s , eos inophi l s , and basophi l s

What other conditions are typically seen in association with atopic asthma?Al lergic rhini ti s , eczema

Name several common triggers of atopic asthma:Dust, pol len, food, and animal dander

Name several triggers of nonatopic asthma:Respiratory tract infection, cold, chemica l i rri tants , exercise

What are the two common presenting features of bronchial asthma?1. Dyspnea2. Wheezing

Which four conditions are commonly associated with bronchiectasis?1. Bronchia l obstruction from tumor or foreign body2. Cystic fibros is (CF)3. Kartagener syndrome4. Necrotizing pneumonia

What are the three common presenting features of bronchiectasis?1. Abundant, copious , often foul -smel l ing sputum2. Chronic cough3. Hemoptys is

What are the components of an acinus of the lungs?Alveol i , a i r ducts , respi ratory bronchioles , terminal bronchioles

Name the type of emphysema described below:Enlargement of bronchioles, typically in the upper lobes and apices (most often associated with smoking)Centriacinar emphysemaDestruction and dilation of entire acinus, typically in the lower lobes; associated with (α1antitrypsin deficiencyPanacinar emphysemaDestruction of the distal acinus occurring typically near the pleura and areas of fibrosis that may cause pneumothoraxParasepta l emphysemaIrregular involvement of the acinus that may be related to inflammatory processes of the lungsIrregular emphysema

How does smoking cause emphysema?Smoke par cles recrui t inflammatory cel l s , promote the release and enhance the ac vi ty of elastase, and inhibi t the ac vi ty of α 1-anti tryps in.

What is the role of (α1antitrypsin deficiency in the development of emphysema?The lack of this protease inhibi tor resul ts in the digestion of the elastin in a lveolar wal l s by elastase.

What is the effect of emphysema on the anteroposterior diameter of the chest and the TLC?Both are increased.

Restrictive Lung Disease

What general category of pulmonary disease is characterized by dyspnea, decreased lung volumes, and decreased compliance?

Restrictive lung diseaseWhat is the effect of restrictive lung disease on FEV1/FVC?

↓ FEV1/FVC≥ 0.8What are the two categories of restrictive lung disease?

1. Pulmonary: inters ti tia l lung disease or pneumoniti s , resul ting in poor lung expans ion2. Extrapulmonary: extrapulmonary disease associated with disorders of the chest wal l , pleura , or respi ratory muscles

What are the common causes of pulmonary restrictive lung disease?Idiopathic, connective ti ssue disease, drug-related, sarcoidos is (a l l → poor lung expansion)

What are the common causes of extrapulmonary restrictive lung disease?Neuromuscular disease (pol io), obes i ty, pleura l disease, and scol ios is (a l l → poor breathing mechanics)

Name the specific type of restrictive lung disease described below:65-year-old (y/o) hay farmer with recent exposure to moldy hay presents with chronic dry cough, chest ghtness; physical examina on (PE): bilateraldiffuse rales; bronchoscopy: interstitial inflammation; bronchioalveolar lavage: lymphocyte and mast cell predominanceHypersens i tivi ty pneumoniti s35-y/o man presents with intermittent hemoptysis and hematuria; W/U demonstrates alveolar hemorrhage and acute glomerulonephritis

Goodpasture syndrome40-y/o with progressive hypoxemia and cor pulmonale; lung biopsy demonstrates chronic inflamma on of the alveolar wall in a pa ern consistent withhoneycomb lung; bronchioalveolar lavage: mild eosinophiliaIdiopathic pulmonary fibros is58-y/o former shipbuilder presents with the insidious onset of dyspnea; transbronchial biopsy demonstrates inters al pulmonary fibrosis, ferruginousbodies; chest CT scan demonstrates pleural effusion and dense pleural fibrocalcific plaquesAsbestos is55-y/o miner (nonsmoker) presents with dyspnea and dry cough; pulmonary func on tests (PFTs) show both obstruc ve and restric ve pa ern; chest x-ray (CXR): hilar lymphadenopathy with eggshell calcificationsSi l i cos is60-y/o male with 100 pack-year history of (h/o) smoking presents with pleuri c chest pain, hemoptysis, and dyspnea; PE: dullness to percussion andabsent breath sounds in the right lower lung fieldPleura l effus ion (secondary to mal ignancy)50-y/o former heavy smoker presents with mul ple lung and rib lesions; biopsy: cells (similar to the Langerhans cells of the skin) containing tennisracket-shaped Birbeck granulesEos inophi l i c granuloma30-y/o black female presents with DOE, fever, arthralgia; PE: iri s, erythema nodosum; W/U: eosinophilia, ↑ serum ACE levels; PFT: restric ve pa ern;CXR: bilateral hilar lymphadenopathy interstitial infiltrates; lymph node biopsy: noncaseating granulomasSarcoidos is

Which typically asymptomatic disorder causes visible black deposits in the lungs of coal workers?Anthracos is

Which three medications are known to commonly cause interstitial lung disease?Bleomycin, methotrexate, and amiodarone

Pulmonary Infections

What infection is characterized by fever > 39°C, chills, cough productive of blood-tinged, purulent sputum, pleuritic pain, hypoxia, and lobar infiltrate on CXR?

Acute pneumoniaWhat infection is characterized by fever 39°C, nonproductive cough, GI upset, and diffuse patchy infiltrates on CXR?

Atypica l pneumoniaName the most common organism(s) associated with the pulmonary infection described below:

Community-acquired acute pneumoniaStreptococcus pneumoniaeCommonly follow a viral respiratory infectionStaphylococcus aureus and Haemophilus influenzaeInterstitial pneumoniaMycoplasma pneumoniae (most common), Chlamydia pneumoniaeFungal pneumonia in an AIDS patientPneumocystis jiroveci (previous ly Pneumocystis carinii)Typical pneumonia in neonateStreptococcus agalactiae (group B s treptococcus)Typical pneumonia in an alcoholic after aspirationKlebsiella pneumoniaeAtypical pneumonia in patient with positive cold agglutinin testMycoplasma pneumoniaeAtypical pneumonia in a neonate with trachomaChlamydia trachomatisAtypical pneumonia in a dairy workerCoxiella burnettiAtypical pneumonia in a rabbit hunterFrancisella tularensisPneumonia in a bird owner with splenomegaly and bradycardiaChlamydia psittaciHospitalized patient with lobar pneumoniaStreptococcus pneumoniae> S. aureusPneumonia in an IV drug userStreptococcus pneumoniae, K. pneumoniae, and S. aureusPneumonia in a patient recovering from viral upper respiratory infectionStaphylococcus aureus, H. influenzaeAtypical pneumonia in a spelunker from the Ohio river valleyHistoplasma capsulatumAtypical pneumonia in a patient from the southwestern United StatesCoccidioides immitisAssociated with spread by inhalation of contaminated water droplets from air conditionersLegionella pneumophila

Name the most common causative pathogen(s) of pneumonia for each age group below:NeonatesGroup B s treptococci , Escherichia coli, ListeriaChildren (6 weeks-18 years)RSV and other vi ruses , M. pneumoniae, C. pneumoniae, S. pneumoniaeAdults (18-40 years old)Mycoplasma pneumoniae, C. pneumoniae, S. pneumoniaeAdults (45-65 years old)Streptococcus pneumoniae, H. influenzae, anaerobes , vi ruses , M. pneumoniaeAdults (> 65 years old)Streptococcus pneumoniae, vi ruses , anaerobes , H. influenzae, gram-negative rods

Name four common complications of lobar pneumonia:1. Abscess formation (especia l ly S. aureus and anaerobes)

2. Empyema or spread of infection to the pleura l cavi ty3. Organization of exudate to form scar ti s sue4. Seps is

What type of pulmonary infection is characterized by localized suppurative necrosis of lung tissue?Lung abscess

Name several bacterial pathogens capable of causing lung abscess:Staphylococcus aureus, aerobic and anaerobic s treptococci , gram-negative baci l l i , and anaerobic ora l flora

Cystic Fibrosis

What is the most common lethal genetic disease in Caucasians?

CFWhat is the mode of inheritance of CF?

Autosomal recess ive (chromosome 7)Which membrane protein is defective in CF?

The cystic fibros is transmembrane conductance regulator (CFTR), a chloride channel proteinWhat is the function of CFTR?

Regulation of sodium, chloride, and bicarbonate transport across the plasma membraneWhat is the most common genetic lesion causing CF?

Deletion of three nucleotides encoding phenyla lanine 508 in the CFTR; AF508 prevents the normal express ion of CFTRDescribe how the ΔF508 mutation prevents normal expression of CFTR:

Altered membrane folding precludes glycosylation and transport to the plasma membrane.What is the end result of the ΔF508 mutation of the CFTR gene?

Complete loss of CFTR in the plasma membraneHow does a defect in the CFTR affect exocrine glands?

Altered CFTR activi ty causes the accumulation of hyperviscid mucus that blocks the secretions of exocrine glands .What tests are used to diagnose CF in infants?

Sweat chloride test, measurement of nasa l potentia l di fference, genotyping (defini tive)How does CF typically present in an infant?

Fai lure to thrive, meconium i leus , mother compla ins chi ld “tastes sa l ty”Describe the effect of CF on each of the following organs:

LungsRecurrent pulmonary infec ons , bronchiesctas is ; ↑ RV and TLC in chronic disease; ↓ FEV 1/FVC in acute exacerba on; pulmonaryhemorrhage may occurPancreasVariable defects in pancreatic exocrine function; may cause pancreatic insufficiency and diabetes mel l i tusIntestinesMucus plugs → smal l bowel obstruction; meconium i leus in some infantsLiverPlugging of bi le cannal icul i → ci rrhos isEpididymis and ductus deferensBi latera l absence of the vas deferensSalivary glandsDucta l di lation; squamous metaplas ia of ducta l epi thel ium and glandular atrophy

Which organisms are commonly responsible for pulmonary infections in CF?Pseudomonas aeruginosa, S. aureus, and Pseudomonas cepacia

What is a common nutritional deficiency in CF?Deficiency of the fat-soluble vi tamins (vi tamins A, D, E, and K)

What type of heart disease is common in patients with CF?Cor pulmonale

Lung Cancer

What is the most common cause of cancer deaths in the United States for both men and women?

Lung cancerWhat is the most common type of malignant tumor in the lungs?

Metastas isWhat are the most common primary lung tumors?

Adenocarcinoma is s l ightly more common than squamous cel l carcinomaName the type(s) of primary lung cancer associated with the following features:

Central locationSquamous cel l and smal l (oat) cel l carcinomasPeripheral locationAdenocarcinoma, large cel l carcinoma, bronchioa lveolar carcinomaDysplasia and carcinoma in situ precede development of this tumorSquamous cel l carcinomaStrongest link to smokingSquamous cel l and smal l (oat) cel l carcinomasLeast linked to smoking, frequently seen in nonsmoking womenBronchioa lveolar carcinomaMost aggressive tumorSmal l (oat) cel l carcinomaAssociated with production of PTH-related peptide, hypercalcemiaSquamous cel l carcinomaAssociated with production of ADH (SIADH) and ACTH (Cushing syndrome)Smal l (oat) cel l carcinomaCarcinoembryonic antigen (CEA) positive

AdenocarcinomaSecretion of 5-hydroxytryptamine (5-HT, serotonin)CarcinoidOat-like, dark blue cellsSmal l (oat) cel l carcinomaTumor cells lining alveolar wallsBronchioa lveolar adenocarcinomaGiant pleomorphic cells, poor prognosis, and high likelihood of cerebral metastasisLarge cel l carcinomaTumor at apex of lung causes Horner syndrome or lower brachial plexopathyPancoast tumorRare pleural tumor is found in patients with a h/o exposure to asbestosMal ignant mesothel iomaMost common cancer in patients with h/o exposure to asbestosLung cancer

OTHER PATHOLOGY

Immotile cilia secondary to a dynein arm defect, associated with situs inversus, infertility, bronchiectasis

Kartagener syndromeCauses stridor, toxic presentation, may be life threatening, thumbprint sign on x-ray, caused by H. influenzae

Acute epigloti tti sCauses stridor, nontoxic presentation, steeple sign on x-ray, caused by parainfluenza virus

Laryngotracheobronchiti s , croup

PHARMACOLOGY

For each of the following drugs, provide:1. The mechanism of action (MOA)2. Indication(s) (IND)3. Significant side effects and unique toxicity (TOX) (if any)

AlbuterolMOA: β2-agonis t → bronchodi lator, short actingIND: as thmaTOX: tachycardia , arrhythmias , tremor, hyperglycemiaInhaled corticosteroids (beclomethasone, fluticasone, triamcinolone)MOA: anti -inflammatory; inhibi ts smooth muscle hyperreactivi tyIND: moderate-to-severe asthmaTOX: oropharyngeal thrush, dysphoniaCromolynMOA: anti -inflammatory; inhibi ts his tamine release from mast cel l sIND: prophylaxis for as thmatic attackTOX: laryngeal edema (very rare)IpratropiumMOA: bronchodi lator; chol inergic antagonis tIND: COPD.TOX: Rare—minor systemic mani festations of antichol inergic effectLeukotriene inhibitors (zileuton, zafirlukast, montelukast)MOA: inhibi ts leukotriene synthes is (zi leuton) or blocks leukotriene receptors (zafi rlukast, montelukast)IND: prophylaxis for as thmatic attackTOX: elevation of l iver enzymesTheophyllineMOA: bronchodi lator; exact mechanism unknownIND: as thmaTOX: seizures and arrhythmias Inhaled a lbuterol as needed

What is the drug of choice for mild asthma?Inhaled a lbuterol as needed

Which three classes of bronchodilators are useful in the management of asthma and COPD?1. Antichol inergic agents (eg, ipratropium)2. β-Agonis ts (eg, a lbuterol )3. Theophyl l ine (much less useful )

What is the drug of choice in an asthmatic requiring daily albuterol use?Inhaled glucocorticoids

Describe how glucocorticoids act on airways to control asthma:Glucocor coids reduce inflamma on and decrease the reac vi ty of a i rways to i rri tants such as cold, cigare e smoke, a l lergens , andexercise.

What type of therapy may be required in patients with daily asthma attacks?Systemic s teroid therapy; usual ly with ora l prednisone or IV methylprednisolone

Name three agents used in the treatment of allergic rhinitis:1. Antihis tamines (eg, diphenhydramine, loratadine, terfenadine)2. α-Agonis t aerosols (eg, phenylephrine)3. Corticosteroid nasa l sprays (eg, beclomethasone, fluticasone, triamcinolone)

Which class of bronchodilators is useful in patients who cannot tolerate β-agonists?Antichol inergic agents (eg, ipratropium)

CHAPTER 7Gastroenterology

EMBRYOLOGY

What portion of the GI tract is derived from the embryonic foregut?

From the intra-abdominal esophagus → pancreas (dis ta l to sphincter of Oddi )What portion of the GI tract is derived from the embryonic midgut?

From the second part of the duodenum → proximal two-thi rds of the colon (at the splenic flexure)What portion of the GI tract is derived from the embryonic hindgut?

From the dis ta l one-thi rd of the colon → rectum (proximal to the pectinate l ine)What GI structures are derived from the embryonic ectoderm?

Oropharynx (anterior two-thi rds of tongue, l ips , parotids , tooth enamel), anus , and rectum (dis ta l to the pectinate l ine)Name the embryonic layer supplied by the following arteries:

Celiac trunkForegutInferior mesenteric arteryHindgutSuperior mesenteric arteryMidgut

What embryonic structure gives rise to the anterior two-thirds of the tongue?Fi rs t branchia l arch

What embryonic structure gives rise to the posterior one-third of the tongue?Third and fourth branchia l arches

What disorder results from failure of fusion of the maxillary and medial nasal processes?Cleft l ip

What disorder results from failure of fusion of the nasal septum, lateral palatine processes, and/or median palatine process?Cleft pa late

What is Meckel diverticulum?Pers is tence of the vi tel l ine duct or yolk s ta lk that may conta in ectopic ssue. Remember: “The rule of 2’s”: 2 in long, 2% of popula on, 2 from i leoceca l va lve, presents within 2 years of l i fe, can conta in 2 types of epi thel ia (gastric, pancreatic).

What primordial embryonic structures give rise to the pancreas?Ventra l bud → pancreatic head, uncinate process , main ductDorsa l bud → everything else (body, ta i l , i s thmus, accessory duct)

What congenital defect is caused by abnormal fusion of the ventral and dorsal buds of the pancreas?Annular pancreas (leads to ring around duodenum and obstruction)

What congenital defect results from failure of migration of neural crest cells, causing an absence of parasympathetic ganglion cells in the distal colon?Hirschsprung (congenita l agangl ionic) megacolon

What congenital defect results from incomplete canaliza on of the bile ducts during development and presents shortly a er birth with clay-colored stoolsand jaundice?

Bi l iary atres ia

ANATOMY

What are the four main layers of the wall of the digestive tract?

1. Mucosa2. Submucosa3. Muscularis4. Serosa

Name the enteric plexus associated with the following descriptions:Located between mucosa and inner layer of smooth muscle in GI tract wallSubmucosa l (Meissner) plexusLocated between inner (circular) and outer (longitudinal) layer of smooth muscle in GI tract wallMyenteric (Auerbach) plexusCoordinates motility along the entire gut wallMyenteric plexus = Moti l i tyControls local secretions, absorption, and blood flowSubmucosal plexus = Secretions

Name the lymphoid tissue found in lamina propria and submucosa of small intestine:Peyer patches

What is the major function of the Peyer patch?Detect antigens → secrete IgA into lumen

In the liver, which zone of the portal acinus contains the highest O2concentration and nutrients?Zone 1

Which zone of the portal acinus contains the lowest O2 concentration and nutrients?Zone 3

Name the three major branches of the abdominal aorta and their approximate vertebral level:1. Cel iac trunk (T12)2. Superior mesenteric artery (L1)3. Inferior mesenteric artery (L3)

What structure provides collateral venous drainage to the superior vena cava (SVC) when there is obstruction of the inferior vena cava (IVC)?Azygous vein

Name the four key sites for portal-systemic shunt and the veins involved:

1. Esophagus (left gastric → esophageal )2. Umbi l icus (paraumbi l ica l → superficia l /inferior epigastric)3. Rectum (superior recta l → middle/inferior recta l )4. Posterior abdominal wal l (col ic → lumbar)

Which two vessels merge to form the portal vein?Splenic (inferior mesenteric vein has a l ready joined the splenic vein) and superior mesenteric veins

What important neural structure traverses the parotid gland?Facia l nerve (CN VII)

Which syndromes may result from reinnervation of the CN VII after Bell palsy or surgical severance of nerve?Frey syndrome (sweating occurs a long with sa l ivation), crocodi le tears syndrome (tearing occurs a long with sa l ivation)

What are the muscles of mastication?Three open: masseter, tempora l i s , and media l pterygoid; one close: latera l pterygoid

What types of muscle are found in the esophagus?Proximal one-thi rd = skeleta l ; dis ta l one-thi rd = smooth; middle one-thi rd = both

What is the function of the pylorus?Acts as muscular sphincter to regulate movement of food out of the s tomach and prevents reflux of duodenal contents

Which blood vessel lies posterior to the first part of the duodenum?Gastroduodenal artery (a concern in cases of posterior perforation due to ulcers )

The common bile duct and main pancreatic duct empty into what portion of the duodenum?Second (descending) part

The common bile duct and pancreatic duct drain into the duodenum through what structure?Hepatopancreatic ampul la (of Vater)

What structure controls the release of bile into the duodenum?Sphincter of Oddi

Which two ducts combine to form the common bile duct?1. Common hepatic2. Cystic duct

What structures make up the porta hepatis?Hepatic artery, porta l vein, common bi le duct

What structure keeps the cystic duct open at all times for bile to flow freely in both directions?Spira l va lve (of Heis ter)

What is the “bare area” of the liver?A portion of the diaphragmatic surface of the l iver that i s devoid of peri toneum

What divides the left and right lobes of the liver?Interlobar fi s sure (an invis ible l ine running from the ga l lbladder to the IVC)

What structure supports the duodenum at the duodenojejunal flexure?Suspensory l igament (of Trei tz)

Name the intestinal structures (duodenum, ileum, jejunum, or colon) associated with each of the following characteristics:Long, finger-shaped villiJejunumIntestinal glands (crypts) and 3 cm luminal diameterI leumNo villi, large crypts, and 6 to 9 cm luminal diameterColonLarge, numerous, plicae circularisJejunumAccounts for the terminal three-fifths of the small intestineI leumContains fatty tags (appendices epiploicae)ColonContains prominent Peyer patchesI leumContains long vasa rectaJejunum

What term is used to describe the three longitudinal bands of smooth muscle in the colon?Teniae col i

What is the name for the wall sacculations in the colon that are separated by the plicae semilunaries?Haustra

What anatomic feature divides the upper and lower anal canal?Pectinate l ine

What muscle holds the rectum in a 90° flexure?Puborecta l i s muscle

Describe the arterial supply, venous drainage, and innervation of internal hemorrhoids:Vasculature = superior recta l artery/vein (dra ins to porta l ci rculation); vi scera l innervation → not pa inful

Describe the arterial supply, venous drainage, and innervation of external hemorrhoids:Vasculature = inferior recta l artery/vein (dra ins to systemic ci rculation via IVC); somatic innervation → painful

What type of muscle is found in the internal anal sphincter?Smooth muscle (under involuntary control via autonomic innervation)

What type of muscle is found in the external anal sphincter?Striated muscle (under voluntary control via the pudendal nerve)

What are the boundaries of Hesselbach triangle?Inferior epigastric artery, inguina l l igament, latera l border of rectus abdominus muscle

Name the types of hernia:Peritoneum protrudes through Hesselbach triangle, medial to the inferior epigastric arteryDirect herniaRetroperitoneal structures to enter the thorax because of defective development of pleuroperitonial membraneDiaphragmatic herniaPeritoneum protrudes through both the internal (deep) and external (superficial) inguinal ringsIndi rect hernia

Stomach herniates upward through the esophageal hiatus of the diaphragmHiata l herniaOccurs in infants as a result of failure of processus vaginalis to closeIndi rect herniaProtrudes below the inguinal ligaments and lateral to the pubic tubercle; more common in femalesFemoral hernia

PHYSIOLOGY

Saliva

Which three glands produce saliva?

1. Parotid2. Submandibular3. Subl ingual

Name three important functions of saliva:1. Protection of denta l hea l th by buffering ora l bacteria l acids2. Digestion (s tarches by a-amylase); triglycerides (TGs) by l ingual l ipase3. Lubrication of food with mucins

What determines the relative composition of salivary contents?Flow rate

How is the regulation of saliva production unique?It i s s timulated by both parasympathetic and sympathetic activi ty.

GI Hormones and Secretions

Name the source of the following GI secretory products:

Intrinsic factor (IF)Parieta l cel l s (s tomach)PepsinogenChief cel l s (s tomach)HistamineMast cel l s (s tomach)Gastric acid (H+)Parieta l cel l s (s tomach)GastrinAntra l G cel l s and duodenumBicarbonateSurface mucosa l cel l s (of s tomach and duodenum)SecretinS cel l s (duodenum)SomatostatinD cel l s (duodenum)Cholecystokinin (CCK)I cel l s (duodenum) and jejunumGastric inhibitory peptide (GIP)Duodenum and jejunum

For each of the following substances, state the factors that regulate its secretion:Gastric acid (H+)↑ By his tamine, acetylchol ine (ACh), gastrin; ↓ by prostaglandin, somatostatin, GIPPepsinogen↑ By vagal s timulation (ACh) and low pHGastrin↑ By smal l pep des and amino acids (AAs) (Phe and Trp = most potent), gastric dis ten on, and vagus (via gastrin-releas ingpeptide[GRP]); ↓ by pH 3.0 and secretinBicarbonate in pancreatic secretions↑ By secretin (potentiated by CCK and vagal input)SecretinAcid (H+ and fatty acids [FAs]) in the duodenumSomatostatin↑ By acid; ↓ by vagusCCK↑ By FAs , AAs , and smal l peptidesGIP↑ By FAs , AAs , and ora l glucose

List the most important functions of the following GI secretions:IFBinds vi tamin B12 for uptake in terminal i leum

Gastric acid (H+)Converts peps inogen to peps in and s teri l i zes chyme. Note: inadequate acid production →↑ risk of Salmonella infectionsPepsinogenDigests proteinGastrin↑ Secretion of IF, HCl , and peps inogen; s timulates gastric moti l i ty and growth of gastric mucosa

BicarbonateNeutra l i zes acid → prevents autodigestionSecretin↑ Pancreatic HCO3

– secretion; ↓ gastric acid secretionSomatostatin↓ H+ and peps inogen secretion, ↓ pancreatic and SI secretions , ↓ ga l lbladder contraction, ↓ release of both insul in and glucagonCCK↑ Gal lbladder contraction, ↑ pancreatic enzyme secretion, ↓ gastric emptyingGIP↑ Insul in (especia l ly in response to ora l glucose); ↓ H+ secretion

Which enzyme in the parietal cell catalyzes the production of H and HCO3- from CO2 and H2O?

Carbonic anhydraseHow is H+ secreted into the lumen of the stomach?

H+/K+ ATPase pumps H out of cel l s and Cl - di ffuses concurrently to mainta in electrica l neutra l i ty forming HCl .Which chemical potentiates the actions of ACh and gastrin in stimulating secretion?

Histamine. (This i s why H receptor blockers are so effective in treating ulcers .)Which prototypical drug blocks the effects of histamine at the level of the parietal cell?

Cimetidine (an H receptor blocker)Why does not atropine block vagally mediated gastrin secretion?

Vagal s timulation of acid production i s independent of ACh. (It i s mediated by GRP.)Which GI hormone is released by the small intestine in response to fats, proteins, and carbohydrates?

GIP

GI Motility

What are the two main divisions of the enteric nervous system?

1. Extrins ic (parasympathetic and sympathetic nervous systems)2. Intrins ic (enteric nervous systems)

Parasympathetic innervation of the GI tract occurs via which nerves and has what general effect?Vagus and pelvic → usual ly s timulatory

Sympathetic innervation of the GI tract occurs via which nerves and has what general effect?Fibers originate in spina l cord and synapse in the cel iac and superior mesenteric gangl ia → usual ly inhibi tory.

What term is used to describe the vasovagal reflex that accomodates for food entering the stomach?Receptive relaxation

What reflex mediates colonic motility in response to food in the stomach?Gastrocol ic reflex

What are the two components of the gastrocolic reflex?1. Fast: mediated by the autonomic nervous system2. Slow: mediated by CCK and gastrin

What two centers of the brain are integral to vomiting?1. Chemoreceptor trigger zone (floor of the fourth ventricle)2. The vomiting center of the medul la

What pacemaker cells in the GI tract regulate the basal rate of gut contraction?Inters ti tia l cel l s of Ca ja l (ICC)

What are the frequencies of ICC’s pacemaker activities?Stomach—3/min, duodenum—12/min, i leum—10/min, colon—3/min

Miscellaneous GI Physiology

What part of the pancreas is responsible for synthesizing and releasing zymogens?

Secretory acinar cel l sWhich two chemicals stimulate the release of zymogens?

1. ACh2. CCK

Which hormone acts on the pancreatic ductal cells to increase mucus and HCO3- secretion?

SecretinName the pancreatic enzymes responsible for the following actions:

Starch digestionα-Amylase (secreted in active form)Protein digestionProteases (eg, tryps in, chymotryps in, elastase, carboxypeptidases—secreted as proenzymes)Fat digestionLipase, phosphol ipase A, col ipase

Which enzyme catalyzes the conversion of trypsinogen into trypsin?Enterokinase (a brush-border enzyme)

Why is the conversion of trypsin an important part of protein digestion?Tryps in converts proenzymes to thei r active forms (including tryps inogen to form a pos i tive-feedback loop)

What is the rate-limiting step in carbohydrate digestion?Production of monosaccharides by ol igosaccharide hydrolases (at brush border)

What are the five major components of bile?1. Water (97%)2. Bi le sa l ts3. Phosphol ipids4. Cholesterol5. Bi l i rubin (BR)

What property of bile salts allows them to solubilize lipids into micelles for absorption?They are amphipathic (conta in both hydrophi l i c and hydrophobic regions).

Where is bile produced and stored?It i s produced continuous ly by hepatocytes and s tored in ga l lbladder.

What is the role of intestinal bacteria in the synthesis of bile acids?They convert primary (1°) bi le acids to secondary (2°) bi le acids .

Which membrane protein is essential for bile acids recirculation?Na + -bi le cotransporter in terminal i leum

Why does ileal resection result in steatorrhea?Lack of bi le acid reci rculation → depletion of the bi le acid pool → impaired fat absorption

What types of carbohydrates can be absorbed?Monosaccharides only (eg, glucose, ga lactose, fructose)

By what mechanism are carbohydrates absorbed?Glucose/ga lactose: Na + -dependent cotransport; fructose: faci l i tated di ffus ion

How are fats absorbed?Lipase breaks triglycerides into glycerol and FAs . Short- and medium-chain FAs undergo pass ive diffus ion; long-chain FAs form micel leswith bi le sa l ts for pass ive di ffus ion.

What metabolites of protein can be absorbed by the GI tract and what type of transport molecules are involved in their absorption?AAs, dipeptides , and tripeptides ; Na +-dependent cotransporters

PATHOLOGY

Nonneoplastic Disorders of the Upper GI Tract

Name the nonneoplastic disorder of the upper GI tract with the following pathologic and clinical features:

Common vesicular circumoral lesion with eosinophilic intranuclear inclusionsHerpes labia l i s (usual ly herpes s implex vi rus [HSV]-1)Connective tissue disorder characterized by xerostomia, keratoconjunctivitis sicca, and autoantibodies to SS-A (Ro) and SS-B (La)Sjögren syndromeEsophageal dysmotility caused by inability of the lower esophageal sphincter to relax (due to loss of ganglion cells in the myenteric plexus)Achalas iaType of hiatal hernia in which the stomach and the cardioesophageal junction slide in and out of the thoraxSl iding (axia l ) herniaPharyngeal outpouching involving >1 layer of the esophageal wall; results in food accumulation and chronic halitosisZenker (pharyngoesophageal ) diverticulumRetching-induced laceration of gastroesophageal (GE) junction resulting in hematemesis and mediastinitis; ↑ incidence in alcoholicsMal lory-Weiss tearComplete rupture of the esophagus (all layers), often due to severe retchingBoerhaave syndromeMay result from Chagas disease causing the loss of myenteric plexus in the esophagus2° achalas iaHyperplasia of gastric surface mucosal cellsMénétrier diseasePostvagotomy, unimpeded passage of hypertonic food to SI → distention and diarrheaDumping syndrome

Neoplastic Disorders of the Upper GI Tract

Name the neoplastic disorder of the upper GI tract described by each of the following statements:

Most common salivary gland tumor; contains a mix of epithelial and mesenchymal elementsPleomorphic adenoma (mixed tumor)Irregular white mucosal patches in the mouth of an AIDS patient as a result of Epstein-Barr virus (EBV)Oral ha i ry leukoplakiaAccounts for 95% of oral cancersSquamous cel l carcinomaBenign salivary tumor containing cystic spaces lined by double-layered eosinophilic epithelium (oncocytes) embedded in dense lymphoid tissueWarthin tumor (papi l lary cystadenoma lymphomatosum)Most common esophageal carcinoma; usually in proximal two-thirds; associated with alcohol and tobacco useSquamous cel l carcinomaIntestinal metaplasia of squamous epithelium in distal esophagus in response to prolonged injury (often due to GERD)Barrett esophagusMucin-producing glandular tumor of the distal one-third of the esophagusAdenocarcinomaInfiltrating carcinoma, causing extensive thickening of stomach wall; “leather-bottle stomach”Lini ti s plastica (s ignet ring carcinoma)Gastric carcinoma that has metastasized to the ovaryKrukenberg tumor

Name six important risk factors for gastric carcinoma:These ↑ your “CHANSE” for gastric cancer:1. C hronic gastri ti s2. H elicobacter pylori infection3. A blood type4. N i trosamines5. S ex (men > 50 years old)

6. E ating habi ts (low-fiber diet)Name six important risk factors for esophageal carcinoma:

“ABCDEF”1. A cha las ia2. B arrett esophagus3. C orros ive esophagi ti s4. D iverticul i5. E sophageal webs6. F ami l ia l

What is the name for metastatic spread of gastric cancer to the supraclavicular node?Virchow node

Pediatric GI Disorders

Name the GI disorder commonly diagnosed in the pediatric population with the following findings:

Difficulty with feeding starting from birth, excessive oral secretions, inability to pass NG tube, no gas in abdomen, early pneumoniaTracheoesophageal fi s tulaNonbilious projectile vomiting, abdominal “olive” in epigastric regionPyloric s tenos isBilious vomiting, “double bubble,” associated with Trisomy 21Duodenal atres iaCommonly occurs at ileocecal junction, most common cause of small bowel obstruction in toddlersIntussuseptionFailure to pass meconium in the first 48 hours of birth, abdominal distention, associated with CFMeconium i leus

Gastritis

Name the type of gastritis associated with the following findings:

Autoimmune disorder with autoantibodies to parietal cells and IF, achlorhydria, pernicious anemia, and agingType A (fundal ) chronic gastri ti s (Remember: the 5 A’s for type A)“Coffee-ground emesis” from mucosal inflammationAcute (eros ive) gastri ti sHelicobacter pylori infectionType B (antra l ) chronic gastri ti s (B = bug)Left shoulder painPerforation of ulcer → i rri tation of left diaphragm

Peptic Ulcer Disease

Name the type of peptic ulcer (gastric or duodenal) disease associated with each of the following findings:

Pain is greater with mealsGastric ulcer (pa in i s G reater with meals ) → weight lossPain decreases with mealsDuodenal ulcer (pa in Decreases with meals ) → weight ga inAlmost 100% associated with H. pylori infectionDuodenal ulcerDue to ↓ mucosal protection against gastric acidGastric ulcerAssociated with nonsteroidal anti-inflammatory drug (NSAID) useGastric ulcerHypertrophy of Brunner glandsDuodenal ulcerBlood type ODuodenal ulcerElevated gastrin levelsDuodenal ulcerDue to ↑ gastric acid secretion and/or ↓ mucosal protectionDuodenal ulcer

Name four common complications of peptic ulcer disease:1. Bleed i ng2. Penetration3. Perforation4. Obstruction

Name a classic complication of posterior duodenal ulcers:Mass ive hemorrhage from eros ion of the gastroduodenal artery

Malabsorption

Name the malabsorption syndrome associated with each of the following pathologic and clinical findings:

Gluten sensitivityCel iac disease (nontropica l sprue)Brush-border enzyme deficiency resulting in bacterial digestion of unabsorbed disaccharide, causing osmotic diarrheaDisaccharidase deficiency (#1 = lactase deficiency)Steatorrhea, weight loss, hyperpigmentation, polyarthritis, fever, and lymphadenopathy in an older, white male; infectious etiology

Whipple disease (caused by Tropheryma whippelii)Increased risk of T-cell lymphoma, GI, and breast malignancyCel iac diseaseAutosomal recessive (AR) defect in chylomicron assembly resul ng in an absence of chylomicrons, very low-density lipoproteins (VLDLs), or low-densitylipoprotein (LDL) in bloodAbeta l ipoproteinemiaFlat, proximal intestinal mucosa with marked villous atrophy; lymphocytes and plasma cells in lamina propriaCel iac diseaseDistinctive periodic acid-Schiff (PAS)-positive macrophages in intestinal mucosaWhipple diseaseAssociated with HLA-DQ2 and HLA-DQ8Cel iac diseaseGram-positive actinomycetesWhipple diseaseAcanthocytes (burr cells) in bloodAbeta l ipoproteinemia

Diverticular Disease

What is the most common cause of painless bleeding from the lower GI tract?

Diverticulos isWhat is the prevalence of diverticulosis in the US population older than age 60?

~50%What part of the colon is most frequently affected by diverticulosis?

Sigmoid colonName the disorder characterized by diverticular inflammation causing left lower quadrant (LLQ) pain, anorexia, nausea, and vomiting:

Diverticul i ti sName four complications of diverticulitis:

1. Perforation2. Peri toni ti s3. Abscess4. Obstruction

What part of the GI tract is most commonly affected by ischemic bowel disease?“Watershed” areas (splenic flexure, rectos igmoid junction)Define intussusception.Telescoping of the intestines resul ting in intestina l obstructionDefine volvulus.Complete twis ting of the bowel around i ts mesenteric base

Where does volvulus most commonly occur?Sigmoid colon (more common in elderly)

Name the inflammatory bowel disease which is often characterized by overgrowth of exotoxin-producing bacteria:Pseudomembranous col i ti s

Which organism is responsible for pseudomembranous colitis?Clostridium difficile

What disorder is characterized by nausea, vomiting, anorexia, leukocytosis, and pain at McBurney point?Appendici ti s

Inflammatory Bowel Disease

Ulcerative colitis or Crohn’s disease?

Pancolitis with crypt abscessesUlcerative col i ti s (UC)Fistulas and fissuresCrohn’s disease BothAssociated with ankylosing spondylitisBothAssociated with sclerosing cholangitisUCAmyloidosisCrohn’s diseaseCan lead to toxic megacolonUCLongitudinal ulcersCrohn’s diseasePunched-out aphthous ulcersCrohn’s diseaseIncreased risk of colorectal carcinomaUC >>> Crohn’s diseaseSkip lesionsCrohn’s diseaseCan involve any portion of the GI tract (usually terminal ileum and colon)Crohn’s disease“String sign” on x-ray (due to bowel wall thickening)Crohn’s diseaseAssociated with pyoderma gangreosumBothTransmural inflammation

Crohn’s diseaseNoncaseating granulomasCrohn’s diseaseCobblestone mucosaCrohn’s disease

Neoplastic Disorders of the Lower GI Tract

What is the most common histologic type of GI lymphomas?

95% are B cel l (MALT omas).What is the most common tumor of the appendix?

Carcinoid tumorWhat type of cells give rise to carcinoid tumors?

Neuroendocrine cel l sWhat substances are secreted from carcinoid tumors?

Serotonin, his tamine, and prostaglandinsWhich carcinoid tumors tend to be more aggressive?

I lea l , gastric, and colonicMetastases to which organ result in carcinoid syndrome?

LiverName five clinical findings of carcinoid syndrome:

1. Vasomotor dys function2. GI hypermoti l i ty3. Bronchoconstriction4. Hepatomegaly5. Right-s ided heart va lve s tenos is

What laboratory test is used in the diagnosis of carcinoid syndrome?5-Hydroxyindoleacetic acid (5-HIAA) in urine

Name the types of neoplastic polyp:Usually benign and pedunculated; most commonTubular adenoma (75%)Highly malignant; sessile tumor > ↓ cm with fingerlike projectionsVi l lous adenomaShares features of both other types of polypsTubulovi l lous adenoma

Name five major risk factors for colon cancer:1. Presence of colonic vi l lous adenomas2. Inflammatory bowel disease3. Low-fiber, high animal fat diet4. Age (> 60)5. Pos i tive fami ly/personal his tory

How does colorectal carcinoma classically present?Left s ide les ions → blood in s tool ; right s ide les ions → anemia (from occul t blood loss )

Where is the most common site for colorectal cancer?Sigmoid colon

Name the autosomal dominant (AD) polyposis syndrome associated with each of the following findings:Colonic polyps, osteomas, and soft tissue tumors; associated with abnormal dentitionGardner syndromeHundreds of colonic polyps; malignant potential ~100%Fami l ia l adenomatous polypos is (FAP)Colonic polyps and CNS tumors; malignant potential ~100%Turcot syndromeDefect in DNA repair → many colonic lesions (especially proximal); malignant potential ~50%Hereditary nonpolypos is colorecta l carcinoma (HNPCC)Benign, hamartomas of GI tract; melanotic pigmentation of hand, mouth, and genitalia; no malignant potential (but ↑ risk of other tumors)Peutz-Jeghers syndrome

What are the three tumor syndromes in which the APC gene is mutated?1. FAP2. Gardner syndrome3. Turcot syndrome

Pancreatitis and Pancreatic Cancer

Name nine causes of acute pancreatitis:

“GET SMASHeD”1. Gal l s tones (major cause)2. Ethanol (major cause)3. Trauma4. Steroids5. Mumps6. Autoimmune disorder7. Scorpion s ting8. Hyperl ipidemia9. Drugs (especia l ly ddI)Other causes : i schemia, infections , pancreatic cancer, and peptic ulcer disease

Name seven possible sequelae of acute pancreatitis:1. Progress ion to chronic pancreati ti s

2. Abscess3. Pseudocyst4. Hypocalcemia5. Foca l fibros is and di ffuse fat necros is6. Acute respiratory dis tress syndrome (ARDS)7. Disseminated intravascular coagulation (DIC)

Name four common laboratory abnormalities in acute pancreatitis:1. ↑ Serum amylase (within 24 hours )2. ↑ Serum l ipase (72-96 hours )3. Hypocalcemia4. Glycosuria

Name five causes of chronic pancreatitis:“ABCCD”1. Al cohol i sm (#1 in adults )2. Bi l iary tract disease3. Cystic fibros is (#1 in kids )4. Ca 2+ (hyperca lcemia)5. Di vi sum (pancreas divisum)

Most pancreatic tumors are found in what region of the pancreas?Two-thi rds of pancreatic tumors are found in the pancreatic head.

What is a common clinical presentation of a mass in the pancreatic head?Painless jaundice caus ing malabsorption and Courvois ier (enlarged, pa lpable) ga l lbladder

What is Trousseau syndrome?Migratory thrombophlebi ti s associated with viscera l cancer, commonly pancreatic adenocarcinoma

What are the two most commonly mutated genes causing pancreatic adenocarcinoma?1. K-ras (> 90% mutated)2. p53 (60%-80% mutated)

What is the prognosis for pancreatic adenocarcinoma?Averages 6 months or less (very aggress ive)

Disorders of Bilirubin Metabolism

What type of hyperbilirubinemia results from cholestasis?

ConjugatedName the hereditary hyperbilirubinemia described in each of the following statements:

Mildly ↓ UDP-glucuronyl transferase; asymptomatic; associated with stress in 6% of peopleGi lbert syndromeAR defect causing ↓ canalicular excretion of BR conjugates → grossly black liver; asymptomaticDubin-Johnson syndromeAR absence UDP-glucuronyl transferase → ↑ unconjugated BR → jaundice, kernicterus, early deathCrigler-Najjar syndrome, type IAR defect causing asymptomatic, conjugated bilirubinemiaRotor syndrome

Liver Disorders

What three pathologic characteristics define cirrhosis?

1. Fibros is2. Nodular regeneration of hepatocytes3. Dis ruption of parenchymal archi tecture

Name four common causes of micronodular cirrhosis:1. Chronic a lcohol i sm (think of a micro brewery)2. Heredi tary hemochromatos is3. 1° bi l iary ci rrhos is4. Wi lson disease (hepatolenticular degeneration

Name four common causes of macronodular cirrhosis:1. Hepati ti s B vi rus (HBV)2. Hepati ti s C vi rus (HCV)3. α1-Anti tryps in deficiency4. Wi lson disease

List the effects of hepatic failure on the following body systems:EyeSclera l i cterusNeurologicComa, hepatic encephalopathy (asterixi s , hyperreflexia , behaviora l changes)SystemicPeriphera l edema, malnutri tionSkinJaundice, pa lmar erythema, spider angiomata, caput medusaeReproductiveTesticular atrophy, gynecomastia , loss of pubic ha i rHematopoieticAnemia, bleeding tendency (↓ coagulation factors ), splenomegalyRenalHepatorenal syndrome (ARF 2° to hypoperfus ion)GIFetor hepaticus , asci tes , esophageal varices , hemorrhoids

Name the liver disorder associated with each of the following findings:Mallory bodiesAlcohol ic hepati ti sOcclusion of IVC or hepa c veins with centrilobular conges on → conges ve liver disease; associated with polycythemia, pregnancy, and hepatocellularcarcinomaBudd-Chiari syndromeObliteration of hepatic vein radicals following bone marrow transplantVeno-occlus ive diseaseCopper deposition in liver, kidneys, brain, and cornea → asterixis, basal ganglia degeneration, dementiaWilson disease (hepatolenticular degeneration)AST:ALT ratio > 1.5Alcohol ic hepati ti sLymphoid aggregates and interface hepatitisChronic hepati ti sMay be incidental finding in young woman taking oral contraceptivesHepatic adenomaSinusoidal hepatic dilations from use of anabolic steroidsPel ios is hepatisHemolysis, elevated LFTs, and low platelets in a pregnant womanHELLP syndrome (hepatic disease of pregnancy)Elevated α-fetoprotein (AFP)Hepatocel lular carcinoma

Name the etiology of cirrhosis associated with each of the following findings:Panacinar emphysemaα1-Anti tryps in deficiencyDecreased ceruloplasminWilson disease (hepatolenticular degeneration)Triad of bronze diabetes, skin pigmentation, and micronodular pigment cirrhosisHemochromatos isAntimitochondrial antibodiesPrimary bi l iary ci rrhos isNutmeg liverCongestive heart fa i lureKayser-Fleischer ringsWilson disease (hepatolenticular degeneration)Micronodular fatty liver; portal hypertension, asterixis, jaundice, and gynecomastiaChronic a lcohol abuse↑ Ferritin, transferrin, and total iron; ↓ total iron-binding capacity (TIBC)Hereditary hemochromatos is . Note: tota l body i ron i s sometimes high enough to trigger meta l detectors .

Name the major risk factors for hepatocellular carcinoma:“WATCH for ABC”Wi l son diseaseα1-AntiTryps inCarcinogens (eg, a flatoxin B1, polyvinyl chloride)Hemochromatos isAl cohol ic ci rrhos isHepati ti s BHepati ti s C

Hepatocellular carcinoma tends to spread through which route?Hematogenous

What is the most common type of malignancy in the liver?Metastas is

Which primary tumors tend to metastasize to the liver?Colon > Stomach > Pancreas > Breast > Lung (Remember: “Cancer Sometimes Penetrates Benign Li ver!”)

Name the hepatobiliary disorder associated with the following:Strawberry gallbladderCholesterolos isAssociated with Opisthorchis sinenis (liver fluke) infection in AsiaCholangiocarcinomaTumor arising at the junction of the left and right hepatic ductsKlatskin tumorInflammation of the bile duct; commonly due to Escherichia coli infectionAscending cholangi ti s

Disorders of the Gallbladder

What is Courvoisier law?

An enlarged, pa lpable ga l lbladder with jaundice i s l ikely the resul t of an underlying mal ignancy (o en in head of the pancreas) and notfrom a s tone in the common duct (because ga l lbladder i s typica l ly too scarred from infection).

What is Charcot triad?Spiking fevers with chi l l s , jaundice, and right upper quadrant (RUQ) pa in (bi l iary col ic); s trongly suggests cholangi ti s

What are the risk factors for cholelithiasis?4 F’s: Ferti le, Fat, Forty-year-old Female

What is the most common type of gallstone?Mixed s tone (components of pigment and cholesterol s tones)

Name six complications of cholelithiasis:1. Bi l iary col ic and common bi le duct obstruction2. Cholecysti ti s (acute or chronic)

3. Ascending cholangi ti s4. Acute pancreati ti s5. Gal l s tone i leus6. Mal ignancy (adenocarcinoma)

Viral Hepatitis

Name the hepatitis virus (viruses) associated with the following features:

Fecal-oral transmissionHepati ti s A vi rus (HAV)Water-borne transmissionHepati ti s E vi rus (HEV)Infection may lead to a carrier stateHBV, HCV, and hepati ti s D vi rus (HDV, del ta agent)Defective virus requiring HBsAg as its envelopeHDV (del ta agent)Sexual, parenteral, and transplacental transmissionHBV, HCV, HDVDNA hepadnavirusHBVHigh mortality rate in pregnant womenHEVAssociated with IV drug useHCVLong incubation (~3 months)HBVIncreased risk of hepatocellular carcinomaHBV, HCVImmune globulin vaccine availableHAV, HBV (and HDV)Short incubation (~2 weeks)HAV

Name the hepatitis serologic marker described below:Antigen found on surface of HBV; continued presence suggests carrier stateHBsAgAntigen associated with core of HBVHBcAgAntigen in the HBV core that indicates transmissibilityHBeAgAntibody suggesting low HBV transmissibilityHBeAbActs as a marker for HBV infection during the “window” period (acute phase)IgM-HBcAbProvides immunity to HBVHBsAb

What is the “window” period of a hepatitis infection?Period during acute infection when HBsAg has become undetectable, but HBsAb has not yet appeared

Infectious Diarrhea

Name six infectious causes of bloody diarrhea:

1. Salmonella2. Shigella3. Campylobacter jejuni4. Enteroinvas ive and enterohemorrhagic E. coli5. Yersinia enterocolitica6. Entamoeba histolytica

Name the diarrhea-causing organism associated with the following statements:Most common cause of diarrhea in infantsRotavi rusTen to twelve loose, bloody, and mucous diarrhea stools per day due to ingestion of cystsEntamoeba histolyticaComma-shaped organisms causing rice-water stoolsVibrio choleraSecond to rotavirus as a cause of gastroenteritis in kidsCal icivi ruses (Norwalk-l ike and Sapporo-l ike)Bloody diarrhea; very low ID50 (small numbers of organisms can cause disease); nonmotileShigellaUsually transmitted from pet fecesYersinia enterocoliticaMotile; lactose nonfermenter; causes bloody diarrheaSalmonellaComma- or s-shaped organisms causing bloody diarrhea; grows at 42°CCampylobacter jejuniWatery diarrhea with extensive fluid loss in AIDS patientCryptosporidiumFoul-smelling diarrhea after returning from a camping trip

Giardia lambliaWatery diarrhea caused by antibiotic-induced suppression of colonic floraClostridium difficile

Name the organism responsible for food poisoning from the following items:Reheated riceBacillus cereusReheated meat dishesClostridium perfringensImproperly canned foodClostridium botulinum

Contaminated seafood or raw oystersVibrio parahaemolyticus and Vibrio vulnificus

Meats, mayonnaise, custardsStaphylococcus aureus. Note: s tarts and ends quickly

Undercooked beef productsEscherichia coli O157:H7

Raw poultry, milk, eggs, and meatSalmonella

PHARMACOLOGY

For each of the following drugs, provide:1. The mechanism of action (MOA)2. Indication(s) (IND)3. Significant side effects and unique toxicity (TOX) (if any)

Omeprazole, pantoprazole, lansoprazoleMOA: Irrevers ibly inhibi ts H+/K+-ATPase in parieta l cel l s (proton pump inhibi tor)IND: Peptic ulcer disease, Zol l inger-El l i son syndrome, eros ive esophagi ti sTOX: Liver toxici ty, pancreati ti s , agranulocytos is (rare)Cimetidine, ranitidine, famotidine, nizatidineMOA: H2 receptor antagonis t in parieta l cel lIND: GE reflux diseaseTOX: Endocrine effects (gynecomas a—primari ly caused by cime dine, ga lactorrhea, ↓ sperm count), potent inhibi tor of P-450, ↓ renalcreatinine clearanceMisoprostolMOA: PGE1 ana log →↑ secretion of gastric mucus and bicarbonateIND: Prophylaxis for NSAID-induced peptic ulcer disease (PUD); maintenance of a patent ductus arteriosus (PDA)TOX: Aborti facientAluminum sucrose sulfate (sucralfate)MOA: Polymerizes in s tomach → binds to injured ti ssue, forming a protective coating over ulcers (Note: requires an acidic envi ronment)IND: Peptic ulcer diseaseTOX: Osteodystrophy, osteomalacia (mainly in pts with renal fa i lure)Ondansetron, granisetronMOA: 5-HT3 receptor antagonis tIND: Postoperative nausea/vomiting, chemotherapy-induced emes isTOX: Headache, diarrheaMeclizineMOA: H1 receptor antagonis tIND: Motion s icknessTOX: TeratogenicLoperamide, diphenoxylateMOA: Antimoti l i ty agents (meperidine analogs )IND: DiarrheaTOX: Toxic megacolon in kids (loperamide)MetoclopramideMOA: Stimulates ACh release →↑ upper GI moti l i tyIND: Nausea, i leusTOX: Sedation, diarrhea, and extrapyramidal symptoms (EPS) (especia l ly in kids )LactuloseMOA: Osmotic laxativeIND: Hyperammonemia, colonic lavage precolonoscopyTOX: Dehydration with overuse

What is “triple therapy” for peptic ulcer disease?Proton pump inhibi tor (PPi ), bismuth sa l icylate, and two of the fol lowing an bio cs : metronidazole, amoxici l l in, clari thromycin, ortetracycl ine

What is the most important approach to healing peptic ulcers?Eradicating H. pylori

How long does proton pump inhibition last?48 hours

Name two uses for bismuth subsalicylate:1. Helicobacter pylori infection2. Traveler diarrhea

What two weak bases that are used as antacids have minimal systemic absorption?1. MgOH2

2. AlOH3 (combined to make mi lk of magnes ia)For each of the following antacids list the toxicity/toxicities:

Antacids

Hypokalemia, a l ter absorption of other drugsSodium bicarbonateSystemic a lka l inizationAluminum hydroxideConstipationMagnesium hydroxideDiarrhea

Name the therapy of choice for each of the following disorders:Hereditary hemochromatosisRepeated phlebotomy and deferoxamineWilson diseaseChelation therapy (with penici l lamine)Acute cholecystitis from gallstonesCholecystectomy

CHAPTER 8Reproduction and Endocrinology

EMBRYOLOGY

During embryogenesis, when is male/female phenotypic differentiation complete?

By week 20Which gene determines phenotypic differentiation?

Sry gene (encodes for a protein ca l led testi s -determining factor [TDF])Which cells secrete Mullerian-inhibiting factor (MIF) during fetal development?

Sertol i cel l sWhat factors are required to direct the indifferent embryo into a male phenotype?

TDF, MIF, testosterone, and dihydrotestosterone (DHT)In what structure, within the testes, does spermatogenesis occur?

Semini ferous tubulesWhat molecule is the primary (1°) source of energy for sperm?

FructoseGive the number of chromosomes and amount of DNA found in each of the following cell types:

Spermatogonia46,2N1° spermatocyte/oocyte46,4NSecondary (2°) spermatocyte/oocyte23,2NSpermatid/ovum23,1N

What stage is each cell type halted in and when is the stage finally completed?1° oocyteProphase I, jus t prior to ovulation2° oocyteMetaphase II , at ferti l i zation

Name the structure of the mature sperm associated with each of the following statements:Derived from the Golgi apparatusAcrosomal capContains mitochondriaMiddle piece (neck)Derived from one of the centriolesFlagel lum (ta i l )

Name the congenital disorder that results from each of the following aberrations:Complete lack of fusion of paramesonephric ductsDouble uterus with double vaginaPartial fusion of the paramesonephric ductsBicornate uterusTestes fail to descend into the scrotumCryptorchidism (bi latera l → steri l i ty)Patency of processus vaginalisIf smal l → hydrocele; i f large → congenita l inguina l herniaFailure of urethral folds to closeHypospadias (peni le urethra opens on inferior/ventra l s ide)Faulty positioning of genital tubercleEpispadias (peni le urethra opens on superior/dorsa l s ide)

Name the embryonic structure that gives rise to each of the following tissues:Thyroid glandThyroid diverticulumThymus, inferior parathyroidsThird branchia l pouchSuperior parathyroids and the parafollicular “C” cellsFourth branchia l pouch (fourth i s superior to thi rd)Chromaffin cells of adrenal medullaNeura l crest cel l sAdrenal cortexMesoderm

What is the most common site for ectopic thyroid tissue and why?The tongue; during development, the thyroid migrates caudal ly from the level of the developing tongue.

Which syndrome is caused by a failure of the development of the third and fourth pharyngeal pouches?DiGeorge syndrome

Describe the clinical features of DiGeorge syndrome:“CATCH 22”Cardiac abnormal i ties (eg, tetra logy of Fa l lot, interrupted aortic arch)Abnormal faciesThymic aplas ia/hypoplas ia and T-cel l deficiencyCleft pa lateHypocalcemia → tetany Chromosome 22q11

ANATOMY

Name the five types of endocrine cells found in the pars distalis of the anterior pituitary:

1. Somatotrophs (~50%)2. Mammotrophs3. Thyrotrophs4. Corticotrophs5. Gonadotrophs

Where is antidiuretic hormone (ADH) primarily produced?Supraoptic nucleus of hypothalamus

Where is oxytocin primarily produced?Paraventricular nucleus of hypothalamus

What part of the pituitary stores and releases ADH and oxytocin?Neurohypophys is (posterior pi tui tary)

Where is the hypophyseal portal system located?Adenohypophys is (in the pars tubera l i s )

What is the 1° arterial supply to the gonads?Testicular/ovarian arteries (di rectly from abdominal aorta branches)

What is the 1° venous drainage of the gonads?Left testicular/ovarian vein → left renal vein; right testicular/ovarian vein → inferior vena cava (IVC)

Which lymph nodes filter lymph from the gonads?Deep lumbar/para-aortic nodes

Which lymph nodes filter lymph from the scrotum?Superficia l inguina l nodes

What are the two pathways for venous drainage from the prostate gland?1. Prostatic venous plexus → internal i l iac veins → IVC2. Prostatic venous plexus → vertebra l venous plexus → crania l dura l s inuses

Describe the type of innervation for each phase of the male sexual response cycle:ErectionParasympathetic nerves (Point)EmissionSympathetic nerves (Shoot)EjaculationViscera l autonomic and somatic nerves

Name the structure(s) contained in the following uterine ligaments:Transverse cervical (cardinal) ligamentUterine vesselsSuspensory ligament of ovariesOvarian vessels , lymphatics , autonomic nervesBroad ligamentRound l igaments of the uterus , ovarian l igament, ureters , uterine tubes , and uterine vessels

What is the anatomic relationship of the ureter and the uterine artery?Ureter l ies posterior and inferior to uterine artery. “Water (ureter) under the bridge (uterine artery)”

What is the normal position of the uterus?Anteverted (uterus to cervix) and anteflexed (cervix to vagina)

What is the landmark for a pudendal nerve block?Ischia l spine

Name the part of the fallopian tube described below:Opens into the peritoneal cavityInfundibulumSite of fertilizationAmpul laOpens into the uterine cavityIntramuralMajority of the length of the tubeIs thmus

Describe the venous drainage of the adrenal glands.Right adrenal vein → IVC; left adrenal vein → left renal vein

What type of nerves synapse in the adrenal medulla?Pregangl ionic sympathetic (through splanchnic nerves)

In the adrenal medulla, onto what cells do the preganglionic sympathetic nerves synapse?Chromaffin cel l s → secrete catecholamines

PHYSIOLOGY

Second Messengers

Name the second messenger mechanism used by each of the f ollowing hormones:

Hypothalamic hormones— gonadotropin-releasing hormone (GnRH), thyrotropin-releasing hormone (TRH), growth hormone-releasing hormone (GHRH),oxytocin, ADH (Vj receptor)Inos i tol triphosphate (IP3)Hypothalamic hormones— corticotropin-releasing hormone and ADH (V2receptor)cAMPAnterior pituitary hormones— luteinizing hormone (LH), follicle-s mula ng hormone (FSH), adrenocor cotropic hormone (ACTH), thyroid-s mula ng

hormone (TSH)Cycl ic adenos ine monophosphate (cAMP)Growth hormone (GH)Tyros ine kinase activationInsulinTyros ine kinase activationMelanocyte-stimulating hormone (MSH)cAMPAngiotensin IIIP3

CalcitonincAMPGlucagoncAMPParathyroid hormone (PTH)cAMPInsulin-like growth factor (IGF)-lTyros ine kinase activationAtrial natriuretic peptide (ANP)Cycl ic guanos ine monophosphate (cGMP)Nitric oxidecGMP

Steroid Hormones

How do steroid hormones circulate in the body?

They are bound to speci fic globul in carrier proteins (which ↑ solubi l i ty and del ivery to target organs).How are the cellular effects of steroids mediated?

Steroids pass through the cel l membrane, bind cytoplasmic receptors , and trans locate to the nucleus where they influence geneexpress ion.

Adrenal Glands

Name the three layers of the adrenal cortex and their major secretory products:

GFR (from outs ide to ins ide), ”the deeper you go, the sweeter i t gets : sa l t, sugar, sex”1. Zona Glomerulosa— minera locorticoids2. Zona Fasciculata—glucocorticoids3. Zona Reticulari s—andogrens

How does the secretion of glucocorticoids vary throughout the day?Highest before waking and lowest at midnight

By what mechanism does ACTH increase steroid hormone synthesis?Stimulates cholesterol desmolase

What is the effect of glucocorticoids on ACTH and cortisol secretion?Potent glucocorticoids inhibi t ACTH and corti sol secretion (a negative feedback mechanism).

What test is used to evaluate the response of the hypothalamic-pituitary axis to glucocorticoids?Dexamethasone suppress ion test

Name four general actions of cortisol:1. Stimulates gluconeogenes is (↑ protein catabol i sm and l ipolys is )2. Anti -inflammatory effects (block arachidonic acid pathway)3. Acute exposure enhances immune response; chronic exposure suppresses immune response (inhibi ts interleukin [ILJ-2 production)4. Mainta ins vascular respons iveness to catecholamines

Name three specific actions of aldosterone:1. ↑ Renal Na + reabsorption2. ↑ Renal K+ secretion3. ↑ Renal H+ secretion

Sex Hormones

What is the function of LH in male reproduction?

LH acts on Leydig cel l s → stimulates cholesterol desmolase → testosteroneWhat is the function of FSH in male reproduction?

FSH acts on Sertol i cel l s → spermatogenes is and secretion of inhibin (inhibi ts FSH)How does testosterone regulate LH secretion?

Acts to ↓ release of GnRH from hypothalamus and to ↓ release of LH from adenohypophys isWhich enzyme is required to convert testosterone to its active form, DHT?

5-α-ReductaseWhich enzyme catalyzes conversion of testosterone to estrogen?

AromataseName three androgens (in order of decreasing potency) and where they are produced:

1. DHT (prostate, periphera l ti s sues)2. Testosterone (testes , adrenals )3. Androstenedione (adrenals )

Name five important functions of testosterone:1. Sexual di fferentiation during development2. 2° Sexual development, growth spurt, and fus ion of epiphyseal plates

3. ↑ Libido4. Anabol ic effects5. Spermatogenes is maintenance

What is the average age of menarche?Between 11 and 14 years (in the United States )

How does GnRH release change during puberty?Pulsati le release of GnRH begins and i t upregulates i ts own receptors in the adenohypophys is .

What is the function of LH in female reproduction?LH acts on theca cel l s → stimulates cholesterol desmolase → testosterone

What is the function of FSH in female reproduction?FSH acts on granulosa cel l s → stimulates aromatase → estrogen

What happens to LH:FSH ratio during puberty?LH >> FSH

Where is estrogen made?Blood (via aromatase), ovaries (estradiol ), placenta (estriol ), and testes

What is the relative potency of the major estrogens?Estradiol > estrone > estriol

Which estrogen is an indicator of fetal well-being?Estriol (↑ 1000 × in pregnancy)

Where is progesterone produced?Corpus luteum, adrenal cortex, placenta, and testes

Decide whether each of the following are characteristics of estrogen, progesterone, or both:Development of genitaliaEstrogenGrowth of follicleEstrogenProliferation of endometriumEstrogenMaintains endometriumProgesteroneMaintains pregnancyBothProduces thick cervical mucusProgesteroneHepatic synthesis of transport proteinsEstrogen↓ Myometrial excitabilityProgesteroneDevelopment of breastsBoth↑ Body temperatureProgesteroneSpiral artery developmentProgesteroneLH surgeEstrogenTypical female fat distributionEstrogenUterine smooth muscle relaxation (prevents contractions)ProgesteroneDecrease → endometrial sloughingProgesteronePubertal developmentEstrogen

What are the two main phases of the menstrual cycle and when do they occur?1. Fol l i cular phase (days 1-14)2. Lutea l phase (days 14-28)

Classify the following as characteristics of the f ollicular or luteal phase:Graafian follicle maturesFol l i cular phaseCorpus luteum develops causing the release of estrogen and progesteroneLutea l phase (early)Basal body temperature increasesLutea l phaseOocyte progresses from meiosis I to meiosis IIFol l i cular phaseEndometrial glands grow → spiral arteriesLutea l phaseFSH and LH levels suppressedFol l i cular phaseEndometrial proliferationFol l i cular phaseProgesterone peaksLutea l phaseMensesFol l i cular phase(early)

When does ovulation occur?14 days before menses (regardless of cycle length)

What changes in cervical mucus occur during ovulation?

Cervica l mucus i s thinnest and most penetrable during ovulation.What endocrine change induces ovulation?

Estradiol burst at end of fol l i cular phase causes a pos i tive feedback effect, resul ting in a surge of LH.Which hormone prevents lactation during pregnancy?

High estrogen and progesterone block the effects of prolactin on the breast.Where is hCG made and what is its function?

Syncy otrophoblast of placenta produces hCG to mainta in corpus luteum so i t can produce progesterone during the first trimester un lplacenta can produce progesterone during the second and thi rd trimester.

Name three scenarios in which hCG is elevated:1. Pregnancy (in urine 10 days after ferti l i zation)2. Hydatidi form moles3. Choriocarcinoma

hCG peaks at what gestational age?Gestational week 9

What happens to estrogen and progesterone just prior to delivery?Both increase throughout; near term estrogen:progesterone ratio increases .

What is the average age of menopause?51 (tends to occur earl ier in smokers )

Name four hormonal changes characteristic of menopause:1. ↓ Estrogen2. ↑↑ FSH3. ↑ LH4. ↑ GnRH

Name four clinical findings characteristic of menopause:Menopause wreaks ”HAVOC” on your body1. Hot flashes2. Atrophy of Vagina3. Osteoporos is4. Coronary artery disease ri sk increases

Anterior Pituitary

Name the hormones produced by the anterior pituitary:

”FLAT PiG” plus melanotropin (MSH)FSHLHACTHTSHProlactinGH

Name four compounds derived from proopiomelanocortin (POMC):1. ACTH2. MSH3. β-Lipotropin4. β-Endorphin

Which hormones share a common α-subunit?T.S.H. and TSH = The SexHormones (LH, FSH, hCG) and TSH

Which glycoprotein subunit determines hormone specificity?β-Subunit

How is GH regulated?↑ By GHRH, s leep, s tress , exercise, hypoglycemia, and puberty; ↓ by somatomedins , somatostatin, obes i ty, pregnancy, and hyperglycemia

Name four direct actions of GH (somatotropin):1. ↓ Glucose uptake into cel l s (diabetogenic)2. ↑ Lipolys is3. ↑ Protein synthes is in muscle4. ↑ Production of somatomedins (IGF)

Name three effects of IGF on growth:1. ↑ Linear growth (puberta l growth spurt)2. ↑ Lean body mass3. ↑ Organ s ize

Which clinical syndrome results from GH hypersecretion before puberty?Increased l inear growth (gigantism)

Which clinical syndrome results from GH hypersecretion after puberty?Acromegaly (may include glucose intolerance)

Name four functions of prolactin:1. Stimulates mi lk production2. Inhibi ts ovulation3. Stimulates breast development (with estrogen)4. Inhibi ts spermatogenes is

How is prolactin regulated?↑ By estrogen (pregnancy), breas eeding, s leep, s tress , TRH, dopamine (DA) antagonis ts ; ↓ by DA and DA agonis ts , somatosta n, andprolactin

What are the effects of excess prolactin?Galactorrhea, ↓ l ibido, and fa i lure to ovulate

How does prolactin inhibit ovulation?Inhibi ts GnRH synthes is and release

Posterior Pituitary

Which hormones are released from the posterior pituitary and what are their function?

Oxytocin—milk ejection and uterine contraction; vasopress in (ADH)— ↑ H2O reabsorption in kidneysHow is the release of oxytocin regulated?

↑ By suckl ing, di lation of the cervix, and orgasmHow is the release of ADH regulated?

↑ By high serum osmolari ty, volume contrac on, pa in, nausea, nico ne, opiates ; ↓ by low serum osmolari ty, ethyl a lcohol , ANP, and a-agonis ts

Thyroid

Which cells are responsible for thyroid hormone synthesis?

Fol l i cular cel l sThyroid hormones are synthesized from which two molecules?

1. Iodide (I -)2. Tyros ine

Which enzyme catalyzes the oxidation of I- to I2?Thyroid peroxidase

What is the relative proportion of T4:T3 released into the bloodstream?T4 = 90%, T3 = 10%

Which thyroid hormone has the greatest biological activity?T3 >> T4 (T4 converted into T3 by l iver and kidneys)

Which protein produced by and used entirely within the thyroid is used to produce and store T4 and T3?Thyroglobul in

Which carrier protein is necessary for delivery of thyroid hormone to the tissues?Thyroxine-binding globul in (TBG)

How is thyroid hormone production regulated?↑ By TRH and TSH; T3 down-regulates TRH receptors in AP →↓ TSH

List four key functions of thyroid hormone:Remember the 4 B’s of T3

1. Bra in maturation2. Bone and carti lage growth (synergis tic with GH)3. β-Agonis t effects (↑ cardiac output [CO], heart rate [HR], s troke volume [SV])4. ↑ BMR (by ↑ Na +/K+ ATPase →↑ O2 consumption)

Calcium Regulation

What is meant by a ”negative Ca 2+balance?”

Intestina l Ca 2+ absorption i s less than Ca 2+ excretion.Which is the most important hormone in the regulation of Ca2+ levels and where is it made?

PTH is synthes ized and secreted by the chief cel l s of the parathyroid gland.What is the major stimulus for PTH secretion?

↓ Serum Ca 2+ (a l so mi ldly ↓ Mg2+

Which hormone is stimulated in response to ↑ PTH levels and ↓ serum Ca2+?Vitamin D (1,25-(OH)2

- choleca lci ferol i s active form.)Which enzyme in the kidney catalyzes the activation of vitamin D?

1α-HydroxylaseWhat factors increase 1α-hydroxylase activity?

↓ Serum Ca 2+, ↑ PTH, ↓ serum phosphateWhat clinical syndrome results from vitamin D deficiency?

Kids → rickets ; adul ts → osteomalaciaWhich hormone is released in response to ↑ serum Ca2+ and where is it made?

Calci tonin i s synthes ized and secreted by the parafol l i cular ”C” cel l s of the thyroid gland.What is the effect of calcitonin on bones?

Blocks PTH-mediated resorption of bonesList the effects of PTH on each of the following organs:

Kidneys↓ Phosphate reabsorption (by ↑ urinary cAMP), ↑ Ca 2+ reabsorption in dis ta l convoluted tubule of kidneysIntestines↑ Ca 2+ absorption (by s timulating 1α-hydroxylase to activate vi tamin D)Bone↑ Resorption (brings Ca 2+ and phosphate into extracel lular fluid [ECF])

List the effects of vitamin D on each of the f ollowing organs:Kidneys↑ Ca 2+ and phosphate reabsorptionIntestines↑ Ca 2+ and phosphate absorptionBone↑ Resorption

Pancreas

Name the three major cell types in the pancreatic islets of Langerhans, their location, and function:

1. Alpha (outer rim) → glucagons2. Beta (centra l rim of i s let) → insul in3. Delta (intermixed) → somatostatin (SS) and gastrin

List three major actions of glucagon on the liver and adipose tissue:1. ↑ Glycogenolys is and gluconeogenes is2. ↑ Lipolys is and ketoacid production3. ↑ Urea production

List five major functions of insulin:1. ↓ Blood glucose2. ↑ Fat depos i tion (-1l ipolys is )3. ↓ Blood amino acids4. ↓ Blood K+

5. ↑ Protein synthes isWhat is the 2° structure of insulin?

An α-chain and a (β-chain, joined by two disul fide bridgesWhat is C-peptide and why is it important?

A connec ng pep de removed from proinsul in, packaged and secreted with insul in; serves as a useful monitor of (β-cel l func on andexogenous insul in adminis tration

What are some factors that ↑ insulin secretion?↑ Blood glucose (major), ↑ A As , ↑ FAs , GH, corti sol , glucagon, ACh

How does glucose trigger insulin release?Binds to GLUT-2 receptor on (3-cel l s → closes K+ channels → cel l depolarization → opens Ca 2+ channels → insul in secretion

Which drugs mimic the action of glucose on (β-islet cells?Sul fonylurea drugs

What is the effect of starvation and obesity on insulin receptor expression?↑ In s tarvation; ↓ in obes i ty

PATHOLOGY

Male Reproductive System

Name the male genitourinary disease characterized by each of the following statements:

Intractable, painful erection; associated with venous thrombosis, trazodone, and sickle cell diseasePriapismInflammation of the glans associated with poor hygieneBalani ti sBent penis due to acquired fibrous tissue formationPeyronie diseaseTwisting of the testicular vasculature; may be spontaneous or the result of traumaTors ionCollection of serous fluid in the tunica vaginalisHydrocelePalpable, ”bag of worms” dilation of multiple veins of the pampinif orm venous plexus of the spermatic cordVaricoceleAcute purulent urethritis caused by gram-negative diplococciGonorrheaFever, chills, and dysuria; tender, boggy prostate; > 10 WBCs per high-power field on examination of prostatic secretionsAcute bacteria l prostati ti s

What are the possible sequelae of cryptorchidism?5 to 10 × ↑ risk of germ cel l tumors (GCTs), atrophy, s teri l i ty, and inguina l hernias

What is the treatment for cryptorchidism?Orchiopexy. Note: ↓ ri sk of s teri l i ty, but no ↓ risk of mal ignancy

What are the most common etiologies of orchitis?Mumps vi rus (1 week postparotidi ti s )

Name the most likely organism(s) responsible for epididymitis in the following groups:Pediatric patientsGram-negative rodsSexually active, 35-year-old (y/o)Neisseria gonorrhoeae, Clamydia trachomatisOlder menEscherichia coli, Pseudomonassp.

Name the penile neoplasm associated with each of the following:Human papillomavirus (HPV) types 6 and 11Condyloma acuminateSingle, grayish plaque on shaft/scrotum; associated with ↑ risk of visceral malignancyBowen diseasePremalignant, multiple wart-like, reddish lesions with HPV type 16 viral sequenceBowenoid papulos isHPV types 16,18, 31, and 33Squamous cel l carcinoma of the penisSingle erythematous plaque representing carcinoma in situ of the penis, often on glans

Erythroplas ia of QueyratWhat category of testicular tumors accounts for ~95% of all cases and has a peak incidence of 15- to 34-y/o?

GCTsName the testicular tumor associated with each of the following:

Malignant, painless enlargement of testis; most common GCT, radiosensitiveSeminomaMalignant, chemosensitive GCTNonseminoma GCTMalignant, painful GCT that has peak incidence in childhood; ↑ α-fetoprotein (AFP)Yolk sac tumor (endodermal s inus tumor)Malignant GCT made of 2+ embryonic layers and multiple tissue types; more common in kidsTeratomaMalignant, aggressive GCT with > 1 neoplastic patternMixed GCTBenign, androgen-producing stromal tumor with intracytoplasmic Reinke crystalsLeydig cel l tumor (inters ti tia l )Malignant, hemorrhagic tumor arising from trophoblastic cells; ↑ β-hCGChoriocarcinomaMost common testicular cancer in older menTesticular lymphoma

Benign prostatic nodular hyperplasia or prostatic carcinoma?Commonly affects peripheral zoneProstatic carcinomaCaused by age-related increase in DHT, testosterone, and estrogenNodular hyperplas iaAssociated with bladder distention and urinary tract infections (UTIs)Nodular hyperplas iaEnlarged, firm, nodular prostate on digital rectal examination (DRE)Prostatic carcinomaCommonly presents with nocturia and hesitancyNodular hyperplas iaPrimarily affects central zone Primarily affects corpora amylaceaNodular hyperplas ia↑ Total prostate-specific antigen (PSA), with ↓ fraction of free PSAProstatic carcinoma↑ Total PSA, with proportionate ↑ in fraction of free PSANodular hyperplas ia

Hypermethylation of which gene is commonly associated with prostatic carcinoma?GSTP1

What do the following findings suggest in a patient with prostatic carcinoma?↑ Prostatic acid phosphataseCapsule of the prostate has been penetrated.↑ Alkaline phosphataseOsteoblastic les ions from bony metastas is

Female Reproductive Pathology

Name the gynecologic infectious disorder characterized by each of the following features:

Clue cells in pap smear; ⊕ “whiff test”Bacteria l vaginos is (eg, Gardnerella vaginitis)Thick, white discharge with vulvovaginal pruritis; most common form of vaginitisCandidias isFever, vomiting, diarrhea with desquamating rash; caused by exotoxins from Staphylococcus aureusassociated with tampon useToxic shock syndromeSoft, painful ulcerative lesion caused by Haemophilus ducreyiChancroidFirm, painless chancre caused by a spirochete Treponema pallidum1° syphi l i sSmall papule/ulcer associated with lymphadenopathy and caused by C. trachomatisserotypes LI, L2, or L3Lymphogranuloma venereumDonovan bodies on biopsyGranuloma inguina leMost common STD; frequent cause of pelvic inflammatory disease (PID) (though often asymptomatic); associated with Reiter syndromeChlamydia l cervici ti s (types D-K)Sexually transmitted infection often associated with extragenital manifestations (eg, proctitis, arthritis, and neonatal conjunctivitis)GonorrheaSTD resulting in benign venereal warts caused by HPV types 6 and 11Condyloma acuminatumPainful vesicles/ulcers; cytologic evidence of multinuclear giant cells with viral inclusionsHerpes geni ta l i s (most often HSV type 2)STD caused by flagellated protozoan; #2 cause of vaginitisTrichomonias isCommonly caused by C. trachomatis or N. gonorrhoeae;findings may include cervical mo on tenderness, salpingi s, endometri s, or tubo-ovarianabscessPID—at ↑ risk for ectopic pregnancy and inferti l i ty

Name the female reproductive disorder associated with each of the following features:Triad of 2° amenorrhea, obesity, and hirsutism, ↑ testosterone, ↑ LH, ↓ FSHPolycystic ovary (Stein-Leventhal ) syndrome

Menstrual disorder associated with excessive bleeding during or between menstrual periodsDysfunctional uterine bleedingCyclic pain and bleeding from proliferation of ectopic endometrial tissueEndometrios isIslands of endometrium found in the myometrium that may cause the uterus to grow to two to four times its normal sizeAdenomyos isChocolate cystsEndometrios is (in ovaries )

Name the tumor of the vulva or vagina associated with each of the following statements:Eczematous lesion; biopsy shows large cells in epidermis with marginal clearingPaget disease of vulva . Note: not a lways associated with underlying adenocarcinomaRare, malignant tumor of vagina associated with maternal use of diethylstilbesterol (DBS) during pregnancyClear cel l adenocarcinomaAssociated with HPV types 16,18, 31, 33, and 45; #1 malignancy of vulva; ↑ in older womenSquamous cel l carcinoma of vulva“Bunch of grapes” protruding from vagina; usually in girls 5-y/o, desmin positiveSarcoma botryoidesAccounts for 95% of neoplasms of vagina; usually from extension of cervical cancerSquamous cel l carcinoma of vagina

Name the uterine tumor associated with each of the following statements:Neoplastic changes in the endometrium occurring at squamocolumnar junction; associated with HPV infectionCervica l intraepi thel ia l neoplas ia (CIN)Invasive carcinoma evolving from CINSquamous cel l carcinoma of the cervixVery common, benign, estrogen-sensitive smooth muscle tumor of the uterus; usually 20- to 40-y/oLeiomyoma (fibroid)Most common gynecologic malignancy; associated with prolonged estrogen exposure; usually 55- to 65-y/oEndometria l carcinomaHighly aggressive, bulky tumor with areas of necrosis; arises de novo; ↑ incidence in blacksLeiomyosarcoma

What HPV types are commonly associated with a high risk of squamous cell carcinoma?Types 16,18, 31, 33

What are the HPV viral proteins associated with squamous cell carcinoma?E6 and E7

What role do HPV viral proteins play in the development of invasive cervical carcinoma?Proteins E6 and E7 bind to and inactivate gene products of p53and Kb,respectively.

What are the distinguishing histopathologic features of carcinoma in situ (CIN 3)?Atypica l changes extending through enti re thickness of the epi thel ium

Name four factors associated with increased risk for invasive cervical carcinoma:1. Early sexual activi ty2. Multiple sex partners3. ↓ Socioeconomic s tatus4. Cigarette smoking

What is the most effective screening tool for cervical cancer?Routine pap smears

Name five factors that predispose to endometrial carcinoma:1. Nul l ipari ty2. Obes i ty3. Diabetes4. Unopposed estrogen exposure (eg, es trogen-producing tumors , hormone replacement therapy [HRT])5. Tamoxi fen

How does endometrial carcinoma typically present?Postmenopausa l vagina l bleeding

Name the ovarian cyst associated with each of the following statements:Distention of unruptured graaf ian follicle; may be associated with ↑ estrogen endometrial hyperplasiaFol l i cular cystHemorrhage into persistent corpus luteum; menstrual irregularityCorpus luteum cystDue to gonadotropin stimulation; often bilateral/multiple; associated with choriocarcinoma and molesTheca-lutein cyst

Name the ovarian tumor associated with each of the following statements:Malignant tumor of epithelial origin; two-thirds are bilateral; psammoma bodiesSerous cystadenocarcinomaBenign adenoma; frequently bilaterally; lined with fallopian tubelike epitheliumSerous cystadenomaMalignant GCT that is homologous to seminoma and may occur in childhoodDysgerminomaMalignant tumor of epithelial origin; can rupture and cause pseudomyxoma peritoneiMucinous cystadenocarcinomaBenign multilocular cyst lined by mucus-secreting epitheliumMucinous cystadenomaBenign GCT with elements from multiple embryonic layers; most common GCTMature teratoma (dermoid cyst)Benign tumor; cells resembling bladder transitional epitheliumBrenner tumorGCT with Schiller-Duval bodies; ↑ AFP

Yolk sac (endodermal s inus) tumorGCT associated with struma ovarii (mature thyroid tissue)Teratoma (monodermal )

Malignant, aggressive tumor arising from syncitiotrophoblastic cells; ↑ serum β-hCGChoriocarcinomaStromal tumor associated with Meigs syndrome (ascites, hydrothorax)Thecoma-fibromaBenign, estrogen-secreting tumor; Call-Exner bodiesGranulosa-theca tumorStromal tumor secreting androgens and causing virilizationSertol i -Leydig cel l tumorMetastatic tumor from gastric adenocarcinoma; signet ring cellsKrukenberg tumor

Most ovarian carcinomas are associated with an increase in what serologic marker?CA-125

What two genes are associated with a predisposition to ovarian cancer?1. BRCA12. HNPCC

What is the most common type of ovarian neoplasm in women older than 20 years?Epithel ia l cel l neoplasms (~75% of a l l ovarian cancers )

What is the most common type of ovarian neoplasm in women younger than 20 years?Germ cel l neoplasms

Breast Disorders

Name the breast disease associated with each of the following statements:

Benign, firm, painless, rubbery mass; most common tumor 25-y/oFibroadenomaInflammatory lesion caused by S. aureus;often occurs during nursingAcute masti ti s”Blue-domed” cysts; usually bilateral; breast tenderness during menstruationFibrocystic changeBenign tumor of lactiferous ducts; most common cause of serous or bloody discharge in females 35-y/oIntraducta l papi l lomaLarge, malignant form of fibroadenomaCystosarcoma phyl lodesInvasive, malignant tumor; cells arranged in linear fashion; bloody discharge; often bilateralInvas ive lobular carcinomaMalignant, firm mass with cells in glands; most common carcinoma of breastInvas ive ducta l carcinomaEczematous lesion of nipple or areola containing large cells with marginal clearingsPaget disease of the breast

Name six risk factors for breast cancer:1. Age > 45-y/o2. Early menarche and late menopause (↑ span of reproductive period)3. Fami ly his tory of (1° relative with his tory of [h/o]) premenopausa l breast cancer4. Personal h/o breast cancer5. Inheri ted mutation (eg, HER-2/neu oncogene)6. Obes i ty and high animal fat dietNote: ri sk i s not increased by fibroadenoma or nonhyperplastic cysts .

Which disease is almost always associated with Paget disease of the breast?Ducta l carcinoma in s i tu

In which quadrant of the breast are most cancers located?~50% found in upper, outer quadrant

What is the significance of estrogen/progesterone receptors on breast cancer?Presence of estrogen and progesterone receptors reflects good prognos is (because tumor wi l l l i kely respond to hormonal therapy).

What is the single most important prognostic factor in breast cancer?Lymph node involvement (metastatic spread)

What two tumor suppressor genes are associated with a genetic predisposition for breast cancer?1. BRCA12. BRCA2

What is the general function of the BRCA1 and BRCA2 genes?DNA repair

Trastuzumab (Herceptin) treatment of breast cancers is directed at the protein product of which overexpressed gene?HER2/neu

Pregnancy

Name the disorder of pregnancy associated with each of the following statements:

An ovum without DNA → “honeycombed uterus” and “cluster of grapes” appearance; formed from intrauterine prolifera on of trophoblasts and cys cswelling of chorionic villi; ↑ serum β-hCGHydatidi form moleTear in placental membranes → sudden peripartal respiratory distress → shock → deathAmniotic fluid embol ismPlacental attachment directly to myometrium → impaired separation and massive bleeding at deliveryPlacenta accretaPlacental attachment to the lower uterine segment, extending to or obstructing the inner cervical os; painless bleeding in any trimesterPlacenta previaPremature detachment of a normally situated placenta; painful bleeding often in third trimesterAbruptio placentae

Hydatidiform mole with 46, XX genotype and markedly elevated β-hCG; no embryo; paternal chromosomesComplete moleHydatidiform mole with triploid genotype and ↑ β-hCG; fetal parts may be present; (2 sperm + 1 egg)Incomplete moleMalignant, aggressive tumor that may arise from mole, ectopic, or normal pregnancyGestational choriocarcinomaToxemia of pregnancy with triad of HTN, edema, and proteinureaPreeclamps iaAssociation with preeclampsia; can lead to death by cerebral hemorrhage or ARDSHemolys is , Elevated LFTs ,Low Platelets (HELLP syndrome)Preeclampsia plus convulsions; DIC may be presentEclamps ia

What are some causes of polyhydramnios (> 1.5-2 L of amniotic fluid)?Maternal diabetes , esophageal/duodenal atres ia , anencephaly, Down syndrome

What are some causes of oligohydramnios (0.5 L of amniotic fluid) and what sequence can result?Genitourinary obstruction (esp. posterior urethra l va lves in boys), bi latera l renal agenes is ; can resul t in Potter syndrome/sequence

Name the six clinically important and dangerous infections of pregnancy:“ToRCHeS”1. Toxoplasma2. Rubel la3. CMV4. and 5. HSV and HIV6. Syphi l i s

Pituitary Disorders

Name the pituitary disorder associated with each of the following statements:

Most common pituitary adenomaProlactinoma (prolactin-secreting adenoma)Deficiency of GnRH → lack of 2° sexual characteristics; associated with anosmiaKal lmann syndromePolyuria, polydipsia, hypernatremia from ↓ ADH; associated with a pituitary or hypothalamic injuryCentra l/neurogenic diabetes ins ipidusPanhypopituitarism 2° to brain tumor, ischemia, or traumaSimmonds disease. Note: > 75% of cel l s must be destroyed before cl inica l ly evident.Hypopituitarism caused by postpartum pituitary necrosisSheehan syndromeSomatotrophic adenoma causing excess GH and IGF-1Acromegaly (adults )/gigantism (kids )Pituitary hypersecretion of ADH→ hyponatremia, ↓ urine output, mental status changesSyndrome of inappropriate antidiuretic hormone (SIADH)

What are the most important hormones to replace in Sheehan syndrome or pituitary apoplexy?Cortisol and thyroid hormones

Name the endocrine/renal disorder associated with the following statements:↓ sNa, ↑ Uosm with Uosm > SosmSyndrome of inappropriate ADH (SIADH)High-normal sNa, ↑ Sosm, ↓ Uosm but no change in Uosm with H2O deprivation testDiabetes ins ipidus (DI)Low-normal sNa, ↓ Uosm but ↑ Uosm toward normal w/H2O deprivation testPrimary polydips ia

What are the two common presentations of a pituitary tumor?1. Mass effect (bi tempora l hemianopia , crania l nerve [CN] pa ls ies )2. Endocrine effects (amenorrhea, ga lactorrhea, hyperthyroidism, ↓ l ibido)

Thyroid

What are the symptoms and physical findings of hypothyroidism?

Cold intolerance, ↓ HR, hypertens ion, hypercholesterolemia, pericardia l effus ion, periorbi ta l myxedema, hypoac ve deep tendonreflexes , coarse, dry skin; ha i r loss , weight ga in, constipation, amenorrhea, ↓ pi tch of voice, depress ion

What are symptoms and physical findings of hyperthyroidism?Heat intolerance, hypertens ion, ↑ CO, ↑ HR, pa lpi ta ons , cardiomegaly (long-term), s taring gaze, l id lag, ↑ sympathe c ac vi ty, finetremor, warm, mois t, and flushed skin; fine ha i r; Graves disease → pre bia l myxedema, weight loss despi te hyperphagia , ↑ mo l i ty,menstrua l abnormal i ties , osteoporos is , anxiety

Name four laboratory findings common in hyperthyroidism:1. ↓ TSH (in 1°)2. ↑ free T4

3. ↑ tota l T4

4. ↑ T3 uptakeName four laboratory findings common in hypothyroidism:

1. ↑ TSH (very sens i tive for 1°)2. ↓ free T4

3. ↓ tota l T4

4. ↓ T3 uptakeName the thyroid disorder associated with each of the following statements:

Child with coarse facial features, short stature, mental retardation, and umbilical hernia

Congenita l hypothyroidism (cretinism)Goiter occurring with high frequency in iodine-deficient areasEndemic goi terPainless enlargement of thyroid of autoimmune etiology; Hiirthle cells and germinal centers; hypothyroidHashimoto thyroidi ti sTriad of diffuse thyroid hyperplasia, ophthalmopathy, dermopathy; hyperthyroidGraves diseaseNormal thyroid replaced by fibrous tissue; hypothyroidRiedel thyroidi ti sPostviral, painful inflammation of thyroid; associated with HLA-B35Subacute (granulomatous , de Querva in) thyroidi ti sPainless goiter that may occur in the postpartum period; lymphocytic infiltrates without germinal centersSubacute lymphocytic (pa inless ) thyroidi ti sThryoid-stimulating immunoglobluin (TSI) and TSH-receptor antibody (AB)Graves diseaseThyroid peroxidase antibodyHashimoto diseaseExtreme thyroid enlargement (> 2 kg) causing mass effects; most patients euthyroidMultinodular goi terMost common thyroid carcinomaPapi l lary carcinomaCalcitonin-secreting tumor with amyloid depositsMedul lary carcinomaBiopsy shows “Orphan Annie” nuclei, “fingerlike” projections, and psammoma bodiesPapi l lary carcinomaCarcinoma presenting as a single nodule with uniform folliclesFol l i cular carcinomaAggressive carcinoma of older patients with pleomorphic cells; dismal prognosisAnaplastic (undi fferentiated) carcinomaCarcinoma associated with Hashimoto thyroiditisLymphoma

What type of nodules is more likely to be benign: hot or cold?Hot

Which gene is associated with medullary carcinoma of the thyroid?RET

What is the function of the two genes associated with papillary carcinoma of the thyroid?Tyros ine kinase receptors1. RET2. NTRK1

Name the multiple endocrine neoplasia (MEN) syndrome characterized by the following features:Pheochromocytoma, thyroid medullary carcinoma, and parathyroid adenomasMEN 2 (Sipple syndrome)Tumors of the pituitary, pancreatic islet cells, and parathyroid glandMEN 1 (Wermer syndrome)Mutation of RET oncogene on chromosome lOqMEN 2Tumors in MEN 2 plus tall, thin habitus, prominent lips, and ganglioneuromas of the tongue and eyelidsMEN 3 (MEN 2b)Mutation of MEN 1 gene on chromosome 11 qMEN1Autosomal-dominant (AD) inheritanceAl l MEN syndromes

Parathyroid Disorders

Name the parathyroid disorder associated with each of the following statements:

Caused by chronic renal failure or ↓ vitamin D2° hyperparathyroidismMost commonly due to parathyroid adenomas1° hyperparathyroidismEtiologies include congenital gland absence, surgically induced, and autoimmune destructionHypoparathyroidismDue to autonomous hormone-secreting adenoma, often occurs after correction of chronic renal failure3° hyperparathyroidismAutosomal-recessive (AR) end-organ resistance to PTH → short stature and short third/fourth metacarpalsPseudohypoparathyroidism

What four systems are primarily targeted by hyperparathyroidism?1. Pa inful bones : ostei ti s fibrosa cystica , osteoporos is2. Renal s tones : nephrol i thias is , nephroca lcinos is3. Abdominal groans : constipation, peptic ulcer disease (PUD), pancreati ti s4. Psychic moans : depress ion, lethargy, seizures

Up to 20% of parathyroid adenomas are commonly associated with altered activity of which gene?PRAD1

Adrenal Disorders

Name the four etiologies for hypercorticism (Gushing syndrome):

1. Exogenous glucocorticoids (most common overa l l )2. Pi tui tary ACTH hypersecretion (eg, adenoma)3. Hypersecretion of corti sol (eg, adrenal hyperplas ia)4. Paraneoplastic (ectopic) ACTH secretion from a tumor

What is the most common cause of endogenous hypercortisolism?Cushing syndrome (pi tui tary adenoma)

What finding specific to an ACTH-producing pituitary adenoma differentiates it from adrenal cortisol-producing and ectopic ACTH-producing tumors?↓Cortisol level a fter high-dose dexamethasone suppress ion test

In most cases of congenital adrenal hyperplasia, deficiency in which enzyme is associated with defec ve conversion of progesterone to 11-deoxycorticosterone?

21-HydroxylaseName nine clinical findings of Gushing syndrome:

1. Hyperglycemia (insul in res is tance)2. Vi ri l i zation and menstrua l i rregulari ties in women3. Moon facies4. Trunca l obes i ty5. Buffa lo hump6. Skin changes (thinning, s triae)7. Osteoporos is8. Immune suppress ion9. Proximal muscle weakness

Name the adrenal disorder associated with each of the following statements:Aldosterone-secreting adenoma causing HTN and hypokalemic, metabolic alkalosisConn syndrome(1° hypera ldosteronism)Endotoxin-mediated massive adrenal hemorrhageWaterhouse-Friderichsen syndrome (N. meningitidis)Deficiency of aldosterone and cortisol due to adrenal atrophy or destruction1° chronic adrenocortica l insufficiency (Addison disease)Hypothalemic pituitary axis (HPA) disturbance causing failure of ACTH secretion2° adrenocortica l insufficiencyBilateral hyperplasia of zona glomerulosa caused by stimulation of renin-angiotensin-aldosterone (RAA) system2° hypera ldosteronismResults from rapid steroid withdrawal or sudden ↑ in glucocorticoid requirements1° acute adrenocortica l insufficiency (adrenal cri s i s )Chromaffin cell tumor usually in adults; results in episodic hyperadrenergic symptomsPheochromocytomaMalignant, “small blue cell” tumor of medulla in kids associated with N-myc oncogene amplificationNeuroblastoma

How are 1° and 2° adrenocortical insufficiencies differentiated?Hyperpigmentation i s absent in 2° adrenocortica l insufficiency (POMC is not increased).

Measurement of what substance can differentiate between 1° and 2° hyperaldosteronism?Renin (↑ in 2° hypera ldosteronism)

What is the drug of choice for hyperaldosteronism?Spironolactone (a ldosterone antagonis t)

What is the “rule of 10’s” for pheochromocytomas?10% mal ignant10% bi latera l10% extra-adrenal10% pediatric10% fami l ia l10% ca lci fied

Besides MEN, name three familial syndromes associated with pheochromocytomas:1. Sturge-Weber syndrome2. von Reckl inghausen syndrome3. von Hippel -Lindau syndrome

What substances are secreted from pheochromocytomas and how are they detected?Epinephrine and norepinephrine; detected by ↑ urinary secre on of catecholamines and their metabol i tes (metanephrine, VMA, and soforth)

Adrenogenital Syndromes

Name the enzyme deficiency responsible for each of the following androgenital syndromes:

↓ Sex hormones, ↓ cortisol, ↑ mineralocorticoids; HTN, hypokalemia, phenotypic female without maturation17α-Hydroxylase deficiency↓ Cortisol, aldosterone, and corticosterone; ↑ sex hormones; virilization, HTNl lβ-Hydroxylase deficiency

↓ Cortisol, aldosterone, and corticosterone; ↑ sex hormones; virilization, hypotension, hyperkalemia, “salt wasting”21-Hydroxylase deficiency

What is the phenotypic result of androgen insensitivity syndrome (testicular feminization)?“Hairless woman”; XY with undescended testes but female/ambiguous geni ta l ia

List the three syndromes associated with 21-hydroxylase deficiency:1. Sa l t-wasting adrenogenita l i sm2. Simple vi ri l i zation adrenogenita l i sm (ambigui ty and adrenal hyperplas ia)3. Asymptomatic

Diabetes

Describe the acute presentation of type 1 diabetes niellitus (DM).

Polydips ia , polyuria , polyphagia , weight loss , and diabetic ketoacidos is (DKA) i f extremeWhat causes hyperglycemia in type 1 DM?

Lack of insul in from decreased β-cel l massWhat is the proposed mechanism of islet cell destruction in type 1 DM?

Environmenta l triggering of autoimmunity to i s let β-cel l sWhat is the theorized cause of type 2 DM?

Obes i ty increases insul in res is tance and causes derangement of β-cel l insul in secretion.What are three ways to diagnose DM?

1. Fasting serum glucose > 126 (100-126 = impaired fasting glucose)2. Ora l glucose tolerance test > 200 (148-200 = impaired glucose tolerance)3. Symptoms + random glucose > 200

What is HbA1c and what is it used for?Percent of glycosylated hemoglobin in blood; correlates with glycemic control over the las t 90 to 120 days

Type 1 or type 2 DM?Relatively common in the US populationType 2Younger average age of onsetType 1Strong, polygenic predispositionType 2Associated with obesityType 2Insulin treatment required from the time of diagnosisType 1Associated with HLA-DR3 and -DR4Type 1May present with hyperosmolar comaType 2May present initially as DKAType 1Amyloid deposition in the isletsType 2

What two cleavage products of proinsulin are stored in the granules of β-islet cells?1. Insul in2. C-peptide

Which membrane protein mediates glucose uptake by β-islet cells?GLUT-2

Increased cytoplasmic concentration of which ion stimulates the secretion of insulin?Calcium

By what mechanism is the cytoplasmic concentration of calcium increased in β-cells following uptake of glucose?Decreased ac va on of adenos ine triphosphate (ATP)-sens i ve potass ium channels leads to membrane depolariza on, resul ng in aninflux of extracel lular ca lcium via a vol tage-dependent ca lcium channel .

Glucose uptake in which organ is independent of insulin?The bra in

What signal transduction pathway mediates the mitogenic effects of insulin binding?MAP-kinase pathway

Ac va on of the PI-3K signal transduc on pathway by insulin results in the transloca on of what molecule to the surface of myocytes in order to transportglucose into myocytes?

GLUT-4List four potential effects of advanced glycation end products (AGEs) in type 1 diabetes:

1. Cross -l inking of col lagen2. Accumulation of extracel lular matrix proteins res is tant to proteolytic degradation3. Generation of reactive oxygen species4. NF-κB activation

What is the most common cause of death in diabetics?Myocardia l infarction (MI) due to accelerated atheroscleros is

What is the most common morphologic change in the microvasculature of a diabetic?Diffuse thickening of the basement membrane

Describe the long-term effect(s) of DM on each of the f ollowing organ systems:Cardiovascular (large vessels)Atheroscleros is → cerebrovascular accident (CVA), MI, periphera l vascular disease (PVD)Renal/urinaryGlomerular: glomeruloscleros is , proteinuriaVascular: arterioscleros is → HTN, chronic renal fa i lureInfectious: UTIs , pyelonephri ti s , necrotizing papi l l i ti sNervousMotor and sensory periphera l neuropathy, autonomic degenerationEyeRetinopathy, cataract formationSkinXanthomas, cutaneous infections , poor wound heal ing, fungal infections

Name four causes of 2° DM:1. Pancreatic disease (eg, hemochromatos is , pancreati ti s , pancreatic carcinoma)2. Pregnancy (gestational diabetes )3. Gushing syndrome4. Other endocrine disorders (eg, acromegaly, glucagonoma, hyperthyroidism)

Pancreatic Endocrine Tumors

Name the islet cell tumor associated with each of the following statements:

Most common islet cell tumorInsul inoma (β-cel l tumor)2° DM, necrolytic migratory erythemaGlucagonoma (α-cel l tumor)Associated with Zollinger-Ellison syndromeGastrinomaAssociated with watery diarrhea, hypokalemia, and achlorhydria (WDHA) syndromeVIPoma2° DM, cholelithiasis, steatorrheaSomatostatinoma (δ-cel l tumor)Clinically characterized by Whipple triadInsul inoma (β-cel l tumor)

Name the clinical findings of Whipple triad:Hypoglycemia, concurrent centra l nervous system (CNS) dys function, and reversa l of symptoms with glucose

What is Zollinger-Ellison syndrome?Hypersecretion of gastric HC1, recurrent PUD, and hypergastinemia

PHARMACOLOGY

Pituitary

For each of the following drugs, provide:

1. The mechanism of action (MOA)2. Indication(s) (IND)3. Significant side effects and uniquetoxicity (TOX) (if any)OctreotideMOA: somatostatin analogIND: acromegaly, secretory diarrhea from VIPoma, carcinoid symptoms, high-output fi s tulasLeuprolideMOA: GnRH analog. Note: given pulsati le = agonis t; given continuous = antagonis tIND: continuous →prostate cancer, endometrios is , uterine fibroids . Pulsatile → inferti l i tyTOX: antiandrogenic effects , nausea, vomitingOxytocinMOA: ↑ uterine contractionIND: induce/reinforce labor; control uterine hemorrhageTOX: uterine rupture, hypertens ive cri s i sDesmopressinMOA: vasopress in analogIND: centra l diabetes ins ipidus , nocturnal enures isTOX: HTN, overhydration, coronary constriction

Thyroid and Parathyroid

For each of the f ollowing drugs, provide:1. The mechanism of action (MOA)2. Indication(s) (IND)3. Significant side effects and uniquetoxicity (TOX) (if any)

Levothyroxine (T4)MOA: synthetic T4 → converted to T3

IND: hypothyroidism (maintenance replacement)TOX: nervousness , pa lpi tations , ↑ HR, heat intolerancePropylthiouracil/methimazoleMOA: blocks thyroid peroxidase, propyl thiouraci l a l so ↓ periphera l convers ion of T4 → T3

IND: hyperthyroidism (propyl thiouraci l okay in pregnancy)TOX: rash, agranulocytos is (rare), +ANCA vascul i ti s , hepatotoxici ty, lupus l ike syndromeRisedronateMOA: bi sphophonate (inhibi ts osteoclastic function)IND: os teoporos is , Paget disease, metastatic bone cancer, hyperparathyroidismTOX: GI upset, esophagi tsCalcitoninMOA: ↓ bone resorption, ↓ serum Ca 2+and phosphateIND: acute hyperca lcemia, Paget disease, osteoporos is

Which other drug is used to treat the symptoms of hyperthyroidism?Propanolol (β-blocker)

Diabetes

Sulfonylureas (eg, tolbutamide, glyburide, chlorpropamide)

MOA: pancreas , closes K+ channels in (β-cel l membrane → depolarization → ↑ Ca 2+ influx → insul in release

IND: type 2 diabetes (not used in type 1 because i t requires some res idual (β-cel l activi ty)TOX: hypoglycemia, weight ga in

Metformin (Glucophage)MOA: l i ver, ↓ gluconeogenes is ; ↑ glycolys is ; ↓ postprandia l glucoseIND: newly diagnosed diabetic (no i s let cel l function required)TOX: potentia l ly l i fe-threatening lactic acidos is , ↓ vi tamin B12 absorption, contra indicated in renal insufficiency

Glitazones (eg, rosiglitazone, pioglitazone)MOA: ↑ target cel l response to insul inIND: type 2 diabetes (monotherapy or combination)TOX: hepatotoxici ty (trogl i tazone), upper respiratory infec on (URI), weight ga in, edema, conges ve heart fa i lure, and anemia(ros igl i tazone)

α-Glucosidase inhibitor (eg, acarbose)MOA: inhibi ts α-glucos idase at brush border → ↑ glucose absorptionIND: type 2 diabetesTOX: GI upset (flatulence, diarrhea)

InsulinMOA: tyros ine kinase activi ty →↑ glycogen and protein synthes is , triglyceride (TG) s torage, and K+ uptakeIND: type 1 and refractory type 2 diabetes ; l i fe-threatening hyperka lemiaTOX: hypoglycemia and rare hypersens i tivi ty reaction

For each of the following types of insulin, state the peak and duration of action:Insulin lisproPeak = 30 to 60 minutes ; duration = 3 to 4 hoursNPH insulinPeak = 8 to 12 hours ; duration = 18 to 24 hoursLente insulinPeak = 8 to 12 hours ; duration = 18 to 24 hoursUltralente insulinPeak = 8 to 16 hours ; duration = 18 to 28 hoursInsulin glarginePeak = none (peakless ); duration 20 to 24+ hours

Adrenal Gland

Describe the mechanism of action of corticosteroids:

Cor costeroids bind to cytoplasmic receptors , pass into the nucleus complexed with their receptors , and act as transcrip on factors forspeci fic target genes .

How do glucocorticoids affect arachidonic acid metabolism?Glucocorticoids inhibi t phosphol ipase A2, blocking the release of arachidonic acid and ↓ production of prostaglandins and leukotrienes .

Select the most appropriate corticosteroid for each of the following clinical situations:Diagnosis of Gushing syndromeDexamethasoneAutoimmune disorderPrednisoneAddison diseaseHydrocorti sone (+/- fludrocorti sone)Relief of inflammationPrednisone, corti soneAsthma/allergiesBeclomethasone or triamcinolone aerosolUsed to ↑ fetal lung maturityBetamethasoneCancer chemotherapyPrednisone

List the effect of glucocorticoids on each of the f ollowing systems:Metabolic↑ Gluconeogenes is ; net ↑ in fat depos i tion in prototypica l areasMuscle↑ Muscle protein catabol i sm; myopathy → weaknessBone↑ Bone catabol i smImmuneInhibi ts cel l -mediated immunity; ↑ PMNs; ↓ lymphocytes , basophi l s , eos inophi l s , and monocytes ; ↓ leukocyte migrationPsychBehaviora l changes , psychosesGI↓ Res is tance to ulcersVascular↓ Capi l lary permeabi l i ty →↓ edema at s i tes of inflammation

What is the consequence of abrupt discontinuation of corticosteroid use?Acute adrenal insufficiency syndrome (potentia l ly letha l )

Name eight commonly observed effects of long-term corticosteroid use:1. Osteoporos is2. HTN3. Insul in res is tance4. Edema5. Psychoses6. Peptic ulcers7. ↑ Susceptibi l i ty to infections

8. CataractsWhat classic toxicity is associated with excess corticosteroid use?

la trogenic Gushing syndromeFor each of the following drugs, provide:1. The mechanism of action (MOA)2. Indication(s) (IND)3. Significant side effects and uniquetoxicity (TOX) (if any)

SpironolactoneMOA: spi ronolactone binds estrogen receptorsIND: hi rsutism (eg, in PCOS), prostate/breast cancerTOX: gynecomastia , thrombocytopenia , hepatotoxici tyClomipheneMOA: blocks negative feedback → ↑ GnRH →↑ LH and FSH → ovulationIND: inferti l i tyTOX: ovarian enlargement, multiple bi rths , hot flashesTamoxifenMOA: competes for estrogen receptors → blocks binding to ER ⊕ cel l sIND: breast cancerTOX: ↑ ri sk of endometria l carcinoma, hot flashesFinasterideMOA: 5α-reductase inhibi tor →↓ DHTIND: benign prostatic hyperplas ia (BPH), ha i r lossTOX: impotence, gynecomastiaFlutamideMOA: competi tive androgen receptor blockerIND: prostatic carcinomaMifepristone (RU486)MOA: competi tive progesterone receptor blockerIND: abortionTOX: GI upset, metrorrhagiaDinoprostoneMOA: PGE2 ana log causes cervica l di lation and uterine contractionIND: induction of labor; abortionRitodrine, terbutalineMOA: β2-agonis ts relax the uterusIND: preterm laborTOX: maternal and feta l tachycardia , fluid retention, and hyperglycemiaSildenafil, vardenafilMOA: inhibi ts phosphodiesterase type 5 →↑ cGMP → smooth muscle relaxation of corpus cavernosumIND: erecti le dys functionTOX: hypotens ion i f a l so taking a ni trate, priapism, headache, color vis ion changes

Name five benefits of oral contraceptives (OCPs):1. ↓ Risk of ovarian and endometria l cancer2. ↓ Genitourinary infections3. Regulates menstrua l cycle4. Low fa i lure rate (i f taken appropriately)5. ↓ Risk of ectopic pregnancy

Name five disadvantages of OCPs:1. Requires da i ly pi l l ingestion2. Not protective aga inst STDs3. ↑ Triglycerides4. ↑ Risk of hepatic adenoma5. Induces hypercoagulable s tate

Name four contraindications to OCPs:1. Pregnancy2. H/o thromboembol ism or s troke3. H/o breast cancer or endometria l cancer4. Smoking (in women > 35-y/o)

CHAPTER 9Renal and Genitourinary

EMBRYOLOGY

Name the structure(s) derived from each of the following:

Nephrogenic cord (three structures)1. Pronephros2. Mesonephros3. MetanephrosUreteric budC CUP (”see you pee”): Col lecting tubules , Ca lyces , Ureter, PelvisGenital RidgeGonadsReproductive structures from the mesonephric (Wolffian) duct”SEED” (in males only):Seminal ves icles , Epididymis , Ejaculatory duct, Ductus deferensReproductive structures from the paramesonephric (Miillerian) ductIn women only:Fal lopian tubes , uterus , superior portion of vaginaMetanephrosDefini tive adult kidney

Name the embryologic tissue layer that following components of the genitourinary system develop from:Kidneys (ie, nephrons)Mesoderm (intermediate mesoderm)Collecting tubules, calyces, pelvis, uretersMesoderm (intermediate mesoderm)Bladder, urethraEndoderm (cloaca)

Name the parts of the external genitalia in males and females derived from the following embryologic structures:Genital tubercleMales : glans penis ; females : glans cl i torisUrogenital sinusMales : corpus spongiosum, bulbourethra l (Cowper) glands , prostate gland; females : vestibular bulbs , Barthol in and Skene glandsUrogenital foldsMales : ventra l shaft of penis ; females : labia minoraLabioscrotal swellingMales : scrotum; females : labia majora

Describe the following congenital disorders:Exstrophy of the bladderCongenita l defect in the anterior wal l of the bladder and adjacent abdominal wal l caus ing the bladder to be exposed at bi rth; associatedwith epispadiasHorseshoe kidneyFus ion of the lower (or upper) poles of kidneys → kidney fa i l s to ascend → often malrotated and remains in pelvisHypospadiasPeni le abnormal i ty resul ting in urethra opening on the ventra l (inferior) s ide of penis ; most common congenita l peni le abnormal i tyEpispadiasPeni le abnormal i ty resul ting in urethra l opening on the dorsa l (superior) s ide of penis ; associated with bladder exstrophyRenal agenesisResul ts from fa i lure of the ureteric bud to form; associated with Potter syndromePotter syndromeRenal agenes is → ol igohydramnios → l imb deformities and pulmonary hypoplas ia

What structure limits the ascent of a fused horseshoe kidney?Inferior mesenteric artery

What substance secreted by the testesSuppresses development of paramesonephric ducts in males?Müler ian-inhibi ting substancePromotes development of the mesonephric ducts?Feta l androgens

Describe the gonads and external genitalia of the following:Female (XX) infant with congenital adrenal hyperplasia (CAH)Female gonads with mascul inized geni ta l iaMale (XY) infant with androgen-insensitivity syndromeMale gonads with female external geni ta l ia , but no uterus or fa l lopian tubes

ANATOMY

Name the structures contained in the retroperitoneal space:

Ureters , kidneys , adrenals , pancreas , duodenum (2nd, 3rd, and 4th parts ), ascending/descending colon, rectum, aorta , IVCWhat is the approximate vertebral level of the kidneys?

T12 to L3 (The right kidney i s s l ightly lower because of l iver.)Name the renal blood vessel that runs posterior to the superior mesenteric artery (SMA) and anterior to the aorta:

Left renal veinWhat vessel does the left gonadal vein drain into?

Left renal vein

What important structures do the ureters pass under on their way to the bladder?Females : uterine artery; males : ductus deferens ; water (ureters ) under the bridge (uterine artery, ductus deferens)

Which terminal vessels carry blood toward the glomerulus?Afferent arterioles

What specialized cells are found between the afferent and efferent glomerular arterioles and have receptors for angiotensin II (AT II) and atrial natriure cpeptide (ANP)?

Lacis cel l s (extraglomerular mesangia l cel l s )What makes up the juxtaglomerular apparatus (JGA)?

JG cel l s , lacis cel l s , and the macula densaWhat hormone do the JG cells secrete?

ReninWhat anatomic feature makes stress incontinence more common in women?

External urethra l sphincter does not completely surround female urethra .Name the sensory and motor components of the micturition reflex:

Pelvic splanchnic nerves (parasympathetic S2-S4)

PHYSIOLOGY

What percentage of body weight is total body water (TBW)?

~60%What proportion of TBW is accounted for by intracellular fluid (ICF) and extracellular fluid (ECF)?

Two-thi rds ICF, one-thi rd ECF (60 = 40 + 20. Rule: 40% body weight i s ICF, 20% is ECF.)Name the major cations and anions found in ICF:

Cations = K+, Mg2+; anions = proteins , organophosphates (eg, ADP, ATP)Name the major cations and anions found in ECF:

Cations = Na +; anions = CF, HCO−3

What are the relative proportions of plasma and interstitial fluid in ECF?One-fourth plasma volume, three-fourths inters ti tia l volume

What is considered normal ECF osmolality?~290 mOsm

Clearance of what chemical can be used to measure glomerular filtration rate (GFR)?Inul in (Inul in i s freely fi l tered, but nei ther secreted nor reabsorbed.)

What is a useful clinical measure to estimate GFR?Creatinine clearance

What is considered normal GFR?~120 mL/min

Effective renal plasma flow (ERPF) can be measured by calculating the clearance of what substance?Para-aminohippuric acid (PAH). PAH is fi l tered and secreted.

Complete the following calculations:UxV/Px (urine concentration X urine volume/plasma concentration) =Renal clearance (Cx)ERPF/(1 - hematocrit) =Renal blood flow (RBF)GFR/RPF (renal plasma flow) =Fi l tration fraction (FF)GFR × (plasmax) =Fi l tered loadVurine flow − (Uosm V/Posm) =

Free water clearanceDefine GFR using the Starling formula.

Kf[(PGC − PBS) − (πGC − πBS)], where Kf = fi l tration coefficient, GC = glomerular capi l lary, BS = Bowman spaceList the three components of the glomerular filtration barrier:

1. Fused basement membrane (negative charge barrier)2. Fenestrated capi l lary endothel ium (s ize barrier)3. Epi thel ia l or podocyte foot process layerIf Cx > GFR then there is net ...tubular secretion of substance XIf Cx

tubular absorption of substance XIf Cv = GFR then there is ...no net tubular absorption or secretion of substance X

What proportion of total cardiac output does RBF account for?~25%

How is RBF maintained at a constant level?Autoregulation (via myogenic response to s tretch and the activi ty of the JGA)

How do the following substances affect the renal arterioles, RPF, GFR, and FF?ProstaglandinsProstaglandins di late afferent arterioles : ↑ RPF, ↑ GFR, thus FF s tays constantAT IIAT II constricts efferent arterioles : ↓ RPF, ↑ GFR, thus FF increases

What effect do the following have on RPF, GFR, and FF?Afferent arteriole constrictionDecreased RPF and GFR, no change in FFEfferent arteriole constrictionDecreased RPF, increased GFR, increased FFIncreased plasma protein concentration

No change in RPF, decreased GFR andFFDecreased plasma protein concentrationNo change in RPF, increased GFR andFF

What is the maximum plasma glucose concentration at which glucose will no longer be reabsorbed?~200 to 250 mg/dL (Concentrations of > 250 mg/dL glucose wi l l be lost in the urine.)

What is the transport maximum (Tm)) for glucose?At 350 mg/min, carriers are saturated (Tm = renal threshold/GFR).

What factors cause K+ to shift out of cells?↓ Insul in, (β-blockers , acidos is , digi ta l i s , extreme exercise, cel l l ys i s , hyperosmolari ty

What factors cause K+ to shift into cells?Insul in, (β-agonis ts , a lka los is

List four causes of decreased distal K+ secretion:1. Low-K+ diet2. Hypoaldosteronism3. Acidos is4. K+-sparing diuretics

List six causes of increased distal K+ secretion:1. High-K+ diet2. Hypera ldosteronism3. Alka los is4. Thiazide diuretics5. Loop diuretics6. Luminal anions

Describe a clinical scenario which might lead to each of the following physiologic changes:Isosmotic volume contractionDiarrheaIsosmotic volume expansionIsotonic fluid infus ion (eg, normal sa l ine IV)Hyperosmotic volume contraction (two scenarios)1. Profuse sweating2. Diabetes ins ipidusHyperosmotic volume expansionHigh NaCl intake, infus ion of hypertonic sa l ineHypo-osmotic volume contractionAdrenal insufficiency (hypoaldosteronism)Hypo-osmotic volume expansionSyndrome of inappropriate antidiuretic hormone (SIADH)

FUNCTIONAL REGIONS OF THE NEPHRON

Name the part of the nephron where each of the following processes occur:

Site of reabsorption of all glucose and amino acids, and the majority of bicarbonate, sodium, and waterProximal convoluted tubule (PCT)Site of active reabsorption K+, Na+, Cl−

Thick ascending loop of HenleSite of 50% of urea reabsorptionPCT (pass ively)Site of active reabsorption of Na+, Cl-

Early dis ta l convoluted tubule (DCT)Site of action of thiazide diureticsDCTSite of action of K+-sparing diureticsCol lecting tubulesSections that are impermeable to waterThick ascending loop of Henle, col lecting ducts (in absence of ADH)Site where ammonia is excreted to act as a buffer for secreted H+ ionsPCTPortion of the nephron which is impermeable to Na+ and passively reabsorbs waterThin descending loop of HenleAldosterone-sensitive site where Na+ is exchanged for K+ or H+

Col lecting tubulesSite of active Ca2+ reabsorption that is controlled by PTHEarly DCTSite of action of loop diureticsThick ascending loop of HenleSection where reabsorption of water is regulated by vasopressin (ADH)Col lecting tubulesSite of action of carbonic anhydrase inhibitorsPCTSection which contains principal cells and intercalated cellsCol lecting tubulesSite of 85% of all phosphate reabsorption (via cotransport)PCT

ENDOCRINE FUNCTIONS OF THE KIDNEYS

List the four main endocrine functions of the kidneys:

1. PTH-mediated convers ion of 25-OH vi tamin D to 1, 25-OH vi tamin D by 1α-hydroxylase2. Secretion of renin by JG cel l s in response to arteria l pressure changes and in response to Na + and Cl - del ivery to the macula densa3. Secretion of prostaglandins (to increase GFR by di lating afferent arteriole)4. Secretion of erythropoeitin in response to hypoxia by peri tubular endothel ia l cel l s

Name the effect of each of the following on the kidneys:ANP↓ Na + reabsorption, ↑ GFRAldosterone↑ Na + reabsorption, ↑ K+ secretion, ↑ H+ secretion in col lecting tubuleVasopressin (ADH)↑ Na +/K+/2Cl - transporters in thick ascending l imb, ↑ water permeabi l i ty in principa l cel l s (via aquaporins ), ↑ urea absorp on incol lecting duct

Where is angiotensin-converting enzyme (ACE) primarily found?Lung capi l laries

Briefly describe the renin-angiotensin-aldosterone cascade:↓ BP (↓ stretch of cel l s in afferent arterioles ) → secre on of renin → convers ion of angiotens inogen to angiotens in I (AT I) → AT Iconverted to AT II by ACE

What is the function of AT II?Increases intravascular volume and ↑ vascular tone → ↑ BP

Name six specific actions of AT II:1. Causes potent vasoconstriction2. Releases vasopress in and adrenocorticotropic hormone (ACTH) from the pi tui tary3. Promotes release of a ldosterone from adrenal cortex4. Stimulates hypothalamus to increase thi rs t5. Stimulates catecholamine release from the adrenal medul la6. Increases Na + and HCO3

- reabsorption in proximal tubuleWhat cells in the kidneys produce and secrete erythropoietin?

Peri tubular capi l lary cel l sWhat stimulates erythropoietin production and release?

Hypoxia

PATHOLOGY

Polycystic Kidney Disease

Name the disease characterized by multiple 3 to 4 cm renal cysts, bilateral enlargement of the kidneys, and chronic renal failure in adults:

Autosomal dominant (adult) polycystic kidney disease (ADPKD)What are the signs and symptoms of ADPKD?

Flank pa in, hypertens ion, hematuria , and UTIWhich gene is most commonly associated with ADPKD?

PKD1 (85% of cases )Name three extra renal manifestations of ADPKD:

1. Liver cysts (40%)2. Berry aneurysms (10%-30%)3. Mitra l va lve prolapse (25%)

What is the prognosis of patients with autosomal recessive polycystic kidney disease (ARPKD)?Poor; majori ty die in infancy or early chi ldhood

What type of renal lesions are characteristic of ARPKD?Multiple cyl indrica l cysts found perpendicular to cortex in an enlarged kidney

Which of the cystic renal diseases is associated with an increased risk of renal cell carcinoma?Dia lys is -associated cystic disease

Nephrotic and Nephritic Syndromes

What are the classic features of the following:

Nephrotic syndromeMass ive proteinuria (>3.5 g/d), hypoalbuminemia, hyperl ipidemia, edemaNephritic syndromeHypertens ion, azotemia, RBC casts , ol iguria , hematuria

Glomerulonephropathies

What term describes the type of immune deposits found outside of the glomerular BM (GBM) but within the podocytes?

Subepithel ia lWhat term describes the type of immune deposits found outside the endothelium but inside the GBM?

Subendothel ia lName the immunof luorescence pattern of deposition of immunoglobulins and/or complement associated with the following diseases:

Membranous glomerulonephritis (GN)Granular or “s tarry-sky”

Goodpasture syndromeLinearIgA nephropathy (Berger disease)Mesangia lPoststreptococcalGranular or “s tarry-sky”

Name the electron microscopic (EM) appearance of the immunoglobulin and/or complement deposits in the following diseases:Membranous GN“Spike and dome” subepithel ia l depos i tsMembranoproliferative GN (MPGN)Subendothel ia l humps, “tram track”Poststreptococcal GNSubepithel ia l humps

Name the glomerulopathy most closely associated with each of the following statements:X-linked syndrome of glomerulonephritis, lens dislocation, nerve deafness, and posterior cataractsAlport syndromeNodular glomerulosclerosis, glomerular capillary basement membrane thickeningDiabetic glomeruloscleros is (Kimmelstiel -Wi lson disease)Commonly associated with HIV infection, heroin abuse, sickle cell disease, and obesityFocal -segmenta l glomerulonephri ti s (FSGN)Diffuse loss of foot processes of the visceral epithelial cells on EM, but normal appearance on light microscopyMinimal change disease (l ipoid nephros is )Apple green birefringence on Congo red stainAmyloidos isC-ANCAWegener granulomatos isMost common cause of nephrotic syndrome in kidsMinimal change disease (l ipoid nephros is )Electron dense deposits in the GBM proper and autoantibody to C3 nephritic factorMPGN type II (aka dense depos i t disease)Crescents seen in glomeruli on light microscopyRapidly progress ive glomerulonephri ti s (RPGN)Most common cause of end-stage renal diseaseDiabetic glomeruloscleros isBasement membrane thickening and splitting (train tracks)MPGN type ISyndrome of hematuria and hemoptysis caused by anti-GBM antibodiesGoodpasture syndromeMesangial widening, recurrent hematuria, and proteinuriaIg A nephropathyAssociated with hepatitis CMPGNResponds well to steroidsMinimal change disease (aka s teroid respons ive nephropathy)X-linked recessive defect in collagen type IVAlport syndromeAsymptomatic familial hematuriaThin membrane disease (GBM is only 50%-60% of normal thickness .)“Wire loop lesions”Systemic lupus erythematosus (SLE)— lupus nephropathy (di ffuse prol i ferative pattern)Henoch-Schönlein purpuraIgA nephropathyIrregularly thick GBM with splitting of lamina densa seen on EMAlport syndromeUpper respiratory vasculitis and granulomasWegener granulomatos is

Renal Tubular Acidosis

Name the type of renal tubular acidosis (RTA) associated with the following statements:

Decreased bicarbonate reabsorptionType II (proximal )Dysfunction of principal cellsType IVDecreased acidification and acid excretionType I (dis ta l )HyperkalemiaType IVNongap metabolic acidosisTypes I, I I , IVFanconi syndromeType II (proximal )Aldosterone deficiency/resistanceType IVMost common RTAType IV

Acid-Base Disturbances

Name the simple acid/base disturbance and the associated compensatory response:

pH > 7.4, Pco2 > 40 mm HgMetabol ic a lka los is → hypoventi lationpH 7.4, Pco2 > 40 mm Hg

Respiratory acidos is → renal HCO3- reabsorption

pH > 7.4, Pco2 40 mm Hg

Respiratory a lka los is → renal HCO3- secretion

pH 7.4, Pco2 40 mm HgMetabol ic acidos is → hyperventi lation

What is the formula for calculating anion gap?Na + - (Cl - + HCO-

3)What is a normal anion gap range?

8 to 12 mEq/LList three common causes of nongap metabolic acidosis:

1. Diarrhea2. Renal tubular acidos is3. Hyperchloremia

Name the most common cause of respiratory acidosis:Hypoventi lation (which can be caused by acute/chronic lung disease, sedatives , weakening of respi ratory muscles )

List three common causes of respiratory alkalosis:1. Hyperventi lation2. Early aspi rin ingestion3. Gram-negative seps is

List five common causes of metabolic alkalosis:1. Excess ive vomiting2. Diuretic abuse3. Antacid use4. Hypera ldosteronism5. Gushing syndrome

List nine possible causes of anion gap metabolic acidosis:“MUD FILERS”1. Methanol2. Uremia3. Diabetic ketoacidos is4. Para ldehyde5. Isoniazid (INH) or Iron tablet overdose6. Lactic acidos is7. Ethylene glycol or Ethanol8. Rhabdomyolys is (mass ive)9. Sa l icylate toxici ty

Acute Renal Failure

List the three main types of acute renal failure (ARF):

1. Prerenal2. Intrins ic3. Postrenal

List four common prerenal causes of ARF:1. Hypovolemia2. Decreased RBF (eg, decreased cardiac output, renal artery s tenos is )3. High periphera l vascular res is tance4. Drugs (eg, diuretics , NSAIDs)

List five common causes of intrinsic ARF:1. Acute tubular necros is (i schemic, nephrotoxic, seps is )2. Acute glomerulonephri ti s3. Autoimmune vascul i ti s (eg, lupus , scleroderma)4. Inters ti tia l nephri ti s5. Hemolytic uremic syndrome (chi ldren)

What is the general mechanism for the development of postrenal ARF?Any ou low obstruc on (benign prosta c hyperplas ia [BPH], bladder-neck, s tones , prior gynecologic surgery, bi latera l ureteric). Note:postrenal ARF accounts for 5% of a l l causes of ARE

Name the type of ARF associated with each of the following:Oliguria and FeNa 1%PrerenalOliguria and FeNa > 1%Intrins icHyaline urine castsPrerenalMuddy brown/granular castsIntrins ic (muddy brown casts are particularly associated with ATN)Blood urea nitrogen (BUN): creatinine ratio > 20PrerenalUrine osmolality > 500 mOsmPrerenal

List the effects of uremia on each of the f ollowing organs or systems:

Nervous systemAsterixi s , confus ion, seizures , comaCardiovascular systemFibrinous pericardi ti sHematologic systemCoagulopathy, immunosuppress ionGastrointestinal systemNausea, vomiting, gastri ti sSkinPruri ti sEndocrine systemGlucose intolerance

List six nonuremic complications of renal failure:1. Metabol ic acidos is2. Hyperka lemia → arrhythmias3. Na + and H2O excess → pulmonary edema and congestive heart fa i lure (CHF)4. Hypocalcemia → osteodystrophy (from fa i lure to secrete active vi tamin D)5. Anemia (↓ EPO secretion)6. Hypertens ion (from renin hypersecretion)

Which type of ATN often causes GBM rupture?Ischemic

What substances cause direct injury to the proximal tubules?Nephrotoxins : drugs (aminoglycos ides), toxins , and mass ive amounts of myoglobin (typica l ly in the se ng of a crush injury or s trenuousexercise)

Renal Infections

What infection commonly presents with flank pain, costovertebral angle tenderness, fever, dysuria, pyuria, bacteriuria?

Acute pyelonephri ti sWhat are the two major causes of pyelonephritis?

1. Ascending infection2. Hematogenous seeding

What are the most common organisms responsible for acute pyelonephritis?Escherichia coli (most common), Proteus, Klebsiella, Enterobacter, Pseudomonas. (Think enteric gram-negative rods .)

What is the greatest risk factor for pyelonephritis?Ves icoureteric reflux (or incompetence)

List five possible sequelae of acute pyelonephritis:1. Abscess2. Necrotizing papi l l i ti s3. Renal scars4. Perinephric abscess5. Pyonephros is

What condition is characterized by broad renal scarring, deformed calyces, progressive loss of renal parenchyma, and thyroidization of kidneys?Chronic pyelonephri ti s

What finding on microscopic examination of urine is pathognomonic for pyelonephritis?WBC casts

Tubulointerstitial Diseases

Name the tubulointerstitial disease associated with each of the following clinical scenarios:

Abuse of phenacetin-containing compoundsRenal papi l lary necros isPenicillin and NSAID use; associated with eosinophiliaInters ti tia l nephri ti sAR syndrome characterized by dysfunction of proximal tubules, leading to impaired reabsorption of amino acids, glucose, phosphate, and bicarbonateFanconi syndromeAR disease characterized by impaired tryptophan reabsorption; clinically resembles pellagraHartnup diseaseAR syndrome characterized by impaired active chloride reabsorption in the loop of Henle; clinically mimics the actions of loop diureticsBartter syndrome

Name the five diseases associated with renal papillary necrosis:1. Diabetes mel l i tus2. Acute pyelonephri ti s3. Analges ic nephropathy4. Sickle cel l disease5. Urinary tract obstruction

Renal Vascular Disease

Name the renal vascular pathology associated with each of the following statements:

Macroscopically, kidneys have a fine granular surface; hyaline arteriolosclerosis, interstitial f ibrosis, glomerular sclerosisBenign nephroscleros isHyperplastic arteriolitis (onion skinning); presents with diastolic BP > 130, encephalopathy, proteinuria, hematuria, and papilledemaMal ignant nephroscleros isPediatric syndrome characterized by ARF, microangiopathic hemolytic anemia, thrombocytopenia, and hypertensionClass ic hemolytic-uremic syndrome (HUS)

Syndrome characterized by ARF, microangiopathic hemoly c anemia, hypertension, fever, and mental status changes; associated with ADAMTS-13defectThrombotic thrombocytopenic purpura (TIP)

What is the most common cause of HUS?Escherichia coliO157:H7 (75% of cases )

What three groups of patients are at increased risk of developing renal failure from benign nephrosclerosis?1. African Americans2. Diabetics3. Patients with hypertens ion

What type of infarction typically occurs in the kidneys?”White,” due to end-organ type arteria l supply of kidneys

What conditions are associated with diffuse cortical necrosis?Obstetric catastrophes (eg, severe placenta l abruption) and septic shock

Name two major causes of renal artery stenosis (RAS):1. Atheroscleros is (70%)2. Fibromuscular dysplas ia (30%)

What group of people are most likely to have secondary hypertension due to f ibromuscular dysplasia-induced RAS?Women in thei r 20’s and 30’s

Urolithiasis

Name the type of renal calculi associated with each of the following statements:

Radiopaque (two types)1. Ca lcium stones2. Struvi te s tonesStaghorn calculiStruvi te (aka ammonium magnes ium phosphate)Infection with urease-positive bacteriaStruvi te (think Proteus)Most common type of calculusCalciumGout (or any other hyperuricemic condition)Uric acid s tonesElevated PTHCalciumSecondary to inherited defect of amino acid transporterCystine (due to cystinuria)

What are the signs and symptoms associated with urinary stone disease?Renal col ic (flank pa in radiating to groin), hematuria , and pyelonephri ti s

Renal and Urinary Tract Tumors

What must be ruled out in an older adult with hematuria?

Urinary tract mal ignancies (renal and bladder)Name the renal or urinary neoplasm associated with each of the following statements:

Large, palpable flank mass in a toddlerWilms tumor (aka nephroblastoma) or neuroblastomaMost common tumor of the renal pelvisTrans i tional cel l carcinomaBenign tumor of kidney often associated with tuberous sclerosisAngiomyol ipomaMost common renal malignancyRenal cel l carcinomaClear cellsRenal cel l carcinomaAssociated with phenacetin abuse, cigarette smoking, cyclophosphamide, and aniline dyesTrans i tional cel l carcinomaLarge, benign tumor of kidney, composed of eosinophilic cells; arises from the intercalated cells of collecting ductOncocytomaSecondary polycythemiaRenal cel l carcinomaAssociated with Schistosoma haematobiumSquamous cel l carcinoma of the bladderInvades renal vein and sometimes IVCRenal cel l carcinomaPainless hematuria in an older patient who smokesTrans i tional cel l carcinoma of the bladderClassically manifests clinically with hematuria, flank pain, and a flank massRenal cel l carcinoma

Which gene is most commonly altered in clear cell carcinomas?VHL (von Hippd-Lindau) gene (mutated or hypermethylated)

Which gene is most commonly altered in papillary renal cell carcinomas?MET

Which hormone is secreted by 5% to 10% of all renal cell carcinomas?Erythropoeitin

Renal cell carcinomas (RCC) are associated with ectopic production of which other hormones?ACTH, renin, parathyroid hormone receptor protein (PTHrp), prolactin

Why is RCC often refractory to chemotherapy?Express ion of P-glycoprotein, a marker of multidrug res is tance

Name the syndrome characterized by Wilms tumor plus each of the following findings:Gonadal dysgenesis, nephropathy associated with WT1 gene mutationsDenys-Drash syndromeWT1 gene mutation, aniridia, GU malformations, and mental/motor retardationWAGR complexOrganomegaly, hemihypertrophy, macroglossia, gigantism, neonatal hypoglycemia associated with the WT2 geneBeckwith-Wiedemann syndrome

Name the most common genetic defect associated with each of the following:Transitional cell carcinomaChromosome 9p and 9q a l terationsRenal cell carcinomaAlterations of chromosome 3 affecting the VHLgeneWilms tumorDeletion of WTLgene (tumor suppressor gene on l lplS)Beckwith-Wiedemann syndromeWT2 gene deletion (11p15.5)

What is the inheritance pattern of Beckwith-Wiedemann syndrome?Genomic (paternal ) imprinting

Urinary System

Name the urinary disease associated with each of the following statements:

Michaelis-Gutmann bodiesMalakoplakiaFibrous masses over the sacrum and lower aorta that encroach on the ureters causing obstruction and hydronephrosisScleros ing retroperi toni ti sDisease characterized by chronic bacterial cystitis with yellow and gray plaques in the mucosa of the bladderMalakoplakia

How does chronic urinary outlet obstruction affect the bladder wall?Chronic obstruction causes bladder wal l hypertrophy

What are the common symptoms of lower urinary tract infection (cystitis)?Dysuria , frequency, urgency

What is the most common source of bacteria that causes cystitis?Colon (E. coli = 80%)

Why are women at 10 times the risk of men for developing a urinary tract infection (UTI)?The female urethra i s shorter and more l ikely to be colonized with feca l flora .

List the most common UTI organisms:”SEEKS PP”Serratia marcescensEscherichia coliEnterobacter cloacaeKlebsiella pneumoniaeStaphylococcus saprophyticusProteus mirabilisPseudomonas aeruginosa

Which UTI-causing bacterium is frequently nosocomial, drug-resistant, and may produce a red pigment?Serratia marcescens

Isomorphic red cells in the urine suggest bleeding from what portion of the urinary tract?Lower urinary tract (ie, nonglomerlular bleeding)

What type of trauma commonly causes extravasation of urine into the superficial perineal space?Straddle injury (rupture of urethra below urogenita l diaphragm)

Name the four types of urinary incontinence:1. Stress2. Urge3. Overflow4. Tota l

Name the type of voiding dysfunction or incontinence associated with the following characteristics:A small urinary bladder and detrusor overactivity resulting in bladder wall thickeningHypertonic neurogenic bladderA large urinary bladder and detrusor areflexia that may result in overflow incontinenceAtonic neurogenic bladderSmall amounts of urine leakage associated with coughing, laughing, or strainingStress incontinenceLeakage of urine associated with sudden, strong need to urinateUrge incontinence

PHARMACOLOGY—RENAL

For each of the f ollowing drugs, provide the following:1. The mechanism and locationof action (MLOA)2. Indication(s) (IND)3. Significant side effects and uniquetoxicity (TOX) (if any)

Furosemide, ethacrynic acidMLOA: inhibi tor of Na +/K+/2CF cotransporter in thick ascending loop of HenleIND: diures is , hypertens ion, CHF, and other edematous s tates (asci tes , nephrotic syndrome, etc.)

TOX: “CHIA DOG”CalciuriaHypokalemic metabol ic a lka los isInters ti tia l nephri ti sAl lergy to sul faDehydrationOtotoxici tyGoutSpironolactoneMLOA: competi tive a ldosterone receptor antagonis t; acts in col lecting duct (CD)IND: diures is , CHFTOX: hyperka lemic metabol ic acidos is , gynecomastia , antiandrogen effectsTriamterene and amilorideMLOA: block Na +/K+ channels in col lecting ductIND: used in combination with other diuretics (especia l ly loop and thiazide diuretics ) for thei r K+-sparing propertiesTOX: leg cramps (triamterene)AcetazolamideMLOA: carbonic anhydrase inhibi tor (CAI) →↓ HCO3

- reabsorption in PCT →↑ urine osmolari tyIND: diures is , glaucomaTOX: metabol ic acidos is , ammonia toxici ty, neuropathy. Note: sul fa a l lergyMannitolMLOA: ↑ tubular fluid osmolari ty →↑ urine flow; acts in PCT, thin descending l imb, and CDIND: diures is , elevated intracrania l pressure (ICP)TOX: pulmonary edema, dehydrationThiazides—chlorothiazide, hydrochlorothiazideMLOA: inhibi tor of NaCl reabsorption in early DCT →↑ urine osmolari tyLOA: DCTIND: hypertens ion, CHFTOX: may cause hyperglycemia, hyperl ipidemia, hyperca lcemia, and a l lergic response in patients sens i tive to sul fa drugsACE inhibitors—captopril, lisinopril, enalaprilMLOA: inhibi tor of ACE →↓ levels of AT II (→↑); prevents inactivation of bradykinin (potent vasodi lator)IND: hypertens ion, renal protective effects in diabetics , CHFTOX: ”CAPTOPRIL”CoughAngioedemaProteinuriaTaste changesHypOtens ionPregnancy i ssues (feta l renal toxici ty)RashIncreased renin and K+

Lowers AT IIContraindicated in bilateral RASLosartanMLOA: AT II receptor antagonis tIND: hypertens ionTOX: dizziness , headache; teratogenicDemeclocyclineMLOA: ADH antagonis t in CDIND: SIADH

Name the diuretic of choice in the following situations:Diuresis in sulfa-allergic patientEthacrynic acidHypercalcemiaLoop diuretics (eg, furosemide)HyperaldosteronismSpironolactoneNephrogenic DIThiazide diuretics (eg, chlorothiazide)Severe edematous statesLoop diuretics (eg, furosemide)Elevated ICPMannitolAltitude sicknessAcetazolamideCalcium stonesThiazide diuretics (eg, chlorothiazide)Used in shock to maintain RBFMannitol

Which diuretics decrease blood pH?CAIs and K+-sparing diuretics

Which diuretics increase blood pH?Loop and thiazide diuretics

Which diuretics increase urinary Ca2+?Loop diuretics and spi ronolactone

Which diuretics decrease urinary Ca2?Thiazide diuretics and ami loride

Which diuretics increase urinary NaCl?Al l classes

CHAPTER 10Hematology and Oncology

EMBRYOLOGY

From what embryonic layer are angioblasts derived?

MesodermName the site(s) of red blood cell (RBC) production during each of the following developmental phases:

First trimesterYolk sacSecond trimesterLiver and spleenThird trimesterCentra l and periphera l skeletonPostpartumAxia l skeleton

PATHOLOGY

Coagulopathy

What is the most common inherited hypercoagulable state?

Factor V LeidenThe factor V Leiden mutation inhibits factor V cleavage by which protein?

Protein CWhat is the effect of estrogen, endogenous or exogenous, on thrombosis?

Estrogen induces a hypercoagulable s tate.What clotting factors require vitamin K for their synthesis?

Factors I I , VII , IX, X, and proteins C and SWhat common laboratory test is used to assess the intrinsic coagulation system?

Partia l thromboplastin time (PTT)What common laboratory test is used to assess the extrinsic coagulation pathway?

Prothrombin time (PT)What commonly used anticoagulant interferes with the extrinsic pathway?

Warfarin. Remember: “WEPT“—Warfarin affects the Extrins ic pathway and i s monitored by PT.The use of the chelator, ethylene diamine tetra acetate (EDTA), as an anticoagulant, in tubes is designed to inactivate what factor in the blood?

CalciumWhat inhibitor of the coagulation cascade inactivates factors Va and VIIIa?

Protein CWhat are the two major functions of von Willebrand factor?

1. Transport of factor VIII2. Linkage of platelets and col lagen

Dysfunction of what clotting component results in mucous membrane hemorrhage, petechiae, purpura, and prolonged bleeding time?Platelet abnormal i ties

Dysfunction of what clotting component results in hemarthroses, purpura, and prolonged PT and/or PTT?Coagulation factor abnormal i ties

Name the coagulopathy associated with the following clinical and pathologic features:Most common hereditary bleeding disordervon Wi l lebrand diseaseMost common type of hemophiliaHemophi l ia A (factor VIII deficiency)Most common cause of acquired platelet dysfunctionAspirin useAutosomal dominant (AD) disorder causing increased bleeding timevon Wi l lebrand diseaseAutosomal recessive (AR) defect in platelet adhesion caused by lack of GpIIb/IXBernard-Soul ier diseaseAR defect in platelet aggregation caused by deficiency of GpIIb/GpIIIa, a membrane receptor responsible for binding fibrinogenGlanzmann thrombastheniaCommon presentation includes hemarthroses and easy bruising, ↑ PTT, normal PTHemophi l ia (A and B)Prolonged bleeding time with normal platelet count, normal PT, and ↑ PTTvon Wi l lebrand diseaseProlonged PT, PTT, ↑ bleeding time, thrombocytopenia, presence of fibrin split productsDisseminated intravascular coagulation (DIC)Normal PT, PTT, normal platelet count, ↑ bleeding timeAspirin useSyndrome characterized by antiplatelet antibodies (commonly anti-Ib-IX or IIb-IIIa), often post-viral infectionsIdiopathic thrombocytopenia (ITP)Syndrome characterized by thrombocytopenia, microangiopathic hemolytic anemia, fever, and neurologic symptomsThrombotic thrombocytopenic purpura (TTP)

Widespread hyaline microthrombi in arterioles and capillaries causing schistocytes and helmet cellsMicroangiopathic hemolytic anemiaSplenomegaly, epistaxis, petechiae, generalized tender lymphadenopathy, serum antibodies (+)SLE-autoimmune thrombocytopenia

Anemias

List five of the most common causes of microcytic anemia:

1. Iron deficiency2. Lead poisoning3. Chronic disease (sometimes normocytic)4. Sideroblastic5. Thalassemia

List four of the most common causes of normocytic anemia:1. Sickle cel l anemia2. Aplastic anemia3. Acute blood loss4. Hemolytic anemia

List five of the most common causes of macrocytic anemia:1. Liver disease2. Vi tamin B12 deficiency2. Folate deficiency4. Alcohol i sm5. Hypothyroidism

What is the primary site of iron absorption? Vitamin B12? Folate?Duodenum; terminal i leum (need intrins ic factor); jejunum (respectively)

What is the major cause of iron deficiency anemia in adults?Menorrhagia or gastrointestina l (GI) bleeding (ie, colon cancer, ulcers )

What are two common clinical complaints in an anemic patient?1. Dyspnea on exertion2. Fatigue

Which amino acid substitution in the β-globin gene is most commonly seen in sickle cell anemia?Glu6 → Val

What is the major type of hemoglobin in homozygous sickle cell anemia?HbS

In sickle cell disease, what is the mechanism of ischemic necrosis of the bones, lungs, liver, brain, spleen, or penis?↓ O2 tens ion → abnormal RBCs s ickle → microvascular occlus ions

What is the major regulatory enzyme in heme biosynthesis?Aminolevul inate (ALA) synthase

What two common enzyme deficiencies can cause hemolytic anemia?1. Glucose-6-phosphate dehydrogenase (G6PD)2. Pyruvate kinase

What hemolytic anemia is associated with exposure to oxidant stress?G6PD deficiency

What two common RBC membrane defects can cause hemolytic anemia?1. Heredi tary spherocytos is2. Paroxysmal nocturnal hematuria (PNH)

What common cause of atypical pneumonia is associated with cold autoimmune hemolytic anemia?Mycoplasma pneumoniae

What type of anemia is caused by lack of intrinsic factor (IF) secretion by gastric parietal cells?Pernicious anemia (vi tamin B12 deficiency)

What two autoimmune diseases of the GI tract can cause megaloblastic anemia?1. Pernicious anemia (due to lack of IF production)2. Crohn disease of the dis ta l i leum (due to lack of IF-B12 complex reabsorption)

What parasites are capable of causing megaloblastic anemia?Diphyllobothrium latum (by depleting B12) and Giardia lamblia (by depleting folate)

How does gastric resection cause megaloblastic anemia?Parieta l cel l s , which are respons ible for IF production may be removed when the gastric fundus i s resected.

What type of malignancy is associated with pernicious anemia?Gastric carcinoma

Name three medications capable of causing autoimmune hemolytic anemia:1. Penici l l in2. Cephalosporins3. Quinidine

What types of malignancies are associated with autoimmune hemolytic anemia?Leukemias and lymphomas

What laboratory tests are seen in hemolytic anemia?↑ Unconjugated bi l i rubin, ↑ urine urobi l inogen, ↓ hemoglobin, hemoglobinuria , ↓ haptoglobin, hemos ideros is

Name two commonly used medications that can cause aplastic anemia:1. Nonsteroida l anti -inflammatory drugs (NSAIDs)2. Chloramphenicol

What happens to the total iron binding capacity (TIBC), serum iron concentration, and percent saturation of transferrin in each of the following diseases?Iron deficiency anemia↑ TIBC, ↓ serum i ron, ↓ % saturationAnemia of chronic disease↓ TIBC, ↓ serum i ron, normal saturationIron overload

Normal TIBC, ↑ serum i ron, maximal saturationName the type(s) of anemia associated with the following clinical and pathologic features:

Most common type of anemiaIron deficiency anemiaAbnormal Schilling testPernicious anemia

ABO incompatibility, lymphoid neoplasm, Raynaud phenomena, anti-i antibodiesCold autoimmune hemolytic anemia

Atrophic glossitisPernicious anemia

AutosplenectomySickle cel l anemia

Basophlilic stipling of erythrocytes, blue/gray discoloration at gumline, wrist/foot dropAnemia from lead poisoning

Celiac sprueFolate deficiency anemia (megaloblastic)

Chronic atrophic gastritisPernicious anemia

Colon cancerIron deficiency anemia (early) and anemia of chronic disease (late)

Crescent-shaped erythrocytes and Howell-Jolly bodiesSickle cel l anemia

Deficiency of α- or β-globin gene synthesisThalassemia

Deficiency of decay accelerating factorParoxysmal nocturnal hemoglobinuria

Demyelination of the dorsal and lateral tracts of the spinal cordPernicious anemia

End-stage liver diseaseMacrocytic anemia

Helmet cells, burr cells, triangular cellsMicroangiopathic anemia (2° to DIC, thrombo c thrombocytopenic purpurahemoly c-uremic syndrome [TTPHUS], or mechanica l heartva lves )

Hypersegmented polymorphonuclears (PMNs)Vitamin B12 or folate deficiency anemia

Increased serum lactate dehydrogenase (LDH)Hemolytic anemia

Microcytosis, atrophic glossitis, esophageal webs (Plummer-Vinson syndrome)Iron deficiency anemia

Pancytopenia and fatty infiltration of bone marrowAplastic anemia

Systemic lupus erythematosus (SLE), chronic lymphocytic leukemia (CLL), lymphomas, drugs; + direct Coombs test (due to IgG autoantibodies)Warm autoimmune hemolytic anemia

AD deficiency of spectrin, positive osmotic fragility testHereditary spherocytos isReduced erythropoietinAnemia of chronic diseaseRinged sideroblastsSideroblastic anemiaSusceptibility to infection by encapsulated organismsSickle cel l anemia (from splenic autoinfarction)Transient normocytic anemiaAnemia of acute blood loss

What is the genetic defect in thalassemias?Spl icing defect, caus ing decreased α or β sub-chain production

Name the type of thalassemia responsible for each of the following findings:β-Thalassemia associated with growth retarda on, frontal bossing, and hepatosplenomegaly (HSM) (from extramedullary hematopoesis), jaundice, andiron overload (2° to transfusions), and ↑ Hgb Fβ-Thalassemia major (β–/β–); Note: β-tha lassemia minor (β+/β-), typica l ly asymptomaticα-Thalassemia associated with mild microcytic anemia, usually asymptomaticα-Thalassemia minor (two a l leles affected); Note: when only one a l lele involved (carrier s tate) → no anemiaα-Thalassemia associated with pallor, splenomegaly, chronic hemolytic anemia, and intraerythrocytic inclusionsHgb H disease (three a l leles affected)α-Thalassemia associated with stillborn fetusHydrops feta l i s (a l l four a l leles affected)

White Blood Cell (WBC) Neoplasms

What are the two major categories of lymphoma?

1. Hodgkin disease (HD)2. Non-Hodgkin lymphoma (NHL)

Name the general type of lymphoma (HD or NHL) associated with the following clinical and pathologic features:Interleukin (IL)-5 secreting Reed Sternberg (RS) cellsHDCommonly arises from B cells NHLNHLConstitutional symptoms including Both fever, night sweats, weight lossBoth

Mediastinal lymphadenopathy, contiguous spreadHDMany cases associated with Epstein-Barr virus (EBV)HDPeripheral lymphadenopathy, noncontiguous spreadNHLBimodal age distribution, but most common in young menHDPeak incidence from 20 to 40 years of ageNHLAssociated with immunosuppression including AIDSNHLPainful lymphadenopathy with alcohol consumptionHD

Name the type of HD associated with each of the following clinical and pathologic findings:Most common typeNodular scleros isAbundance of RS cellsMixed cel lulari tyLacunar cells and collagen bandingNodular scleros isCommonly seen in older patients with HDMixed cel lulari tyWidely disseminated disease with poor prognosisLymphocyte depletionMore common in femalesNodular scleros isAbundance of lymphocytesLymphocyte predominanceCommonly seen in men 35 years presenting with cervical or axillary lymphadenopathyLymphocyte predominanceHigh proportion of RS cells relative to lymphocytesLymphocyte depletion

Name the type of NHL associated with each of the following clinical and pathologic findings:Overexpression of cyclin D1Mantle cel l l ymphomaStarry-sky appearance on histopathologyBurki tt lymphomaClinically similar to CLL; characterized by nodules of small lymphocytesSmal l cel l l ymphocytic lymphomaOlder adults with BCL2 gene mutationsFol l i cular lymphomaOften appears at extranodal sites and can cause small bowel obstructionDiffuse large cel l l ymphomaChild presenting with enlarging mandibular massBurki tt lymphomaCommon in children who present with a mediastinal mass and a syndrome similar to that of acute lymphocytic leukemia (ALL)Lymphoblastic lymphomaTdT+ lymphocytesPrecursor B- or T-cel l acute lymphoblastic leukemia/lymphomaEndemic in Africa and associated with EBVBurki tt lymphoma

Name the specific leukemia associated with the following age brackets:0 to 14 years oldALL15 to 39 years oldAML40 to 59 years oldAML and CML> 60 years oldCLL

Name the specific leukemia associated with each of the following findings:Very high white cell counts, often >200,000Chronic myelogenous leukemia (CML)Isolated lymphocytosisCLLTdT+ lymphoblastsALLLarge, immature myeloblasts predominateAcute myelogenous leukemia (AML)Neoplastic pre-T or pre-B lymphocytes, CD10 ©ALLCD19/CD20 and CD5 © malignant cellsCLLBone marrow is replaced with myeloblastsAMLPhiladelphia chromosome t(9:22)CMLAuer rods

AML (M3 > M2 > M1)Low leukocytic alkaline phosphataseCML (a lso seen in paroxysmal nocturnal hematuria)Associated with fatigue, thrombocytopenia, signs of anemia, frequent infections, leukemia cutis, and DICAML (M3)Bone pain, fever, generalized lymphadenopathy, HSM, and signs of central nervous system (CNS) spreadALLExcellent prognosis if treated earlyALLMay progress to AMLCMLMost responsive to therapyALLAssociated with prior exposure to radiationCMLPeripheral leukocytes containing tartarate-resistant acid phosphatase (TRAP) and cytoplasmic projectionsHairy cel l leukemia (Remember: “TRAP the Hairy beast”)

Myeloproliferative Disorders

Name the four chronic myeloproliferative disorders:

1. CML2. Polycythemia vera3. Essentia l thrombocytos is4. Myelofibros is with myeloid metaplas ia

Name the myeloproliferative disorder associated with each of the following clinical and pathologic findings:↑ RBC mass and low/normal erythropoietinPolycythemia veraTear drop deformity of erythrocytes, bone marrow hypercellularityMyelofibros is with myeloid metaplas iaPlethoric complexion, pruritus after showering, blurred vision, splenomegaly, and epistaxisPolycythemia veraErythromelalgia (throbbing or burning of hands and feet)Essentia l thrombocytos isBasophiliaPolycythemia veraWidespread extramedullary hematopoesis with megakaryocytic proliferation in the bone marrowMyelofibros is with myeloid metaplas iaHyperviscosity syndromePolycythemia vera and essentia l thrombocytos isPeripheral thrombocytosis, bone marrow megakaryocytosis, and splenomegalyEssentia l thrombocytos is

Name the plasma cell disorder associated with the following clinical and pathologic findings:Bone pain, osteopenia, pathologic fractures, and “punched-out” lytic lesions on x-rayMultiple myelomaRussel bodies and “plymphocytes” (plasmacytoid lymphocytes)Waldenstrom macroglobul inemiaBence-Jones proteinuriaMultiple myeloma and Waldenstrom macroglobul inemiaSmall M-spike on plasma electrophoresis in an otherwise healthy patientMonoclonal gammopathy of undetermined s igni ficance (MGUS)Hypercalcemia; renal insufficiencyMultiple myelomaHyperviscosity syndromeWaldenstrom macroglobul inemiaLarge M-spike on plasma electrophoresisMultiple myeloma and Waldenstrom macroglobul inemiaPrimary amyloidosisMultiple myeloma

Peripheral Blood Smear

Name the condition associated with each of the following peripheral blood smear findings:

Defective PMN degranulation → accumulation of giant granules in PMNs and other leukocytesChediak-Higashi anomalyAtypical lymphocytesInfectious mononucleos isLarge numbers of Auer rods (Faggot cells)AML (M3 subtype)Basophilic stipplingLead poisoningBurr cells (echinocytes)Burns , uremiaDumbbell-shaped bilobed nucleiPelger-Huet anomalyGiant plateletsBernard-Soul ier disease, May-Heggl in anomaly

Heinz bodies, bite cellsG6PD deficiencyHelmet cells, schistocytesMicroangiopathic hemolytic anemia (DIC, TTP, HUS)Howell-Jolly bodiesAspleniaHypersegmented PMN nucleiMegaloblastic anemiaIntracytoplasmic ringsMalaria , babes ios isLymphocytic cerebriform nucleiSézary syndromeNucleated erythrocytesHemolytic anemiaRouleau formationMultiple myeloma and Waldenstrom macroglobul inemiaSmall plateletsWiskott-Aldrich syndromeSmudge cellsCLLSpherocytesHereditary spherocytos isSpur cells (acanthocytes)Abeta l ipoproteinemia, l iver diseaseTarget cells (codocytes)Thalassemias , i ron deficiency anemia, l iver disease, s ickle cel l anemiaTeardrop cells (dacryocytes)Myelofibros isToxic granulations in leukocytes (Dohle bodies)Seps is

Chromosomal Translocations

Name the neoplasm associated with the following chromosomal translocations and genes/gene products:

t(11:14) protooncogene under Ig promoterMantle cel l l ymphomat(11:22) EWS (EWS-FLI1 fusion protein)Ewing sarcomat(14:18) BCL2Fol l i cular lymphomat(15:17) (PML/RAR-β [retinoic acid receptor alpha])AML (M3 subtype, treated with retinoic acid)t(3:6) VHLvon Hippel -Lindau syndrome (t[3:8] and t[3:11] are two common variants seen in VHL)t(8:14) c-myc and IgHBurki tt lymphoma (t[8:22] and t[2:8] are two common variants seen in Burki tt lymphoma)t(9:22) bcr-abl fusion proteinCML (treated with imatinib)

Tumor Markers

Name the neoplasm associated with the following tumor markers:

α1-ATLiver cancer, yolk-sac tumorsα-Fetoprotein (AFP)Germ cel l tumors , hepatocel lular carcinomaAlkaline phosphataseMetastatic bone involvement, Paget diseaseβ-HCGHydati form moles , Choriocarcinoma, Gestational trophoblastic tumorsCA-125Ovarian cancersCarcinoembryonic antigen (CEA)Colon, pancreatic, and other cancers of the GI tractProstate-specific antigen (PSA)Prostate cancerS-100Melanoma, neura l tumors

Cancer Genetics

For each of the following tumor suppressor genes, state the function and the malignancy/malignancies they are associated with:

APCFunction: regulation of β-catenin in the Wnt/β-catenin s igna l ing pathway—promotes cel l adhes ion and regulates cel l prol i ferationAssociated malignancies: fami l ia l adenomatos is polypos is (FAP) and many GI cancersBRCA1 and BRCA2

Function: DNA repair and transcriptional regulationAssociated malignancies: breast and ovarian cancersNF1 and NF2Function: regulates s igna l transduction through the ras pathwayAssociated malignancies: neurofibromas , schwannomas; (NF-2 a lso associated with meningioma)p16Function: regulates cel l cycle by inhibi ting cycl in-dependent kinasesAssociated malignancies: pancreatic and esophageal carcinomas and mal ignant melanomap53Function: regulates cel l death and prol i feration in response to DNA damageAssociated malignancies: most human cancersRbFunction: regulates trans i tion from G1 to S in the cel l cycle by sequestering E2Fs , a fami ly of transcription factorsAssociated malignancies: retinoblastoma, osteosarcomaWT1Function: inhibi ts transcription of genes promoting cel l prol i ferationAssociated malignancies: Wi lms tumor

For each of the following oncogenes, state the function and the malignancy/malignancies they are associated with:ablFunction: promotes cel l prol i feration through tyros ine kinase activi tyAssociated malignancies: CML, ALLBCL2Function: overexpress ion prolongs cel l surviva l by inhibi ting apoptos isAssociated malignancies: fol l i cular and undi fferentiated lymphomascyclin DFunction: promotes cel l prol i feration by s timulating the phosphorylation of pRbAssociated malignancies: l ymphoma, breast, l i ver, and esophageal cancers cycl in DCDK4Function: promotes cel l prol i feration by phosphorylating pRbAssociated malignancies: sarcoma, gl ioblastoma multi forme, mal ignant melanomaHER2/neuFunction: ampl i fi cation promotes cel l prol i feration by enhancing growth factor s igna l transductionAssociated malignancies: breast, ovarian, lung, s tomach cancersmycFunction: promotes cel l prol i feration by transcriptional activation of speci fic genesAssociated malignancies: Burki tt lymphoma, smal l cel l carcinoma of the lung, neuroblastomarasFunction: s igna l transduction through the MAP kinase pathwayAssociated malignancies: colon cancer and many other human cancersretFunction: receptor tyros ine kinase that promotes cel l prol i feration in response to growth factorsAssociated malignancies: multiple endocrine neoplas ia 2A and 2B; fami l ia l medul lary thyroid carcinomasisFunction: β-cha in of PDGF which promotes cel l prol i ferationAssociated malignancies: as trocytomas , osteosarcomas

Miscellaneous Oncology

Name the type of neoplasm associated with the following diseases:

Actinic keratosisSquamous cel l carcinomas of skinAIDSAggress ive mal ignant lymphomas, Kapos i sarcoma, bra in lymphomasBarrett esophagitisEsophageal adenocarcinomaChronic atrophic gastritisGastric adenocarcinomaCirrhosisHepatocel lular carcinomaDown syndromeALLDysplastic nevusMal ignant melanomaImmunodeficiency statesNHLMyasthenia gravisThymomaPaget disease of boneSecondary osteosarcoma and fibrosarcomaPlummer-Vinson syndrome Tuberous sclerosisSquamous cel l carcinomas of esophagusUlcerative colitisColonic adenocarcinomaXeroderma pigmentosaSquamous and basa l cel l carcinomas of skin

List the four major differences between benign and malignant neoplasms:1. Di fferentiation and anaplas ia2. Rate of growth

3. Loca l invas ion4. Metastases (most important di fference)

What are the three ways a tumor can spread?1. Invas ion of lymphatic system (eg, carcinoma of the breast)2. Hematogenous (typica l of sarcomas)3. Seeding of body cavi ty (eg, ovarian cancer)

What four classes of genes are the primary targets for genetic mutation leading to cancer?1. Protooncogenes (promote cel lular growth)2. Tumor suppressor genes (inhibi t cel lular growth)3. Genes that regulate and mediate apoptos is4. Genes that regulate and mediate DNA repair

What three genetic mechanisms can lead to the activation of protooncogenes?1. Point mutations2. Chromosomal rearrangements3. Ampl i fi cation

What three chromosomal abnormalities are characteristic of tumor cells?1. Ampl i fi cation2. Deletion3. Trans location

What three factors influence tumor growth?1. Doubl ing time of tumor cel l s2. Cel l prol i feration3. Cel l death (apoptos is )

Name the six small, round, blue cell tumors of childhood:1. Ewing sarcoma2. Lymphoma3. Neuroblastoma4. Medul loblastoma5. Rhabdomyosarcoma6. Primitive neuroectodermal tumors

Chemical Carcinogens

What are the two steps in the process of chemical carcinogenesis?

1. Ini tiation: cel l s undergo i rrevers ible genetic mutation.2. Promotion: chemica ls promote growth of ini tiated cel l s .

Name the neoplasm associated with each of the following chemical, viral, and microbial carcinogens and types of radiant energy:AflatoxinHepatocel lular carcinomaAniline dyesBladder cancerAromatic hydrocarbonsLung cancer (aromatic hydrocarbons found in cigarettes )AsbestosMal ignant mesothel iomaEBVBurki tt lymphoma, nasopharyngeal cancer, B-cel l l ymphoma in AIDS patients , some types of HDEstrogenBreast carcinoma and endometria l carcinomaHelicobacter pyloriGastric adenocarcinoma and margina l zone lymphoma (MALToma)Hepatitis B virusHepatocel lular carcinomaHuman papilloma virusCervica l squamous cel l carcinoma and geni ta l wartsHuman T-cell lymphocytic virus-1 (HTLV-1)T-cel l leukemia/lymphomaIonizing radiation NitrosaminesMyeloid leukemias and thyroid cancersPolyvinylchloride (PVC) assemblyHepatic hemangiosarcomaUltraviolet radiationSkin cancersSchistosoma haematobiumSquamous cel l carcinoma of the urinary bladder

Paraneoplastic Syndromes

Name the process of profound weight loss and weakness due to muscle and fat loss in a patient with an advanced neoplasm:

Cancer cachexiaName the tumor associated with the following paraneoplastic syndromes:

Acanthosis nigracansMany types of vi scera l mal ignanciesCarcinoid syndromeCarcinoid and neuroendocrine carcinomas of the bronchi or GI tractClubbing of fingersPulmonary or thoracic mal ignancies

Cushing syndromeAdrenocorticotropic hormone (ACTH)-secreting pi tui tary adenoma or a corti sol -secreting adrenal adenomaDICAML (M3)HypercalcemiaParathyroid hormone (PTH)-secreting squamous cel l carcinoma of the lungLambert-Eaton myasthenic syndromeSmal l cel l carcinoma of the lungThrombophlebitisPancreatic or lung adenocarcinomaSyndrome of inappropriate antidiuretic hormone (SIADH)Antidiuretic hormone (ADH)-secreting smal l cel l carcinoma of the lung

PHARMACOLOGY

Medication for Anemia

Name the drug of choice for each of the following groups of anemic patients:

Adolescent girls with heavy periods and pregnant womenIronPatients on methotrexate who develop megaloblastic anemiaFolateElderly patients with atrophic gastritis who develop megaloblastic anemiaCyanocobalamin (vi tamin B12)Patients with anemia secondary to end-stage renal diseaseErythropoietinPatients with extremely low WBC count and prone to infectionG-CSF, GM-CSF

Chemotherapeutic Agents

For each of the following drugs, provide:1. The mechanism of action (MOA)2. Indication(s) (IND)3. Significant side effects and unique toxicity (TOX) (if any)

MethotrexateMOA: folate analog that inhibi ts dihydrofolate reductase and, consequently, S phase of the cel l cycleIND: leukemia, lymphoma, carcinoma, sarcomaTOX: myelosuppress ion (revers ible with leucovorin)5-FluorouracilMOA: inhibi ts pyrimidine synthes is which inhibi ts S phase progress ionIND: colon cancer and other sol id tumorsTOX: myelosuppress ion and phototoxici tyCytarabineMOA: inhibi ts pyrimidine synthes is which inhibi ts S phase progress ionIND: acute leukemiasTOX: neurotoxici ty6-Mercaptopurine or 6-thioguanineMOA: inhibi ts purine synthes is which inhibi ts S phase progress ionIND: leukemia, lymphomaTOX: myelosuppress ion, hepatotoxici tyBusulfanMOA: DNA a lkylating agentIND: pa l l iative role in CML treatmentTOX: pulmonary fibros is , hyperpigmentationCyclophosphamideMOA: DNA a lkylating agentIND: NHL, breast and ovarian carcinomasTOX: hemorrhagic cysti ti s (prevented with mesna), myelosuppress ionNitrosureas (carmustine, lomustine, streptozocin)MOA: DNA a lkylating agent capable of cross ing the blood-bra in barrierIND: bra in tumorsTOX: CNS toxici ty including dizziness and ataxiaCisplatinMOA: DNA a lkylating agentIND: tes ticular cancer, female reproductive tract cancers , bladder and testicular cancersTOX: nephrotoxici ty and acoustic nerve damageDoxorubicin/adriamycinMOA: interca lates into DNA and inhibi ts DNA repl icationIND: Hodgkin disease, myeloma, sarcoma, sol id tumorsTOX: cardiotoxici ty, a lopecia , myelosuppress ionBleomycinMOA: interca lates into DNA and causes DNA strand breaksIND: tes ticular cancer, lymphomasTOX: pulmonary fibros is , myelosuppress ion

EtoposideMOA: inhibi ts topoisomerase, caus ing double-s tranded breaks in DNAIND: smal l cel l carcinoma of lung, prostate cancer, testicular cancerTOX: Bone marrow suppress ion, hypotens ionPrednisoneMOA: may trigger apoptos isIND: CLL, HD, lymphomasTOX: cushingoid reaction, immunosuppress ion, cataracts , acne, osteoporos is , hypertens ion (HTN), peptic ulcers , hyperglycemiaTamoxifen/raloxifeneMOA: es trogen receptor agonis t/antagonis tIND: breast cancerTOX: increased ri sk of endometria l cancerVinblastine/vincristineMOA: binds tubul in to inhibi t formation of the mitotic spindleIND: HD, lymphoma, Wi lms tumor, choriocarcinomaTOX: vincris tine: periphera l neuropathy and para lytic i leus ; vinblastine: myelosuppress ionPaclitaxelMOA: binds tubul in to inhibi t formation of the mitotic spindleIND: ovarian and breast cancersTOX: myelosuppress ion, hypersens i tivi ty

Anticoagulants

What test is used to monitor anticoagulation in a patient treated with heparin?

PTTWhat test is used to monitor anticoagulation in a patient treated with warfarin?

PTWarfarinMOA: causes synthes is of dys functional vi tamin K-dependent clotting factors (I I , VII , IX, X)IND: chronic treatment and prophylaxis of DVTTOX: hemorrhage, teratogenic, osteoporos isHeparinMOA: increases PTT by activating anti thrombin II IIND: acute treatment of DVT, pulmonary embolus , angina, myocardia l infarction (MI), i schemic s trokeTOX: hemorrhage, Heparin-Induced Thrombocytopenia Syndrome (HITS)Low-molecular-weight heparin (LMWH) (eg, enoxaparin)MOA: s imi lar to heparinIND: anticoagulation outs ide of the hospi ta l , commonly for DVTThrombolytics (streptokinase, urokinase, tissue plasminogen activator, anistreplase)MOA: faci l i tate the convers ion of plasminogen to plasmin, which cleaves thrombin and fibrin clotsIND: acute therapy for MI and i schemic s trokeTOX: hemorrhageClopidogrel and ticlopidineMOA: antiplatelet agent that blocks adenos ine diphosphate (ADP) receptors to inhibi t platelet aggregationIND: acute coronary syndrome, interventional cardiology procedures , trans ient i schemic attacks , and s trokeTOX: hemorrhage, leukopenia , diarrheaEptifibatide and tirofibanMOA: prevent platelet aggregation by blocking glycoprotein IIb/II Ia receptorsIND: acute coronary syndromesTOX: hemorrhage

CHAPTER 11Skin and Connective Tissues

EMBRYOLOGY/ANATOMY/HISTOLOGY

Name the embryonic structures that give rise to each of the following:

EpidermisEctodermMelanocytesNeura l crest cel l s

Name the layers of the epidermis:“Cal i fornians Li ke Gi rl s in String Bi kinis” (from surface → base)Stratum CorneumStratum Lucidum (lacking in thin skin of face and geni ta l ia )Stratum GranulosumStratum SpinosumStratum Basa l i s

What epidermal dendritic cells are responsible for antigen presentation?Langerhans cel l s

Name the type of collagen described by the following statements:Found in Bone, tendons, skin, fascia, cornea, and dentin; replaces reticulin later in wound repairType I (Remember: bONE)Found in Cartilage, nucleus pulposus, and ocular vitreous bodyType II (Remember: carTWOlage)Found in skin, blood vessels, uterus, fetal tissues, and involved in early wound repair (granulation tissue)Type II I (Reticul in)Component of Basement membrane or basal laminaType IV (Remember: type IV, under the floor/basement membrane)Found at epiphyseal platesType XAccounts for ~90% of collagen in the bodyType I

Mnemonic for collagen types I to IV“Be Cool , Read Books”

Name the epithelial cell specialization described by the following statements:Allows adjacent cells to communicate via connexonsGap junction. Note: miss ing in cancer cel l sConnects cells to underlying extracellular matrixHemidesmosomesExtends around entire perimeter; contains E-cadherin and actin filamentsZona adherensPrevents diffusion across intracellular space; extends around entire perimeterZona occludens (tight junction)Small, discrete sites of attachment; contains desmoplakin and keratinMacula adherens (desmosome)

Helical array of polymerized dimmers of α and β tubulinMicrotubules

How are the internal structures of cilia organized?9 + 2 arrangement of microtubules (9 doublets around 2 centra l microtubules )

Which enzyme causes bending of cilia and how does it work?Dynein i s an adenos ine triphosphatase (ATPase) that l inks the 9 doublets and causes bending by di fferentia l s l iding.

Name the cytoskeletal elements that perform the following functions:Microvilli, muscular contraction, cytokinesis, adhering junctionActin and myos inCilia, flagella, mitotic spindle, neurons, centriolesMicrotubulesVimentin, desmin, cytokeratin, glial fibrillary acid protein, neurofilamentsIntermediate fi laments

Where are apocrine sweat glands found?Axi l la , mons pubis , and anal regions

PATHOLOGY

Give the dermatologic term for each of the following descriptions:

Flat, nonpalpable, circumscribed lesion 1 cm in diameter; different color than surrounding skinMaculeA macule > 1 cm in diameterPatchPalpable, solid, elevated skin lesion Papule ≤5 mm in diameterPapuleA papule > 5 mm in diameterPlaqueFluid-containing blister 0.5 cm in diameter

Ves icleFluid-containing blister > 0.5 cm in diameterBul laPus-filled, raised areaPustuleSolid, round lesion; diameter = thickness, > 5 mm in diameterNodule

Name the dermatologic disorder characterized by the following descriptions:Autosomal recessive (AR) defect in melanin synthesis → predisposition to multiple skin disordersAlbinism (oculocutaneous)Acquired loss of epidermal melanocytes → depigmented white patchesViti l igoMasklike facial hyperpigmentation associated with pregnancyMelasmaTan-brown, evenly pigmented, localized overgrowth of melanin-forming cells of the skin present at birth with benign behavior and variable histologyNevocel lular nevus (mole)Often multiple, atypical, irregularly pigmented lesions and on non-sun-exposed skin, that have the potential to transform into malignant melanomaDysplastic nevusEruption of comedones and pustules; ↑ during puberty and adolescence; associated with proliferation of PropionibacteriumAcne vulgarisUmbilicated, pearly, dome-shaped papules typically occurring in the genitals; caused by poxvirus infectionMol luscum contagiosumBenign papilloma caused by HPV infection, most commonly found on dorsum of hand; koilocytes are characteristicVerruca vulgaris (common wart)Common benign neoplasm of older adults; sharply demarcated, tan-brown plaques with a “pasted on” appearanceSeborrheic keratos is (seni le keratos is )Benign, flesh-colored, dome-shaped, > 1 cm nodule with central kera n-filled plug-craterlike that resembles squamous cell carcinoma; may resolvewithout treatmentKeratoacanthomaYellowish papules or nodules that tend to occur on the eyelids; associated with hypercholesterolemiaXanthoma (on the eyel ids = xanthelasma)Accumulation of excessive dermal collagen that can occur following skin trauma; results in large, raised tumorlike scarKeloidProliferation of Langerhans cells; electron microscopy (EM) shows Birbeck granulesHistiocytos is XA T-cell lymphoproliferative disease arising in the skin; initially simulates eczema or other inflammatory dermatosesMycos is fungoides (cutaneous T-cel l l ymphoma)Rough, scaling epidermal lesion, usually 1 cm, due to chronic sunlight exposure; may be a precursor for squamous cell carcinomaActinic keratos isThickened, hyperpigmented skin in the flexural areas; may be suggestive of visceral malignancyAcanthos is nigricansCapillary hemangioma appearing as a purple-red area on the face or neckPort-wine s ta in

Name the inflammatory dermal lesion associated with each of the following findings:Pruritic, inflammatory disorders due to infection, atopy, chemicals, UV light, or repeated traumaDermati ti sCharacteristic “target” macule or papule; associated with infections, drugs, cancers, and autoimmune diseaseErythema multi formeSilvery scaling plaques over the knees, Psoriasis elbows, and scalpPsorias is“Saw toothing” of rete ridgesLichen planusMunro microabscesses in the stratum corneumPsorias isPruitic eruption commonly on flexor surfaces. Associated with asthma and allergic rhinitisAtopic dermati ti s (eczema)Wickham striaLichen planusPurple, pruritic, polygonal, papulesLichen planusCharacteristic rete elongation and parakeratosisPsorias isFever combined with erosions and hemorrhagic crusting of mucosal surfacesStevens-Johnson syndromeType IV hypersensitivity reaction following exposure to allergens such as poison ivy or poison oakAl lergic contact dermati ti sSometimes associated with severe destructive rheumatoid arthritis-like lesions of the fingersPsorias is (psoriatic arthri ti s )

Name the blistering dermal disease associated with each of the following descriptions:Subepidermal bullae causing detachment of the entire thickness of the epidermisBul lous pemphigoidPruritic subepidermal blisters occurring in groups; eosinophils and IgA deposits at tips of dermal papillae; seen in patients with celiac diseaseDermati ti s herpeti formisIntraepidermal/suprabasal blisters that often rupture; may be fatalPemphigus vulgarisImmunofluorescence demonstrates linear deposition of complement and antibodies to hemidesmosome proteins BPAG1 and BPAG2Bul lous pemphigoidIgA and IgG antibodies to glutenDermati ti s herpeti formis

Antibodies (Abs) to the desmosomal protein desmoglein 3 in the macula adherensPemphigus vulgaris

Name the neurocutaneous syndrome characterized by each of the following features:Port-wine stains of the face, ipsilateral glaucoma, retinal lesions, and hemangiomas of the meningesSturge-Weber syndromeHypopigmented macules (ash-leaf spots), adenoma sebaceum, seizures, and mental retardationTuberous scleros isMultiple organ hemangioblastomas, cysts, and paragangliomas throughout the bodyvon Hippel -Lindau diseaseCafé au lait spots, acoustic neuromas, and meningiomasNeurofibromatos is

Which syndrome is characterized by mycosis fungoides, erythema, and scaling?Sézary syndrome, cutaneous T-cel l l ymphoma

Connective Tissue Disorders

Name the connective tissue disorder characterized by each of the following descriptions:

Immune complex deposi on in almost any organ, characteris c bu erfly malar rash, wire loop lesions in kidney, Libman-Sacks endocardi s, ANA, an -dsDNA, anti-Smith antibodiesSystemic lupus erythematousCalcinosis, Raynaud phenomenon, esophageal dysfunction, sclerodactyly, telangiectasias; anticentromere antibodiesCREST syndromeAutosomal dominant (AD) mutation in the fibrillin-1 gene (FBN1) on chromosome 15qMarfan syndromeMost common form is an AD defect in collagen type I synthesis; may be confused with child abuseOsteogenes is imperfectaProximal muscle weakness, elevated serum creatine kinase, less often associated with malignancyPolymyos i ti sAbnormal collagen synthesis causing bleeding tendency, hypermobile joints, and hyperextensible skinEhlers -Danlos syndromeBlue sclera and brittle bonesOsteogenes is imperfectaProximal muscle weakness, heliotrope rash, more often associated with malignancyDermatomyos i ti sAntinuclear ribonucleoprotein (anti-nRNP) antibodiesMixed connective ti ssue diseaseWidespread visceral involvement; anti-Scl-70 antibodiesDiffuse sclerodermaTriad of xeropthalmia (dry eyes), xerostomia (dry mouth), arthritis; anti-Ro, anti-La antibodiesSjögren syndrome

Skin Cancer

Name the skin malignancy associated with each of the following statements:

The most common skin malignancyBasa l cel l carcinomaAssociated with excessive sunlight exposure; may arise from dysplastic nevus cellsMal ignant melanomaActinic keratosis is a precursor lesionSquamous cel l carcinomaLocally aggressive, ulcerating, and hemorrhagic; almost never metastasesBasa l cel l carcinomaOccurs in sun-exposed areas and tends to involve the lower part of the faceSquamous cel l carcinomaOccurs in sun-exposed areas and tends to involve the upper part of the faceBasa l cel l carcinomaHistopathology characterized by “keratin pearls”Squamous cel l carcinomaHistopathology shows darkly staining cells with palisading nucleiBasa l cel l carcinomaRadial phase precedes vertical growth phaseMal ignant melanomaAssociated with arsenic and radiation exposureSquamous cel l carcinoma

What are the characteristics worrisome for malignant melanoma?ABCDE’sAsymmetryBorder (i rregular)ColorDiameterEvolving

What is the most important prognostic factor in malignant melanoma?Depth of invas ion

What clinical variant of malignant melanoma has the poorest prognosis?Nodular melanoma

What clinical variant of malignant melanoma often appears on the hands and feet of dark-skinned people?Acra l -lentiginous melanoma

Miscellaneous Skin Disorders

Name the dermatologic finding(s) associated with each of the following diseases:

Gastric adenocarcinomaAcanthos is nigricansAddison diseaseHyperpigmentation and s triaeRheumatic feverErythema marginatumKawasaki syndromeErythematous pa lms and soles ; dry, red l ips ; desquamation of fingertipsInsulin resistance, diabetes mellitus type 2Acanthos is nigricansSézary syndromeMycos is fungoides (lymphoma of the skin-s imulating eczema)Severe chronic renal failureUremic frostBacterial endocarditisOsler nodes (tender, ra ised les ions on pads of fingers or toes) and Janeway les ions (smal l , erythematous les ions on pa lms or soles )Xeroderma pigmentosumDry skin and melanomaHenoch-Schönlein purpuraPurpuric les ions on extensor surfaces of arms, legs , buttockHypothyroidismCool , dry skin with coarse bri ttle ha i rGraves diseaseWarm, mois t skin with fine ha i r; pretibia l myxedemaGraft-versus-host diseaseMaculopapular rashvon Recklinghausen disease (NF-1)Multiple café au la i t spotsFamilial hypercholesterolemiaXanthomasSystemic lupus erythematosusMalar rash and photosens i tivi tyPellagraDermati ti s

Name the dermatologic finding(s) associated with each of the following infectious diseases:AnthraxVes icular papules covered by black escharParvovirus B19Erythema infectiosum (slapped cheek appearance)Lyme diseaseErythema chronicum migransPrimary syphilisPainless chancreSecondary syphilisRash over pa lms and soles , condyloma latumRocky Mountain spotted feverRash over pa lms and soles (migrates centra l ly)Congenital cytomegalovirusPinpoint petechia l “blueberry muffin” rashHPV (in genital region)Condylomata acuminataHerpes simplex virus (type 1)Painful ves icles (at border of l ips )LeprosyHypopigmented, anesthetic skin patches

CHAPTER 12Musculoskeletal

ANATOMY

Name the peripheral nerve or region of the brachial plexus injured in each of the following scenarios:

Erb-Duchenne or Waiter’s tip palsyUpper trunk of the brachia l plexus (C5, C6)Klumpke injury due to sudden upward jerk of the arm; associated with Horner syndromeLower trunk of the brachia l plexus (C8, T1)Claw hand (impaired wrist flexion and adduction)Ulnar nerveWrist dropRadia l nerveVague pain in wrist with tingling, burning sensation in hand (carpal tunnel syndrome)Median nerveDeltoid paralysisAxi l lary nerveWinged scapula after mastectomy (paralyzed serratus anterior)Long thoracic nerveFoot dropCommon peroneal nerveLoss of ability to plantarflexTibia l nerveAnterior shoulder dislocationAxi l lary nervePositive distal tingling on percussion of anterior wrist (Tinel) and tingling on forced flexion (Phalen) testsMedian nerve (at wris t)Anterior hip dislocation resulting in loss of adduction of the thighObturator nerveLoss of ability to rise from a seated position or climb stairs due to loss of gluteus maximus functionInferior glutea l nervePositive Trendenlenburg sign (when contralateral leg is raised, contralateral hip falls secondary to ipsilateral gluteus medius weakness)Superior glutea l nervePosterior hip dislocationSciatic nerve

Name the fracture associated with the following statements:Laceration of the deep brachial artery and/or radial nerveMidshaft fracture of the humerusGreatest risk of upper extremity compartment syndrome in children (Volkman contracture)Supracondylar fracture of the humerusYoung person with fall on outstretched hand, tenderness in the anatomic snuffboxScaphoid fractureCaused by closed fist striking a hard objectBoxer’s fracture or fracture of the fi fth metacarpalElderly woman falling on an outstretched hand with the wrist extendedCol les fracture (fracture of the dis ta l radius with dorsa l displacement of hand)Associated with foot drop due to injury of common peroneal nerveFracture of fibular neckLower extremity fracture caused by landing on foot from a large vertical dropCalcaneal fracture (must check lumbar spine x-rays for associated fracture)

List the muscles that make up the hypothenar eminence:Opponens digi ti minimi , Abductor digi ti minimi , Flexor digi ti minimi (OAF)

List the muscles that make up the thenar eminence:Opponens pol l i ci s , Abductor pol l i ci s brevis , Flexor pol l i ci s brevis

What are the functions of the thenar and hypothenar muscles?Oppose, Abduct, and Flex

What muscles ADduct at the metacarpophalangeal (MCP) joints?Palmar interosseous muscles (PAD)

What muscles ABduct at the MCP joints?Dorsa l interosseous muscles (DAB)

What ligaments make up the borders of the anatomic snuffbox?Extensor pol l i ci s longus , extensor pol l i ci s brevis , abductor pol l i ci s longus

What artery passes through the anatomic snuffbox?Radia l artery

How can one test for radial or ulnar artery patency?Al len test (used before arteria l blood sampl ing from the radia l artery)

What structures are damaged by lateral impact to the knee/twisting injury?The terrible triad: anterior cruciate l igament (ACL), media l col latera l l igament (MCL), and media l mens icus

What does a positive anterior drawer/Lachman sign suggest?Torn ACLWhat does abnormal passive abduction (valgus instability) of the leg suggest?Torn MCLWhat is the most common site for a clavicular fracture?

Middle one-thi rdWhat is the term for increased pressure within a fascial compartment that causes damage to muscles and neurovascular structures?

Compartment syndromeWhat are the six P’s of compartment syndrome?1. Pain out of proportion/pain with pass ive s tretch2. Parasthes ias3. Para lys is4. Pal lor5. Pulselessness6. Poiki lothermia (cold)What ligaments can be stretched or torn by inversion of the ankle?Anterior ta lofibular l igament (most common, “ Always Tears Fi rs t”), ca lcaneofibular (second most common), posterior ta lofibular l igament(least common)What common carpal bone fracture can lead to avascular necrosis?Scaphoid fractureWhat are the muscles of the rotator cuff?Supraspinatus , Infraspinatus , Teres minor, Subscapularis (SITS) musclesWhat term is used to describe the syndrome of pain on extension of the wrist and fingers?Latera l epicondyl i ti s (tennis elbow)

Name the syndrome characterized by sensory loss of the medial forearm and hand, disappearance of radial pulse on turning head away from affected side,with atrophy of thenar, hypothenar, and interosseous muscles:

Thoracic outlet syndrome (often due to cervica l accessory rib)Fracture that presents with leg in abduction, external rotation, and appearing shorter than the contralateral leg

Femoral neck fractureInjury that presents with an internally rotated and adducted leg appearing shorter than the contralateral leg?

Posterior dis location of the hip

PHYSIOLOGY

State the key cellular events in excitation-contraction coupling in skeletal muscle:

1. Action potentia ls cause depolarization of T tubules .2. Ca 2+ release from sarcoplasmic reticulum.3. Ca 2+ binds troponin C, causes conformational change.4. Tropomyos in moves to expose actin-binding s i te.5. Actin and myos in interact to generate contracti le force.6. Ca 2+ reuptake by sarcoplasmic reticulum.

How does Ca2+ activate contraction in smooth muscle cells?By binding to ca lmodul in, activating myos in l ight-chain kinase (MLCK)

Name the type(s) of muscle fiber associated with each of the following cellular or histologic features:Peripherally located nucleusSkeleta l muscle fibersCentrally located nucleusSmooth and cardiac muscle fibersDistinct banding patternCardiac and skeleta l muscle fibers ; bands are appearance of sarcomeresCapacity to regenerateSmooth muscle fibersZ-linesCardiac and skeleta l muscle fibers ; Z-l ines are borders that separate sarcomeresGap junctionsSmooth muscle fibersIntercalated disksCardiac muscle fibersSynapse with peripheral nervesSkeleta l muscle fibersInositol triphosphate (IP3)-mediated calcium releaseSmooth muscle fibersCa2+-mediated calcium releaseCardiac muscle fibersVoltage-mediated, T-tubule-mediated calcium releaseSkeleta l muscle fibersTroponin is the major calcium-binding proteinCardiac and skeleta l muscle fibers

Name the type of skeletal muscle fiber associated with each of the following features:Slow twitchType 1—one slowFast twitchType 2Abundant lipid storesType 1—fatRed colorType 1—redWhite colorType 2Primarily uses anaerobic metabolism, few mitochondriaType 2Primarily uses aerobic metabolism, many mitochondria

Type 1—oxAbundant glycogen storesType 2Generation of a sustained forceType 1Generation of a sudden movementType 2

Mneumonic for type 1 fibersRemember: “One Slow, Fat, Red Ox”

MUSCULOSKELETAL AND CONNECTIVE TISSUE PATHOLOGY

Nonneoplastic Bone Disorders

Name the bone disease associated with the following clinical and pathologic features:

AD disorder characterized by short limbs due to narrow epiphyseal plates, normal torso, enlarged head, frontal bossing, and bow legsAchondroplas ia , most common cause of dwarfi smA group of disorders characterized by abnormalities of type 1 collagenOsteogenes is imperfectaInadequate proline and lysine hydroxylation of procollagenScurvyVitamin D deficiency → failure of bone mineralizationRickets (chi ldren) or osteomalacia (adults )Vitamin C deficiency → bone lesions, bleeding from gums. and petechial hemorrhagesScurvyAssociated with blue sclera and multiple fracturesOsteogenes is imperfecta↑ Parathyroid hormone (PTH) → “Brown tumor” of bone (fibrosis, giant cells, osteoclasts hemorrhagic debris, and cyst formation)Ostei ti s fibrosa cystica (von Reckl inghausen disease of bone)Slow paramyxovirus infection of osteoblasts and osteoclasts that results in multiple fractures and ↑ serum alkaline phosphatasePaget disease of bone (ostei ti s deformans)Marrow fibrosis, moth-eaten bones on x-ray (XR), and metastatic calcificationsOstei ti s fibrosa cystica (von Reckl inghausen disease of bone), caused by hyperparathyroidismMultiple lytic lesions of spine and skullMultiple myelomaProgressive decrease in bone mass most pronounced in menopausal womenOsteoporos isHereditary disorder of increased bone density caused by defective osteoclast function and bone overgrowthOsteopetros isLateral curvature of spine, usually with rotational component; most common in adolescent femalesScol ios isSubperiosteal hemorrhage, failure of ephiphyseal cartilage replacement by osteoid, osteoporosisScurvy (vi tamin C deficiency)Delayed fontanelle closure, “Rachitic rosary” (thickening of costochondral junction), and Harrison grooveRickets (vi tamin D deficiency)Marked cortical thinning and attenuation of bone trabeculaeOsteogenes is imperfectaPoor calcification of bone leading to skeletal abnormalities, including bowing of legs, craniotabes, and pigeon breast deformity in childrenRicketsMosaic pattern of lamellar bone, increased serum alkaline phosphatasePaget disease of bone (ostei ti s deformans)Bone infection → sequestrum and involucrum around the inflammatory focusPyogenic osteomyel i ti sInfarction of osteocytes leading to joint pain and osteoarthritisAvascular necros isAvascular necrosis of the head of the femur; typically presents with painless limp in obese adolescentLegg-Calve-Perthés diseasePartial avulsion of tibial tuberosity; typically presents as knee pain in an active adolescentOsgood-Schlatter diseaseBacterial infection of bone, causing an acute febrile illness and localized pain and inflammation at the site of infectionPyogenic osteomyel i ti sGranulomatous disease caused by spread of tuberculosis to spinePott diseaseVertebral compression fractures causing pain, kyphosis, and loss of height in the elderlyOsteoporos is

What is the most likely etiology of Paget diease?Slow reaction to paramyxovirus infection of osteoblasts

What are the three phases of Paget disease of bone?1. Osteolytic phase: bone reabsorption by large osteoclasts2. Mixed phase: osteoblastic and osteoclastic activi ty resul ts in mosaic pattern of bone3. Burnt-out phase: osteoblastic activi ty predominates

Name six complications of Paget disease of bone:1. High-output cardiac fa i lure due to intraosseous atrioventricular (AV) shunting2. Hearing loss3. Leonitias is ossea4. Osteosarcoma

5. Osteoarthri ti s6. Bone pa in from fractures

Osteomyelitis

Name the most common organism(s) responsible for pyogenic osteomyelitis in each of the following patients:

Otherwise healthy adultStaphylococcus aureusIntravenous drug userPseudomonas spp.Sickle cell anemia patientSalmonellaNewbornStreptococci spp. or Escherichia coliChildStaphylococcus aureusMost common mechanism of seeding for adultsTraumaticMost common mechanism of seeding for childrenHematogenous

Neoplasia of Bone

Name the bone tumor associated with the following clinical and pathologic findings:

Most common bone tumorMetastatic tumors to boneMost common primary bone tumorMultiple myelomaMost common benign tumor of boneOsteochondroma or exostos isMost common primary malignant tumor of boneOsteosarcomaBenign sessile tumor attached to the bone surface, usually affecting skull and facial bonesOsteomaBenign tumor (2 cm) that is painful at night (due to excess PGE2), relieved with aspirin and common in males 25 years old (y/o)Osteoid osteomaMalignant tumor typically occurring in the metaphyseal region prior to epiphyseal closure in patients 25 y/oOsteosarcomaMushroom-shaped, laterally protruding tumor that may result from lateral displacement of the growth plateOsteochondroma or exostos isBenign tumor composed primarily of mature hyaline cartilageChondromaBenign tumor composed of fibrous trabeculae of woven bone resembling Chinese charactersFibrous dysplas iaNodule of hyaline cartilage encased in reactive boneEnchondromaMalignant, painful, small round blue cell tumor of childhood occurring typically in the appendicular skeleton (may also affect ribs)Ewing sarcoma/primitive neuroectodermal tumor (PNET)Malignant tumor composed of multinucleated giant cells within a fibrous stroma, occurring in the epiphyses of long bonesGiant cel l tumor of boneLacelike pattern of bone produced by tumor cellsOsteosarcomaHomer-Wright pseudorosettes, onion skin appearance on x-rayEwing sarcoma/PNETMalignant tumor of cartilage found in the central skeletonChondrosarcomaDouble bubble or soap bubble appearance on XRGiant cel l tumor of boneBenign but painful bone tumor that appears as a radiolucent nidus surrounded by dense boneOsteoid osteoma or osteoblastomaCodman triangle on x-ray forms as tumor, causing periosteal elevationOsteosarcomaOnion skin appearance on x-rayEwing sarcoma

What is the most common genetic defect associated with Ewing sarcoma/PNET?t(11;22)(q24;q12) trans location (85%) associated with an EWS-FLI1 fus ion gene product

Mutation of what gene increases the risk of osteosarcoma by 1000 X x?Rb gene

What malignancies are most likely to metastasize to bone?Breast, lung, thyroid, kidney, and prostate (BLT with a Kosher pickle)

What pediatric disease is characterized by the triad of skull lesions, diabetes insipidus, and exophthalmos?Hand-Schül ler-Chris tian disease or his tiocytos is X

What pediatric disease is characterized by polyostotic fibrous dysplasia, cafe au lait spots, precocious puberty, and other endocrine disorders?McCune-Albright syndrome

Arthritis

Name the arthritic joint disease associated with the following clinical and pathologic findings:

Most common type of arthritisOsteoarthri ti sMost common type of infective arthritis and most common cause of arthritis in sexually active adultsGonococca l arthri ti sAnti-IgG Fc antibodiesRheumatoid arthri ti s (RA); anti -IgG Fc antibodies are ca l led rheumatoid factorSubcutaneous rheumatoid nodulesRAHeberden (DIP) and Bouchard (PIP) nodesOsteoarthri ti sPIP and MCP involvementRA (Note: RA a lmost never affects DIP)Most commonly affects great toe (podagra)GoutPresents as pain in weight-bearing joints after use, improves with restOsteoarthri ti sInfrequent complication of psoriasis asymmetrically affecting DIP and PIP joints in the lower extremitiesPsoriatic arthri ti sSwan-neck and boutonniere deformityRAChronic low back pain, rigidity, fixation of spine causing a condition referred to as bamboo spineAnkylos ing spondyl i ti sInfection causing migratory polyarthritis and erythema chronicum migrans; may lead to pericarditis and aseptic meningitisLyme diseaseArthritic joint disease associated with inflammatory bowel diseaseAnkylos ing spondyl i ti sPresents as morning stiffness improving with use, symmetric joint involvement, systemic symptomsRAArthritis in the absence of systemic symptomsOsteoarthri ti sExtraskeletal manifestations of this arthritic disease include pulmonary fibrosis, aortic insufficiency, and cauda equina syndromeAnkylos ing spondyl i ti sTriad of conjunctivitis or anterior uveitis, urethritis, and arthritisReactive arthri ti s , previous ly know as Rei ter syndrome (Remember: can’t see, can’t pee, can’t climb a tree)Tophi (nodules of fibrous tissue) and crystals may be found on Achilles tendon or at external earGoutOsteophyte formation at edge of articular surface and at sites of ligamentous attachmentOsteoarthri ti sFilling of the joint space by granulation tissue (ie, a pannus)RAFrayed fragments of cartilage and osteophytes released into synovium forming “joint mice”Osteoarthri ti sAssociated with hyperuricemia, thiazide diuretic use, and urate kidney stonesGoutEburnation of bone due to cartilage erosionOsteoarthri ti sMarginal erosion of subchondral boneRACystic changes in bones underlying affected jointsOsteoarthri ti sPrecipitation of urate crystals in joints resulting in an inflammatory responseGoutPrecipitation of calcium pyrophosphate dihydrate crystals in joints → inflammatory responsePseudogout (aka ca lcium pyrophosphate depos i tion disease [CPPD])Infective arthritis common in individuals with complement deficiencies; associated with ↑ susceptibility to meningitisGonococca l arthri ti sAssociated with deficiency of hypoxanthine guanine phosphoribosyl transferease (HGPRT)Lesch-Nyhan syndrome (X-l inked disease which may include gout as one of i ts mani festations)Positively birefringent, rhomboid-shaped crystalsPseudogout = Pos i tively bi refringentNegatively birefringent, needle-shaped crystals in synovial fluidGoutInfective monoarticular arthritis typically causing knee pain and a pustular rashGonococca l arthri ti sInfective polyarticular arthritis caused by Borrelia burgdorferiLyme diseaseMonoarticular arthritis with gram-positive organisms on synovial fluid analysisNongonococca l septic arthri ti sStress, alcohol binge, or a large meal may precipitate an attack of this type of arthritisGoutTreatment for acute gout attackIndomethacin (NSAID), Colchicine (use l imited by GI s ide effects )Treatment for chronic goutAl lopurinol (inhibi ts xanthine oxidase), probenecid (inhibi ts reabsorption of uric acid)

Anticytokine therapy for RA is directed at which two cytokines?1. Interleukin (IL)-12. Tumor necros is factor (TNF)

What is the factor produced by activated T cells and fibroblasts that promotes bone destruction by osteoclasts?Receptor Activator of Nuclear Factor k B Ligand (RANKL), a l so known as TNF-related activation-induced cytokine (TRANCE)

What group of diseases is associated with high incidence (90%) in HLA-B27-positive patients?Seronega ve spondyloarthropathies (ankylos ing spondyl i s , reac ve arthri s , arthri s associated with inflammatory bowel disease[IBD])

Name four extra-articular manifestations of RA:1. Pleura l and pericardia l effus ions2. Acute vascul i ti s3. Inflammatory les ions of lungs , pleura , myocardium, pericardium, periphera l nerves , and eyes4. Amyloidos is (in severe, long-term disease)

What chemotactic factors are generated by urate crystal activation of complement?C3a and C5a

How is tissue injury mediated in gout?Release of lysosomal enzymes from neutrophi l s

Why are leukemia, multiple myeloma, and other neoplastic processes associated with gout?High cel l turnover releases uric acid and predisposes to an attack of gout.

What is the renal complication of longstanding gout?Urate nephropathy

Which syndrome is characterized by cutaneous pigmentation, leg ulcerations, splenomegaly, neutropenia, and RA?Fel ty syndrome

What two bacteria most commonly cause nongonococcal septic arthritis?Staphylococcus aureus and Streptococcus spp.

CHAPTER 13Behavioral Sciences

LIFE CYCLE

Development

At what age is an average child expected to do each of the following:

Hold his/her head up3 monthsSit up without support6 monthsCrawl9 monthsWalk12 monthsRide a tricycle36 months (tricycle at 3 years )Display stranger anxiety7 monthsUse a pincer grasp9 months (Remember: an ups ide-down pincer grasp forms the number “9”)Say their first word12 months (1 word at 1 year)Use two-word combinations24 months (2 words at 2 years )Use three-word sentences36 months (3 words at 3 years )Understand object permanence12 to 24 monthsToilet training30 to 36 months“No” phase, repeated temper tantrums24 months (address tantrums by ignoring behavior)Show abstract reasoning (formal operations)Adolescence/puberty

At what age are the following reflexes considered normal:Babinski0 to 12 monthsPalmar0 to 2 monthsRooting0 to 3 months

Name the components of the Apgar score:APGAR (0, 1, or 2 in each category)Appearance/color (blue/pale, trunk pink, a l l pink)Pulse (0, 100, >100)Grimace (0, grimace, grimace + cough)Activi ty/muscle tone (l imp, some, active)Respiratory effort (0, i rregular, regular)

Determine the Apgar score for these patients:Newborn with a pink trunk, heart rate (HR) = 50, a grimace and cough when stimulated, strong muscle tone, and an irregular respiratory effort7A blue newborn, HR = 30, a grimace when stimulated, appears limp, and has no respiratory effort2

What is the definition of low birth weight?Less than 2500 g

Name four sequelae of low birth weight:1. Infections2. Respiratory dis tress syndrome3. Pers is tent feta l ci rculation4. Necrotizing enterocol i ti s

What term describes the act of a child reverting to a more primitive mode of behavior due to stress?Regress ion

Name four long-term effects of depriving affection from infants:The 4 W’s:1. Weak2. Wordless3. Wary4. Wanting

What term describes depression in an infant due to sustained separation from the caregiver?Anacl i ti c depress ion (resul ts in impaired socia l , emotional , and phys ica l development)

Name three findings that are suggestive of each type of abuse listed below:

Physical child abuse1. Fractures at di fferent s tages of heal ing2. Cigarette burns3. Retina l hemorrhage/detachment (32% of kids 5 years old [y/o] are phys ica l ly abused)Sexual child abuse1. Genita l/anal trauma2. STDs3. UTIs (25% of kids 8 y/o are sexual ly abused)Elder abuse1. Evidence of depleted finances2. Poor hygiene3. Spi ra l fractures

Aging

Name the changes found in the elderly in each of the following categories:

PsychiatricDepress ion and anxiety more common; ↑ suicide rateSexualMen: s lower erection/ejaculation, ↑ refractory period; women: vagina l shortening, thinning, and drynessNote: sexual interest does not decrease.Sleep patterns↓ Rapid eye movement (REM), s low-wave s leep; ↑ s leep latency, awakeningsCognitive↓ Learning speed; intel l igence s tays the same

Name three conditions that would qualify normal bereavement as pathologic grief:1. Prolonged grief (> 1 year)2. Excess ively intense grief (s leep dis turbances , s igni ficant weight loss , suicida l ideations)3. Grief that i s delayed, inhibi ted, or denied

Name the Kübler-Ross stages of dying:Denia l , anger, barga ining, depress ion, acceptance. Note: one or more s tages can occur at once and not necessari ly in this order.

PHYSIOLOGY

Sleep

Name the sleep stage associated with each of the following descriptions:

Light sleep, peacefulness, ↓ HR and BPStage 1Deepest non-REM sleep; sleepwalking, bedwettingStages 3 and 4Deeper sleep; EEG shows sleep spindles and K-complexes; occupies ~half of total sleep time in young adultsStage 2Beta waves (↑ frequency, ↓ amplitude) onlyAwake (eyes open) and a lert REMBeta, alpha, and theta waves → “sawtoothing”REMAlpha wavesAwake (eyes closed)Dreaming, loss of motor tone, ↑ brain O2use, erections; occurs every 90 minutesREMDelta (slow) waves(↓ frequency, ↑ amplitude)Stages 3 and 4

Name the sleep disorder described in each of the following statements:Abnormal behavior associated with sleep or sleep-wake transitions (eg, sleep terrors, enuresis, somnabulism)ParasomniaDisturbance in amount, quality, or timing of sleep (eg, insomnia, narcolepsy)DyssomniaNighttime respiratory effort against an impeded airway, resulting in 10-second lapses in breathing and chronic fatigueObstructive s leep apnea

What is a useful drug for night terrors and sleepwalking?Benzodiazepines (shortens s tage 4 s leep)

Which drug treats enuresis in children by decreasing stage 4 sleep?Imipramine

What is the main neurotransmitter involved in REM sleep?Acetylchol ine (ACh)

Name four physiologic changes that occur in REM sleep:1. Increased and variable pulse2. Increased and variable BP3. Peni le/cl i tora l tumescence4. Rapid eye movements

Name four clinical findings of narcolepsy:1. Sleep para lys is (brief para lys is upon awakening)2. Hypnagogic (go ing to s leep) and hypnopompic (waking) ha l lucinations3. ↓ REM latency (s leep episodes a l l s tart in REM)

4. Cataplexy (sudden loss of muscle tone, especia l ly with extreme emotion)How is narcolepsy treated?

Stimulants (eg, amphetamines), scheduled napsWhat three sleep pattern changes are typical of depressed patients?

1. Reduced s low-wave s leep2. ↓ REM latency3. “Terminal insomnia” (early-morning awakenings)

Sexuality

Name the four stages of the sexual response cycle:

1. Exci tement2. Plateau3. Orgasm4. Resolution

Name three possible etiologies of sexual dysfunction:1. Drugs (eg, selective serotonin reuptake inhibi tors [SSRIs ], ethanol [ETOH], antihypertens ives )2. Disease (eg, depress ion, diabetes , myocardia l infarction [MI])3. Psychologic (eg, avers ion, hypoactive des i re, premature ejaculation)

What disorder of sexual function is characterized by painful spasm of the outer one-third of the vagina during intercourse or pelvic examination?Vaginismus

What is a paraphilia?Unusual sexual activi ties or sexual des i re for unusual objects (eg, pedophi l ia , voyeurism, and so forth)

Behavioral Neurochemistry

For each of the following diseases, describe the associated neurotransmitter activity:

Schizophrenia↑ Dopamine (DA) and 5-hydroxytryptamine (5-HT)Depression↓ Norepinephrine (NE) and 5-HTAnxiety↑ NE, ↓ γ-aminobutyric acid (GABA), and 5-HTAlzheimer dementia↓ ACh in Al zheimer dementiaParkinson disease↓ DAHuntington disease↓ GABA and ACh

SUBSTANCE ABUSE

What is the lifetime prevalence of substance abuse/dependence?

13%How is “dependence” defined?

Withdrawal occurs i f substance i s s topped and patient has tolerance to substance.Excluding tobacco and caffeine, what is the most commonly abused substance?

AlcoholWhat is a short, useful alcoholism screening tool?

“CAGE” questionsHave you fel t the need to Cut down?Have you ever fel t Annoyed by cri ti ci sm of your drinking?Have you ever fel t Gui l ty about drinking?Have you ever had an Eye opener?

What is the major complication of alcohol withdrawal and when is it most likely to occur?Del i rium tremens (DTs); peak occurrence i s 2 to 7 days . Note: DTs are a medica l emergency.

What is the mortality rate of DTs?15% to 20%

Name three gastrointestinal (GI) complications of alcoholism:1. GI bleeding (from ulcers , gastri ti s , esophageal varices , or Mal lory-Weiss tears )2. Pancreati ti s3. Liver disease

Which syndrome of anterograde amnesia, confabulations, ataxia, and nystagmus results from thiamine deficiency in chronic alcoholics?Wernicke-Korsakoff syndrome— Remember: Wobbly-Konfabulations (associated with bi latera l mammil lary body necros is )

What am I high on?Central nervous system (CNS) and respiratory depression, euphoria, pinpoint pupils, nausea, and ↓ GI motilityOpioids ; inspect for track marks a long veinsPsychomotor agitation, dilated pupils, euphoria, ↑ HR and BP, prolonged wakefulness and attention, delusions, ↑ pain thresholdAmphetaminesAll of the above, plus tactile hallucinations, angina, and sudden cardiac deathCocaineBelligerence, psychomotor agitation, nystagmus, ataxia, homicidality, psychosis, delirium, ↑ HR, and feverPhencycl idine hydrochloride (PCP)Delusions, visual hallucinations, postuse “flashbacks”Lysergic acid diethylamide (LSD)Euphoria, ↑ appetite, dry mouth, paranoid delusions, erythematous conjunctiva

Mari juanaDisinhibition, emotional lability, slurred speech, ataxia, blackouts, comaAlcohol

What is the drug of choice for opioid overdose?Naloxone and na l trexone (competi tively inhibi t opioid receptors )

Which drug is used for heroin detoxification?Methadone (long-acting ora l opioid)

PSYCHOLOGY

What are the three parts of Freud’s structural theory of the mind?

1. Id2. Superego3. Ego

Which of these parts is described by the following statements:Represents conscience and moral valuesSuperegoControlled by primitive wishes and pleasures; represents instinctive sexual and aggressive drivesIdBridges unconscious mind and external worldEgo

What Freudian term encompasses repressed sexual feelings of a child for the opposite-sex parent, plus a rivalry with the same-sex parent?Oedipus complex

What type of insight therapy, developed by Freud, may be useful for chronic personality problems?Psychoanalys is

What describes a scenario in which a patient’s unconscious feelings from past relationships are experienced in the present relationship with the physician?Transference reaction

What reaction occurs when the physician unconsciously reexperiences feelings about his/her parents (or other important persons) with the patient?Countertransference reaction

Name the four mature ego defense mechanisms:1. Suppress ion2. Al truism3. Subl imation4. HumorNote: defense mechanisms are automatic and unconscious .

Name the type of learning that is described by each of the following statements:Reflexive response is elicited by a learned stimulusClass ica l conditioningTendency of an organism to follow the first thing they see after birthImprintingBehavior is eliminated when not reinforcedExtinctionBehavior is determined by its consequencesOperant conditioningUnwanted behavior is paired with painful stimulusAvers ive conditioningType of classical conditioning where the subject learns that it cannot escape a painful stimulusLearned helplessness

In what type of reinforcement schedule is a reward presented after a random, unpredictable number of responses?Variable ratio (s lowest extinction when not rewarded); example = s lot machine

In what type of reinforcement schedule is a reward presented after every response?Continuous (rapid extinction when not rewarded); example = vending machine

Criteria for mental retardation begin below what IQ level?Less than 70 (2 SD below the mean of 100)

PSYCHIATRY

Name the type of amnesia described below:

Inability to remember things that occurred before insult to CNSRetrograde amnes iaInability to remember things that occurred after a CNS injury → no new memory formationAnterograde amnes iaComplication of electroconvulsive therapy (ECT)Retrograde amnes iaThiamine deficiency causing bilateral mammillary body necrosis; seen in alcoholicsKorsakoff (anterograde) amnes ia

Delirium or dementia?Waxing and waning level of consciousnessDel i riumRapid onset; transientDel i riumCharacterized by memory lossDementia (think deMEMtia )Associated with disturbances in sleep wake cycleDel i riumOften irreversibleDementia

Associated with changes in sensorium Delirium (hallucinations and illusions)Del i rium

Name four major causes of delirium:“HIDE”:1. Hypoxia2. Infection (often UTIs )3. Drugs4. Electrolyte abnormal i ties

What is the most common etiology for dementia?Dementia of Alzheimer type (DAT) = 70% to 80% of cases

Name some other etiologies for dementia:“DEMENTIASS”Degenerative diseases (Parkinson, Huntington)Endocrine (thyroid, pi tui tary, parathyroid)Metabol ic (electrolytes , glucose, hepatorenal dys function, ETOH)Exogenous (CO poisoning, drugs , heavy meta ls )Neoplas iaTraumaInfection (encephal i ti s , meningi ti s , cerebra l abscess , syphi l i s , prion diseases , HIV, Lyme disease)Affective disorders (depress ion may mimic dementia)Stroke (multi -infarct dementia , i schemia, vascul i ti s ). Note: vascular causes ~10% of dementiasStructure (normal -pressure hydrocephalus [NPH]).Note: NPH is one of the few revers ible causes of dementia .

Mood Disoders

Name the nine key features of major depressive disorder (MDD):

“SIG E CAPSS”1. Sleep changes (insomnia/hypersomnia)2. Inabi l i ty to experience pleasure, Interest decreases3. Gui l t or feel ings of worthlessness4. Energy ↓ (fatigue)5. Concentration ↓, indecis iveness increases6. A ppeti te dis turbance with weight change (> 5% body weight in 1 month)7. Psychomotor changes (agi tation or retardation)8. Suicida l ideations9. Sadness (depressed mood for most of day)

What features are required to make the diagnosis of MDD?Two episodes (involving five of the above nine including # 1 or 2) of impaired functioning for 2 weeks, separated by 2 months

What is the suicide rate in MDD?Approximately 15% to 30%

Name the risk factors for suicide:“SAD PERSONS”Sex—male (women > attempts ; men > actua l suicides )Age (↓ 15-24 and the elderly)Depress ionPrevious attempts = #1 ri sk factorEthanolRational thoughtSi cknessOrganized planNo spouseSocia l support lacking

What is the first-line pharmacotherapy for MDD?SSRIs

Name two other alternate pharmacotherapies:1. Tricycl ic antidepressants (TCAs)2. Monoamine oxidase inhibi tors (MAOIs )

What is a safe, effective treatment for refractory MDD?ECT

What is the distinctively abnormal, elevated, expansive mood that lasts > 1 week or is severely impairing (eg, requiring hospitalization)?Manic episode

What are the seven key features of mania?“DIG FAST” (at least three of the fol lowing for diagnos is ):1. Di s tractibi l i ty2. Insomnia3. Grandios i ty4. Fl ight of ideas or racing thoughts5. Psychomotor Agi tation6. Speech that i s pressured (hyperverbal )7. Thoughtlessness (↑ pleasurable activi ties with ↑ consequences)

Name the following mood disorders:Chronic disorder > 2 years (alternating hypomania and mild depression); no period of euthymia > 2 months and no significant impairmentCyclothymiaLess severe features for several days that are not impairing; no psychotic featuresHypomaniaHistory of major depressive episodes and at least one hypomanic episodeBipolar I I disorder

Manic episodes that often alternate with depressive episodesBipolar I disorder

Somatoform Disorders

Name the somatoform disorder associated with each of the following descriptions:

Preoccupation with an imagined physical defect, causing significantly impaired social and occupational functioningBody dysmorphic disorderMultiple, unrelated physical complaints leading to excessive medical attention seeking and severely impaired functioningSoma za on disorder (requires four pa in, two GI, one sexual/GU, and one pseudoneurologic compla ints ). Note: cannot be inten onal orfakeProlonged preoccupa on with concerns of having a serious illness (despite nega ve medical work-ups) and exaggerated a en on to bodily or mentalsensationsHypochondrias isConscious simulation of physical or psychologic illness solely to receive attention from medical personnelFacti tious disorder (Munchausen syndrome). Note: technica l ly not a somatoform disorder because i t i s intentionalIntentionally simulating illness for personal gain (usually financial)Mal ingering. Note: a l so not a somatoform disorder; suspect in cases involving l i tigationSudden onset of motor/sensory neurologic disorder following traumatic emotional eventConvers ion disorder

Name the type of gain associated with each of the following descriptions:Interpersonal or social advantages gained indirectly from illnessSecondary ga inBenefits of illness on the patient’s internal psychic economyPrimary ga inAdvantage gained by the caretakerTertiary ga in

Name the type of anxiety disorder associated with each of the following:Maladaptive reaction to environmental or psychologic stress that interferes with functioning and does not remit after the stress endsAdjustment disorderMarked, persistent fear of an object/situation that is excessive and unreasonable → stimulus is avoided; patient has insight (treat by exposure therapy)Speci fic phobiaOccurs after a person is subjected to a traumatic event; lasts >1 month; may be debilitatingPosttraumatic s tress disorder (PTSD)Symptoms of PTSD that occur within 4 weeks of the stressor and last 4 weeksAcute s tress disorderMoments of intense fear characterized by palpitations, choking sensation, GI upset, perspiration, chest pain, and chillsPanic disorderExcessive worrying for the majority of days over the past 6 months that causes significant impairmentGenera l i zed anxiety disorderRecurrent, intrusive, senseless thoughts and impulses; plus the repetitive behaviors driven by the will to decrease the anxiety caused by themObsess ive-compuls ive disorder (OCD)

Name the dissociative disorder characterized by the following statements:Two or more separate personalities in one individual; ↑ in women and sexually abusedDissociative identi ty disorder (formerly multiple personal i ty disorder)A complete, often transient, inability to remember important personal informationDissociative amnes iaAmnesia plus sudden wandering from home and taking on a different identityDissociative fugue

Psychoses

Give the appropriate term for each of the following psychotic symptoms:

False belief or wrong judgment held with conviction despite incontrovertible evidence to the contraryDelus ionFalse perception of an actual external stimulusI l lus ionThought disorder whereby ideas are not logically connected to those that occur before or afterLoose associationMisinterpreting others’ actions or environmental cues as being directed toward one’s self when, in fact, they are notIdeas of referenceSubjective perception of an object or event when no such external stimulus existsHal lucination

Name the psychotic disorder characterized by each of the following findings:Two or more psychotic symptoms and disturbed behavior for > 6 months; results in impaired functioningSchizophreniaPsychotic symptoms lasting 1 to 6 monthsSchizophreni form disorderPsychotic symptoms lasting > 1 day, but 1 month (often with obvious precipitating psychosocial stressor)Brief psychotic disorderFixed, nonbizarre delusional system; without other thought disorders or impaired functioningDelus ional disorderSymptoms of major mood disorder as well as of schizophrenia (with psycho c features occurring before mood disturbance); chronic social andoccupational impairmentSchizoaffective disorderClouded consciousness, predominantly visual hallucinations, often occurring in inpatient settingPsychotic disorder due to a genera l medica l condition

Adopting the delusional system of a psychotic personShared psychotic disorder (Fol ie-à-deux)

Name the five subtypes of schizophrenia:1. Disorganized2. Catatonic3. Paranoid4. Undi fferentiated5. Res idual

Give two examples of positive symptoms of schizophrenia:1. Hal lucinations2. Delus ions

Positive symptoms respond best to what type of drugs?Typica l antipsychotics

Give four examples of negative symptoms that are characteristic of schizophrenia:The 4 A’s:1. Affect flat2. Alogia3. Anhedonia4. Avol i tion

Negative symptoms respond best to what type of drugs?Atypica l antipsychotics . Note: worse prognos is i f negative symptoms predominate

Childhood Disorders

Name the disorder of childhood described by each of the following statements:

Repe ve behaviors (in pa ent 18 y/o) that violate social norms; may exhibit physical aggression, cruelty to animals, vandalism, and robbery, alongwith truancy, cheating, and lyingAtypica l antipsychotics . Note: worse actions)Recurrent pattern of negativistic, hostile, and disobedient behavior toward authority figures; loss of temper and defiance (but not theft or lying)Oppos i tional defiant disorder (predominantly words)Developmentally inappropriate degrees of inattention, impulsiveness, and hyperactivity at home, in school, and in social situationsAttention defici t hyperactivi ty disorder (ADHD)Pervasive developmental disorder (PDD) with stereotyped movements and nonprogressive impairments in social interac ons, communica on, andbehaviorAutismProgressive syndrome of autism, dementia, ataxia, and purposeless hand movements; regression of development; mainly in girlsRett syndromePDD with severe impairment in social skills and repe ve behaviors, leading to impaired social and occupa onal func oning but without significantdelays in language developmentAsperger disorder

Personality Disorders

List the three cluster A personality disorders:“Weird”1. Paranoid2. Schizoid3. Schizotypal

List the four cluster B personality disorders:“Wild”1. His trionic2. Borderl ine3. Antisocia l4. Narciss i s tic

List the three cluster C personality disorders:“Worried”1. Avoidant2. Obsess ive-compuls ive3. Dependent

Name the personality disorder characterized by each of the following statements:Socially inhibited, sensitive to rejection, feels inferiorAvoidant (C)Peculiar appearance, interpersonal awkwardness, “magical” or odd thought patterns, no psychosisSchizotypal (A)Impulsive, unstable mood, chaotic relationships, feels empty and alone, self-mutilation, females >> malesBorderl ine (B)Sense of entitlement, grandiosity, lacks empathy for others, insists on special treatment when illNarciss i s tic (B)Suspicious and distrustful, uses projection as primary defense mechanismParanoid (A)Lacks self-confidence, submissive, and clingyDependent (C)Unable to maintain intimate relationships, extroverted, melodramatic, sexually provocativeHistrionic (B)Disregards and violates rights of others, criminality, males > females; if 18 y/o = conduct disorderAntisocia l (B)Lifelong pattern of voluntary social withdrawal, no psychosis, shows minimal emotions

Schizoid (A)

PHARMACOLOGY—PSYCHOPHARMACOLOGY

For each of the following drugs, provide:1. The mechanism of action (MOA)2. Indication(s) (IND)3. Significant side effects and unique toxicity (TOX) (if any)

Antidepressants

TCAs (imipramine, clomipramine, amitriptyline, desipramine, nortriptyline, doxepin, amoxapine)MOA: prevents reuptake of NE and 5-HTIND: depress ion, enures is (imipramine), depress ion in elderly (nortriptyl ine), OCD (clomipramine), depress ion with psycho c features(amoxapine), neurologic pa inTOX: seda on (des ipramine i s least seda ng), an chol inergic effects , letha l in overdose → respiratory depress ion, hyperpyrexia , and Tri-C’s: Cardiac arrhythmia, Convuls ions , ComaSSRIs (fluoxetine, paroxetine, sertraline, citalopram, fluvoxamine, escitalopram)MOA: selectively blocks reuptake of 5-HT (usual ly requires 2-3 weeks to take effect)IND: depress ion, premenstrua l syndrome (fluoxetine), OCD (fluvoxamine)TOX: agi tation, insomnia , sexual dys function, “serotonin syndrome” with MAOIs (muscle rigidi ty, hyperthermia, cardiovascular col lapse)BupropionMOA: heterocycl ic agent, mechanism not wel l knownIND: depress ion, smoking cessationTOX: agi tation, seizures , insomnia (↓ sexual s ide effects )TrazodoneMOA: mainly inhibi ts serotonin reuptakeIND: depress ionTOX: postura l hypotens ion, sedation, priapismVenlafaxineMOA: inhibi ts 5-HT and NE reuptakeIND: depress ion, genera l i zed anxiety disorderTOX: s timulant effects , minimal effects on P-450MirtazapineMOA:5-HT2 receptor antagonis t and α2-antagonis t →↑ NE and 5-HT releaseIND: depress ionTOX: sedation, ↑ appeti te, ↑ cholesterolMAOIs (TIP: T ranylcypromine, I socarboxazid, phenelzine)MOA: nonselective MAOIsIND: a typica l depress ions , anxiety disorders , pa in disorders , eating disorderTOX: hypertens ive cri s i s with tyramine or meperidine ingestion, “serotonin syndrome” with SSRIsLithiumMOA: prevents generation of inos i tol triphosphate (IP3) and diacylglycerol (DAG) 2° messenger systemsIND: bipolar disorder (prevents and treats acute mania)TOX: hypothyroidism, nephrogenic DI, teratogenes is (Ebstein anomaly)CarbamazepineMOA: blocks sodium channels and inhibi ts action potentia lsIND: bipolar disorder, mixed episodes , and rapid cycl ing formTOX: hematologic SEs , elevated LFTs , teratogenes is (neura l tube defects )Valproic acidMOA: increases CNS levels of GABAIND: bipolar disorder, mixed manic episodes , and rapid cycl ing formTOX: hepatotoxici ty, thrombocytopenia , teratogenes is (neura l tube defects )

Antipsychotics

What is the name for the stereotyped oral-facial movements that occur as a result of long-term antipsychotic use?

Tardive dyskines iaDescribe the chronology of extrapyramidal side effects from neuroleptic medications:

Rule of 4’s: 4 hours—acute dystonia , 4 days—akines ia , 4 weeks—akathes ia , 4 months—tardive dyskines ia (usual ly i rrevers ible)What is the characteristic triad of neuroleptic malignant syndrome (NMS)?

Muscle rigidi ty, autonomic instabi l i ty, and hyperpyrexiaWhat is the treatment for NMS?

Dantrolene and DA agonis tsFor each of the following drugs, provide:1. The mechanism of action (MOA)2. Indication(s) (IND)3. Significant side effects and unique toxicity (TOX) (if any)

Typical antipsychotics—high potency (haloperidol, perphenazine, trifluoperazine)MOA: block D2 DA receptors (less blockade of α2, muscarinic, and his taminic receptors )IND: schizophrenia , psychos is (especia l ly pos i tive symptoms)TOX:↑ neurologic (eg, extrapyramidal ) SEs , NMS, tardive dyskines ia , Parkinsonism, hyperprolactinemiaTypical antipsychotics—low potency (chlorpromazine, thioridazine)MOA: block D2 DA receptors (more blockade of α2, muscarinic, and his taminic receptors )IND: schizophrenia , psychos is

TOX:↓ neurologic SEs , ↑ an chol inergic and endocrine SEs ; cardiac conduc on defects and re nal pigmenta on (thioridazine), cornealand lenticular depos i ts (chlorpromazine)Atypical antipsychotics (clozapine, risperidone, olanzapine, quetiapine)MOA: block 5-HT2; block D4 and D1 > D2 receptorsIND: schizophrenia , psychos is (especia l ly negative symptoms); OCD/anxiety disorder (olanzapine)TOX:↓ an chol inergic and extrapyramidal symptoms (EPS); metabol ic syndrome (hyperl ipidemia, glucose intolerance), agranulocytos is(clozapine)

MEDICAL ETHICS

What are four key components to informed consent?

The patient must:1. Understand the heal th impl ications of thei r diagnos is2. Be informed of ri sks , benefi ts , and a l ternatives to treatment3. Be aware of outcome i f they do not give thei r consent4. Have the right to withdraw consent at any time

Name four exceptions to informed consent:1. Patient not lega l ly competent to make decis ions2. In an emergency (impl ied consent)3. Patient waives the right to informed consent4. Therapeutic privi lege—withholding info that would severely harm the patient or undermine decis ion-making capaci ty i f revealed

What are five situations in which parent/legal guardian consent is not required to treat a minor?1. Emergencies2. STDs3. Prescription of contraceptives4. Treatment of ETOH/drug treatment5. Care during pregnancy

What four criteria qualify a minor as emancipated?1. Sel f-supporting2. In the mi l i tary3. Being married4. Having chi ldren that they support

What type of directive is based on the incapacitated patient’s prior statements and decisions?Oral advance di rective (substi tuted judgment s tandard)

What type of written advance directive gives instructions for the patient’s future health care should he/she become incompetent to make decisions?Living wi l l

What type of document allows the patient to designate a surrogate to make medical decisions in case the patient loses decision-making capacity?Durable power of attorney

Name the ethical responsibility of the physician described by each of the following statements:Requires physicians to “do no harm”NonmaleficenceRequires physicians to act in the best interests of the patientBeneficence (may confl i ct with patient autonomy)Demands respect for patient privacy and autonomyConfidentia l i ty

Name five exceptions to confidentiality:1. Suspected chi ld and/or elder abuse2. Suicida l/homicida l patient3. Impaired automobi le driver4. Speci fic infectious diseases—phys ician duty to report to publ ic officia ls or individuals at ri sk5. Tarasoff decis ion-law requiring phys ician to di rectly inform/protect potentia l victim from harm

What elements are required in order to prove a malpractice claim?The “4 D’s”: must prove that the phys ician showed Deri l i c on (devia on from standard of care) of a Duty that caused Da ma ges Di rectly tothe patient

What is the legal standard of death?Fai lure to meet cardiorespiratory cri teria and i rrevers ible cessation of a l l bra in functions (including bra instem)

BIOSTATISTICS

For each description, name the proper term and the equation used to calculate the value described below:

Probability that a person without the disease will be correctly identifiedSpeci fici ty TN/(TN + FP)Probability that a person who tests positive actually has the diseasePos i tive predictive va lue TP/(TP + FP)Probability that a person who has a disease will be correctly identifiedSens i tivi ty TP/(TP + FN)Total number of cases in a population at any given timePrevalence TP + FN/(enti re population)Number of new cases that arise in a population over a given time intervalIncidence Preva lence × duration of disease (approximately)Used in case-control studies to approximate the relative risk if the disease prevalence is too highOdds ratio TP × TN/FP × FNUsed in cohort studies to compare incidence rate in exposed group to that in unexposed groupRelative ri sk (TP/[TP + FP])/(FN/[FN + TN])Probability that patient with a negative test actually has no diseaseNegative predictive va lue TN/(FN + TN)

How are incidence and prevalence related?Incidence × disease duration = preva lencePrevalence > incidence for chronic diseases ; preva lence = incidence for acute diseases

What quality is desirable for a screening tool?High sens i tivi ty (SNOUT—SeNs i tivi ty rules OUT)

What quality is desirable for a confirmatory test?High speci fici ty (SPIN—Speci fici ty rules IN)

Name four ways to reduce bias:1. Use of placebo2. Bl inded s tudies (s ingle, double)3. Crossover s tudies (each subject i s own control )4. Randomization

Name the type of statistical distribution described below:Asymmetry with tail to the rightPos i tive skewTwo peaksBimodalScores cluster in the high endNegative skewBell-shapedNormal (Gauss ian)

What percent of the area under a normal curve falls within 1, 2, and 3 standard deviations (SD) of the mean?68% (1 SD), 95% (2 SD), 99.7% (3 SD)

How is standard error of the mean (SEM) calculated?

/square root of sample s i zeName the term for each of the following descriptions:

Refers to the reproducibility of a testRel iabi l i tyRefers to the appropriateness of a test (whether the test measures what it is supposed to)Val idi tyAbsence of random variation in a test; consistency and reproducibility of a testPrecis ionCloseness of a measurement to the truthAccuracyTest that compares the difference between two meanst-testTest that analyzes the variance of three or more variablesAnalys is of variance (ANOVA)Test that compares percentages or proportionsX2 (chi -squared test)Absolute value that indicates the strength of a relationshipr (a lways between -1 and 1)Observational study where the sample is chosen based on presence/absence of risk factorsCohort s tudy (eg, prospective, his torica l )Experimental study comparing benefits of two or more alternative treatmentsCl inica l tria lObservational study where the sample is chosen based on disease presence/absenceCase-control s tudy (usual ly retrospective)Assembling data from multiple studies to achieve great statistical powerMeta-analys isHypothesis postulating that there is no difference between groupsNul l hypothes is (H0)Error of mistakenly rejecting H0 (stating that there is a difference when there really is not)Type I error (α)Error of failing to reject H0 (stating there is no difference when there really is)Type II error (β)Probability of rejecting H0 when it is in fact falsePower (1-β)Probability of making an α errorP va lue

PUBLIC HEALTH/EPIDEMIOLOGY

Name four reportable STDs:

1. Gonorrhea2. Syphi l i s3. Hepati ti s B4. Acquired immunodeficiency syndrome (AIDS) (nonreportable = HIV, chlamydia , geni ta l herpes)

What is the leading cause of mortality in each of the following age groups:1 y/oCongenita l anomal ies1 to 14 y/oUnintentional injuries15 to 24 y/oUnintentional injuries (mostly car accidents )25 to 64 y/o

Cancer (#1 lung, #2 breast/prostate, #3 colon)65 years and olderHeart disease

What is 1° disease prevention?Aims to prevent disease occurrence (eg, vaccination, education)

What is 2° disease prevention?Early detection of disease; screening programs

What is 3° disease prevention?Reduces disabi l i ty resul ting from disease (eg, phys ica l therapy for s troke)

What is the current definition of “obese”?BMI> 30, BMI> 25 i s overweight

Approximately what percentage of the US population is obese?~30%, > 50% overweight

What is the divorce rate in the United States?~50%

Name three risk factors for divorce:1. Teenage marriage2. Mixed rel igions3. Low SES. Note: peaks during second and thi rd years

What is the criterion to qualify for hospice care?Medica l ly anticipated death within 6 months

What federal program addresses health care needs of the elderly?MedicarE (Elderly)

What federal and state program addresses health care needs of the underprivileged?Medica iD (Destitute)

What 1996 law helps to protect rights to health coverage during events such as changing or losing jobs, pregnancy, moving, or divorce?Health insurance portabi l i ty and accountabi l i ty act (HIPAA)

CHAPTER 14Make the Diagnosis

NUTRITION

1-year-old (y/o) impoverished child presents with lethargy and poor wound healing; physical examina on (PE): low-grade fever, pallor, oral mucosalpetechiae, and bleeding gums.

Scurvy50-y/o with history of (h/o) long-term treatment of psoriasis with methotrexate presents with nausea and fa gue; PE: within normal limits (WNL); workup(W/U): MCV > 100, hypersegmented neutrophils, and normal vitamin B12 levels.

Fol ic acid deficiency42-y/o alcoholic presents with ataxia and shortness of breath (SOB); PE: nystagmus, cardiomegaly with flow murmur, ↓ DTRs, ↓ peripheral sensa on, andhepatomegaly.

Beriberi48-y/o Scandinavian woman presents with weakness, ataxia, and SOB; PE: ↓ balance and vibratory sensa on in lower extremi es; W/U: MCV > 100,hypersegmented PMNs, and positive Schilling test.

Pernicious anemia

MICROBIOLOGY AND INFECTIOUS DISEASES

4-y/o with a superficial skin infection consisting of erythematous pustules with honey-colored scabs.

Impetigo, most commonly due to Staphylococcus aureus6-y/o child with poor hygiene presents with complaints of severe perianal itching that is worse at night; W/U: a “scotch tape test” reveals eggs visualizedunder the microscope.

Pinworm infection36-y/o man presents with flu-like symptoms since moving to a farm in Ohio 2 months ago; PE: fever and generalized lymphadenopathy; chest x-ray (CXR):bilateral hilar adenopathy.

Histoplasmos isPa ent with recent h/o an bio c use presents with fever, bloody diarrhea, and abdominal pain; PE: tender abdominal examina on, guiaic + stool; completeblood count (CBC): leukocytosis; colonoscopy: tan nodules seen attached to erythematous bowel wall with superficial erosions.

Pseudomembranous col i ti s (Clostridium difficile col i ti s )18-y/o student returns to clinic with a rash a er being treated with ampicillin for fever and sore throat; PE: tonsilar exudates and enlarged posterior cervicallymph nodes; W/U: ↑ lymphocytes and ⊕ heterophile antibody test.

Infectious mononucleos is due to Epstein Barr vi rus (EBV)28-y/o with h/o treatment for 2° syphilis 5 hours ago with intramuscular (IM) penicillin presents with fever, chills, muscle pain (myalgias), and headache.

Jari sch-Herxheimer reaction33-y/o northern European with h/o ea ng raw fish presents with shortness of breath and weakness; W/U: megaloblas c anemia, operculated eggs on stoolexamination.

Diphyllobothrium latum infection (with vi tamin B12 deficiency)17-y/o swimmer presents with pain and discharge from the left ear; PE: movement of tragus is extremely painful; Gram stain shows gram-negative rods.

Otiti s externa (most l ikely due to Pseudomonas aeruginosa)10-y/o with sickle cell disease and recent h/o prodromal illness presents with sudden-onset pallor, fa gue, and tachycardia; CBC: pancytopenia withreticulocytes 1%.

Parvovirus B19 aplastic cri s i s44-y/o parrot owner presents with fever, chills, headache, and cough; PE: Horder spots on abdomen and splenomegaly; CXR: bilateral, interstitial infiltrates.

Ps i ttacos is33-y/o epilep c with recent loss of consciousness presents with fever and cough with purulent, putrid sputum; W/U: Gram stain reveals mixed oral flora;CXR: consolidation of right lower lobe.

Aspiration pneumonia50-y/o man presents with a fever of unknown origin. One month ago he had a dental procedure. He has a history of rheuma c heart disease; PE: significantfor a heart murmur, splinter hemorrhages, Janeway lesions, Roth spots, and Osler nodes.

Subacute bacteria l endocardi ti s due to Streptococcus viridans9-y/o with recent viral prodomal illness presents to ER vomi ng and lethargic a er being given aspirin for fever; W/U: impaired liver func on; computedtomography (CT): cerebral edema.

Reye syndrome15-y/o new pet owner presents with painful axillary lumps and fever; PE: cervical lymphadenopathy.

Catscratch disease2-month-old with maternal h/o rash and flu in first trimester presents with a rash and failure to a ain milestones; PE: microcephaly, cataracts, jaundice,continuous machinery-like murmur at left upper sternal border, and hepatosplenomegaly (HSM).

Congenita l rubel la8-y/o from Connec cut presents with fever, rash, headache, and joint pain a er playing in the woods; PE: dis nc ve macule with surrounding 6 cm target-shaped lesion.

Lyme disease35-y/o man with h/o urinary catheteriza on presents with fever, chills, dysuria, and perineal pain; PE: swollen, tender, hot prostate; urinalysis (UA): WBCsand culture ⊕ for Escherichia coli.

Acute bacteria l prostati ti s38-y/o man with h/o asthma presents with recurrent fever, wheezing, and cough produc ve of brown mucous plugs; CBC: eosinophilia, high IgE ters; CXR:mucoid impaction of dilated central bronchi.

Al lergic bronchopulmonary aspergi l los is6-y/o unvaccinated child presents with rhinorrhea, cough, conjunctivitis; PE: oral lesions are noted which are bluish with an erythematous border.

Meas les37-y/o man with a recent h/o upper respiratory infec on (URI) presents with fever and severe chest pain; PE: fric on rub and Kussmauls sign; W/U: ↑ ESR,normal cardiac enzymes, and diffuse ST elevations on ECG.

Acute pericardi ti s4-y/o presents with barking cough, fever, and rhinorrhea; PE: respiratory distress and tachypnea; x-ray of soft tissues of neck reveals “steeple sign.”

Croup18-y/o sexually active woman with h/o treatment for purulent vaginitis presents with a fever and tenderness, warmth, and swelling of her right knee.

Gonococca l arthri ti sNewborn presents with rash and maternal h/o intrauterine growth retarda on (IUGR) and flu during first trimester; PE: petechial rash, choriore ni s,microcephaly, ↓ hearing, and HSM; W/U: ↓ platelets and periventricular calcifications on head CT.

Congenita l cytomegalovi rus (CMV)25-y/o West Virginian man presents with fever, headache, myalgia, and a petechial rash that began peripherally but now involves his whole body, even hispalms and soles; W/U: shows +OX19 and OX2 Weil-Felix reaction.

Rocky Mounta in spotted fever10-y/o presents with fevers and a pruritic rash spreading from the trunk to the arms; PE: vesicles of varying stages.

Varicel la29-y/o athlete presents with a red, pruri c skin erup on with an advancing peripheral, creeping border on the forearm; W/U shows septate hyphae in KOHscraping.

Tinea corporis (ringworm)8-y/o presents with fever, photophobia, stiff neck, and headache; PE: ⊕ Kernig and Brudzinski signs; lumbar puncture shows normal glucose and ↓ WBCs.

Vira l meningi ti s21-y/o presents with nausea, vomiting, bloating, and foul-smelling stools on returning from a camping trip; W/U: binucleate, flagellated trophozoites in stool.

Giardias is14-y/o adolescent boy with h/o recent travel to Mexico now presents with jaundice and dark yellow urine; PE: icterus and firm hepatomegaly; W/U: ↑bilirubin (BR), ↑ LFTs with alanine transaminase (ALT) > aspartate transaminase (AST).

Hepati ti s A infection26-y/o sexually ac ve, na ve Caribbean presents with painless, beefy-red ulcers of the genitalia and inguinal swelling; W/U: Donovan bodies on Giemsa-stained smear.

Granuloma inguina lePa ent presents with sudden onset of severe watery diarrhea, vomi ng, and abdominal discomfort 4 hours a er ea ng potato salad at a picnic; thesymptoms resolve spontaneously within 24 hours.

Staphylococcus aureus-induced diarrhea6-y/o with recent h/o sore throat presents with mul ple joint pain and swelling, fever, and SOB; PE: erythema marginatum, subcutaneous nodules, and apicalsystolic murmur; W/U: ↑ ESR and CRP.

Acute rheumatic fever25-y/o woman presents with malodorous vaginal discharge; W/U: visualiza on of the discharge under the microscope demonstrates mul ple vaginalepithelial cells covered in bacteria.

Bacteria l vaginos is due to Gardenella vaginalis73-y/o presents with a painful, unilateral, vesicular rash in the distribution of the CN V1; cornea shows diminished sensation and stains with fluorescein.

Herpes zoster ophthalmicus29-y/o missionary with h/o travel to rural India presents with high fever and right upper quadrant (RUQ) pain; PE: severe hepatomegaly and dullness overright lower lung; abdominal CT: large, cavitating lesion in liver.

Amebic l iver abscess25-y/o sexually ac ve man presents with dysuria, irrita on, and cloudy discharge; W/U: a Gram stain of the discharge reveals neutrophils, but no organismsare visualized.

Nongonococca l urethri ti s , most l ikely due to Chlamydia trachomatis31-y/o obese woman presents with pruri s in her skin fold beneath her pannus; PE: whi sh curd-like concre ons beneath the abdominal panniculus; W/Ushows budding yeast on 10% KOH preparation.

Cutaneous candidias is

NEUROSCIENCE

55-y/o man presents with lower extremity weakness and muscle atrophy; PE: positive Babinski reflex, upper extremity hyperreflexia, and spasticity.

Amyotrophic latera l scleros is (ALS)65-y/o presents with a gradual decline in memory and inability to complete ac vi es of daily living; W/U: CT shows marked enlargement of ventricles anddiffuse cortical atrophy.

Alzheimer disease65-y/o woman with h/o spinal metastases from breast cancer presents with pain radia ng down the back of her leg, saddle anesthesia, urinary reten on; PE:absent ankle jerk reflexes; W/U: CT shows large bony fragment in lumbar spinal canal.

Cauda equina syndrome65-y/o man with h/o caro d atherosclerosis presents with aphasia and right-sided weakness; PE: dense right-hemiparesis, posi ve Babinski on right; W/U:CT shows left middle cerebral artery (MCA) territory infarction and edema.

Left MCA cerebrovascular accident20-y/o presents with nausea, vomi ng, and headache 2 hours a er being hit in the temple with a baseball; pa ent lost consciousness ini ally but recoveredquickly; W/U: CT shows lens-shaped, right-sided hyperdense mass adjacent to temporal bone.

Epidura l hematoma40-y/o with h/o Campylobacter enteri s 1 week ago presents with ascending symmetric muscle weakness; CSF shows ↑ protein, normal cellularity(albuminocytologic dissociation).

Gui l la in-Barré syndrome37-y/o man with family history (FH) of a father who died at 45 with worsening tremor and demen a presents with poor memory, depression, choreiformmovements, and hypotonia; W/U: MRI demonstrates marked atrophy of the caudate nucleus.

Huntington disease25-y/o with h/o bilateral temporal lobe contusions 1 week ago presents with a sudden increase in appetite, sexual desire, and hyperorality.

Klüver-Bucy syndrome30-y/o woman with insidious onset of diplopia, scanning speech, paresthesias, and numbness of right upper extremity and urinary incon nence; W/U: MRIshows discrete areas of periventricular demyelination and CSF analysis is positive for oligoclonal bands.

Multiple scleros is65-y/o woman with h/o neurofibromatosis type 2 presents with headache, right-sided leg jerking, and worsening mental status; PE: papilledema and right-sided pronator drift; W/U: CT scan shows dural-based, enhancing, left-sided softball-sized tumor.

Meningioma55-y/o with a h/o squamous cell carcinoma of the lung presents with nausea, vomi ng, headache, and diplopia; PE: papilledema, le oculomotor palsy, right

pronator drift; MRI: multiple round, hyperintense cortical and cerebellar lesions.Metastases to bra in

30-y/o woman presents with unilateral throbbing headache, nausea, photophobia, scotoma; similar symptoms occur monthly at the same me of hermenstrual cycle.

Migra ine65-y/o with urinary incon nence, loss of short-term memory, and demen a; PE: wide based, magne c gate; W/U: CT scan shows massively dilatedventricular system.

Normal pressure hydrocephalus60-y/o presents with gradual onset of pill-rolling tremor; PE: masked facies, stooped posture, festinating gait, cogwheel muscle rigidity.

Parkinson disease30-y/o presents with loss of libido, galactorrhea, and irregular menses; PE: bitemporal hemianopia; W/U: nega ve beta human chorionic gonadotropin (β-hCG).

Prolactinoma (prolactin-secreting pi tui tary adenoma)45-y/o presents with the gradual onset of sharp pain radia ng from his bu ocks down his leg that began 2 weeks ago when he began to li a heavy box; PE:positive straight leg raise test.

Sciatica from acute herniation of a lumbar disc50-y/o with h/o polycys c kidney disease presents with “worst headache of life,” photophobia, nausea; PE: right eye deviated down and out; W/U: CSF isxanthrochromic.

Subarachnoid hemorrhage from ruptured berry aneurysm32-y/o man with h/o Arnold-Chiari malforma on presents with bilateral upper extremity muscle weakness; PE: loss of pain and temperature sensa on, ↓DTR in upper extremities and scoliosis; MRI shows central cavitation of the thoracic spinal cord.

Syringomyel ia75-y/o alcoholic man on warfarin for h/o atrial fibrilla on presents with declining mental status, headache, and papilledema; CT shows crescenteric,hypodense, 2 cm fluid collection along convexity.

Chronic subdura l hematoma36-y/o woman with family h/o renal cell carcinoma presents with gait disturbance and blurred vision; PE: re nal hemangiomas, nystagmus, cerebellarataxia, dysdiadokinesia; MRI shows two cerebellar cystic lesions.

von Hippel -Lindau diseaseA 50-y/o with h/o alcoholism presents with psychosis, opthalmoplegia, and ataxia; MRI: mamillary body atrophy and diffuse cortical atrophy.

Wernicke encephalopathy

CARDIOVASCULAR

56-y/o woman presents with dyspnea on exer on (DOE); PE: loud S1, delayed P2, and a diastolic rumble; W/U: transesophageal echocardiogram showsmobile, pedunculated left atrial mass.

Atria l myxoma60-y/o presents with chest pain relieved by si ng up and leaning forward; PE: pericardial fric on rub; ECG: diffuse ST segment eleva on; echocardiogram:pericardial effusion with thickening of the pericardium.

Acute pericardi ti s65-y/o man presents with 1-week h/o fever, DOE, and orthopnea; PE: new, blowing holosystolic murmur at apex radia ng into le axilla; W/U: bloodcultures show Viridans spp. streptococci; echocardiogram: oscillating mass attached to mitral valve.

Acute infective endocardi ti s60-y/o presents with dyspnea and palpita ons; PE: 20 mm Hg decline in systolic BP with inspira on (pulsus paradoxus), ↓ BP, jugular venous disten on,diminished S1 and S2; echocardiogram: large pericardial effusion.

Tamponade58-y/o man with Marfan syndrome presents with the abrupt onset of “tearing” chest pain radia ng to the back; PE: ↓ BP, asymmetric pulses, decliningmental status; CXR: widened mediastinum.

Aortic dissection70-y/o diabe c with hypercholesterolemia presents with angina, syncope, DOE, and orthopnea; PE: diminished, slowly rising caro d pulses, crescendo-decrescendo systolic murmur at second interspace at the right upper sternal border.

Aortic s tenos is80-y/o diabe c with HTN and a h/o rheuma c heart disease presents with le -sided weakness; PE: pulses are irregularly irregular; ECG: absence of P wavesand irregularly irregular QRS complexes.

Atria l fibri l lation (leading to embol ic s troke)70-y/o with h/o coronary artery disease (CAD) presents with worsening DOE, orthopnea, and paroxysmal nocturnal dyspnea; PE: jugular venous disten on, S3

gallop, positive hepatojugular reflex, bibasilar rales, and peripheral edema; CXR: cardiomegaly, bilateral pleural effusions.Congestive heart fa i lure (CHF)

50-y/o alcoholic presents with worsening DOE, orthopnea, and paroxysmal nocturnal dyspnea; PE: laterally displaced apical impulse; ECG: four-chamberdilation, mitral and tricuspid regurgitation.

Alcohol ic di lated cardiomyopathy35-y/o man with FH of sudden cardiac death presents with DOE and syncope; PE: double apical impulse, S4 gallop, holosystolic murmur at apex and axilla;ECG: left ventricular hypertrophy and mitral regurgitation.

Hypertrophic cardiomyopathy40-y/o black man with a h/o HTN presents with chest pain, dyspnea, and severe headache; PE: BP = 210/130 mm Hg in all four extremi es, flame-shapedretinal hemorrhages, papilledema; W/U: negative vanillylmandelic acid (VMA), urine catecholamines, and cardiac enzymes.

Mal ignant hypertens ion35-y/o woman with a h/o rheuma c fever presents with worsening DOE and orthopnea; PE: loud S1, opening snap and low-pitched diastolic murmur at theapex; CXR: left atrial enlargement.

Mitra l s tenos is65-y/o man presents with substernal pressure for the past hour with radia on of the pain into the jaw and le arm, nausea, diaphoresis, and dyspnea; PE: S4

gallop; W/U: ↑ serum troponin and CK-MB; ECG: ST segment elevation in leads aVL, V1 to V6.Anterior myocardia l infarction

PULMONARY

7-y/o with h/o environmental allergies presents in acute respiratory distress; PE: tachypnea, expiratory wheezes, intercostal retrac ons, accessory muscle

usage during respiration; CXR: hyperinflation; CBC: eosinophilia.Bronchia l as thma

60-y/o with a 50 pack-year h/o smoking presents with fever and cough produc ve of thick sputum for the past 4 months; PE: cyanosis, crackles, wheezes;W/U: Hct = 48, WBC = 12,000; CXR: no infiltrates.

Chronic bronchi ti s60-y/o with a 50 pack-year h/o smoking presents with DOE and dry cough, but no chest pain; PE: ↓ breath sounds, ↑ heart rate (HR), hyperresonant chest,distant S1 and S2; CXR: hyperlucent lung fields.

Emphysema60-y/o with 50 pack-year h/o smoking presents with fa gue, DOE, hoarseness, anorexia; PE: miosis, ptosis, anhydrosis, dullness to percussion at right apex;CXR: large, hilar mass extending into the right superior pulmonary sulcus.

Pancoast tumor (most l ikely bronchogenic squamous cel l carcinoma, caus ing Horner syndrome)60-y/o 4 days s/p total knee replacement has a sudden onset of tachycardia, tachypnea, sharp chest pain, hypotension; W/U: arterial blood gas (ABG) showsrespiratory alkalosis; ECG: sinus tachycardia; lower extremity venous duplex ultrasound (U/S): clot in right femoral vein.

Pulmonary embol ism (most l ikely from DVT)30-y/o black woman presents with DOE, fever, arthralgia; PE: iri s, erythema nodosum; W/U: eosinophilia, ↑ serum ACE levels; PFT: restric ve pa ern;CXR: bilateral hilar lymphadenopathy, interstitial infiltrates; lymph node biopsy: noncaseating granulomas.

Sarcordos is40-y/o white man presents with chronic rhinosinusi s, ear pain, cough, dyspnea; PE: ulcera ons of nasal mucosa, perfora on of nasal septum; W/U: ↑ C-ANCA; U/A: red cell casts; biopsy of nasal lesions: necrotizing vasculitis and granulomas.

Wegener granulomatos is

GASTROENTEROLOGY

20-y/o woman presents with bloody diarrhea and joint pain; PE: abdominal tenderness, guaiac ⊕ stool; laboratory values: ↑ ESR and CRP, HLA-B27 +;colonoscopy: granular, friable muscosa with pseudopolyps throughout the colon.

Ulcerative col i ti s (UC)28-y/o pa ent with h/o of UC presents with severe abdominal pain, disten on, and high fever; PE: severe abdominal tenderness; CBC: leukocytosis;abdominal x-ray (AXR): dilated (> 6 cm) transverse colon.

Toxic megacoloncirrho c pa ent presents with massive hematemesis; PE: jaundice, ↓ BP, ↑ HR, ascites; W/U: stems pancytopenia, ↑ ALT and AST; EGD: ac vely bleedingvessel with numerous cherry red spots.

Esophageal varices38-y/o man with recent h/o fa gue, excessive thirst, and impotence presents with hyperpigmenta on of his skin; PE: cardiomegaly, HSM; W/U: ↑ glucose,ferritin, transferrin, and serum iron.

Hemochromatos is (heredi tary)19-y/o woman with recent h/o behavioral disturbance presents with jaundice and res ng tremor; PE: pigmented granules in cornea and HSM; W/U: ↓ serumceruloplasmin.

Wilson disease29-y/o with h/o intermi ent jaundice since receiving blood transfusion a er motor vehicle accident (MVA) 2 years ago; PE: RUQ tenderness, hepatomegaly;W/U: negative HBV serology.

Hepati ti s C infection31-y/o woman presents with 10-month h/o foul-smelling, greasy diarrhea; PE: pallor, hyperkeratosis, mul ple ecchymoses, and abdominal disten on; W/U:abnormal D-xylose test.

Cel iac disease60-y/o white man presents with steatorrhea, weight loss, arthritis, and fever; small bowel biopsy shows PAS ⊕ macrophages and gram-positive bacilli.

Whipple disease19-y/o Jewish woman with h/o chronic abdominal pain presents with recurrent UTIs and pneumaturia; PE: diffuse abdominal pain; CT: enterovesical fistula;colonoscopy: skip lesions of linear ulcers and transverse fissures giving cobblestone appearance to mucosa.

Crohn’s disease21-y/o man presents with hematemesis after ingestion of aspirin and seven shots of whiskey; PE: diaphoretic, ↑ HR, epigastric tenderness; EGD: edematous,friable, reddened gastric mucosa.

Acute gastri ti spa ent with h/o pep c ulcer disease (PUD) presents with melena; PE: ↑ HR, diaphore c, diffuse abdominal pain; W/U: nasogastric tube (NGT) aspirate isbloody; EGD: visible bleeding vessel distal to the pylorus.

Bleeding duodenal ulcer40-y/o obese, mother of four children presents with constant RUQ pain radia ng to right scapula, N/V; PE: fever, tenderness, and respiratory pause inducedby RUQ palpation, painful palpable gallbladder; W/U: ↑ WBC, ↑ ALP; U/S: thickened gallbladder wall, pericholecystic fluid with gallstones present.

Acute cholecysti ti s39-y/o man presents with dull, steady epigastric pain radia ng to the back a er an alcohol binge, N/V; PE: fever, ↑ BP, epigastric tenderness, guarding, anddistention; W/U: ↑↑ amylase/lipase, ↑ WBC; AXR: ⊕ sentinel loop and colon cutoff sign.

Acute pancreati ti s65-y/o black man with h/o smoking presents with anorexia, weight loss, pruri s, and painless jaundice; PE: palpable, nontender, distended gallbladder,migratory thrombophlebitis; W/U: ↑ direct BR, ALP, carcinoembryonic antigen (CEA), and CA 19-9.

Pancreatic adenocarcinoma60-y/o black man with h/o gastroesophageal reflux disease (GERD) presents with weight loss and dysphagia; EGD: par ally obstruc ng mass near GEjunction.

Esophageal carcinoma65-y/o presents with severe worsening le lower quadrant (LLQ) pain, N/V, and diarrhea; PE: fever, LLQ tenderness, local guarding, and rebound tenderness;W/U: ↑ WBC; abdominal CT: edematous colonic wall with localized fluid collection.

Diverticul i ti s30-y/o woman presents with periumbilical pain which has now migrated to the RLQ followed by anorexia, N/V; PE: low-grade fever, local RLQ guarding,rebound tenderness, RLQ tenderness on LLQ palpation; W/U: β-hCG negative, ↑ WBC with left shift.

Appendici ti s55-y/o presents with colicky abdominal pain, small-caliber stools, and occasional melena; PE: cachexia, abdominal discomfort, guaiac ⊕ colonoscopy showsobstructing mass seen in ascending colon.

Right-s ided colon carcinoma80-y/o woman presents with halitosis, dysphagia, and regurgita on of undigested foods; W/U: barium swallow shows posterior midline pouch greater than 2cm in diameter arising just above the cricopharyngeus muscle.

Zenker diverticulum55-y/o Asian woman with h/o HBV presents with dull RUQ pain; PE: weight loss, painful hepatomegaly, ascites, jaundice; W/U: ↑ ALT/AST, ↑ α-fetoprotein;abdominal CT: mass seen in right lobe of liver.

Hepatocel lular carcinoma55-y/o with h/o choledocholithiasis presents with fever, chills, and RUQ pain; PE: jaundice; W/U: ↑ WBC, BR, and ALP; U/S: stone in common bile duct.

Cholangi ti s43-y/o man presents with epigastric pain, diarrhea, and recurrent pep c ulcers; PE: epigastric tenderness; W/U: ↑ fas ng gastrin levels, paradoxic ↑ ingastrin with secretin challenge; Octreotide scan: detects lesion in pancreas.

Zol l inger-El l i son syndrome72-y/o presents with recurrent, low-grade, painless hematochezia; PE: guaiac ⊕ stool; colonoscopy reveals slightly raised, discrete, scalloped lesion withvisible draining vein in right colon.

Angiodysplas ia63-y/o Japanese man with h/o atrophic gastri s presents with weight loss, indiges on, epigastric pain, and vomi ng; PE: supraclavicular lymph node; W/U:anemia, ⊕ fecal occult blood.

Gastric carcinoma48-y/o with chronic watery diarrhea, hot flashes, and facial redness; PE: shows II/VI right-sided ejec on murmur; W/U: ↑ 5-hydrocyindoleace c acid (5-HIAA) in urine.

Carcinoid syndrome40-y/o presents with dysphagia, regurgita on, and weight loss; W/U: barium swallow demonstrates dilated esophagus with distal narrowing (bird beakappearance).

Achalas ia16-y/o with strong FH of colorectal CA presents with rectal bleeding and abdominal pain; W/U: anemia; flexible sigmoidoscopy: > 100 adenomatous polypsvisualized.

Fami l ia l adenomatous polypos is (FAP)34-y/o bulimic presents with sudden-onset retrosternal pain after vigorous vomiting; upper GI series shows extravasation of contrast into mediastinum.

Boerhaave syndrome44-y/o heavy smoker presents with heartburn and regurgita on that is worse when lying down and is relieved with antacids; upper GI series reveals mildhiatal hernia.

GERD5-day-old infant presents with abdominal distention and failure to pass meconium until after a rectal examination; XR shows massively dilated colon.

Hirschsprung disease39-y/o woman with h/o rheumatoid arthri s presents with fa gue and pruritus; PE: jaundice and HSM; W/U reveals ↑↑ ALP and γ-glutamyl transpep dase(GGT) and presence of antimitochondrial antibodies.

Primary bi l iary ci rrhos is2-week-old first-born male infant presents with projectile vomiting and dehydration; PE: visible peristalsis and palpable knot in epigastrum.

Hypertrophic pyloric s tenos is50-y/o with long h/o retrosternal pain drinks and smokes despite undergoing treatment for GERD; biopsy of distal esophagus shows metaplasia.

Barrett s esophagus34-y/o woman with factor V Leiden deficiency presents with abdominal disten on and jaundice; PE: pi ng pedal edema, markedly visible leg veins, HSM,and absent hepatojugular reflex; U/S shows obstruction of hepatic veins.

Budd-Chiari syndrome7-month-old infant presents with vomi ng and currant jelly-appearing stools; PE: right-sided, sausage-shaped, palpable mass in abdomen; barium enema(BE) shows telescoping of intestines.

Intussusception

REPRODUCTIVE/ENDOCRINE

31-y/o presents with loss of libido, galactorrhea, and irregular menses; PE: bitemporal hemianopia; W/U: negative β-hCG.

Prolactinoma7-month-old with history of mul ple infec ons turns cyano c when aggravated; PE: abnormal facies, cle palate, heart murmur; W/U: hypocalcemia,tetrology of Fallot.

DiGeorge syndrome (22ql l )Patient presents to clinic with polyuria and polydipsia; W/U: urine specific gravity 1.005, urine osmolality 200 mOsm/kg, hypernatremia.

Diabetes ins ipidus30-y/o white woman presents with weight loss, tremor, and palpita ons; PE: brisk DTRs, ophthalmopathy, pre bial myxedema; W/U: ↓ TSH, ↑ T 4, ↑ T3

index.Graves disease

40-y/o woman presents with fa gue, cons pa on, and weight loss; PE: puffy face, cold dry hands, coarse hair, and enlargement of thyroid gland; W/U: ↑TSH, ↓ T3 and T4, ⊕ antimicrosomal and antithyroglobulin AB.

Hashimoto disease32-y/o woman with h/o recurrent PUD presents with episodes of hypocalcemia and nephrolithiasis; W/U: fas ng hypoglycemia, ↑ gastrin levels, andhypercalcemia.

Multiple endocrine neoplas ia (MEN) 170-y/o presents with episodal hypertension, nephrolithiasis, and diarrhea; PE: ↑ BP, thyroid nodule; W/U: ↑ calcitonin levels, ↑ urinary catecholamines.

MEN 2Female patient presents with bone pain, kidney stones, depression, and recurrent ulcers; W/U: hypercalcemia, hypophosphatemia, and hypercalciuria.

Hyperparathyroidism35-y/o woman presents with weightgain, irregular menses, and HTN; PE: ↑ BP, ↑ weight in face and upper back, hirtuism, mul ple ecchymoses; W/U: ↑ACTH levels and suppression with high-dose dexamethasone suppression test.

Cushing disease50-y/o woman presents with HTN, muscle weakness, and fatigue; W/U: hypokalemia, hypernatremia, and metabolic alkalosis.

Conn syndrome30-y/o woman presents with progressive weakness, weight loss, N/V; PE: hyperpigmenta on of skin, ↓ BP; W/U: hyperkalemia, hyponatremia, andeosinophilia.

Addison disease40-y/o presents with episodes of HA, diaphoresis, palpitations, and tremor; PE: ↑ BP, ↑ HR; W/U: ↑ in urinary VMA and homovanillic acid.

Pheochromocytoma

17-y/o white adolescent with h/o diabetes mellitus (DM) presents with diffuse abdominal pain, N/V, and slight confusion; PE: ↓ BP, shallow, rapid breathingpattern; W/U: glucose = 300, hypokalemia, hypophosphatemia, and metabolic acidosis.

Diabetic ketoacidos is (DKA)-DM type 160-y/o diabetic obese patient found at home confused and disoriented; PE: ↓ BP, ↑ HR; W/U: glucose > 1000.

Hyperosmolar hyperglycemic non-ketotic (HHNK)-DM type 250-y/o woman presents with h/o weakness, blurred vision, and confusion several hours a er meals, which improves with ea ng; W/U: ↑ fas ng levels ofinsulin and hypoglycemia.

Insul inoma (with Whipple triad)Newborn presents with ambiguous genitalia; PE: lethargy and ↓ BP; W/U:↓ Na+, ↑ K+, ↓ 17 α-OH-progesterone, ↑ ACTH, and karyotype of 46, XX.

Congenita l adrenal hyperplas ia (21α-hydroxylase deficiency)7-y/o girl presents with breast buds andmonthly vaginal bleeding; PE: height and weight >> 95 percen le, full pubic and axillary hair; hand x-ray showsadvanced bone age.

Precocious puberty21-y/o woman presents with no h/o menarche; PE: normal breast ssue, no axillary or pubic hair, vagina ending in blind pouch, no palpable cervix or uterus;karyotype shows 46, XY.

Androgen insens i tivi ty (testicular feminization) syndrome45-y/o with recent h/o coarsening of facial features presents with headaches and states that his shoes no longer fit; PE: enlarged jaw, tongue, hands, andfeet, and bitemporal hemianopia.

Acromegaly65-y/o smoker with h/o lung cancer presents with fatigue and oliguria; W/U: ↓ Na+, ↓ serum osmolarity, ↑↑ urine osmolarity.

Syndrome of inappropriate antidiuretic hormone (SIADH)22-y/o sexually ac ve woman presents with dysuria, dyspareunia, vulvar pain for the past 3 days. PE: so ulcer on labia majora, inguinal lymphadenopathy;W/U: culture grows Haemophilus ducreyi.

Chancroid20-y/o sexually ac ve woman presents with crampy abdominal pain and purulent vaginal discharge; PE: fever and adnexal tenderness; W/U: ↑ WBCs, ↑ESR, and combined infection with C. trachomatis and Neisseria gonorrhoeae.

Pelvic inflammatory disease (PID)18-y/o woman with 3 days of vaginal pruritis; PE: thick, white discharge; W/U: budding yeast on KOH preparation.

Vagina l candidias is28-y/o woman postpartum day 1 with excessive hemorrhage in labor becomes weak and loses consciousness. PE: hypotension; W/U: ↓ cor sol, ↓ TSH, ↓fT4, ↓ LH, ↓ FSH.

Sheehan syndrome (pi tui tary apoplexy)31-y/o woman with h/o PID presents with sudden-onset nausea and LLQ pain; PE: ↓ BP, ↑ HR, rebound tenderness in LLQ; W/U shows ⊕ β-hCG and fluid incul-de-sac on U/S.

Ruptured ectopic pregnancy28-y/o man presents with gynecomastia and painless lump in his left testicle for 3 months; PE: firm 4 cm mass on left testis; W/U: ↑↑ serum hCG and AFP.

Testicular carcinoma35-y/o man with sensation of heaviness in left scrotum; appears like a “bag of worms” on examination.

Varicocele62-y/o obese nun with h/o menopause at age 57 presents with vaginal bleeding for the past 4 months; PE shows normal-sized uterus; Pap smear revealsabnormal endometrial cells.

Endometria l carcinoma52-y/o obese patient presents with numbness in hands and feet; PE: ↑ BP and retinopathy; W/U: ↑ HbA1c and glycosuria.

DM type 255-y/o woman presents with an itching, scaling, oozing rash over her le nipple; PE: serosanguinous discharge and eczematous redness of le nipple withaxillary lymphadenopathy.

Paget disease of the breast4-y/o girl presents with a “bunch of grapes” protruding from her vagina. W/U: desmin positive.

Sarcoma botryoides37-y/o woman presents with dysmenorrhea, dyspareunia, menorrhagia, and pain coinciding with her menstrual cycle; PE: nodularity of uterosacral ligamentsand cul-de-sac.

Endometrios is22-y/o woman with 6-month h/o amenorrhea presents for infer lity evalua on; PE: obesity, hirsu sm; W/U: ↑ LH:FSH ra o, ↑ testosterone, and enlargedovaries on U/S.

Polycystic ovarian syndrome (Stein-Leventhal syndrome)44-y/o with recent h/o thyroidectomy presents with muscle cramping; PE: circumoral numbness, positive Trousseau and Chvostek signs.

Hypoparathyroidism23-y/o woman marathon runner presents with lack of menses for 5 months; PE shows no signs of pregnancy; W/U: nega ve β -hCG, normal prolac n andthyroid hormones.

Secondary amenorrhea28-y/o black woman at 35 weeks gestation in her first pregnancy presents with swollen legs; PE: ↑ BP and pitting pedal edema; W/U: 3+ proteinuria.

Preeclamps ia2-y/o child presents with developmental delay; PE: macroglossia, short stature, and protuberant abdomen; W/U: ↑ TSH, ↓ T3 and T4.

Congenita l hypothyroidism (cretinism)51-y/o woman with 9-month h/o amenorrhea presents with fatigue and flushing of skin; PE: atrophic vaginal mucosa; W/U: ↑ FSH and LH.

Menopause24-y/o woman presents with painless lump in her le breast; PE: small, firm, palpable, and freely mobile, rubbery mass in the upper-outer quadrant of thebreast.

Fibroadenoma19-y/o man is brought to you for failure of pubertal matura on; PE: anosmia, ↓ muscle mass, no axillary or pubic hair, and hypogonadism; W/U: ↓ LH andFSH.

Kal lmann syndrome9-y/o girl presents with muscle cramps; PE: rounded face with flat nasal bridge, abnormal den on, posi ve Trousseau and Chvostek sign, and shortenedthird and fourth metacarpals.

Albright heredi tary osteodystrophy (pseudohypoparathyroidism)22-y/o pregnant woman at 27 weeks with painless vaginal bleeding that stopped after an hour. W/U: placenta overlying the cervical os on ultrasound.

Placenta previa19-y/o woman with h/o recent hyda diform mole presents with vaginal bleeding, nausea, and vomi ng; PE: vascular growth at cervical os and enlarged

uterus; W/U: ↑↑ β-hCG.Choriocarcinoma

35-y/o man presents for sterility evaluation; PE: eunuchoid body habitus, small testicles, and gynecomastia; karyotype reveals 47, XXY.Kl inefel ter syndrome

RENAL AND GENITOURINARY

Pa ent hospitalized for CHF is started on an aminoglycoside for a UTI and develops oliguria, N/V, and malaise; PE: ↑ BP and asterixis; serum electrolytes: ↑creatinine (Cr), K+; UA: “muddy brown” casts, FeNa+ > 3%.

Acute renal fa i lure (drug-induced ATN)70-y/o black man with h/o of life-long DM presents with peripheral edema, SOB, and oliguria; PE: auscultatory rales, pi ng edema, myoclonus, and uremicfrost; serum electrolytes: ↑ Cr, hyperkalemia, hypocalcemia, hyperphosphatemia.

Chronic renal fa i lureTeenage female presents with fever, chills, and flank pain; PE: costovertebral angle (CVA) tenderness; UA: leukocyte esterase ⊕ , 30 WBC/hpf with WBC casts.

Pyelonephri ti s32-y/o man presents with pain and hematuria; PE: ↑ BP, palpable kidney, and midsystolic ejec on click; abdominal U/S: mul ple cysts of renal parenchyma;cerebral angiogram: unruptured berry aneurysm.

Adult polycystic kidney disease20-y/o man presents with significant blood loss following a trauma and begins to have decreased urine output. W/U: oliguria, FeNa 1%, and BUN:Cr > 20.

Acute renal fa i lure—prerenal12-y/o girl with recent h/o sore throat presents with low urine output and dark urine; PE: periorbital edema; W/U: hematuria, ↑ BUN and Cr, ↑antistreptolysin O (ASO) titer.

Poststreptococca l glomerulonephri ti s45-y/o Asian man with h/o hepa s B presents with malaise, edema, and foamy urine; PE: anasarca; W/U: proteinuria (> 3.5 g/day), hyperlipiduria;hyperlipidemia and hypoalbuminemia.

Membranous glomerulonephri ti sMale infant is born with flattened facies, joint position abnormalities, and hypoplastic lungs. Oligohydramnios was noted prior to delivery.

Potter sequence—secondary to renal agenes is80-y/o man presents with urinary hesitancy, nocturia, and weak urinary stream; PE: diffusely enlarged rubbery prostate; serum electrolytes: ↑ Cr, UA isWNL.

Benign prostatic hyperplas ia (BPH)68-y/o man, who is a smoker, presents with flank pain and hematuria; PE: fever, palpable kidney mass; W/U: hypercalcemia, polycythemia.

Renal cel l carcinoma20 y/o man presents with acute onset of left testicular pain and N/V; PE: swollen, tender testicle in transverse position, absent cremasteric reflex on left side;Doppler: no flow detected in left testicle.

Testicular tors ion65-y/o man, who is a smoker, presents with painless hematuria and occasional urinary urgency and frequency; PE: unremarkable; urine cytology posi ve formalignant cells.

Bladder—urothel ia l carcinoma85-y/o man presents with back pain, weight loss, and weak urinary stream; PE: palpable firm nodule on digital rectal examination (DRE); W/U: ↑ PSA.

Prostate cancer25-y/o Asian man presents with N/V and colicky right flank pain; PE: acute distress and CVA tenderness; W/U: hematuria and discrete radiopaci es onabdominal XR.

Renal s tones45-y/o with documented h/o aor c atheromatous plaques presents with recent-onset, severe le flank pain, and hematuria; abdominal CT: wedge-shapedlesion in the left kidney.

Renal infarct3-y/o boy presents with h/o flank mass found recently by his mother while bathing him; PE: palpable mass in le flank; abdominal CT: large mass growingout of left kidney.

Wilms tumor55-y/o with long h/o DM presents with increasing fatigue and edema; PE: ↑ BP, retinopathy, and pitting edema; W/U: severe proteinuria and glycosuria.

Diabetic nephropathy (glomeruloscleros is )21-y/o sexually active woman presents with frequency and dysuria; PE: suprapubic tenderness; W/U: E. coli-positive urine cultures.

Urinary tract infection (UTI)25-y/o man presents with hemoptysis, dark urine, and fatigue; PE: bilateral crackles at lung bases; W/U: oliguria, hematuria, and anti-GBM antibodies.

Goodpasture syndrome7-y/o presents in stupor after ingesting antifreeze; PE: Kussmaul respirations and mental status changes; W/U reveals anion gap of 21 mEq/L.

Metabol ic acidos is (ethylene glycol toxici ty)6-y/o boy presents with hematuria and worsening vision; PE: corneal abnormali es, re nopathy, sensorineural hearing loss; W/U: hematuria withdysmorphic red cells.

Alport syndrome3-y/o boy with h/o recent URI presents with facial edema; PE: ascitic fluid in abdomen and pedal edema; W/U reveals 4+ proteinuria and ↓ serum albumin.

Minimal change disease

HEMATOLOGY/ONCOLOGY

50-y/o with h/o bone marrow transplant for chronic myelogenous leukemia (CML) 3 weeks ago presents with severe pruri s, diarrhea, and jaundice; PE:violaceous rash on palms and soles; W/U: ↑ BR, ALT, and AST.

Graft-versus-host disease1-y/o Greek child presents with pallor and delayed milestones; PE: skeletal abnormali es, splenomegaly; peripheral blood smear (PBS): hypochromicmicrocytic RBCs, target cells, fragmented RBCs; skull XR: “hair-on-end” appearance.

β-Thalassemia10-y/o with a h/o recurrent chest pain presents with fever and bilateral leg pain; PE: febrile, mul ple leg ulcers; PBS shows sickle-shaped erythrocytes; Hbelectrophoresis shows HbS band.

Sickle cel l anemia60-y/o with headache, vertigo, blurry vision, pruritus, joint pain; PE: ↑ BP, plethoric splenomegaly; W/U: Hct = 60, mild leukocytosis, and hyperuricemia.

Polycythemia vera4-y/o boy with a 1-week h/o fever, pallor, headache, and bone tenderness; PE: fever, HSM, and generalized, nontender lymphadenopathy; PBS revealsabsolute lymphocytosis with abundant TdT+ lymphoblasts.

Acute lymphoblastic leukemia27-y/o presents with 2-month h/o fa gue, oropharyngeal candidiasis, pseudomonal UTI, and epistaxis; PE: numerous petechiae and ecchymoses of skin,gingival mucosal bleeding, guaiac ⊕ stools; W/U: ↑ WBC; PBS shows >30% myeloblasts with Auer rods.

Acute myelocytic leukemia17-y/o man presents with a 2-month h/o fever, night sweats, and weight loss; PE: nontender, cervical lymphadenopathy, and HSM; CBC: leukocytosis; CXR:bilateral hilar adenopathy; lymph node biopsy: Reed-Sternberg cells.

Hodgkin disease60-y/o man presents with fa gue and anorexia; PE: generalized lymphadenopathy and HSM; W/U: WBC = 250,000, posi ve direct Coombs test; PBS: small,round lymphocytes predominate with occasional smudge cells.

Chronic lymphocytic leukemia10-y/o African child presents with a 3-week h/o a rapidly enlarging, painless mandibular mass; CBC: mild anemia and leukopenia; cytogene cs reveal at(8:14) translocation; excisional biopsy: “starry-sky” pattern.

Burki tt lymphoma35-y/o presents with a 3-year h/o mild weight loss, anorexia, worsening DOE; PE: splenomegaly; CBC: mild anemia, WBC = 125,000; PBS: granulocytosis with10% myeloblasts; cytogenetics reveal a t(9:22) translocation.

Chronic myelocytic leukemia55-y/o with a recent h/o streptococcal pneumonia presents with bone pain and weight loss; W/U: mild anemia, hypercalcemia; PBS: roleau forma on; UA:Bence-Jones proteinuria; serum electrophoresis: M spike; XR: cranial “punched-out” lesions.

Multiple myeloma18-y/o woman develops dyspnea and declining mental status 1 hour a er a C-sec on complicated by excess blood loss; PE: mucosal bleeding, large clot inthe vaginal vault; W/U: ↑ D-dimer, ↑ PT/PTT, ↓ antithrombin III, and thrombocytopenia.

Disseminated intravascular coagulation7-y/o with h/o viral URI 1 week ago presents with epistaxis; PE: petechial hemorrhages of nasal mucosa and extremi es; W/U: ↓ platelets, normal PT andPTT; bone marrow biopsy: ↑↑ megakaryocytes.

Idiopathic thrombocytopenic purpura8-y/o with a h/o vomi ng and diarrhea a er ea ng a hamburger last week presents with fa gue, periorbital edema, and oliguria; PE: purpuric rash; CBC: ↓platelets; PBS: burr cells, helmet cells; UA: RBC casts, proteinuria, hematuria.

Hemolytic uremic syndrome8-y/o with a h/o environmental allergies presents with a painful rash on the legs, abdominal discomfort, joint pain; UA: hematuria and RBC casts; renalbiopsy: glomerular mesangial IgA deposits.

Henoch-Schonlein purpura8-y/o boy presents with a swollen painful knee; FH: maternal grandfather died from hemorrhage a er a cholecystectomy; PE: cutaneous ecchymoses; W/U:gross blood in swollen knee joint, ↑ PTT, normal PT and platelet count, ↑ bleeding time.

Hemophi l ia A2-y/o boy with a h/o recurrent epistaxis presents with the third episode of o s media in 4 months; PE: eczematous derma s; W/U: thrombocytopenia, ↓IgM, ↑ IgA.

Wiskott-Aldrich syndromeNewborn develops jaundice rapidly during the first day of life; PE: HSM; W/U: severe anemia, ⊕ indirect Coombs test in both mother and newborn.

Rh incompatibi l i ty16-y/o adolescent with a h/o menorrhagia presents with fa gue; PE: mul ple cutaneous bruises; guiaic ⊕ stools; W/U: ↑ bleeding me, ↓ factor VIII, normalplatelet count, PT, and PTT.

von Wi l lebrand disease

SKIN AND CONNECTIVE TISSUES

30-y/o man with h/o recurrent sinusitis presents for sterility evaluation; PE: heart sounds are best heard over right side of chest.

Kartagener syndrome9-y/o with h/o easy bruising and hyperextensible joints presents to the ER after dislocating his shoulder for the fifth time this year.

Ehlers -Danlos syndrome5-y/o presents to the ER with his sixth bone fracture in the past 2 years; PE: bluish sclera and mild kyphosis; XR: fractures with evidence of osteopenia.

Osteogenes is imperfecta8-y/o with a long h/o severe sunburns and photophobia presents to the dermatologist for evaluation of several lesions on the face that have recently changedcolor and size.

Xeroderma pigmentosum6-y/o boy presents to the ophthalmology clinic with sudden ↓ visual acuity; PE: unusual body habitus, long and slender fingers, pectus excavatum, andsuperiorly dislocated lens.

Marfan syndrome36-y/o with h/o celiac disease presents with clusters of erythematous vesicular lesions over the extensor surfaces of the extremities.

Dermati ti s herpeti formis5-y/o patient presents with honey-colored crusted lesions at the angle of his mouth; Gram stain of pus: gram-positive cocci in chains.

Impetigo29-y/o HIV-positive patient presents with multiple painless pearly-white umbilicated nodules on the trunk and anogenital area.

Mol luscum contagiosum68-y/o fair-skinned farmer presents with large, telangiectatic, and ulcerated nodule on the bridge of the nose.

Basa l cel l carcinoma11-y/o presents with bilateral wrist pain and a rash; PE: erythematous, reticular skin rash of the face and trunk with a “slapped-cheek appearance.”

Erythema infectiosum43-y/o woman presents with difficulty swallowing; PE: bluish discoloration of the hands and shiny, tight skin over her face and fingers.

Progress ive systemic scleros is (scleroderma)5-y/o Asian boy presents with fever and diffuse rash including the palms and soles; PE: cervical lymphadenopathy, conjunc val injec on, and desquama onof his fingertips; echocardiogram reveals dilation of coronary arteries.

Kawasaki syndrome (mucocutaneous lymph node syndrome)33-y/o patient presents with itchy, purple plaques over her wrists, forearms, and inner thigh; PE: Wickham striae.

Lichen planus25-y/o woman with h/o Raynaud phenomenon presents with arthralgias and myositis; W/U reveals esophageal hypomotility and ↑ anti-nRNP titers.

Mixed connective ti ssue disease

MUSCULOSKELETAL

25-y/o man presents with morning s ffness, heel pain, and photophobia; PE: ↓ lumbar spine extension and lateral flexion, tenderness on lumbar spinousprocesses and iliac crests; W/U: HLA-B27 ⊕ ; XR: bamboo spine.

Ankylos ing spondyl i ti s7-y/o girl presents with a limp, anorexia, and spiking fevers; PE: salmon-pink linear rash on trunk and extremi es and swelling of bilateral hip, knee, elbow,and wrist joints; W/U: ↑ ESR.

Juveni le rheumatoid arthri ti s50-y/o woman presents with long-standing h/o morning s ffness and diffuse joint pain; PE: boutonierre and swan neck deformi es of fingers, shouldertenderness, and ↓ range of motion (ROM), symmetric and bilateral knee swelling; W/U: RF positive.

Rheumatoid arthri ti s25-y/o man with a h/o urethri s 2 weeks ago presents with unilateral knee pain, s ffness, and eye pain; PE: conjunc vi s, edema, and tenderness of leknee, mucoid urethral discharge; W/U: urethral swab ⊕ for Chlamydia.

Reiter syndrome45-y/o woman presents with dry eyes and dry mouth, PE: parotid gland enlargement, bibasilar rales; W/U: ⊕ ANA, RF, SS-A/Ro titers.

Sjögren syndrome70-y/o woman presents with pain in hands that is worse with ac vity, PE: Heberden and Bouchard nodes, bony enlargement at DIP and PIP joints, bilateralknee effusions; W/U: negative RF, normal ESR; hand XR: joint space narrowing, osteophytes.

Osteoarthri ti s72-y/o woman presents with a 6-week h/o morning s ffness in neck and shoulders; PE: low-grade fever, tenderness to palpa on, and decreased ROM inneck, shoulder, hip joints; W/U: ↓ ESR and CRP, but negative RF.

Polymyalgia rheumatica50-y/o man presents with acute onset of sharp pain in the le great toe; PE: severe tenderness, swelling, and warmth of the le MTP joint; synovial fluidanalysis shows negatively birefringent crystals.

Gout25-y/o woman with a 1-week h/o pain in several joints presents with swelling, redness, and pain in her right knee; PE: pustular lesions on palms, right kneeshows erythema, tenderness, and ↓ ROM; W/U: gram-negative diplococci in synovial fluid.

Gonococca l arthri ti s28-y/o woman presents with difficulty keeping her eyelids open and holding her head up during the day; PE: weakness of facial muscles, deltoids; W/U: an -ACh titer +; CXR: anterior mediastinal mass.

Myasthenia gravis (associated with thymoma)20-y/o black woman presents with fa gue, arthralgias, Raynaud phenomenon, and pleuri c chest pain; PE: bu erfly malar facial rash; W/U: ↓ platelets,proteinuria, and ⊕ ANA, anti-dsDNA, and anti-Smith Abs.

Systemic lupus erythematosus (SLE)20-y/o with a h/o developmental delay presents with facial weakness; PE: cataracts, marked weakness in muscles of hand, neck, and distal leg withsustained muscle contraction; genetic testing: CTG repeat expansion within DMPK gene.

Myotonic dystrophy

BEHAVIORAL SCIENCE

20-y/o woman presents with excessive anxiety about a variety of events for more than half of the days for the last 7 months.

Genera l i zed anxiety disorder8-y/o boy presents with a 5-month h/o regular bedwetting episodes; PE is unremarkable and fasting glucose is WNL.

Enures is28-y/o man who systematically checks each lock in his house every time before leaving, often causing him to be over an hour late for meetings.

Obsess ive-compuls ive disorder29-y/o man presents with a 9-month h/o insatiable urges to rub himself against strangers, which he has regretably acted on several times.

Frotteurism22-y/o woman college student who is 20% below her ideal body weight complains of not having any menstrual cycles and “feeling fat.”

Anorexia nervosa26-y/o woman medical student is convinced for the past 9 months that she has SLE and despite numerous nega ve workups, she fears she will have to dropout of school.

Hypochondrias is17-y/o woman presents with complaints of “feeling fat” and h/o ea ng dinner alone in her bedroom; PE shows normal height and weight, dental erosions,and scars on the back of her hand.

Bul imia21-y/o woman with no h/o trauma presents to the ER because she cannot feel or move her legs; W/U is WNL; detailed history reveals that her boyfriend leher this morning.

Convers ion disorder43-y/o alcoholic with h/o confabula on and amnesia presents to ER a er falling down; PE shows nystagmus and ataxic gait; W/U reveals macrocy canemia.

Wernicke-Korsakoff syndrome6-y/o presents with 8-month h/o hyperactivity, inattentiveness, and impulsivity both at school and at home; PE and W/U are WNL.

Attention defici t hyperactivi ty disorder (ADHD)33-y/o woman presents to your office distressed after turning down a lucrative job offer because of the requirement to speak in front of people.

Socia l phobia9-y/o boy with 2-year h/o involuntary tics is brought to your office because he has recently been shouting obscenities.

Tourette syndrome33-y/o woman nurse presents with recent occurrences of hypoglycemia; PE reveals mul ple crossed scars on abdomen; W/U shows insulin/C-pep de ra o >1.0.

Facti tious disorder (Munchausen syndrome)33-y/o is referred to you for reports of random episode of shou ng and screaming during the night; he has no recollec on of the event and denies havingnightmares.

Sleep terror disorder16-y/o with h/o sudden-onset daytime sleep attacks with loss of muscle tone and audiovisual hallucinations while waking and falling asleep.

Narcolepsy19-y/o with an 8-month h/o deteriora ng grades and social withdrawal presents with auditory hallucina ons; PE shows odd thinking pa erns, tangen althoughts, and flattened effects.

Schizophrenia24-y/o with h/o depression presents with ↓ need for sleep and auditory hallucina ons; PE reveals easy distrac bility and pressured speech; W/U showsnormal TSH and negative toxicology screen.

Bipolar disorder—type 1 (manic episode)48-y/o woman presents with recent h/o early morning waking, ↓ appe te, feelings of guilt, and loss of interest in her usual hobbies over the past 3 months;PE and laboratory results are WNL.

Major depress ive disorder68-y/o veteran presents with complaints of vivid flashbacks, hypervigilance, and difficulty falling asleep for the past several years; pa ent appears veryanxious during the PE.

Posttraumatic s tress disorder6-month-old child presents with unyielding crying; PE reveals mul ple bruises in different stages of healing and bilateral re nal hemorrhages; XR showsmultiple, healing fractures along posterior ribs.

Shaken baby syndrome43-y/o woman from Boston finds herself in Utah, but does not remember why she is there or how she got there; PE and W/U are WNL; detailed historyreveals that her son suddenly died 1 week ago.

Dissociative fugue3-y/o boy with h/o of poor cuddling presents with severely delayed language and social development; PE reveals below normal intelligence with unusualcalculating abilities and repetitive behaviors.

Autism

APPENDIXAbbreviations

AA

amino acidAb

antibodyABG

arteria l -blood gasACE

angiotens in-converting enzymeACEi

ACE inhibi torACh

acetylchol ineACL

anterior cruciate l igamentACTH

adrenocortiotropic hormoneAD

autosomal dominantADH

antidiuretic hormoneADHD

attention defici t hyperactivi ty disorderADP

adenos ine diphosphateAFP

α-fetoproteinAg

antigenAIDS

acquired immunodeficiency syndromeALL

acute lymphocytic leukemiaALP

alka l ine phosphataseALS

amyotrophic latera l scleros isALT

alanine transaminaseAML

acute myelogenous leukemiaANA

antinuclear antibodyANOVA

analys is of varianceANS

autonomic nervous systemAR

autosomal recess iveARB

angiotens in receptor blockerARDS

acute respiratory dis tress syndromeASA

aspirinASD

atria l septa l defectASO

antis treptolys in OAST

aspartate transaminaseATN

ischemic acute tubular necros isATP

adenos ine triphosphateATPase

adenos ine triphosphataseAV

atrioventricularAXR

abdominal x-rayAZT

azidothymidineBAL

Bri ti sh anti -Lewis i te

BMbasement membrane

BPblood pressure

BPGbisphophoglycerate

BPHbenign prostatic hyperplas ia

BRbi l i rubin

BUNblood urea ni trogen

Bxbiopsy

CAcancer/carcinoma

CADcoronary artery disease

cAMPcycl ic adenos ine monophosphate

cANCAcytoplasmic pattern of antineu trophi l cytoplasmic antibodies

CBCcomplete blood count

CCKcholecystokinin

CEAcarcinoembryonic antigen

CFcystic fibros is

CFTRcystic fibros is transmembrane regulator

cGMPcycl ic guanos ine monophosphate

CHFcongestive heart fa i lure

CINcervica l intraepi thel ia l neoplas ia

CKserum creatine kinase

CLLchronic lymphocytic leukemia

CMLchronic myelogenous leukemia

CMVcytomegalovi rus

CNcrania l nerve

CNScentra l nervous system

COcardiac output

CoAcoenzyme A

COMTcatechol -O-methyl transferase

COPDchronic obstructive pulmonary disease

COXcyclooxygenase

Cp

concentration in plasmaCPK

creatine phosphokinaseCr

creatinineCRP

C-reactive proteinCSF

cerebrospina l fluidCT

computed tomographyCV

cardiovascularCVA

cerebrovascular accident or costovertebra l angleCXR

chest x-rayD or DA

dopamined

day(s )d/o

disorderDAG

diacylglycerolDES

diethyls ti lbesterolDHT

dihydrotestosteroneDI

diabetes ins ipidusDIC

disseminated intravascular coagulationDIP

dis ta l interphalangeal jointDKA

diabetic ketoacidos isDM

diabetes mel l i tusDNA

deoxyribonucleic acidDOE

dyspnea on exertionDRE

digi ta l recta l examinationds

double s trandedDSM

diagnostic and s tati s tica l manualDTR

deep tendon reflexDTs

delerium tremensDVT

deep venous thrombos isE

epinephrineEBV

Epstein-Barr vi rusECF

extracel lular fluidECG

electrocardiogramECT

electroconvuls ive therapyEDV

end-diastol ic volumeEEG

electroencephalogramEF-2

elongation factor 2EGD

esophagogastroduodenoscopyELISA

enzyme-l inked immunosorbent assayEM

electron microscopyEOM

extraocular muscleEPO

erythropoetinEPS

extrapyramidal symptomsER

endoplasmic reticulum or emergency roomESR

erythrocyte sedimentation rateESV

end-systol ic volumeETOH

ethanolFAs

fatty acidsFAD

flavin adenine dinucleotide (oxidized)FADH2

flavin adenine dinucleotide (reduced)

FAPfami l ia l adenomatous polypos is

FENa+

fractional excretion of sodiumFFP

fresh frozen plasmaFH

fami ly his toryFSH

fol l i cle-s timulating hormoneFN

fa lse negativesFP

fa lse pos i tivesFTA-ABS

fluorescent treponemal antibody-absorbedFx

fractureG3P

glucose-3-phosphateG6PD

glucose-6-phosphate dehydrogenaseGABA

γ-aminobutyric acidGBM

glomerular basement membraneGCT

germ cel l tumorGFR

glomerular fi l tration rateGGT

γ-glutamyl transpeptidaseGH

growth hormoneGI

gastrointestina lGi

G protein, inhibi toryGMP

guanos ine monophosphateGN

glomerulonephri ti sGnRH

gonadotropin-releas ing hormoneGs

G protein, s timulatoryGSE

genera l somatic efferentGTP

guanos ine triphosphateGU

genitourinaryGVE

genera l vi scera l efferentHA

headacheHb

hemoglobinHBV

hepati ti s B vi rushCG

human chorionic gonadotropinHDL

high-dens i ty l ipoproteinHGPRT

hypoxanthine guanine phosphoryl transferaseHHV

human herpesvi rusHIV

human immunodeficiency vi rusHMG-CoA

hydroxymethylglutaryl -CoAh

hour(s )h/o

his tory ofHPETE

hydroperoxyeicosatetraenoic acid

HPVhuman papi l lomavirus

HRheart rate

HRThormone replacement therapy

HSMhepatosplenomegaly

HSVherpes s implex vi rus

HTLVhuman T-cel l leukemia/lymphoma vi rus

HUShemolytic-uremic syndrome

HTNhypertens ion

IBDinflammatory bowel disease

ICAMintracel lular adhes ion molecule

ICFintracel lular fluid

ICPintracrania l pressure

IFintrins ic factor

Igimmunoglobul in

ILinterleukin

IMintramuscular

INDindication(s )

INHisoniazid

IOPintraocular pressure

IP3

inos i tol triphosphateIPV

inactivated pol io vaccineIUGR

intrauterine growth retardationIV

intravenousIVC

inferior vena cavaIVIG

IV immunoglobul inJG

juxtaglomerularJVD

jugular venous dis tens ionL

leftLAD

left anterior descendingLCA

left coronary arteryLDH

lactate dehydrogenaseLDL

low-dens i ty l ipoproteinLES

lower esophageal sphincterLFT

l iver function testLH

leutinizing hormoneLLQ

left lower quadrantLLSB

left-lower s ternal borderLMN

lower motor neuronLP

lumbar punctureLPS

l ipopolysaccharideLT

leukotrieneLUSB

left-upper s ternal borderLUQ

left upper quadrantLV

left ventricleMAOI

monoamine oxidase inhibi torMCA

middle cerebra l arteryMCP

metacarpel phalangealMCL

media l col latera l l igamentMCV

mean corpuscular volumeMEN

multiple endocrine neoplas iaMHC

major his tocompatibi l i ty complexMI

myocardia l infarctionMLF

media l longi tudina l fasciculusmm

mil l imeters or musclesMMR

meas les , mumps, rubel laMOA

mechanism of actionMPTP

1-methyl -4-phenyl -1,2,3,6-tetrahydropyridineMRI

magnetic resonance imagingMS

multiple scleros isMTP

metatarsa l -phalangealMVA

motor vehicle accidentNAD

nicotinamide adenine dinucleotide (oxidized)NADH

nicotinamide adenine dinucleotide (reduced)NADP

nicotinamide adenine dinucleotide phosphate (oxidized)NADPH

nicotinamide adenine dinucleotide phosphate (reduced)NE

norepinephrineNSAID

nonsteroida l anti -inflammatory drugN/V

nausea/vomitingOAA

oxaloacetic acidOCP

ora l contraceptive pi l l sOPV

ora l pol io vaccinePABA

para-aminobenzoic acidPAF

platelet activi ty factorPAH

para-aminohippuric acidPALS

periarteria l lymphoid sheathPAN

polyarteri ti s nodosaP-ANCA

perinuclear pattern of antineutrophi l cytoplasmic antibodiesPAS

periodic acid-Schi ff (s ta in)PBS

periphera l blood smear

PCAMplatelet cel l adhes ion molecule

PCI2

prostacycl in I2

PCLposterior cruciate l igament

PCPphencycl idine hydrochloride

PCRpolymerase chain reaction

PCWPpulmonary capi l lary wedge pressure

PDApatent ductus arteriosus

PDEphosphodiesterase

PDGFplatelet-derived growth factor

PEphys ica l examination

PFKphosphofructokinase

PFTpulmonary function tests

PGprostaglandin

PIDpelvic inflammatory disease

PIPproximal interphalangeal

PIP2

phosphatidyl inos i tol -4,5-bisphosphonatePK

pyruvate kinasePKU

phenylketonuriaPMN

polymorphonuclearPNH

paroxysmal nocturnal hemoglobinuriaPNS

periphera l nervous systemPPi

pronton-pump inhibi torPPRF

parapontine reticular formationPSA

prostate-speci fic antigenPT

prothrombin timePTH

parathyroid hormonePTT

partia l thromboplastin timePVD

periphera l vascular diseaseR

rightRA

right atriumRAA

Renin angiotens in a ldosteroneRBC

red blood cel lRBF

renal blood flowRCA

right coronary arteryREM

rapid eye movementRER

rough endoplasmic reticulumRF

rheumatoid factorRLQ

right lower quadrantROM

range of motion

RPFrenal plasma flow

RPRrapid plasma reagin

RRrespiratory rate

RSVrespiratory syncytia l vi rus

RUQright upper quadrant

RVright ventricle

RVHright ventricular hypertrophy

ssecond(s )

S1(2,3,4)1stheart sound (2nd, 3rd, 4th)

SAs inoatria l

SCsubcutaneous or s ickle cel l

SDstandard deviation

SEMstandard error of the mean

SESsocioeconomic s tatus

SGOTserum glutamic oxa loacetic transaminase

SGPTserum glutamic pyruvate transaminase

SLEsystemic lupus erythematosus

SMXsul famethoxazole

SOBshortness of breath

sss ingle s tranded

SSAspecia l somatic afferent

SSPEsubacute scleros ing panencephal i ti s

SSRIselective serotonin reuptake inhibi tor

STDsexual ly transmitted disease

SVstroke volume

SVAspecia l vi scera l a fferent

SVTsupraventricular tachycardia

Sxsymptom(s)

s/pstatus post

t1/2

hal f-l i feT3

tri iodothyronineT4

thyroxineTB

tuberculos isTBW

tota l body waterTG

triglycerideTIBC

tota l i ron binding capaci tyTMP

trimethoprimTN

true negativesTNF

tissue necros is factor

TNMtumor node metastas is

TOXtoxici ty

TPtrue pos i tives

tPAtissue plasminogen activator

TPRtota l periphera l res is tance

TRHthyrotropin-releas ing hormone

TSHthyroid-s timulating hormone

TSStoxic shock syndrome

TTPthrombotic thrombocytopenic purpura

Txtreatment/therapy

TXAthromboxane

UAurina lys is

UMNupper motor neuron

UGIupper GI

URIupper respiratory infection

UTIurinary tract infection

U/Sultrasound

Vd

volume of dis tributionVCAM

vascular cel l adhes ion moleculeVDRL

venereal disease research laboratoryVfib

ventricular fibri l lationVHL

von Hippel -LindauVLDL

very low-dens i ty l ipoproteinVMA

vani l lylmandel ic acidV/Q

venti lation/perfus ion ratioVSD

ventricular septa l defectvWF

von Wi l lebrand factorVZV

varicel la -zoster vi rusWBC

white blood cel lWNL

within normal l imitsXL

x-l inkedXR

x-rayy/o

year-oldZE

Zol l inger-El l i son5-FU

5-fluorouraci l5-HIAA

5-hydrocyindoleacetic acid5-HT

5-hydroxytryptamine (serotonin)↑

High or increases↓

Low or decreases→

Leads to or causes1°/2°/3°

primary/secondary/tertiary~

Approximately⊕

pos i tive>>>

much greater thanmuch less than

Index

A

Abeta l ipoproteinemiaAbruptio placentaeAbscessAbsorption, drugAcanthos is nigricansAccuracyACE inhibi tors . See Angiotens in-converting enzyme inhibi torsAcebutololAcetaminophen (Tylenol )AcetazolamideAcetylchol ine (ACh)Acetylchol inesterase poisoningACh. See Acetylchol ineAchalas iaAchondroplas iaAcid-base dis turbancesAcidos isAcinus of the lungAcne vulgarisAcquired immunodeficiency syndrome (AIDS)AcromegalyACTH. See Adrenocorticotropic hormoneActinomycesActivatorAcute postinfectious glomerulonephri ti sAcute renal fa i lure (ARF)Acyclovi rAD disorders . See Autosomal dominant disordersAddison diseaseAdenocarcinomaAdenomasAdenos ine deaminase deficiencyAdenos ine triphosphate (ATP)AdenovirusADH. See Antidiuretic hormoneADHD. See Attention defici t hyperactivi ty disorderAdipose ti ssueAdjustment disorderADPKD. See Autosomal dominant polycystic kidney diseaseAdrenal glandsAdrenocortica l insufficiencyAdrenocorticotropic hormone (ACTH)Adrenogenita l syndromesAdrenoleukodystrophyAdriamycinAdult respi ratory dis tress syndrome (ARDS)African s leeping s icknessAgingAgonis tAIDS. See Acquired immunodeficiency syndromeAlanineAlbendazoleAlbinismAlbuterolAlcohol

abuseteratogenic effects

Alcohol ic di lated cardiomyopathyAldosteroneAlka l inze urineAlka los isAlkaptonuriaAl lergic bronchopulmonary aspergi l los isAl lopurinolα-Glucos idase inhibi torα-Methyldopaα-TocopherolAlport syndromeALS. See Amyotrophic latera l scleros isAluminum hydroxideAluminum sucrose sul fate

Alzheimer diseaseAmantadineAmenorrheaAmi lorideAmino acidsAminocaproic acidAminoglycos idesAminolevul inate synthaseAmiodaroneAmitriptyl ineAmnes iaAmniotic fluid embol ismAmoxapineAmoxici l l inAmphetamineAmphotericinAmpici l l inAmprenavirAmyloidosesAmyotrophic latera l scleros is (ALS)ANA. See Antinuclear antibodiesAnacl i ti c depress ionAnalges icsAnalys is of variance (ANOVA)Anaphylactic shockAnaplastic carcinomaAnatomic snuffboxAnatomy

cardiovascular systemconnective ti ssuesendocrine systemgastrointestina l systemgenitourinary systemmusculoskeleta l systemreproductive systemrespiratory systemskin

Androgen insens i tivi ty syndromeAnemiaAnemic infarctsAnencephalyAnestheticsAneurysmsAnginaAngiodysplas iaAngiotens in I IAngiotens in receptor blockers (ARBs)Angiotens in-converting enzyme (ACE) inhibi torsAnkylos ing spondyl i ti sAnorexia nervosaANOVA. See Analys is of varianceAntacidsAnterior pi tui taryAnthracos isAnthraxAntiacetylchol ine esteraseAntiangina l agentsAntiarrhythmicsAntibiotics . See also specific drugsAntichol inergic agentsAntichol inesterasesAnticoagulantsAntidepressantsAntidiuretic hormone (ADH)Antiepi lepticsAntihis taminesAntihypertens ivesAntinuclear antibodies (ANA)Antiparkinsonian agentsAntiphosphol ipid antibody syndromeAntipsychoticsAntisocia l personal i ty disordersAntivi ra l agents . See also specific drugsAnxietyAortic dissectionAortic regurgi tationAortic s tenos isApgar scoreAplas ia

Apol ipoproteinApoptos isAppendici ti sApraxiaAR disorders . See Autosomal recess ive disordersArachidonic acidArbovirusesARBs . See Angiotens in receptor blockersArenaviridaeARF. See Acute renal fa i lureArgyl l -Robertson pupi lArnold-Chiari mal formationArsenicArterioloscleros isArtery(ies )

CNScoronaryin s troke

Arthri ti sAsbestos isAscending cholangi ti sAschoff bodiesAscorbic acidASD. See Atria l septa l defectAsperger syndromeAspergillus sp.Aspirin (sa l icylates )AsthmaAstrovi rusAtaxia-telangiectas iaAtelectas isAtenololAtheroscleros isATP. See Adenos ine triphosphateAtracuriumAtria l fibri l lationAtria l myxomaAtria l natriuretic peptideAtria l septa l defect (ASD)AtrophyAtropineAttention defici t hyperactivi ty disorder (ADHD)Auerbach plexus . See Myenteric plexusAutismAutoantibodiesAutoimmune connective ti ssue disordersAutomatici tyAutonomicsAutoregulationAutosomal dominant (AD) disordersAutosomal dominant polycystic kidney disease (ADPKD)Autosomal recess ive (AR) disordersAvers ive conditioningAvoidant personal i ty disordersAxonal transportAzi thromycinAzole fami lyAZT. See Zidovudine

B

B lymphocyteBabesiaBabinski reflexBacillusBacteria . See also specific bacteria

binding and phagocytos is ofgram-pos i tiveinfectious diseases fromtaxonomy

Bacteria l vaginos isBacteroidesBactrimBalani ti sBang diseaseBarbi turatesBarrett’s esophagusBartter syndrome

Basa l cel l carcinomaBasophi lBeckwith-Wiedemann syndromeBeclomethasoneBehaviora l neurochemistryBeneficenceBenign prostatic nodular hyperplas iaBenzodiazepinesBenztropineBeriberiBernard-Soul ier diseaseBerry aneurysmα-Agonis tsβ-Blockersβ-Endorphinβ-Lactam antibioticsβ-Lactamase inhibi torsBetamethasoneBethanecholBiasBicarbonateBicuspid aortic va lveBi leBi l iary atres iaBi l i rubinBiostatis ticsBiotinBipolar disorderBismuth subsa l icylateBlack feverBleomycinBlood pressure maintenanceBody dysmorphic disorderBoerhaave syndromeBone

fracturesneoplas ia ofnonneoplastic disordersosteomyel i ti s

Borderl ine personal i ty disordersBordetella pertussisBorreliaBotul inumBrachia l plexusBra in tumorsBranchia l archesBreast disordersBretyl iumBrief psychotic disorderBrodmann areasBromocriptineBronchiectas isBronchi ti sBronchodi latorsBrucellaBubonic plagueBuccopharyngeal membraneBudd-Chiari syndromeBuerger diseaseBul imiaBul laBul lous pemphigoidBunyaviridaeBupivaca ineBupropionBurnsBusul fan

C

Ca lci toninCalciumCalcium channel blockersCalcium regulationCal icivi ruscAMP. See Cycl ic adenos ine monophosphateCampylobacter jejuniCancer. See specific types and organs

Candida albicansCandidias isCarbamazepineCarbidopaCarbohydratesCarbon monoxideCarcinogensCarcinoid syndromeCarcinoid tumorsCarcinoma. See specific types and organsCardiac contracti l i ty and outputCardiac electrophys iologyCardiac infectious disordersCardiac muscleCardiac tumorsCardiogenic shockCardiomyopathy. See also Hypertrophic cardiomyopathyCardiovascular system

anatomydisorders

aneurysmsarterioloscleros isatheroscleros iscardiac tumorscardiomyopathycase presentationscongenita l heart diseasedissectionheart fa i lurehypertens ioninfectiousischemic heart diseasemurmurspericardi ti svascular tumorsvascul i tides

embryologypharmacology

antiangina l agentsantiarrhythmicsantihypertens ivesautonomicsinotropic supportl ipid-lowering agents

phys iologyCase-control s tudyCatscratch diseaseCauda equina syndromeCCK. See CholecystokininCDR. See Complementary determining regionCeftriaxoneCelecoxibCel iac diseaseCel l changes/accumulationsCel l cycleCel l injuryCel l membranesCel lular adaptationsCentra l nervous system (CNS)

arteriescerebel lum, tha lamus, hypothalamus ofcerebra l cortexcrania l nervesenergy sourceslymphomaspina l tracts

CephalosporinsCerebel lar cortexCerebel lumCerebra l cortexCerebra l i schemiaCerebrospina l fluid (CSF)Cervica l carcinomaCervica l intraepi thel ia l neoplas ia (CIN)CF. See Cystic fibros isChancroidChediak-Higashi diseaseChemotherapeutic agentsCHF. See Congestive heart fa i lure

Chi ld abuseChi ldhood psychiatric disordersChi -squared testChlamydiaChloramphenicolChloroquineChlorpromazineChlorpropamideCholangiocarcinomaCholangi ti sCholecysti ti sCholecystokinin (CCK)Cholel i thias isCholesterolCholesterolos isCholestyramineChondromaChondrosarcomaChoriocarcinomaChorionChromosomesChronic granulomatous diseaseChronic obstructive pulmonary disease (COPD)Churg-Strauss syndromeCimetidineCIN. See Cervica l intraepi thel ia l neoplas iaCiprofloxacinCirrhos isCisplatinCi ta lopramCitric acid cycleClass ica l conditioningClavulanic acidClearanceCleft l ip, pa lateCl indamycinCl inica l testingClofazimineClofibrateClomipheneClomipramineClonidineClopidogrelClostridiumClozapineCMV. See Cytomegalovi rusCNS. See Centra l nervous systemCoagulopathyCobalaminCocaineCoccidioides immitisCoccidioidomycos isCohort s tudyColchicineCol lagenCol lecting tubuleColonColon carcinomaCommon bi le ductCommon variable immunodeficiencyCompartment syndromeComplement systemComplementary determining region (CDR)ComplementationConduction veloci tyCondyloma acuminatumConesConfidentia l i tyConfi rmatory testCongenita l heart diseaseCongestive heart fa i lure (CHF)Conjunctivi ti sConn syndromeConnective ti ssues

anatomydisordersembryologyhis tology

Construction apraxia

Contrecoup injuryConvers ion disorderCOPD. See Chronic obstructive pulmonary diseaseCor pulmonaleCori cycleCoronary arteriesCoronary occlus ionCorticosteroidsCorti solCorti soneCorynebacteriumCountertransference reactionCoup injuryCourvois ier lawCOX pathwayCoxsackie vi rusesCrania l nervesCraniopharyngiomaCREST syndromeCretinismCreutzfeldt-Jacob diseaseCrigler-Najjar syndromeCrocodi le tears syndromeCrohn’s diseaseCromolynCroupCryptococcos isCryptorchidismCryptosporidiumCSF. See Cerebrospina l fluidCyanideCyanocobalaminCycl ic adenos ine monophosphate (cAMP)CyclophosphamideCyclothymiaCystic fibros is (CF)Cysticercos isCystinuriaCystosarcoma phyl lodesCytarabineCytogenetic disordersCytokinesCytomegalovi rus (CMV)

D

Dandy-Walker mal formationDantroleneDapsoneddC. See Za lci tabineddI. See Didanos ineDeafnessDeathDecerebrate rigidi tyDecorticate rigidi tyDeferoxamineDelavi rdineDeletion disorderDel i riumDel i rium tremensDelus ionDemeclocycl ineDementiaDemyel inating disordersDenta l cariesDenys-Drash syndromeDepress ionDermati ti sDermatomyos i ti sDES. See Diethyls ti lbestrolDes ipramineDesmopress inDevelopmenta l s tagesDexamethasoned4T. See StavudineDiabetesDiabetic glomeruloscleros isDiabetic ketoacidos is

Dia lys isDiaphragmDiarrheaDiastol ic blood pressureDiazepamDIC. See Disseminated intravascular coagulationDidanos ine (ddI)DiethylcarbamazineDiethyls ti lbestrol (DES)DiGeorge syndrome. See Thymic aplas iaDigi ta l i sDigoxinDi lantinDi lated cardiomyopathyDi l tiazemDimercaprolDinoprostoneDiphenhydramineDiphenoxylateDiphyllobothrium latumDisaccharide deficiencyDiscoid lupus erythematosusDisease preventionDisopyramideDissectionDisseminated intravascular coagulation (DIC)Dissociative identi ty disorderDis tribution, drugDisul fide bondsDisul fi ramDiureticsDiverticular diseaseDivorceDNA

mutationsrepl icationtransfer proceduresvi ruses

DobutamineDopamineDose-response curveDown syndromeDoxazos inDoxepinDoxorubicinDoxycycl ineDrug development. See also PharmacologyDubin-Johnson syndromeDucta l carcinomaDumping syndromeDuodenal atres iaDuodenal ulcerDuodenumDurable power of attorneyDysfunctional uterine bleedingDyssomniaDystrophic ca lci fi cation

E

Ebstein anomalyEBV. See Epstein-Barr vi rusECF. See Extracel lular fluidEchinococcus granulosusEchothiophateEclamps iaEctodermEctopic pregnancyEdemaEdrophoniumEfavi renzEffective doseEfficacyEgoEhlers -Danlos syndromeEjection fractionElder abuseEl imination, drug

ELISA. See Enzyme-l inked immunosorbent assayEmbol ismEmbryology

cardiovascular systemconnective ti ssuesendocrine systemgastrointestina l systemgenera lgeni tourinary systemnervous systemred blood cel l productionreproductive systemrespiratory systemskin

EmphysemaEncainideEncephal i ti sEncephaloceleEnchondromaEndocardi ti sEndocrine system

anatomydisorders

adrenaladrenogenita l syndromescase presentationsdiabetespancreatic endocrine tumorsparathyroidpi tui tarythyroid

embryologykidneypharmacology

adrenal glanddiabetesparathyroidpi tui tarythyroid

phys iologyEndodermEndometria l carcinomaEndometrios isEndotoxinEnhancerEnkephal insEntamoeba histolyticaEnteric nervous systemEnterobacterEnterobius vermicularisEnterococcusEnures isEnzyme-l inked immunosorbent assay (ELISA)EnzymesEos inophi lEos inophi l i c granulomaEpendymomaEphedrineEpidemiologyEpididymisEpididymiti sEpidura l hematomaEpiglotti ti sEpi lepsyEpinephrineEpispadiasEpstein-Barr vi rus (EBV)Epti fibatideErb-Duchenne pa lsyErythema infectiosumErythema multi formeErythromycinErythropoietinEscherichia coliEsci ta lopramEsmololEsophageal atres iaEsophageal carcinomaEsophageal varices

EsophagusEssentia l fructosuriaEssentia l thrombocytos isEstrogenEthacrynic acidEthicsEthosuximideEthylene glycolETOH. See Hepatic ethanolEtopos ideEwing sarcomaExotoxinsExstrophy of the bladderExtinctionExtracel lular fluid (ECF)Exudate

F

Fabry diseaseFacia l nerveFacti tious disorderFactor V LeidenFactor XIIaFa l lopian tubeFami l ia l adenomatous polypos is (FAP)Fami l ia l hypercholesterolemiaFamotidineFanconi syndromeFAP. See Fami l ia l adenomatous polypos isFat digestionFat embol iFata l fami l ia l insomniaFat-soluble vi taminsFel ty syndromeFemale reproductive system disordersFeta l a lcohol syndromeFeta l hydantoin syndromeFibroadenomaFibrocystic changeFibrous dysplas iaFi lovi ridaeFinasterideFi rs t-pass metabol i smFitz-Hugh and Curti s syndromeFleca inideFlexor withdrawal reflexFluconazoleFlucytos ineFlumazeni lFluorescent treponemal antibody-absorption test (FTA-ABS)FluoroquinolonesFluoxetineFlutamideFluticasoneFluvoxamineFocal -segmenta l glomerulonephri ti s (FSGN)FolateFol ic acid deficiencyFol l i cle-s timulating hormone (FSH)Fol l i cular carcinomaFomepizoleForegutFoscarnetFracturesFragi le X syndromeFrancisellaFrank-Starl ing relationshipFreudian theoryFrey syndromeFriedreich ataxiaFrotteurismFructose intoleranceFSGN. See Foca l -segmenta l glomerulonephri ti sFSH. See Fol l i cle-s timulating hormoneFTA-ABS. See Fluorescent treponemal antibody-absorption testFungiFurosemide

G

Ga i t apraxiaGalactosemiaGal l bladder disordersGal l s toneGanciclovi rGardner syndromeGardnerella vaginalisGastric acidGastric carcinomaGastric inhibi tory peptide (GIP)Gastric ulcerGastrinGastri ti sGastrocol ic reflexGastrointestina l (GI) system

alcohol i sm influencinganatomydisorders

case presentationsdiverticular diseasegal l bladdergastri ti sinfectious diarrheainflammatory bowel diseasel ivermalabsorptionneoplasticnonneoplasticpancreatic cancerpancreati ti spediatricpeptic ulcer diseasevi ra l hepati ti s

embryologypharmacologyphys iology

Gaucher diseaseGemfibrozi lGenesGenita l iaGenitourinary system. See also Kidney(s )

anatomydisorders

acid-base dis turbancesacute renal fa i lurecase presentationsglomerulonephropathiesnephrotic and nephri tic syndromespolycystic kidney diseaserenal infectionsrenal tubular acidos isrenal tumorsrenal vascular diseasetubulointersti tia l diseasesurinary systemurinary tract tumorsurol i thias is

embryologypharmacologyphys iology

GentamicinGenuGERDGerminomaGestational choriocarcinomaGFAP. See Gl ia l fibri l lary acidic proteinGFR. See Glomerular fi l tration rateGH. See Growth hormoneGHRH. See Growth hormone-releas ing hormoneGI lymphomasGI system. See Gastrointestina l systemGiant cel l tumor of boneGiardia lambliaGigantism. See Increased l inear growthGi lbert syndromeGIP. See Gastric inhibi tory peptide

Glanzmann thrombastheniaGl ia l fibri l lary acidic protein (GFAP)Gl ioblastoma multi formeGl i tazonesGlomerular fi l tration rate (GFR)GlomerulonephropathiesGlucagonGlucocorticoidsGluconeogenes isGlucose-6-phosphate dehydrogenaseGlutamine synthetaseGlyburideGlycineGlycogen s torage diseasesGlycogenolys isGlycolys isGlyconeogenes isGoiterGolgi tendon organsGolgi tendon reflexGonococca l arthri ti sGonorrheaGoodpasture syndromeGoutGraftsGraft-versus-host diseaseGram-pos i tive bacteriaGranisetronGranuloma inguina leGrapefrui tGraves diseaseGriseofulvinGrowth hormone (GH)Growth hormone-releas ing hormone (GHRH)GTP. See Guanos ine triphosphateGuanethidineGuanos ine triphosphate (GTP)Gui l la in-Barré syndrome

H

HAART. See Highly active anti retrovi ra l therapyHaemophilus ducreyiHaemophilus influenzaeHageman factor. See Factor XIIaHal f-l i fe, drugHal lucinationHaloperidolHalothanehand-foot-and-mouth diseaseHand-Schül ler-Chris tian diseaseHansen diseaseHantavi rusHaptenHartnup diseaseHashimoto diseaseHaustraHealth insurance portabi l i ty and accountabi l i ty act (HIPAA)Heart

amyloidos is influencingdiseaseenergy sourcesfa i luresounds

Helicobacter pyloriHELLP syndromeHelminthsHemangioblastomaHemangiosarcomaHematologyHemeHemichol iniumHemochromatos isHemodynamic dys functionHemoglobinHemolytic uremic syndrome (HUS)Hemophi l iaHemorrhages

Hemos iderinHemostas isHenoch-Schönlein purpuraHeparinHepatic adenomaHepatic ethanol (ETOH)Hepatic fa i lureHepati ti sHepati ti s AHepati ti s BHepati ti s CHepati ti s EHepatocel lular carcinomaHereditary hemorrhagic telangiectas iaHereditary nonpolypos is colorecta l carcinoma (HNPCC)HerniasHerpes geni ta l i sHerpes labia l i sHerpes s implex vi rus (HSV)Herpes zoster ophthalmicusHerpesvi ridaeHHV. See Human herpesvi rusHigh a l ti tude, accl imatization toHighly active anti retrovi ra l therapy (HAART)HIPAA. See Heal th insurance portabi l i ty and accountabi l i ty actHirschsprung diseaseHis tamineHis tiocytos is XHis tonesHistoplasma capsulatumHistoplasmos isHis trionic personal i ty disordersHIV. See Human immunodeficiency vi rusHLA subtypesHNPCC. See Heredi tary nonpolypos is colorecta l carcinomaHodgkin diseaseHomocystinuriaHormones . See also specific hormonesHorner syndromeHorseshoe kidneyHospice careHot-tub fol l i cul i ti sHPV. See Human papi l lomavirusHSV. See Herpes s implex vi rusHTLV. See Human T-cel l leukemia/lymphoma vi rusHTN. See Hypertens ionHuman herpesvi rus (HHV)Human immunodeficiency vi rus (HIV)Human papi l lomavirus (HPV)Human T-cel l leukemia/lymphoma vi rus (HTLV)Hunter syndromeHuntington diseaseHurler syndromeHUS. See Hemolytic-uremic syndromeHydra lazineHydroceleHydrocephalusHydrocorti soneHydrops feta l i sHydroxylase deficienciesHypera ldosteronismHyperbi l i rubinemiaHyperca lcemiaHypercortici sm (Cushing syndrome)HyperemiaHyper-IgM syndromeHyperparathyroidismHyperplas iaHyperpolarizationHypersens i tivi ty reactionsHypertens ion (HTN)HyperthyroidismHypertrophic cardiomyopathyHypertrophyHypervariable regionHypochondrias isHypoglycemiaHypomaniaHypoparathyroidism

Hypoplas iaHypospadiasHypothalamusHypothenar eminenceHypothyroidismHypovolemic shockHypoxic encephalopathy

I

Ibuti l ideICAM. See Intercel lular adhes ion moleculeI-cel l diseaseICF. See Intracel lular fluidIdIdeas of referenceIdiopathic pulmonary fibros isIdiopathic thrombocytopeniaIF. See Intrins ic factorIg. See Immunoglobul insIL-. See InterleukinsIleumIl lus ionImipramineImmune system

autoantibodiescel l sdisordershypersens i tivi ty reactionsimmunoglobul ins intransplant rejection and

ImmunodeficienciesImmunoglobul ins (Ig)ImpetigoImprintingIncidenceInclus ionsIncreased l inear growth (gigantism)Indinavi rIndomethacinInfarctsInfectious diarrheaInfectious diseases . See also specific diseases

bacteria lcardiaccase presentationsfungalprotozoalvi ra l

Infective endocardi ti sInflammationInflammatory bowel diseaseInfluenzaInformed consentInheri ted diseasesInotropic supportInsul inInsul inomaIntention tremorIntercel lular adhes ion molecule (ICAM)Interferon-αInterleukins (IL-)Intersti tia l nephri ti sIntestinesIntracel lular fluid (ICF)Intracrania l hemorrhageIntraducta l papi l lomaIntranuclear ophthalmoplegiaIntrins ic factor (IF)IntussusceptionInvers ion disorderIodoquinolIpratropiumIronIschemic heart diseaseIschemic/hypoxic cel l injuryIsocarboxazidIsoniazid

Isonicotinic acidIsoproterenolIsotretinoinIvermectin

J

Jari sch-Herxheimer reactionJaundiceJC vi rusJejunumJob syndromeJointsJuveni le pi locytic as trocytoma

K

Ka l lmann syndromeKapos i sarcomaKartagener syndromeKawasaki diseaseKeloidKeratoacanthomaKeratos isKetamineKetoconazoleKidney(s )

disordersendocrine functions ofhorseshoepharmacology

Kimmelstiel -Wi lson diseaseKlebsiellaKl inefel ter syndromeKlüver-Bucy syndromeKorsakoff psychos isKrabbe diseaseKübler-Ross s tages of dyingKuruKwashiorkor

L

Labeta lolLactobacillusLactose intoleranceLactuloseLacunar infarctionsLambert-Eaton syndromeLamivudine (3TC)LamotrigineLangerhans cel l sLansoprazoleLarge cel l carcinomaLaryngotracheobronchiti sLatera l epicondyl i ti sLearned helplessnessLeft MCA cerebrovascular accidentLegg-Calvé-Perthes diseaseLegionellaLeiomyomaLeiomyosarcomaLeishmaniaLeprosyLesch-Nyhan SyndromeLethal doseLeukemiaLeukocytesLeukodystrophiesLeukotriene inhibi torsLeuprol ideLevodopaLevofloxacinLevothyroxineLH. See Luteinizing hormone

Lichen planusLidoca ineLi fe cycleLigamentsLini ti s plasticaLipid metabol i smLipid-lowering agentsLipofuscinLipogenes isListeriaLi thiumLiver

disordersenergy sourcesmetabol i sm andnutmeg

Living wi l lLoa loaLoading doseLobular carcinomaLocal anestheticsLocked-in syndromeLoose associationLoperamideLoratadineLosartanLow birth weightLung(s ). See also Respiratory system

capaci ty and volumecompl iance and res is tancedisorderszones

Luteinizing hormone (LH)Lyme diseaseLymph nodesLymphocytesLymphogranuloma venereumLymphomas

M

Macrol idesMacrophagesMaculeMad cow diseaseMagnes ium hydroxideMajor depress ive disorder (MDD)MalabsorptionMalabsorption syndromesMalakoplakiaMalariaMalassezia furfurMale reproductive system disordersMale sexual response cycleMal ignant melanomaMal ignant mesothel iomaMal ingeringMal lory-Weiss tearMalpracticeManiaMannitolMAOIsMaple syrup urine diseaseMarasmusMarcus Gunn pupi lMarfan syndromeMari juanaMass ive pulmonary embolusMasti ti sMcCune-Albright syndromeMDD. See Major depress ive disorderMeas les vi rus (rubeola)MebendazoleMeckel diverticulumMecl izineMeconium i leusMedica idMedica l ethics

MedicareMedul loblastomaMefloquineMeissner corpuscleMeissner plexus . See Submucosa l plexusMelaninMelanomaMelarsoprolMelasmaMelatoninMEN syndrome. See Multiple endocrine neoplas ia syndromeMénétrier diseaseMeningesMeningiomaMeningi ti sMeningoceleMenopauseMenstrua l cycleMeperidine6-mercaptopurineMerkel tacti le discMesnaMesodermMeta-analys isMetabol ic acidos isMetabol i sm

carbohydrate, protein, and l ipiddrug

Metabol i tesMetachromatic leukodystrophyMetaplas iaMetastasesMetastatic ca lci fi cationMetforminMethadoneMethanolMethemoglobinMethimazoleMethotrexateMethylene blueMethylprednisoloneMetoclopramideMetoprololMetronidazoleMexi letineMGUS. See Monoclonal gammopathy of undetermined s igni ficanceMHC moleculesMI. See Myocardia l infarctionMichael i s -Menten kineticsMiconazoleMicrobiologyMicrogl iaMicroscopic polyangi i ti sMidazolamMidgutMifepris toneMigra ineMi l i tary TBMinimal change diseaseMinorsMinoxidi lMirtazapineMisoprostolMitra l regurgi tationMitra l s tenos isMitra l va lve

prolapseMixed connective ti ssue diseaseMolecular biology techniquesMolesMol luscum contagiosumMonoclonal gammopathy of undetermined s igni ficance (MGUS)MonocytesMononucleos isMontelukastMood disordersMorphineMosaicismMotor fibers

Mucor sp.Mucormycos isMulti -infarct dementiaMultiple endocrine neoplas ia (MEN) syndromeMultiple myelomaMultiple scleros isMumps vi rusMural thrombusMurmursMusculoskeleta l system

anatomydisorders

arthri ti scase presentationsneoplas ia of bonenonneoplastic bone

phys iologyMutationsMyasthenia gravisMycobacterium avium-intracellulareMycobacterium lepraeMycobacterium scrofulaceumMycobacterium tuberculosisMycologyMycoplasma pneumoniaeMycos is fungoidesMyelofibros isMyelomeningoceleMyeloprol i ferative disordersMyenteric (Auerbach) plexusMyocardia l infarction (MI)Myos i ti sMyotonic dystrophy

N

Na lbuphineNaloxoneNaltrexoneNarciss i s tic personal i ty disordersNarcolepsyNE. See NorepinephrineNecros isNegative predictive va lueNeisseria gonorrheaNeisseria meningitidesNelfinavi rNeocortexNeonata l respi ratory dis tress syndrome (RDS)Neoplas ia of boneNeoplastic disordersNeoplastic polypNeostigmineNephron, functional regions ofNephroscleros isNephrotic and nephri tic syndromesNervous system. See also Centra l nervous system

disordersbra in tumorscongenita ldemyel inatingencephal i ti sintracrania l hemorrhageleukodystrophiesmeningi ti smiscel laneousneurocutaneous syndromesneurodegenerativeseizuresspina l cordstroke

embryologyentericparasympatheticpharmacology

analges icsanestheticsantiepi leptics

antiparkinsonian agentssympathetic

NeuroblastomaNeurocutaneous syndromesNeurodegenerative disordersNeurofibromatos isNeurogenic shockNeuroleptic mal ignant syndrome (NMS)NeurotransmittersNeutrophi l sNevirapineNiacinNiclosamideNiemann-Pick diseaseNifedipineNifurtimoxNitrateNitratesNitric oxide (NO)NitrofurantoinNitropruss ideNitrosoureasNitrous oxideNizatidineNMS. See Neuroleptic mal ignant syndromeNO. See Ni tric oxideNocardiaNoduleNondis junction disorderNongonococca l septic arthri ti sNon-Hodgkin lymphomaNonmaleficenceNonneoplastic bone disordersNonneoplastic GI disordersNonnucleos ide RT inhibi torsNonsteroida l anti -inflammatory drugs (NSAIDS)Norepinephrine (NE)Northern blotNortriptyl ineNorwalk vi rusNSAIDS. See Nonsteroida l anti -inflammatory drugsNucleos ide RT inhibi torsNul l hypothes isNutmeg l iverNutri tionNystatin

O

Oat cel l carcinomaObeseObsess ive-compuls ive disorder (OCD)Obstructive lung diseaseObstructive s leep apneaOCD. See Obsess ive-compuls ive disorderOCPs . See Ora l contraceptivesOctreotideOdds ratioOedipus complexOlanzapineOl igodendrocyteOl igodendromaOmeprazoleOnchocerca volvulusOndansetronOperant conditioningOperonOpioidsOpisthorchis sinensisOppos i tional defiant disorderOpsonizationOral advance di rectiveOra l contraceptives (OCPs)Ora l ha i ry leukoplakiaOra l hypoglycemicsOrganophosphatesOrthomyxoviridaeOsel tamivi r

Osgood-Schlatter diseaseOs ler nodesOstei ti s fibrosa cysticaOsteoarthri ti sOsteoblastomaOsteochondromaOsteogenes is imperfectaOsteomaOsteomalaciaOsteomyel i ti sOsteopetros isOsteoporos isOsteosarcomaOti ti sOvarian carcinomaOvarian cystsOvarian neoplasmOvarian tumorsOxytocin

P

P va luePacinian corpusclePacl i taxelPaget diseasePalmar reflexPancoast syndromePancreasPancreatic cancerPancreatic ductPancreatic endocrine tumorsPancreati ti sPanic disorderPantoprazolePantothenatePantothenic acidPapez ci rcui tPapi l lary carcinomaPapovavirusPapuleParacoccidioidomycos isPara influenza vi rusesParaneoplastic syndromesParaphi l iaParas i tologyParasomniaParasympathetic nervous systemParathyroid adenomasParathyroid glandsParathyroid hormone (PTH)Parkinson diseaseParoxetineParoxysmal nocturnal hemoglobinuriaParvovirusPasteurellaPatchPatent ductus arteriosus (PDA)Pathologic griefPCAM. See Platelet cel l adhes ion moleculePCP. See Phencycl idine hydrochloridePDA. See Patent ductus arteriosusPectinate l inePediatric disorders , of gastrointestina l systemPel lagraPelvic inflammatory disease (PID)Pemphigus vulgarisPenici l laminePenici l l inPeni le neoplasmPentamidinePentose phosphate pathwayPeps inogenPeptic ulcer diseasePeptidoglycanPericardi ti sPericardiumPeriphera l blood smear

Permeabi l i tyPerphenazinePersonal i ty disordersPeutz-Jeghers syndromePeyer patchesPeyronie diseasep53P–450PharmacodynamicsPharmacology. See also specific drugs

bas iccardiovascular systemendocrine systemgastrointestina l systemgenitourinary systemnervous systemfor psychiatric disordersrespiratory system

Pharyngi ti sPhencycl idine hydrochloride (PCP)PhenelzinePhenobarbi ta lPhenoxybenzaminePhentolaminePhenyla laninePhenylephrinePhenylketonuriaPhenytoinPheochromocytomaPhobiaPhrenic nervePhys ica l chi ld abusePhys iology

cardiovascular systemendocrine systemgastrointestina l systemgenitourinary systemmusculoskeleta l systemreproductive systemrespiratory systemsleep

PhysostigminePick diseasePicornavi rusesPID. See Pelvic inflammatory diseasePigmentsPi locarpinePindololPinworm infectionPiogl i tazonePiperaci l l inPi tui tary adenomaPitui tary glandPitui tary tumorPlacenta accretaPlacenta previaPlaquePlatelet aggregationPlatelet cel l adhes ion molecule (PCAM)Pleomorphic adenomaPleura l effus ionPneumocystisPneumoniaPol iomyel i ti sPol iovi rusPolycystic kidney diseasePolycystic ovarian syndromePolycythemia veraPolymyalgia rheumaticaPolymyos i ti sPolymyxinsPontiac feverPorta hepatisPorta l -systemic shuntPort-wine s ta in bi rthmarkPosaconazolePos i tive predictive va luePosterior pi tui taryPosttraumatic s tress disorder (PTSD)

Potass ium iodidePotencyPott diseasePotter syndromePoxviridaePPD test. See Puri fied protein derivative testPR interva lPra l idoximePraziquantelPrazos inPrecis ionPrecocious pubertyPrednisonePreeclamps iaPregnancyPresbycus isPreva lencePriapismPrimaquinePrimary bi l iary ci rrhos isPrionsProbenecidProca inamideProca ineProgesteroneProgress ive multi foca l leukoencephalopathyProlactinProlactinomaPropafenonePropofolPropranololPropyl thiouraci lProstaglandinsProstate cancerProstate glandProstatic carcinomaProstati ti sProtamine sul fateProtease inhibi torsProtein(s )

digestion ofmetabol i sm

ProteusProtozoaP-selectinPseudogoutPseudohyperparathyroidismPseudohypoparathyroidismPseudomembranous col i ti sPseudomonasPs i ttacos isPsorias isPsychiatric disorders

chi ldhoodmoodpersonal i typsychopharmacologypsychosessomatoform

PsychiatryPsychoanalys isPsychologyPsychopharmacologyPsychosesPTH. See Parathyroid hormonePTSD. See Posttraumatic s tress disorderPubl ic hea l thPulmonary edemaPulmonary embol ismPulmonary fibros isPulmonary infectionsPulmonary thrombusPulmonic s tenos isPulse pressurePulvinarPupi l lary di lationPuri fied protein derivative (PPD) testPurkinje systemPustule

Pyelonephri ti sPyloric s tenos isPylorusPyogenic osteomyel i ti sPyrantel pamoatePyridostigminePyridoxinePyrimethaminePyruvate dehydrogenase deficiency

Q

Q feverQRS complexQuetiapineQuinidineQuinineQuinolones

R

RA. See Rheumatoid arthri ti sRabiesRabies vi rusRadiationRadioimmunoassay (RIA)Raloxi feneRanitidineRapidly progress ive glomerulonephri ti s (RPGN)RAS. See Renal artery s tenos isRaynaud diseaseRBF. See Renal blood flowRDS. See Neonata l respi ratory dis tress syndrome (RDS)Reactive arthri ti sReactive oxygen speciesReassortmentReceptive relaxationRecombinationRed blood cel l productionRegress ionReiter syndromeRel iabi l i tyREM s leepRenal agenes isRenal artery s tenos is (RAS)Renal blood flow (RBF)Renal cel l carcinomaRenal cri s i sRenal dysgenes isRenal fa i lureRenal infarctRenal infectionsRenal papi l lary necros isRenal s tonesRenal tubular acidos is (RTA)Renal tumorsRenal vascular diseaseRenin-angiotens in-a ldosterone systemReovirusesReproductive system

anatomydisorders

breastcase presentationsfemalemale

embryologyphys iologypregnancy and

ReserpineRespiration, neurologic regulation ofRespiratory syncytia l vi rus (RSV)Respiratory system

anatomydisorders

ARDScase presentations

cystic fibros islung cancerneonata l RDSobstructive lung diseaseother pathologypulmonary edemapulmonary embol ismpulmonary infectionsrestrictive lung disease

embryologypharmacologyphys iology

Restriction fragment length polymorphism analys isRestrictive cardiomyopathyRestrictive lung diseaseRetinolRetrovi rusRett syndromeReye syndromeRh incompatibi l i tyRheumatic feverRheumatic heart diseaseRheumatoid arthri ti s (RA)RhinovirusesRhizopus sp.RIA. See RadioimmunoassayRibavi rinRiboflavinRicketsRickettsiaRi fampinRimantadineRisedronateRisperidoneRitodrineRitonavirRNARocky Mounta in spotted feverRodsRooting reflexRose gardener diseaseRos igl i tazoneRotavi rusRoth spotsRotor syndromeRPGN. See Rapidly progress ive glomerulonephri ti sRSV. See Respiratory syncytia l vi rusRTA. See Renal tubular acidos isRubel la vi rusRubeola . See Meas les vi rusRuffini corpuscle

S

SA node. See Sinoatria l nodeSal icylates . See Aspi rinSa l icyl ic acidSa l ivaSa l ivary glandsSalmonellaSan Joaquin feverSaquinavi rSarcoidos isSarcoma botryoidesSca lded skin syndromeScar formationScarlet feverSchistosomaSchizoaffective disorderSchizoid personal i ty disordersSchizophreniaSchizophreni form disorderSchizotypal personal i ty disordersSchwannomaSciaticaSclerodermaScleros ing retroperi toni ti sScol ios is

Screening toolsScurvySecond messengersSecretinSeizuresSelegi l ineSelenium sul fideSEM. See Standard error of the meanSensory fibersSeptic shockSeronegative spondyloarthropathiesSerotoninSertra l ineSevere combined immunodeficiencySex hormonesSexual chi ld abuseSexual response cycleSexual i tySézary syndromeShaken baby syndromeShared psychotic disorderSheehan syndromeShigella dysenteriaeShockShy-Drager syndromeSIADH. See Syndrome of inappropriate antidiuretic hormoneSi ldenafi lSi l i cos isSimmonds diseaseSinoatria l (SA) nodeSjögren syndromeSkeleta l muscleSkin

anatomydisordersembryologyhis tology

SLE. See Systemic lupus erythematosusSleepSleep terror disorderSmooth muscleSMX. See Sul famethoxazoleSodium bicarbonateSodium stibogluconateSomatization disorderSomatoform disordersSomatostatinSota lolSouthern blotSpeci fici tySpermSpherocytos isSphincter of OddiSpider telangiectas iaSpina bi fidaSpina l cord disordersSpina l tractsSpira l va lveSpironolactoneSpleenSpongi form encephalopathySquamous cel l carcinomaSSRIsStandard error of the mean (SEM)Staphylococcus aureusStaphylococcus epidermidisStarl ing equationStatinsStatis tica l dis tributionsStavudine (d4T)STDsSteatorrheaSteroid hormonesSteroidsStevens-Johnson syndromeStomachStreptococcus sp.

agalactiaepneumoniae

pyogenesviridans

Stretch reflexStriata l motor systemStrokeSturge-Weber syndromeSubarachnoid spaceSubdura l hematomaSubependymal germinal matrix bleedSubmucosal (Meissner) plexusSuboptimal renal perfus ionSubstance abuseSubstance PSuccimerSuccinylchol ineSuicideSul fadoxineSul famethoxazole (SMX)Sul fonamidesSul fonesSul fonylureasSul foxoneSuperegoSuperior vena cava syndromeSuraminSurfactantSympathetic nervous systemSyncopeSyncytiotrophoblastSyndrome of inappropriate antidiuretic hormone (SIADH)Syphi l i sSyringomyel iaSystemic lupus erythematosus (SLE)Systemic scleros isSystol ic blood pressure

T

T lymphocyteTabes dorsa l i sTaenia soliumTakayasu arteri ti sTamoxi fenTamponadeTardive dyskines iaTay-Sachs diseaseTazobactamTB. See Tuberculos isTCAsTEF. See Tracheoesophageal fi s tulaTeichoic acidTempora l arteri ti sTeniae col iTeratogensTerazos inTerbinafineTerbuta l ineTerfenadineTestesTesticular tumorsTestosteroneTetraca ineTetracycl ineTetra logy of Fa l lotThalamusThalassemiaThal idomideT-helper cel lThenar eminenceTheophyl l ineTherapeutic indexThiabendazoleThiamineThiazides6-thioguanineThiopenta lThioridazineThiosul fate

Thoracic outlet syndrome3TC. See LamivudineThrombolyticsThrombos isThrombotic thrombocytopenic purpura (TTP)ThrushThymic aplas ia (DiGeorge syndrome)Thyroid glandThyroidi ti sTiclopidineTinea corporisTinea cruris/capi ti s /corporis/unguium/pedisTinea nigraTinea vers icolorTi rofibanTissue repairTMP. See TrimethoprimTMP-SMXTNF. See Tumor necros is factorTocainamideTolbutamideTors ionTourette syndromeToxic megacolonToxic shock syndromeToxocara canisToxoplasmatPATracheoesophageal fi s tula (TEF)Transference reactionTrans ient i schemic attackTrans i tional cel l carcinomaTransmural infarctionTransplant rejectionTranylcypromineTrastuzumabTrazodoneTrench feverTriamcinoloneTriamtereneTrichinella spiralisTrichomonas vaginalisTrichomonias isTrichophytonTricuspid regurgi tationTricycl ic antidepressantsTri fluoperazineTrigeminal neura lgiaTrimethoprim (TMP)Trisomy 21Troponin testTrousseau syndromeTrypanosomaTryps int-testTTP. See Thrombotic thrombocytopenic purpuraTuberculos is (TB)Tuberous scleros isTubocurarineTubulointersti tia l diseasesTumor markersTumor necros is factor (TNF)Turcot syndromeTylenol . See AcetaminophenType I and II errorsTyramineTyros ine

U

UC. See Ulcerative col i ti sUlcerative col i ti s (UC)Undulant feverUrea cycleUrethri ti sUrinary incontinenceUrinary system. See Genitourinary systemUrinary system disorders

Urinary tract infection (UTI)Urinary tract tumorsUrobi l inogenUrol i thias isUrothel ia l carcinomaUterine l igamentsUterusUTI. See Urinary tract infection

V

VaccinesVaginismusVagini ti sVagus nerveValacyclovi rVal idi tyValproic acidValsa lva maneuverVancomycinVardenafi lVariable ratioVaricel la -zoster vi rus (VZV)VaricoceleVascular cel l adhes ion molecule (VCAM)Vascular tumorsVascul i tidesVasoconstrictionVasodi lationVasopress inVATER syndromeVCAM. See Vascular cel l adhes ion moleculeVecuroniumVenlafaxineVeno-occlus ive diseaseVentra l spina l artery occlus ionVentricular septa l defect (VSD)Verapami lVerruca vulgarisVes icleVestibular schwannomaVestibular systemVibrioVinblastineVincris tineVira l hepati ti sVi rchow nodeViruses . See also specific viruses

antivi ra l agents andinfectious diseasestaxonomy

VitaminsViti l igoVolume, lungVolume of dis tributionVolvulusVon Hippel -Lindau diseaseVon Reckl inghausen diseaseVon Reckl inghausen disease of boneVon Wi l lebrand diseaseVoriconazoleVSD. See Ventricular septa l defectVZV. See Varicel la -zoster vi rus

W

Waiter’s tip pa lsyWaldenstrom macroglobul inemiaWal lerian degenerationWarfarinWaterhouse-Friderichsen syndromeWBC neoplasms. See White blood cel l neoplasmsWegener granulomatos isWei l diseaseWei l -Fel ix reactionWerdnig-Hoffman diseaseWernicke encephalopathy

Wernicke-Korsakoff syndromeWestern blotWhipple diseaseWhite blood cel l (WBC) neoplasmsWi lson diseaseWiskott-Aldrich syndromeWoolsorter’s diseaseWuchereria bancrofti

X

XanthomaXeroderma pigmentosaXL agammaglobul inemia. See X-l inked agammaglobul inemiaXL recess ive disorders . See X-l inked recess ive disordersX-l inked (XL) agammaglobul inemiaX-l inked (XL) recess ive disorders

Y

YawsYel low fever vi rusYersinia

Z

Zafi rlukastZa lci tabine (ddC)Zanamivi rZenker diverticulumZidovudine (AZT)Zi leutonZol l inger-El l i son syndrome